You are on page 1of 268

十年(2013-2022)高考英语真题分项详解

阅读理解·说明文体类
2021-2022 题组

(2022·全国)Can a small group of drones ( 无人机) guarantee the safety and reliability

of railways and, at the same time, help railway operators save billions of euros each year?

That is the very likely future of applying today’s “eyes in the sky” technology to making sure

that the millions of kilometres of rail tracks and infrastructure ( 基础设施) worldwide are safe

for trains on a 24/7 basis.

Drones are already being used to examine high-tension electrical lines. They could do

precisely the same thing to inspect railway lines and other vital aspects of rail infrastructure

such as the correct position of railway tracks and switching points. The more regularly they

can be inspected, the more railway safety, reliability and on-time performance will be

improved. Costs would be cut and operations would be more efficient ( 高 效 ) across the

board.

That includes huge savings in maintenance costs and better protection of railway

personnel safety. It is calculated that European railways alone spend approximately 20 billion

euros a year on maintenance, including sending maintenance staff, often at night, to inspect

and repair the rail infrastructure. That can be dangerous work that could be avoided with

drones assisting the crews’ efforts.

By using the latest technologies, drones could also start providing higher-value services

for railways, detecting faults in the rail or switches, before they can cause any safety

problems. To perform these tasks, drones for rail don’t need to be flying overhead. Engineers

are now working on a new concept: the rail drones of the future. They will be moving on the

track ahead of the train, and programmed to run autonomously. Very small drones with

advanced sensors and AI and travelling ahead of the train could guide it like a co-pilot. With

their ability to see ahead, they could signal any problem, so that fast-moving trains would be

able to react in time.

157. What makes the application of drones to rail lines possible?

A. The use of drones in checking on power lines.

学科网(北京)股份有限公司
B. Drones’ ability to work at high altitudes.

C. The reduction of cost in designing drones.

D. Drones’ reliable performance in remote areas.

158. What does “maintenance” underlined in paragraph 3 refer to?

A. Personnel safety. B. Assistance from drones.

C. Inspection and repair. D. Construction of infrastructure.

159. What function is expected of the rail drones?

A. To provide early warning. B. To make trains run automatically.

C. To earn profits for the crews. D. To accelerate transportation.

160. Which is the most suitable title for the text?

A. What Faults Can Be Detected with Drones

B. How Production of Drones Can Be Expanded

C. What Difficulty Drone Development Will Face

D. How Drones Will Change the Future of Railways

【答案与解析】本文是一篇说明文。应用今天的“空中之眼”的技术,无人机能在保

证铁路安全可靠的同时又能帮助铁路运营商每年节省数十亿欧元。
157. 推 理 判 断 题 。 根 据 文 章 第 二 段 “ Drones are already being used to examine high-

tension electrical lines. They could do precisely the same thing to inspect railway lines and

other vital aspects of rail infrastructure such as the correct position of railway tracks and

switching points.(无人机已经被用于检查高压电线。他们完全可以做同样的事情来检查

铁路线路和铁路基础设施的其他重要方面,如铁路轨道和换乘点的正确位置 )”可推知,

无人机在高空工作的能力使无人机应用于铁路线路成为可能。故选 B。

158. 词 义 猜 测 题 。 根 据 后 文 “ It is calculated that European railways alone spend

approximately 20 billion euros a year on maintenance, including sending maintenance staff,

often at night, to inspect and repair the rail infrastructure. ( 据统计,仅欧洲铁路公司每年在

铁路维护上的花费就约为 200 亿欧元,其中包括经常在夜间派遣维修人员检查和维修

铁 路 基 础 设 施 )” 可 知 花 在 maintenance 上 的 费 用 是 用 于 “ inspect and repair the rail

infrastructure (检查和维修铁路基础设施”,由此可知“ That includes huge savings in

maintenance costs and better protection of railway personnel safety” 是指大幅节省检修成本

和更好地保护铁路人员安全,划线词和 C 项:Inspection and repair(检修)含义相近。故

选 C。

159. 推理判断题。根据文章最后一段 “ Very small drones with advanced sensors and AI

and travelling ahead of the train could guide it like a co-pilot. With their ability to see ahead,
they could signal any problem, so that fast-moving trains would be able to react in time.( 带有

先进传感器和人工智能的非常小的无人机可以像副驾驶一样在火车前面行驶。 凭借他

们的预见能力,他们可以发出任何问题的信号,以便快速行驶的火车能够及时做出反

应。)”可知对于无人机期待的功能是提前发现问题。故选 A。

160. 主 旨 大 意 题 。 根 据 文 章 主 题 段 第 一 段 “ Can a small group of drones( 无 人

机 )guarantee the safety and reliability of railways and, at the same time, help railway

operators save billions of euros each year? That is the very likely future of applying today’s

“eyes in the sky” technology to making sure that the millions of kilometers of rail tracks and

infrastructure( 基础设施 )worldwide are safe for trains on a24/7 basis.( 一小群无人机能否在

保证铁路安全可靠的同时,帮助铁路运营商每年节省数十亿欧元 ?这很可能是应用今天

的“空中之眼”技术的未来,以确保全球数百万公里的铁路轨道和基础设施全天候安

全运行。 )”以及后文第二段讲到了使用无人机检查电力线路使无人机应用于铁路线路

成为可能;第三段讲到了使用无人机大幅节省维护成本和更好地保护铁路人员安全;第

四段讲到了通过使用最新的技术,无人机还可以开始为铁路提供更高的价值,可知文

章主要讲述了无人机将如何改变铁路的未来,所以 D 项“无人机将如何改变铁路的未

来。”符合文章中心思想,适合作为本文的最佳标题。故选 D。

( 2022· 全国) The Government’s sugar tax on soft drinks has brought in half as much

money as Ministers first predicted it would generate, the first official data on the policy has

shown.

First announced in April, 2016, the tax which applies to soft drinks containing more than

5g of sugar per 100ml, was introduced to help reduce childhood obesity ( 肥胖). It is believed

that today’s children and teenagers are consuming three times the recommended level of

sugar, putting them at a higher risk of the disease.

Initially the sugar tax was expected to make £520m a year for the Treasury. However,

data of the first six months showed it would make less than half this amount. At present it is

expected to generate £240m for the year ending in April 2019, which will go to school sports.

It comes after more than half of soft drinks sold in shops have had their sugar levels cut

by manufacturers ( 制造商 ) so they can avoid paying the tax. Drinks now contain 45 million

fewer kilos of sugar as a result of manufacturers’ efforts to avoid the charge, according to

Treasury figures. Since April drinks companies have been forced to pay between 18p and 24p

for every litre of sugary drink they produce or import, depending on the sugar content.

学科网(北京)股份有限公司
However, some high sugar brands, like Classic Coca Cola, have accepted the sugar tax

and are refusing to change for fear of upsetting consumers. Fruit juices, milk-based drinks

and most alcoholic drinks are free of the tax, as are small companies manufacturing fewer

than 1m litres per year.

Today’s figures, according to one government official, show the positive influence the

sugar tax is having by raising millions of pounds for sports facilities ( 设 施 ) and healthier

eating in schools. Helping the next generation to have a healthy and active childhood is of

great importance, and the industry is playing its part.

161. Why was the sugar tax introduced?

A. To collect money for schools. B. To improve the quality of drinks.

C. To protect children’s health. D. To encourage research in education.

162. How did some drinks companies respond to the sugar tax?

A. They turned to overseas markets. B. They raised the prices of their products.

C. They cut down on their production. D. They reduced their products’ sugar

content.

163. From which of the following is the sugar tax collected?

A. Most alcoholic drinks. B. Milk-based drinks.

C. Fruit juices. D. Classic Coke.

164. What can be inferred about the adoption of the sugar tax policy?

A. It is a short-sighted decision. B. It is a success story.

C. It benefits manufacturers. D. It upsets customers.

【答案与解析】本文是一篇说明文。文章主要介绍了英国政府对软饮料征收的糖税来

解决儿童以及青少年的健康问题,同时该收入用于学校体育。
161. 细节理解题。根据文章第二段中的“ First announced in April, 2016, the tax which

applies to soft drinks containing more than 5g of sugar per 100ml, was introduced to help

reduce childhood obesity . (该税于 2016 年 4 月首次宣布,适用于每 100 毫升含糖超过

5g 的软饮料,旨在帮助减少儿童肥胖)”可知,征收糖税的目的是帮助儿童减少肥胖 ,

保护儿童健康。故选 C 项。

162. 细节理解题。根据第四段中的“ It comes after more than half of soft drinks sold in

shops have had their sugar levels cut by manufacturers so they can avoid paying the tax. (此

前,制造商已经降低了商店中销售的超过一半的软饮料的含糖量,以避免纳税)”可

知,一些饮料公司通过降低了产品的含糖量来避税。故选 D 项。

163. 细节理解题。根据第五段中的“ However, some high sugar brands, like Classic Coca
Cola, have accepted the sugar tax and are refusing to change for fear of upsetting consumers.

Fruit juices, milk-based drinks and most alcoholic drinks are free of the tax, as are small

companies manufacturing fewer than 1m litres per year. (然而,一些高糖品牌,如经典可

口可乐,已经接受了糖税,并拒绝改变,因为担心会惹恼消费者。果汁、以牛奶为原

料的饮料和大多数酒精饮料是免税的,每年生产不到 100 万升的小公司也是免税

的)”可知,糖税主要来自经典可口可乐这些高糖品牌。故选 D 项。

164. 推理判断题。根据文章最后一段中“ Today’s figures, according to one government

official, show the positive influence the sugar tax is having by raising millions of pounds for

sports facilities and healthier eating in schools. (根据一位政府官员的说法,今天的数据

显示了糖税的积极影响,它为学校的体育设施和健康饮食筹集了数百万英镑)”可推

断,糖税政策的实施是一个成功的政策。故选 B 项。

( 2022· 全 国 ) Like most of us, I try to be mindful of food that goes to waste. The

arugula ( 芝麻菜 )was to make a nice green salad, rounding out a roast chicken dinner. But I

ended up working late. Then friends called with a dinner invitation. I stuck the chicken in the

freezer. But as days passed, the arugula went bad. Even worse, I had unthinkingly bought way

too much; I could have made six salads with what I threw out.

In a world where nearly 800 million people a year go hungry, “food waste goes against

the moral grain,” as Elizabeth Royte writes in this month’s cover story. It’s jaw-dropping how

much perfectly good food is thrown away — from “ugly” (but quite eatable) vegetables

rejected by grocers to large amounts of uneaten dishes thrown into restaurant garbage cans.

Producing food that no one eats wastes the water, fuel, and other resources used to grow

it. That makes food waste an environmental problem. In fact, Royte writes, “if food waste

were a country, it would be the third largest producer of greenhouse gases in the world.”

If that’s hard to understand, let’s keep it as simple as the arugula at the back of my

refrigerator. Mike Curtin sees my arugula story all the time — but for him, it's more like 12

bones of donated strawberries nearing their last days. Curtin is CEO of DC Central Kitchen in

Washington, D. C. , which recovers food and turns it into healthy meals. Last year it

recovered more than 807,500 pounds of food by taking donations and collecting blemished

( 有 瑕 疵 的 ) produce that otherwise would have rotted in fields. And the strawberries?

Volunteers will wash, cut, and freeze or dry them for use in meals down the road.

Such methods seem obvious, yet so often we just don’t think. “Everyone can play a part

学科网(北京)股份有限公司
in reducing waste, whether by not purchasing more food than necessary in your weekly

shopping or by asking restaurants to not include the side dish you won’t eat,” Curtin says.

168. What does the author want to show by telling the arugula story?

A. We pay little attention to food waste. B. We waste food unintentionally at times.

C. We waste more vegetables than meat. D. We have good reasons for wasting food.

169. What is a consequence of food waste according to the test?

A. Moral decline. B. Environmental harm.

C. Energy shortage. D. Worldwide starvation.

170. What does Curtin’s company do?

A. It produces kitchen equipment. B. It turns rotten arugula into clean fuel.

C. It helps local farmers grow fruits. D. It makes meals out of unwanted food.

171. What does Curtin suggest people do?

A. Buy only what is needed. B. Reduce food consumption.

C. Go shopping once a week. D. Eat in restaurants less often.

【答案与解析】本文是一篇说明文。文章介绍了我们日常生活中的食物浪费现象以及

华盛顿 DC 中央厨房的首席执行官科廷为解决食物浪费而采取的努力。

168. 推理判断题。根据第一段中的“ Like most of us, I try to be mindful of food that goes

to waste.(像我们大多数人一样,我努力关注那些被浪费的食物 )”及“But as days passed,

the arugula went bad. Even worse, I had unthinkingly bought way too much; could have

made six salads with what I threw out.( 但随着时间的推移,芝麻菜变坏了。更糟糕的是 ,

我不假思索地买了太多东西;我扔掉的东西可以做六份沙拉 )”可推知,作者想通过讲

述芝麻菜的故事来表明我们有时会无意间浪费食物。故选 B。

169. 细节理解题。根据第三段“ Producing food that no one eats wastes the water, fuel, and

other, resources used to grow it. That makes food waste an environmental problem. In fact,

Royte writes, “if food waste were a country, it would be the third largest producer of

greenhouse gases in the world.”( 生产没人吃的食物会浪费用于种植食物的水、燃料和其

他资源。这使得食物浪费成为一个环境问题。事实上,罗伊特写道,“如果食物浪费

是一个国家,它将是世界上第三大温室气体排放国。” )” 可知,根据文中的说法,浪

费食物的一个后果是对环境的危害。故选 B。

170. 细 节 理 解 题 。 根 据 倒 数 第 二 段 中 的 “ Curtin is CEO of DC Central Kitchen in

Washington. D.C., which recovers food and turns it into healthy meals. Last year it recovered

more than 807,500 pounds of food by taking donations and collecting blemished ( 有瑕疵的 )

produce, that otherwise would have rotted in fields. And the strawberries? Volunteers will
wash, cut, and freeze or dry them for use in meals down the road.( 科廷是华盛顿 DC 中央厨

房的首席执行官,该公司把食物复原,变成健康的食物。去年,该组织通过接受捐赠

和收集有瑕疵的农产品,收回了超过 807500 磅的食物,否则这些农产品就会在地里腐

烂。草莓呢?志愿者们将清洗、切割、冷冻或干燥它们,以便在路上的餐食中使用 )”

可知,科廷的公司用人们不想要的食物重新制作食物。故选 D。

171. 细节理解题。根据最后一段中的““ Everyone can play a part in reducing waste,

whether by not purchasing more food than necessary in your weekly shopping or by asking

restaurants to not include the side dish you won’t eat,” Curtin says.(“ 每个人都可以在减少浪

费方面发挥作用,无论是在每周的购物中不购买不必要的食物,还是要求餐馆不包括

你不吃的配菜,”科廷说)”可知,科廷建议人们只买需要的东西来避免浪费食物。故

选 A。

( 2022· 全国) The elderly residents ( 居民 ) in care homes in London are being given

hens to look after to stop them feeling lonely.

The project was dreamed up by a local charity ( 慈 善 组 织 ) to reduce loneliness and

improve elderly people’s wellbeing, It is also being used to help patients suffering dementia, a

serious illness of the mind. Staff in care homes have reported a reduction in the use of

medicine where hens are in use.

Among those taking part in the project is 80-year-old Ruth Xavier. She said: “I used to

keep hens when I was younger and had to prepare their breakfast each morning before I went

to school. ”

“I like the project a lot. I am down there in my wheelchair in the morning letting the

hens out and down there again at night to see they’ve gone to bed.”

“It’s good to have a different focus. People have been bringing their children in to see

the hens and residents come and sit outside to watch them. I’m enjoying the creative

activities, and it feels great to have done something useful.”

There are now 700 elderly people looking after hens in 20 care homes in the North East,

and the charity has been given financial support to roll it out countrywide.

Wendy Wilson, extra care manager at 60 Penfold Street, one of the first to embark on the

project, said: “Residents really welcome the idea of the project and the creative sessions. We

are looking forward to the benefits and fun the project can bring to people here.”

Lynn Lewis, director of Notting Hill Pathways, said: “We are happy to be taking part in

学科网(北京)股份有限公司
the project. It will really help connect our residents through a shared interest and creative

activities.”

172. What is the purpose of the project?

A. To ensure harmony in care homes. B. To provide part-time jobs for the aged.

C. To raise money for medical research. D. To promote the elderly people’s welfare.

173. How has the project affected Ruth Xavier?

A. She has learned new life skills. B. She has gained a sense of achievement.

C. She has recovered her memory. D. She has developed a strong personality.

174. What do the underlined words “embark on” mean in paragraph 7?

A. Improve. B. Oppose. C. Begin. D. Evaluate.

175. What can we learn about the project from the last two paragraphs?

A. It is well received. B. It needs to be more creative.

C. It is highly profitable. D. It takes ages to see the results.

【答案与解析】本文是一篇说明文。主要讲述了旨在减少孤独,改善老年人的健康状

况的项目。
172. 推理判断题。根据文章第二段“ The project was dreamed up by a local charity ( 慈善

组织) to reduce loneliness and improve elderly people’s wellbeing ( 该项目由当地一家慈善

机构构想,旨在减少孤独,改善老年人的健康状况 )”可知,这个项目的目的是为了提

高老年人的幸福。故选 D。

173. 推理判断题。根据文章第五段““ It’s good to have a different focus. People have

been bringing their children in to see the hens and residents come and sit outside to watch

them. I’m enjoying the creative activities, and it feels great to have done something useful.”

(有不同的关注点很好。人们把自己的孩子带进来看母鸡,居民们也来外面坐着看它们 。

我喜欢创造性的活动,做一些有用的事情的感觉很好)”可推知,Ruth Xavier 通过该项

目获得了一种成就感。故选 B。

174. 词义猜测题。根据文章倒数第二段“ Residents really welcome the idea of the project

and the creative sessions. We are looking forward to the benefits and fun the project can bring

to people here. (居民们非常欢迎该项目的想法和创意会议。我们期待这个项目能给这里

的人们带来好处和乐趣)”以及划线处前的“ one of the first ( 第一批人之一)”可知 Wendy

Wilson 是着手这项工程的人之一,划线处的含义与 C 项:“Begin (开始)”含义相近。

故选 C。

175. 推理判断题。根据文章倒数第二段“ Residents really welcome the idea of the project

and the creative sessions. We are looking forward to the benefits and fun the project can bring
to people here. (居民们非常欢迎该项目的想法和创意会议。我们期待这个项目能给这里

的人们带来好处和乐趣 )” 以及最后一段“ Lynn Lewis, director of Notting Hill Pathways,

said: “We are happy to be taking part in the project. It will really help connect our residents

through a shared interest and creative activities.” (“ 诺丁山路径”的负责人林恩 ·刘易斯说:

我们很高兴能参与这个项目。它将通过共同的兴趣和创造性活动真正帮助我们的居民

联系起来)”可知,该项目的反响很好。故选 A。

( 2022· 全 国 ) Human speech contains more than 2,000 different sounds, from the

common “m” and “a” to the rare clicks of some southern African languages. But why are

certain sounds more common than others? A ground-breaking, five-year study shows that

diet-related changes in human bite led to new speech sounds that are now found in half the

world’s languages.

More than 30 years ago, the scholar Charles Hockett noted that speech sounds called

labiodentals, such as “f” and “v”, were more common in the languages of societies that ate

softer foods. Now a team of researchers led by Damián Blasi at the University of Zurich,

Switzerland, has found how and why this trend arose.

They discovered that the upper and lower front teeth of ancient human adults were

aligned ( 对 齐 ), making it hard to produce labiodentals, which are formed by touching the

lower lip to the upper teeth. Later, our jaws changed to an overbite structure ( 结构), making it

easier to produce such sounds.

The team showed that this change in bite was connected with the development of

agriculture in the Neolithic period. Food became easier to chew at this point. The jawbone

didn’t have to do as much work and so didn’t grow to be so large.

Analyses of a language database also confirmed that there was a global change in the

sound of world languages after the Neolithic age, with the use of “f” and “v” increasing

remarkably during the last few thousand years. These sounds are still not found in the

languages of many hunter-gatherer people today.

This research overturns the popular view that all human speech sounds were present

when human beings evolved around 300,000 years ago. ”The set of speech sounds we use has

not necessarily remained stable since the appearance of human beings, but rather the huge

variety of speech sounds that we find today is the product of a complex interplay of things

like biological change and cultural evolution,“ said Steven Moran, a member of the research

学科网(北京)股份有限公司
team.

176. Which aspect of the human speech sound does Damián Blasi’s research focus on?

A. Its variety. B. Its distribution. C. Its quantity. D. Its development.

177. Why was it difficult for ancient human adults to produce labiodentals?

A. They had fewer upper teeth than lower teeth.

B. They could not open and close their lips easily.

C. Their jaws were not conveniently structured.

D. Their lower front teeth were not large enough.

178. What is paragraph 5 mainly about?

A. Supporting evidence for the research results.

B. Potential application of the research findings.

C. A further explanation of the research methods.

D. A reasonable doubt about the research process.

179. What does Steven Moran say about the set of human speech sounds?

A. It is key to effective communication. B. It contributes much to cultural diversity.

C. It is a complex and dynamic system. D. It drives the evolution of human beings.

【答案与解析】本篇是一篇说明文。主要介绍因为饮食的改变导致了现在在世界上一

半的语言中发现了新的语音。
176. 细节理解题。根据文章第一段中的“ More than 30 years ago, the scholar Charles

Hockett noted that speech sounds called labiodentals, such as “f” and "v", were more

common in the languages of societies that ate softer foods. Now a team of researchers led by

Damian Blasi at the University of Zurich, Switzerland, has found how and why this trend

arose. ( 30 多年前,学者 Charles Hockett 注意到,被称为唇齿音的语音,如“ f” 和

“v”,在吃软食物的社会的语言中更常见。现在,瑞士苏黎世大学的 Damian Blasi 领

导的一组研究人员发现了这一趋势产生的方式和原因。)”可知 Damian Blasi 的研究

重点是在语言的演变上。故选 D 项。

177. 细节理解题。根据第三段中的“ They discovered that the upper and lower front teeth

of ancient human adults were aligned , making it hard to produce labiodentals , which are

formed by touching the lower lip to the upper teeth. Later, our jaws changed to an overbite

structure , making it easier to produce such sounds. (他们发现,古人类的上门牙和下门牙

是对齐的,因此很难产生唇齿音,唇齿音是通过下唇接触上牙齿而形成的。后来,我

们的下颚变成了覆盖咬合结构,更容易发出这样的声音)”可知,因为古代成年人的

下颚结构使他们很难发出唇齿音。故选 C 项。
178. 主旨大意题。根据第五段中的“ Analyses of a language database also confirmed that

there was a global change in the sound of world languages after the so Neolithic age, with the

use of “f” and “v”increasing remarkably during the last few thousand years. These sounds are

still not found in the languages of many hunter-gatherer people today. (对语言数据库的分

析也证实,在新石器时代之后,世界语言的发音发生了全球性的变化,在过去几千年

里,“ f” 和“v”的使用显著增加。这些声音在今天许多狩猎采集者的语言中仍然没有

发现)”可知,第五段主要是通过列明数据分析结果来进一步证明研究结果。故选 A

项。
179. 推理判断题。根据文章最后一段中““ The set of speech sounds we use has not

necessarily remained stable since the appearance of human beings, but rather the huge variety

of speech sounds that we find today is the product of a complex interplay of things like

biological change and cultural evolution,” said Steven Moran, a member of the research team.

(研究小组成员 Steven Moran 说:“ 自从人类出现以来,我们使用的语音不一定保持稳

定,我们今天发现的各种语音都是生物变化和文化进化等复杂相互作用的产

物。”)”可知,Steven Moran 认为语音是一个复杂的动态系统。故选 C 项。

( 2022· 全国 ) Over the last seven years, most states have banned texting by drivers,

and public service campaigns have tried a wide range of methods to persuade people to put

down their phones when they are behind the wheel.

Yet the problem, by just about any measure, appears to be getting worse. Americans are

still texting while driving, as well as using social networks and taking photos. Road accidents,

which had fallen for years, are now rising sharply.

That is partly because people are driving more, but Mark Rosekind, the chief of the

National Highway Traffic Safety Administration, said distracted (分心) driving was "only

increasing, unfortunately."

"Big change requires big ideas." he said in a speech last month, referring broadly to the

need to improve road safety. So to try to change a distinctly modern behavior, lawmakers and

public health experts are reaching back to an old approach: They want to treat distracted

driving like drunk driving.

An idea from lawmakers in New York is to give police officers a new device called the

Textalyzer. It would work like this: An officer arriving at the scene of a crash could ask for

the phones of the drivers and use the Textalyzer to check in the operating system for recent

学科网(北京)股份有限公司
activity. The technology could determine whether a driver had just texted, emailed or done

anything else that is not allowed under New York's hands-free driving laws.

"We need something on the books that can change people's behavior,” said Félix W.

Ortiz, who pushed for the state's 2001 ban on hand-held devices by drivers. If the Textalyzer

bill becomes law, he said, "people are going to be more afraid to put their hands on the cell

phone."

187. Which of the following best describes the ban on drivers' texting in the US?

A. Ineffective. B. Unnecessary.

C. Inconsistent. D. Unfair.

188. What can the Textalyzer help a police officer find out?

A. Where a driver came from. B. Whether a driver used their phone.

C. How fast a driver was going. D. When a driver arrived at the scene.

189. What does the underlined word "something" in the last paragraph refer to?

A. Advice. B. Data. C. Tests. D. Laws.

190. What is a suitable title for the text?

A. To Drive or Not to Drive? Think Before You Start

B. Texting and Driving? Watch Out for the Textalyzer

C. New York Banning Hand-Held Devices by Drivers.

D. The Next Generation Cell Phone: The Textalyzer-

【答案与解析】本文一篇说明文。为解决司机在开车时使用手机造成“分神”,引发

交通事故的问题,纽约的一名立法者提出使用 Textalyzer(短信监控器)的技术来监控

司机在开车的时候是否使用了手机。
187. 推 理 判 断 题 。 根 据 第 一 段 中 “ Over the last seven years, most states have banned

texting by drivers, and public service campaigns have tried a wide range of methods to

persuade people to put down their phones when they are behind the wheel.” (在过去的七年

里,大多数州都禁止司机发短信,公共服务活动也尝试了各种各样的方法来说服人们

在 开 车 时 放 下 手 机 。 ) 以 及 第 二 段 中 “ Yet the problem, by just about any measure,

appears to be getting worse.”(然而,无论以何种标准衡量,这个问题似乎都在恶化。)

可知,大多数州使用了各种各样的方法说服司机们在开车的时候放下手机,可是情况

却越来越糟糕。所以各种方法是无效的。 A 选项 ineffective 意为“无效的”,与此相符。

故选 A。

188. 细节理解题。根据倒数第二段中“ The technology could determine whether a driver

had just texted, emailed or done anything else that is not allowed under New York's hands-
free driving laws.”(这项技术可以确定司机是否发了短信、发了邮件,或者做了纽约免

提驾驶法不允许的其他事情。)可知, Textalyzer 能够确定的是司机是否使用了手机发

短信、邮件以及其他驾驶法不允许的行为。 B 选项意为“是否司机使用了手机。”与

此相符,故选 B。

189. 词义猜测题。根据句中的“ We need something on the books that can change people's

behavior.”(我们需要一些能改变人们行为的东西,)可知,something 是能够改变人们

的行为的事情。根据下文的“ If the Textalyzer bill becomes law, he said, "people are going

to be more afraid to put their hands on the cell phone.” (他说,如果 Textalyzer 法案成为法

律,“人们会更害怕拿起手机。)可知,人们的行为会改变的条件是当 Textalyzer 法案

成为法律。收到法律的约束和惩罚,司机们才不会在开车的时候使用手机。故

something 指代的是法律。故选 D。

190. 主旨大意题。纵观全文,第一段和第二段阐述的是“虽然大多数州已经尝试了各

种各样的方法来说服人们在开车时放下手机。可是问题却越来越严重”。第三段中

“That is partly because people are driving more.” (部分原因是开车的人越来越多)可知,

解释了该行为产生的部分原因。第四段至第五段讲述的是为了解决该问题是纽约立法

者提出了一个新的想法即利用 Textalyzer 技术,来监控司机在开车的时候是否使用了手

机。最后一段讲述的是:相关人士呼吁该项技术能够成为真正的法案由此才能真正的

改变人们的行为。故 B 选项 Texting and Driving? Watch Out for the Textalyzer(发短信还

是在开车?防范短信监控器。)适合文章的标题。故选 B。

( 2022· 全 国 ) As we age, even if we’re healthy, the heart just isn’t as efficient in

processing oxygen as it used to be. In most people the first signs show up in their 50s or early

60s. And among people who don’t exercise, the changes can start even sooner.

“Think of a rubber band. In the beginning, it is flexible, but put it in a drawer for 20

years and it will become dry and easily broken,” says Dr. Ben Levine, a heart specialist at the

University of Texas. That’s what happens to the heart. Fortunately for those in midlife, Levine

is finding that even if you haven’t been an enthusiastic exerciser, getting in shape now may

help improve your aging heart.

Levine and his research team selected volunteers aged between 45 and 64 who did not

exercise much but were otherwise healthy. Participants were randomly divided into two

groups. The first group participated in a program of nonaerobic ( 无 氧 ) exercise—balance

training and weight training—three times a week. The second group did high-intensity

学科网(北京)股份有限公司
aerobic exercise under the guidance of a trainer for four or more days a week. After two

years, the second group saw remarkable improvements in heart health.

“We took these 50-year-old hearts and turned the clock back to 30-or 35-year-old

hearts,” says Levine. “And the reason they got so much stronger and fitter was that their

hearts could now fill a lot better and pump ( 泵送 ) a lot more blood during exercise.” But the

hearts of those who participated in less intense exercise didn’t change, he says.

“The sweet spot in life to start exercising, if you haven’t already, is in late middle age

when the heart still has flexibility,” Levine says. “We put healthy 70-year-olds through a

yearlong exercise training program, and nothing happened to them at all.”

Dr. Nieca Goldberg, a spokeswoman for the American Heart Association, says Levine’s

findings are a great start. But the study was small and needs to be repeated with far larger

groups of people to determine exactly which aspects of an exercise routine make the biggest

difference.

191. What does Levine want to explain by mentioning the rubber band?

A. The right way of exercising. B. The causes of a heart attack.

C. The difficulty of keeping fit. D. The aging process of the heart.

192. In which aspect were the two groups different in terms of research design?

A. Diet plan. B. Professional background.

C. Exercise type. D. Previous physical condition.

193. What does Levine’s research find?

A. Middle-aged hearts get younger with aerobic exercise.

B. High-intensity exercise is more suitable for the young.

C. It is never too late for people to start taking exercise.

D. The more exercise we do, the stronger our hearts get.

194. What does Dr. Nieca Goldberg suggest?

A. Making use of the findings. B. Interviewing the study participants.

C. Conducting further research. D. Clarifying the purpose of the study.

【答案与解析】本文是一篇说明文,主要讲的是锻炼对于心脏的好处。
191. 推 理 判 断 题 。 根 据 第 二 段 的 “ “ Think of a rubber band. In the beginning, it is

flexible, but put it in a drawer for 20 years and it will become dry and easily broken,” says Dr.

Ben Levine, a heart specialist at the University of Texas. That’s what happens to the heart.

(“想想橡皮筋。一开始,它是灵活的,但把它放在抽屉里 20 年,它就会变得干燥,很

容易破碎,”德克萨斯大学的心脏专家本 ·莱文博士说。这就是心脏的变化。)”可知,
莱文想通过提到橡皮筋来解释心脏的老化过程,故选 D。

192. 推理判断题。根据第三段的“ The first group participated in a program of nonaerobic

exercise—balance training and weight training—three times a week. The second group did

high-intensity aerobic exercise under the guidance of a trainer for four or more days a week.

(第一组每周参加三次非有氧运动——平衡训练和重量训练。第二组在教练的指导下每

周进行 4 天或更多的高强度有氧运动。)”可知,两组在研究设计上的不同在于运动类型

的不同,故选 C。

193. 细节理解题。根据第三段的“ The second group did high-intensity aerobic exercise

under the guidance of a trainer for four or more days a week. After two years, the second

group saw remarkable improvements in heart health.( 第二组在教练的指导下每周进行 4 天

或更多的高强度有氧运动。两年后,第二组的心脏健康状况有了显著改善。 )”和第四

段的 ““ We took these 50-year-old hearts and turned the clock back to 30-or 35-year-old

hearts,” says Levine.( 莱 文 说 :“ 我 们 把 这 些 50 岁 的 心 脏 的 时 钟 拨 回 30 或 35 岁 的 心

脏。”)”可知,莱文的研究发现了通过有氧运动,中年人的心脏会变得更年轻,故选

A。

194. 推理判断题。根据最后一段的“But the study was small and needs to be repeated with

far larger groups of people to determine exactly which aspects of an exercise routine make the

biggest difference.( 但这项研究的规模很小,需要在更大的人群中重复进行,以确定日

常锻炼的哪些方面会产生最大的影响。)”可知,妮卡·戈德堡博士建议进行进一步的研

究。故选 C。

(2022·浙江)The United States rose to global power on the strength of its technology,

and the lifeblood that technology has long been electricity. By providing long-distance

communication and energy, electricity created the modem world. Yet properly understood, the

age of electricity is merely the second stage in the age of steam, which began a century

earlier.

"It is curious that no one has put together a history of both the steam and electric

revolutions." writes Maury Klein in his book The Power Makers, Steam, Electricity, and the

Men Invented Modem America. Klein, a noted historian of technology, spins a narrative so

lively that at times it reads like a novel.

The story begins in the last years of the 18th century in Scotland, where Watt perfected

"the machine that changed the world". Klein writes, "America did not invent the steam

学科网(北京)股份有限公司
engine, but once they grasped its passwords they put it to more uses than anyone else. "

Meanwhile, over the course of 19th century, electricity went from mere curiosity to a

basic necessity. Morse invented a code for sending messages over an electromagnetic circuit.

Bell then gave the telegraph a voice. Edison perfected an incandescent bulls that brought

electric light into the American home.

Most importantly, Edison realized that success depended on mass electrification, which

he showed in New York City. With help from Tesla, Westinghouse's firm developed a system

using alternating current, which soon became the major forms of power delivery.

To frame his story, Klein creates the character of Ned, a fictional witness to the progress

brought about by the steams and electric revolutions in America during one man's lifetime.

It's a technique that helps turn a long narrative into an interesting one.

198. What is Klein's understanding of the age of electricity?

A. It is closely linked to the steam age.

B. It began earlier than proper thought.

C. It is a little-studied period of history.

D. It will come to an end sooner or later.

199. What can be inferred about Ned?

A. He was born in New York City. B. He wrote many increasing stories,

C. He created an electricity company. D. He lived mainly in the 19th century.

200. What is the text?

A. A biography. B. A book review. C. A short story. D. A science report.

【答案与解析】本文是说明文。文章按照时间顺序讲述了蒸汽时代和电力时代的联系。
198. 推理判断题。根据第二段“ It is curious that no one has put together a history of both

the steam and electric revolutions.” (我很好奇,竟然没有人把蒸汽和电力革命的历史放

在一起。)可知,在 Klein 看来,电力时代和蒸汽时代是有很紧密的联系的。故选 A。

199. 推理判断题。根据最后一段“To frame his story, Klein creates the character of Ned, a

fictional witness to the progress brought about by the steams and electric revolutions in

America during one man's lifetime.” (为了编织他的故事, Klein 创造了一个人物 Ned ,

它是对美国蒸汽和电力革命在一个人的整个人生中的进程的神奇的见证。)和第四段

“Meanwhile, over the course of 19th century, electricity went from mere curiosity to a basic

necessity.”(同时,在 19 世纪,电力从好奇变成了根需。)可知, Ned 见证了蒸汽时代

和电力革命,所以他应该是生活在 19 世纪。故选 D。

200. 推理判断题。根据最后一段“To frame his story, Klein creates the character of Ned, a
fictional witness to the progress brought about by the steams and electric revolutions in

America during one man's lifetime. It's a technique that helps turn a long narrative into an

interesting one.”(为了构建他的故事,克莱因创造了奈德这个角色,一个虚构的人物,

在一个人的一生中见证了蒸汽和电力革命给美国带来的进步。这是一种有助于将长篇

故事变得有趣的技巧。)可知,这篇文章是一篇书评。故选 B。

( 2022· 浙江) The benefits of regular exercise are well documented but there’s a new

bonus to add to the ever-growing list. New researchers found that middle-aged women who

were physically fit could be nearly 90 percent less likely to develop dementia in later life, and

as they did, it came on a decade later than less sporty women.

Lead researcher Dr. Helena Horder, of the University of Gothenburg in Sweden, said :

"These findings are exciting because it’s possible that improving people's cardiovascular (心

血管的)fitness in middle age could delay or even prevent them from developing dementia. "

For the study, 191 women with an average age of 50 took a bicycle exercise test until

they were exhausted to measure their peak ( 最 大 值 的 ) cardiovascular capacity. The

average peak workload was measured at 103 watts.

A total of 40 women met the criteria for a high fitness level, or 120 watts or higher. A

total of 92 women were in the medium fitness category; and 59 women were in the low

fitness category, defined as a peak workload of 80 watts or less, or having their exercise tests

stopped because of high blood pressure, chest pain or other cardiovascular problems.

These women were then tested for dementia six times over the following four decades.

During that time, 44 of the women developed dementia. Five percent of the highly fit women

developed dementia, compared to 25 percent of the women with medium fitness and 32

percent of the women with low fitness.

"However, this study does not show cause and effect between cardiovascular fitness and

dementia, it only shows an association. More research is needed to see if improved fitness

could have a positive effect on the risk of dementia and also to look at when during a lifetime

a high fitness level is most important. " She also admitted that a relatively small number of

women were studied, all of whom were form Sweden, so the results might not be applicable

to other groups.

201. What is on the ever-growing list mentioned in the first paragraph?

A. Positive effects of doing exercises.

学科网(北京)股份有限公司
B. Exercises suitable for the middle-aged.

C. Experimental studies on diseases.

D. Advantages of sporty woman over man

202. Why did the researchers ask the woman to do bicycle exercise?

A. To predict their maximum heart rate.

B. To assess their cardiovascular capacity

C. To change their habits of working out

D. To detect their potential health problems

203. What do we know about Dr Horder's study?

A. It aimed to find a cure for dementia.

B. Data collection was a lengthy process.

C. Some participants withdrew from it.

D. The results were far from satisfactory.

204. Which of the following is the best title for the text?

A. More Women Are Exercising to Prevent Dementia

B. Middle-Aged Women Need to Do More Exercise

C. Fit Women Are Less Likely to Develop Dementia

D. Biking Improves Women's Cardiovascular Fitness

【答案与解析】这是一篇说明文。主要说明了经常锻炼的中年女性在老年时罹患失智

症的几率会大大降低。
201. 细节理解题。根据文章第一段“ New researchers found that middle-aged women who

were physically fit could be nearly 90 percent less likely to develop dementia (失智症) in

later life, and as they did, it came on a decade later than less sporty women.” (新的研究人员

发现,身体健康的中年女性在晚年患痴呆症的可能性要低近 90%,而且确实如此,与

不太喜欢运动的女性相比,她们患痴呆症的时间要晚十年。)可知,第一段提到了健

身锻炼的好处。故选 A 项。

202. 细 节 理 解 题 。 根 据 上 文 “ These findings are exciting because it’s possible that

improving people's cardiovascular fitness in middle age could delay or even prevent them

from developing dementia.”(这些发现令人兴奋,因为在中年时改善人们的心血管健康

可能会延缓甚至防止他们患上痴呆症。)以及本段“ For the study, 191 women with an

average age of 50 took a bicycle exercise test until they were exhausted to measure their peak

cardiovascular capacity.” (在这项研究中,191 名平均年龄为 50 岁的女性进行了自行车

运动测试,直到她们精疲力竭,以测量她们的心血管峰值能力。)可知,做这项实验
的目的是为了测试女性的心血管峰值能力。故选 B 项。

203. 细节理解题。根据文章第五段“ These women were then tested for dementia six times

over the following four decades.” (这些女性在随后的四十年里测试了六次失智症状

况。)可知,此次实验的数据收集是一个长期的过程。故选 B 项。

204. 推理判断题。根据文章第一段“ New researchers found that middle-aged women who

were physically fit could be nearly 90 percent less likely to develop dementia (失智症) in

later life, and as they did, it came on a decade later than less sporty women.” (新的研究人员

发现,身体健康的中年女性在晚年患痴呆症的可能性要低近 90%,而且确实如此,与

不太喜欢运动的女性相比,她们患痴呆症的时间要晚十年。)以及文章后面几段通过

列举实验目的、过程、方式、结果等,说明了身体健康的中年女性在老年时患失智症

的几率会大大降低。故选 C 项。

10

( 2021· 全国) An Australian professor is developing a robot to monitor the health of

grazing cattle, a development that could bring big changes to a profession that's relied largely

on a low-tech approach for decades but is facing a labor shortage.

Salah Sukkarieh, a professor at the University of Sydney, sees robots as necessary given

how cattlemen are aging. He is building a four-wheeled robot that will run on solar and

electric power. It will use cameras and sensors to monitor the animals. A computer system

will analyze the video to determine whether a cow is sick. Radio tags ( 标签 ) on the animals

will measure temperature changes. The quality of grassland will be tracked by monitoring the

shape, color and texture ( 质地) of grass. That way, cattlemen will know whether they need to

move their cattle to another field for nutrition purposes.

Machines have largely taken over planting, watering and harvesting crops such as com

and wheat, but the monitoring of cattle has gone through fewer changes.

For Texas cattleman Pete Bonds, it's increasingly difficult to find workers interested in

watching cattle. But Bonds doesn't believe a robot is right for the job. Years of experience in

the industry - and failed attempts to use technology - have convinced him that the best way to

check cattle is with a man on a horse. Bonds, who bought his first cattle almost 50 years ago,

still has each of his cowboys inspect 300 or 400 cattle daily and look for signs that an animal

is getting sick.

Other cattlemen see more promise in robots. Michael Kelsey Paris, vice president of the

Oklahoma Cattlemen's Association, said a robot could be extremely useful given rising

学科网(北京)股份有限公司
concerns about cattle theft. Cattle tend to be kept in remote places and their value has risen,

making them appealing targets.

57. What is a problem with the cattle-raising industry?

A. Soil pollution. B. Lack of workers.

C. Aging machines. D. Low profitability.

58. What will Sukkarieh's robot be able to do?

A. Monitor the quality of grass. B. Cure the diseased cattle.

C. Move cattle to another field. D. Predict weather changes.

59. Why does Pete Bonds still hire cowboys to watch cattle?

A. He wants to help them earn a living. B. He thinks men can do the job better.

C. He is inexperienced in using robots. D. He enjoys the traditional way of life.

60. How may robots help with cattle watching according to Michael Kelsey?

A. Increase the value of cattle. B. Bring down the cost of labor.

C. Make the job more appealing. D. Keep cattle from being stolen.

【答案与解析】这是一篇说明文。文章讲述了澳大利亚一名教授正在开发一种机器人 ,

用于监测放牧牛的健康状况。
57. 细 节 理 解 题 。 根 据 文 章 第 一 段 “ An Australian professor is developing a robot to

monitor the health of grazing cattle, a development that could bring big changes to a

profession that's relied largely on a low-tech approach for decades but is facing a labor

shortage.(澳大利亚一名教授正在开发一种机器人,用于监测放牧牛的健康状况,这一

开发可能会给畜牧业带来重大变化。几十年来,畜牧业主要依靠低技术手段,但目前

正面临劳动力短缺的问题。)”可知,畜牧产业面临劳动力短缺的问题。故选 B。

58. 细节理解题。根据文章第二段“The quality of grassland will be tracked by monitoring

the shape, color and texture ( 质地) of grass.(通过监测草的形状、颜色和质地来跟踪草地

的质量。)”可知,Sukkarieh 的机器人可以监测草的质量。故选 A。

59. 细节理解题。根据文章第四段“ But Bonds doesn't believe a robot is right for the job.

Years of experience in the industry - and failed attempts to use technology - have convinced

him that the best way to check cattle is with a man on a horse. (但邦兹认为机器人并不适

合这份工作。多年的行业经验——以及使用技术的失败尝试——使他确信,检查牛的

最好方法是让一名男子骑在马上。)”可知,皮特 ·邦兹仍然雇佣牛仔看牛是因为他认

为人能做得更好。故选 B。

60. 细 节 理 解 题 。 根 据 文 章 最 后 一 段 “ Michael Kelsey Paris, vice president of the

Oklahoma Cattlemen's Association, said a robot could be extremely useful given rising
concerns about cattle theft. (俄克拉荷马州养牛人协会副会长迈克尔 ·凯尔西·帕里斯说,

鉴于人们对偷牛的担忧不断上升,机器人可能会非常有用。)”可知,机器人帮助照

看牛,可以防止牛被偷。故选 D。

11

(2021·天津)A trial project by the Montreal Children's Hospital suggested that the use

of medical hypnosis(催眠)can reduce pain and anxiety in patients. The project also resulted in

a reduction in the amount of medicines used to perform medical-imaging imaging( 医学影像)

procedures.

“During the examination children don't move. It works perfectly. It's amazing,“ said

Johanne   L'Ecuyer, a medical-imaging technologist at the hospital.

The project was inspired by a French team from Rouen University Hospital Centre

where examinations are done under hypnosis instead of general anesthesia( 麻醉).

A French medical-imaging technologist-also a hypnotist — was invited to train a few

members in the medical-imaging department of the children's hospital. In all, 80 examinations

were conducted for the project between January and September, 2019, focusing on the

imaging procedures that would cause anxiety.

Hypnosis is not a state of sleep: It is rather a modified( 改变的 )state of consciousness.

The technologist will guide the patient to this modified state—an imaginary world that will

disassociate itself more and more from the procedure that follows.

“The technologist must build up a story with the patient," Ms. L'Ecuyer said. "The

patient is left with the power to choose what he wants to talk about. Do you play sports? Do

you like going to the beach? We establish a subject that we will discuss throughout the

procedure."

Everything that happens next during the procedure must be related to this story — an

injection (注射) becomes the bite of an insect; the heat on the skin becomes the sensation

of the sun and a machine that rings becomes a police car passing nearby.

“The important thing is that the technologist associates what is happening outside the

patient's body with what the patient sees in his head," Ms. L'Ecuyer said. "It requires

creativity on the part of the technologist, imagination, a lot of patience and kindness."

The procedure appealed to the staff a lot when it was introduced in January. It spread

like wildfire that someone from France was here to train the technologists,"   Ms. L'Ecuyer

said. She added that she had a line of staff at her door wanting to take the training.

学科网(北京)股份有限公司
84. One of the results produced by the trial project is ________ .

A. a better understanding of children

B. less use of certain medicines

C. new medical-imaging technology

D. an improved reputation of the hospital

85. The French technologist came to the children's hospital to ________.

A. assist in treating a patient

B. carry out hypnosis training

C. start up a new department

D. learn about the procedure

86. According to Paragraph 5, hypnosis works by ________.

A. creating a perfect world for patients

B. forcing patients into a state of deep sleep

C. putting patients into an unconscious state

D. leading patients' consciousness away from reality

87. What can we learn about the story used in the procedure?

A. It should keep pace with the procedure.

B. It reflects the patient's creativity.

C. It is selected by the technologist.

D. It tells what doctors are doing to the patient.

88. The procedure was received among the staff with ________.

A. uncertainty

B. enthusiasm

C. worry

D. criticism

89. What is the passage mainly about?

A. An easy way to communicate with patients.

B. The standard method of conducting hypnosis.

C. An introduction of medical-imaging technology.

D. The use of hypnosis in medical-imaging procedures.

【答案与解析】本文是一篇说明文。文章主要讲述了蒙特利尔儿童医院的一个试验项

目表明催眠技术的使用可以减轻病人的痛苦和焦虑。一位法国医学影像技术专家——

也是一位催眠师被邀请到儿童医院的医学影像部门培训几位员工。
84. 细节理解题。根据第一段“ The project also resulted in a reduction in the amount of
medicines used to perform medical-imaging( 医学影像)procedures.”(该项目还导致用于医

学影像程序的药品数量减少。)可知,实验的结果之一就是减少某些药物的使用。故

选 B。

85. 细节理解题。根据第三段“ A French medical-imaging technologist--also a hypnotist --

was invited to train a few members in the medical-imaging department of the children's

hospital.” (一位法国医学影像技术专家——也是一位催眠师被邀请到儿童医院的医学

影像部门培训几位员工)可知,法国技术专家来儿童医院是做催眠培训来的。故选

B。

86. 细 节 理 解 题 。 根 据 第 五 段 第 一 句 “ Hypnosis is not a state of sleep: It is rather a

modified (改变的) state of consciousness. The technologist will guide the patient to this

modified state—an imaginary world that will disassociate itself more and more from the

procedure that follows.” (催眠状态不是一种睡眠的状态:而是一种被改变的意识状态 。

技术专家会引导病人进入这种改变的状态——一个想象中的世界,它会越来越脱离接

下来的程序)可知,催眠是引导病人的意识远离现实,进入一个想象中的世界。故选

D。

87. 推理判断题。根据倒数第三段“ Everything that happens next during the procedure

must be related to this story” (催眠过程中接下来发生的一切都必须和这个故事有关)以

及倒数第二段“ The important thing is that the technologist associates what is happening

outside the patient's body with what the patient sees in his head” (重要的是技术专家把病

人身体外发生的事情和他在大脑里看到的联系起来)可知,故事必须跟整个催眠过程

同步。故选 A。

88. 细节理解题。根据倒数第一段“ The procedure appealed to the staff a lot when it was

introduced in January.” (这个程序在一月份开始引进的时候,吸引了很多员工)以及最

后一句“She added that she had a line of staff at her door wanting to take the training.” (她

补充说,有一队员工在她门口等着接受培训)可知,这个催眠程序受到了员工的欢迎 。

A. uncertainty 不确定;B. enthusiasm 热情;C. worry 担心;D. criticism 批评。故选 B。

89. 主旨大意题。根据文章第一段“ A trial project by the Montreal Children's Hospital

suggested that the use of medical hypnosis( 催眠)can reduce pain and anxiety in patients. The

project also resulted in a reduction in the amount of medicines used to perform medical-

imaging imaging( 医学影像 ) procedures.” (蒙特利尔儿童医院的一个试验项目表明催眠

技术的使用可以减轻病人的痛苦和焦虑。一位法国医学影像技术专家——也是一位催

学科网(北京)股份有限公司
眠师被邀请到儿童医院的医学影像部门培训几位员工)可知,全文主要讲述催眠技术

在医学影像程序中的应用。故选 D。

12

(2021· 天津)In the fictional worlds of film and TV, artificial intelligence (Al) has

been described as so advanced that it is indistinguishable from humans. But what if we're

actually getting closer to a world where Al is capable of thinking and feeling?

Tech company UneeQ is heading for its "digital humans", which appear life like on the

screen not only in terms of language, but also because of facial movements: raised eyebrows,

a smile, even a nod. They look close to a human, but not quite.

What lies beneath UneeQ9 s digital humans? Their 3D faces are modeled on actual

human features. Speech recognition enables them to understand what a person is saying, and

natural language processing is used to work out a response. Meanwhile, another Al company,

Soul Machines, is taking a more biological approach, with a "digital brain", that imitates

aspects of the human brain to adjust the emotions "felt" and "expressed" by its "digital

people".

Shiwali Mohan, an Al scientist at the Palo Research Center, is skeptical of these digital

beings. "They're humanlike in their looks and the way they sound, but that in itself is not

being human," she says. "Human qualities also involve how you think, how you approach

problems, and how you break them down; and that takes a lot of algorithmic ( 算 法 )design.

Designing for human-level intelligence is a different attempt than designing images that

behave like humans." She then continues, “If something looks like a human, we have high

expectations of them, but they might behave differently in ways that humans just instinctively

(直觉地)know how other humans react.

Yet the demand is there, with UneeQ seeing high adoption of its digital employees

across the financial, health care, and commercial sectors ( 行 业 ) . "Unless these sectors

make their business models much more efficient digitally, they might be left behind," says

Chetan Dube, UneeQ9s CEO.

Some other companies are taking their digital beings a step further, enabling

organizations and individuals to create digital humans themselves using free-access platforms

they provide. "The biggest motivation for such platforms is to popularize Al," Dube says.

Mohan is cautious about this approach, yet she supports the purpose behind these digital

beings and is optimistic about where they are headed. "As we develop more advanced Al
technology, we would then have to use new ways of communicating with that technology,she

says. "'Hopefully, all of that is designed to support humans in their goals."

105. According to Para. 2, in what respect ( s ) do UneeQ9s "digital humans" resemble

human beings?

A. In the way they move around.

B. In the way they act and react.

C. In observation and analysis.

D. In speech and facial expressions.

106. Soul Machines’digital brain is a technological breakthrough because it .

A. leams to make proper emotional responses

B. tends to imitate human beings' tone vividly

C. recognizes the speech sounds it receives

D. processes the natural language it hears

107. In Mohan's opinion, what human quality is lacking in digital beings?

A. Calculating brain.

B. Language skills.

C. Instinctive judgements.

D. Problem-solving ability.

108. What makes many sectors employ digital humans?

A. The fear of falling behind in efficiency.

B. The urgency to promote e-commerce.

C. The wish to spread digital technology.

D. The need to upgrade the health care system.

109. What does Mohan think of the future of digital beings?

A. It's well planned.

B. It is promising.

C. It is uncertain.

D. It's quite hopeless.

【答案与解析】这是一篇说明文。短文介绍许多科技公司正在推进、研发“数字人

类”,并被很多部门使用。但一些科学家对此表示怀疑,并指出了“数字人类”并不

是真正的人类,有很多缺陷。但前景还是很有希望的。
105. 细节理解题。根据第二段“ Tech company UneeQ is heading for its "digital humans",

which appear life like on the screen not only in terms of language, but also because of facial

学科网(北京)股份有限公司
movements: raised eyebrows, a smile, even a nod.” (科技公司 UneeQ 正在研发一种“数

字人”,这种人在屏幕上看起来栩栩如生,不仅因为语言,还因为面部动作 :眉毛扬起、

微笑,甚至点头。)由此可知,根据第 2 段,UneeQ9s 的“数字人”在言语和面部表

情上方面与人类相似。故选 D 项。

106. 细节理解题。根据第三段“ Meanwhile, another Al company, Soul Machines, is taking

a more biological approach, with a "digital brain", that imitates aspects of the human brain to

adjust the emotions "fblt" and "expressed" by its “digital people”.” (与此同时,另一家 Al

公司,灵魂机器公司,正在采取一种更具生物性的方法,拥有一个“数字大脑”,它

模仿人脑的各个方面来调节其“数字人”的“ fblt”和“表达”的情绪。)由此可知,

灵魂机器的数字大脑是一项技术突破,因为它倾向于做出适当的情绪反应。故选 A 项。

107. 推理判断题。根据第四段“ "They're humanlike in their looks and the way they sound,

but that in itself is not being human," she says. "Human qualities also involve how you think,

how you approach problems, and how you break them down; and that takes a lot of

algorithmic ( 算 法 )design, but they might behave differently in ways that humans just

instinctively (直觉地)know how other humans react.”(“他们的外表和声音都像人类,但

这本身不是人类,”她说。“人类的品质还包括你如何思考、如何处理问题以及如何

分解问题;这需要大量的算法设计。但他们的行为方式可能不同,人类只是本能地知

道其他人的反应。”)由此判断出,在 Mohan 看来,数字人缺少本能的判断。故选 C

项。
108. 细节理解题。根据第五段“Yet the demand is there, with UneeQ seeing high adoption

of its digital employees across the financial, health care, and commercial sectors ( 行

业 ) .Unless these sectors make their business models much more efficient digitally, they

might be left behind”(但需求依然存在, UneeQ 的数字员工在金融、医疗和商业行业的

使用率很高。除非这些行业将它们的商业模式数字化得更高效,否则它们可能会被甩

在后面)由此可知,害怕效率落后让许多行业使用“数字人类”。故选 A 项。

109. 推理判断题。根据最后一段“Mohan is cautious about this approach, yet she supports

the purpose behind these digital beings and is optimistic about where they are headed.” (

Mohan 对这种方法持谨慎态度,但她支持这些数字生物背后的目的,并对他们的未来

持乐观态度)由此判断出,Mohan 数字人的未来是有希望的。故选 B 项。

13

( 2021· 天津) Art is everywhere. Any public space has been carefully designed by an

artistic mind to be both functional and beautiful. Why, then, is art still so widely considered to
be "the easy subject" at school, insignificant to wider society, a waste of time and effort?

Art can connect culture with commercial products in a way that not many other things

can; art generates money and holds significant emotional and cultural value within

communities. When people attend a concert, they are paying for music, sure, maybe even

hotel rooms, meals, and transport, but they also gain an incredible experience, a unique

atmosphere and a memory that will go through the rest of their lives. People don't just want

material things anymore, they want to experience life 一 the arts are a perfect crossover (交

迭)between culture and commerce.

Furthermore, the arts can bring communities together, reducing loneliness and making

people feel safer. Social bonds are created among individuals when they share their arts

experiences through reflection and discussion, and their expression of common values

through artworks in honour of events significant to a nation's experience.

The arts clearly have a pretty positive impact on physical and psychological health. It is

found that people who frequent cultural places or participate in artistic events are more likely

to gain good health compared to those who do not; more engagement with the arts is linked to

a higher level of people's wellbeing. The Royal Society of Public Health discovered that

music and art, when used in hospitals, help to improve the conditions of patients by reducing

stress, anxiety and blood pressure.

Children who are involved with the arts make greater achievements in their education:

those engaged with drama have greater literary ability while others taking part in musical

practice exhibit greater skills in math and languages. Kids with preference for the arts have a

greater chance of finding employment in the future. Participating in the arts is essential for

child development; encouraging children to express themselves in constructive ways could

help to form healthy emotional responses in later life.

Vital to human life, art is celebrated and used by nations across the world for various

purposes. Life without art would be boring and dead still, for art is a part of what makes us

human.

110. Art products differ from most other commercial products because .

A. most people purchase them for collection

B. they are more expensive and less accessible

C. they have both commercial and cultural values

D. their prices may climb up as time passes

学科网(北京)股份有限公司
111. By sharing their arts experiences, community members can .

A. keep the community safe from illnesses

B. develop a stronger tie between them

C. learn to appreciate their own works of art

D. offer honourable solutions to their problems

112. What can we learn about people who are involved in artistic activities?

A. They enjoy better living conditions.

B. They like to compare themselves with others.

C. They are particularly good at both music and art.

D. They tend to be healthier physically and mentally.

113. How does kids' engagement with the arts benefit them?

A. It promotes their academic performance and emotional growth.

B. It gives them more confidence in exhibiting their learning skills.

C. It inspires their creativity in designing their future career.

D. It helps to make responsible people out of them.

114. What is the best title for this passage?

A. How Art Cures Our Hearts

B. Art: A Blessing to Humankind

C. How Art Benefits Communities

D. Art: A Bridge Between Cultures

【答案与解析】本文是一篇说明文。讲述了艺术这一活动对人类的重要意义,是我们

不可或缺的社会生活之一。从赋予商品的商业价值,人类的身心健康和青少年的学术

和情感发展等方面论述了艺术的重要意义。
110. 细节理解题。根据第一段“Any public space has been carefully designed by an artistic

mind to be both functional and beautiful.” (任何公共空间都是由艺术家精心设计的,既

实用又美观。)以及“ Art can connect culture with commercial products in a way that not

many other things can”(艺术可以将文化与商业产品联系起来,这是其他很多东西都做

不到的。)可以看出,可见艺术参与的作品兼具文化价值和商业价值。故选择 C。

111. 细节理解题。根据第三段“ Social bonds are created among individuals when they

share their arts experiences through reflection and discussion,, and their expression of

common values through artworks in honor of events significant to a nation's experience.”

(当个人通过反思和讨论分享他们的艺术经验,并通过艺术作品表达共同的价值观,

以纪念对一个国家的经验有重要意义的事件时,社会纽带就在他们之间建立起来。) ,
可见通过分享经历,可以建立一种特殊的联系和纽带。故选择 B。

112. 细节理解题。根据第四段“ It is found that people who frequent cultural places or

participate in artistic events are more likely to gain good health compared to those who do

not”(研究发现,经常出入文化场所或参加艺术活动的人比不经常出入文化场所或参

加艺术活动的人更容易获得健康 ;),可见参加艺术活动的人们在身心上都相对更加健

康。故选择 D。

113. 细 节 理 解 题 。 根 据 第 五 段 “ Children who are involved with the arts make greater

achievements in their education” (参与艺术的孩子在他们的教育中取得更大的成就)以

及 “ encouraging children to express themselves in constructive ways could help to form

healthy emotional responses in later life.” (有助于在以后的生活中形成健康的情感反

应。)可见艺术活动促进了孩子们的学业表现和情感成长。故选择 A。

114. 主旨大意题。根据最后一段“ Vital to human life, art is celebrated and used by nations

across the world for various purposes. Life without art would be boring and dead still, for art

is a part of what makes us human.” (艺术对人类的生命至关重要,世界各国为各种目的

庆祝和使用艺术。没有艺术的生活将是无聊和死气沉沉的,因为艺术是我们人类的一

部分。)可见艺术是人类重要的部分,文章中也详细描述了艺术对不同人群的好处。

所以短文的最佳标题为“艺术:人类的福祉”。故选择 B。

14

( 2021· 浙 江 ) At the start of the 20th century, an American engineer named John

Elfreth Watkins made predictions about life today. His predictions about slowing population

growth, mobile phones and increasing height were close to the mark. But he was wrong in

one prediction: that everybody would walk 10 miles a day.

Today, in Australia, most children on average fall 2, 000 steps short of the physical

activity they need to avoid being overweight. In the early 1970s, 40 per cent of children

walked to school, while in 2010, it was as low as 15 per cent.

The decline is not because we have all become lazy. Families are pressed for time, many

with both parents working to pay for their house, often working hours not of their choosing,

living in car-dependent neighborhoods with limited public transport.

The other side of the coin is equally a deprivation: for health and well-being, as well as

lost opportunities ( 机 会 ) for children to get to know their local surroundings. And for

parents there are lost opportunities to walk and talk with their young scholar about their day.

Most parents will have eagerly asked their child about their day, only to meet with a

学科网(北京)股份有限公司
“good”, quickly followed by "I'm hungry". This is also my experience as a mother. But

somewhere over the daily walk more about my son's day comes out. I hear him making sense

of friendship and its limits. This is the unexpected and rare parental opportunity to hear more.

Many primary schools support walking school-bus routes ( 路 线 ) , with days of

regular,   parent-accompanied walks. Doing just one of these a few times a week is better than

nothing. It can be tough to begin and takes a little planning-running shoes by the front door,

lunches made the night before, umbrellas on rainy days and hats on hot ones-but it's certainly

worth trying.

118. Why does the author mention Watkins' predictions in the first paragraph?

A. To make comparisons. B. To introduce the topic.

C. To support her argument. D. To provide examples.

119. What has caused the decrease in Australian children's physical activity?

A. Plain laziness. B. Health problems.

C. Lack of time. D. Security concerns.

120. Why does the author find walking with her son worthwhile?

A. She can get relaxed after work. B. She can keep physically fit.

C. She can help with her son's study. D. She can know her son better.

【答案与解析】这是一篇说明文。文章通过否定沃特金斯的预言,进而提出现在人们

时间的紧缺和陪伴的重要性。
118. 推理判断题。通过文章第二段“ Today: in Australia: most children on average fall 2:

000 steps short of the physical activity they need to avoid being overweight. In the early

1970s, 40 percent of children walked to school while in 2010, it was as low as15 percent.( 今

天:在澳大利亚:大多数孩子平均比避免超重所需的体力活动少了 2000 步。在上世纪

70 年代初,40%的孩子步行上学,而在 2010 年,这一比例降至 15%)”说明作者根据沃

特金斯的预言做了相关调查,并且写下了这篇文章。通读全文得知作者在第一段提到

沃特金斯的预言,是为了介绍文章主题。故选 B 项。

119. 细节理解题。通过文章第三段中“ Families are pressed for time: many with both

parents working to pay for their house, often working hours not of their choosing, living in

car-dependent neighborhoods with limited public transport.( 家庭时间紧迫:许多家庭的父

母都在为房子买单而工作,工作时间往往不是他们自己选择的,他们住在公共交通有

限的依赖汽车的社区)”可知,导致澳大利亚儿童体育活动的减少的原因是时间不够。

故选 C 项。

120. 细节理解题。通过文章倒数第二段中“ But somewhere over the daily walk more


about my son’s day comes out. I hear him making sense of friendship and its limits. This is

the unexpected and rare parental opportunity to hear more.( 但是,在每天散步的某个地方,

更多关于我儿子日常的事情出现了。我听到他在理解友谊及其局限性。这是一个意想

不到的难得的机会,家长听到更多 )”说明作者觉得和儿子一起散步能让她更了解她的

儿子。故选 D 项。

( 2021· 浙江) Researchers say they have translated the meaning of gestures that wild

chimpanzees ( 黑猩猩 ) use to communicate. They say wild chimps communicate 19 specific

messages to one another with a "vocabulary" of 66 gestures. The scientists discovered this by

following and filming groups of chimps in Uganda, and examining more than 5,000 incidents

of these meaningful exchanges.

Dr Catherine Hobaiter, who led the research, said that this was the only form of

intentional communication to be recorded in the animal kingdom. Only humans and chimps,

she said, had a system of communication where they deliberately sent a message to another

group member.

"That's what's so amazing about chimp gestures," she said. "They're the only thing that

looks like human language in that respect. ”

Although previous research has shown that apes and monkeys can understand complex

information from another animal's call, the animals do not appear to use their voices

intentionally to communicate messages. This was a significant difference between calls and

gestures, Dr Hobaiter said.

Chimps will check to see if they have the attention of the animal with which they wish to

communicate. In one case, a mother presents her foot to her crying baby, signaling:" Climb on

me. " The youngster immediately jumps on to its mothers back and they travel off together.

"The big message from this study is that there is another species ( 物 种 ) out there. that is

meaningful in its communication, so that's not unique to humans," said Dr Hobaiter.

Dr Susanne Shultz, an evolutionary biologist from the University of Manchester, said the

study was praiseworthy in seeking to enrich our knowledge of the evolution of human

language. But, she added, the results were "a little disappointing".

"The vagueness of the gesture meanings suggests either that the chimps have little to

communicate, or we are still missing a lot of the information contained in their gestures and

actions," she said. "Moreover, the meanings seem to not go beyond what other animal convey

with non-verbal communication. So, it seems the gulf remains. "

学科网(北京)股份有限公司
121. What do chimps and humans have in common according to Dr Hobaiter?

A. Memorizing specific words. B. Understanding complex information.

C. Using voices to communicate. D. Communicating messages on purpose.

122. What did Dr Shultz think of the study?

A. It was well designed but poorly conducted.

B. It was a good try but the findings were limited.

C. It was inspiring but the evidence was unreliable.

D. It was a failure but the methods deserved praise.

123. What does the underlined word "gulf" in the last paragraph mean?

A. Difference. B. Conflict. C. Balance. D. Connection.

124. Which of the following is the best title for the text?

A. Chimpanzee behaviour study achieved a breakthrough

B. Chimpanzees developed specific communication skills

C. Chimpanzees: the smartest species in the animal kingdom

D. Chimpanzee language: communication gestures translated

【答案与解析】这是一篇说明文。主要介绍了科学家们通过跟踪和拍摄乌干达的黑猩

猩群,翻译出了黑猩猩用来交流的手势含义。
121. 细节理解题。根据第二段中的“ Only humans and chimps, she said, had a system of

communication where they deliberately sent a message to another group member. (她说,只

有人类和黑猩猩有一个交流系统,他们故意向其他成员发送信息)”可知,根据 Dr

Hobaiter,黑猩猩和人类的共同点在于故意传递信息,故选 D。

122. 推理判断题。根据第六段中的“ Dr Susanne Shultz, an evolutionary biologist from the

University of Manchester, said the study was praiseworthy in seeking to enrich our

knowledge of the evolution of human language. But, she added, the results were“a little

disappointing”(曼彻斯特大学的进化生物学家苏珊娜·舒尔茨博士说,这项研究是值得

称赞的,它试图丰富我们对人类语言进化的知识。但是,她补充说,结果“有点令人

失望”)”可知,这项研究是一个好的尝试,但是发现的结果是有限的。故选 B。

123. 词义猜测题。首先根据第三段中的“ They’re the only thing that looks like human

language in that respect. (在这方面,它们是唯一看起来像人类语言的东西)”可知,

黑猩猩的手势交流很像我们人类语言的交流方式。但是根据最后一段中的“ Moreover,

the meanings seem to not go beyond what other animals convey with non-verbal

communications.(此外,这些含义似乎并不超越其他动物通过非语言交流所传达的信

息)”可知,黑猩猩手势的交流方式还是和我们语言的交流方式存在不同的,也就是
“ So, it seems the gulf remains. ( 所 以 , 看 来 差 异 依 然 存 在 ) ” , 故 gulf 的 意 思 是

difference,故选 A。

124. 主旨大意题。根据第一段的“ Researchers say they have translated the meaning of

gestures that wild chimpanzees use to communicate. (研究人员表示,他们已经翻译出了

野生黑猩猩用来交流的手势的含义)”以及文章对这方面的讨论可知,文章主要讲科

学家们对黑猩猩手势的研究及一些成果,所以 D 项:黑猩猩语言:翻译出来的交流手

势,这一题目涵盖文章的内容。故选 D。

15

(2021· 全国)If you ever get the impression that your dog can "tell" whether you look

content or annoyed, you may be onto something. Dogs may indeed be able to distinguish

between happy and angry human faces, according to a new study

Researchers trained a group of 11 dogs to distinguish between images( 图像 )of the same

person making either a happy or an angry face. During the training stage, each dog was

shown only the upper half or the lower half of the person's face. The researchers then tested

the dogs' ability to distinguish between human facial expressions by showing them the other

half of the person's face on images totally different from the ones used in training. The

researchers found that the dogs were able to pick the angry or happy face by touching a

picture of it with their noses more often than one would expect by random chance.

The study showed the animals had figured out how to apply what they learned about

human faces during training to new faces in the testing stage. "We can rule out that the dogs

simply distinguish between the pictures based on a simple cue, such as the sight of teeth,"

said study author Corsin Muller. "Instead, our results suggest that the successful dogs realized

that a smiling mouth means the same thing as smiling eyes, and the same rule applies to an

angry mouth having the same meaning as angry eyes."

"With our study, we think we can now confidently conclude that at least some dogs can

distinguish human facial expressions," Muller told Live Science.

At this point, it is not clear why dogs seem to be equipped with the ability to recognize

different facial expressions in humans. "To us, the most likely explanation appears to be that

the basis lies in their living with humans, which gives them a lot of exposure to human facial

expressions," and this exposure has provided them with many chances to learn to distinguish

between them, Muller said.

132. The new study focused on whether dogs can_________.

学科网(北京)股份有限公司
A. distinguish shapes B. make sense of human faces

C. feel happy or angry D. communicate with each other

133. What can we learn about the study from paragraph 2?

A. Researchers tested the dogs in random order.

B. Diverse methods were adopted during training.

C. Pictures used in the two stages were different

D. The dogs were photographed before the lest.

134. What is the last paragraph mainly about?

A. A suggestion for future studies. B. A possible reason for the study findings.

C. A major limitation of the study D. An explanation of the research method.

【答案与解析】这是一篇说明文。一项研究证明狗能够识别人类的面部表情,但目前

还不清楚它们为什么有这种能力,可能的原因是它们长时间与人类共同生活。
132. 细节理解题。根据第二段的“ Researchers trained a group of 11 dogs to distinguish

between images of the same person making either a happy or an angry face.( 研究人员训练了

11 只狗来区分同一个人脸上的表情是高兴还是愤怒 )”可知,该新研究的关注点是狗是

否能够区分人的面部表情。故选 B。

133. 细节理解题。根据第二段的“ During the training stage, each dog was shown only the

upper half or the lower half of the person’s face. The researchers then tested the dogs’ ability

to distinguish between human facial expressions by showing them the other half of the

person’s face or images totally different from the ones used in training.( 在训练阶段,每只

狗只看到人脸的上半部分或下半部分。研究人员随后测试了狗辨别人类面部表情的能

力,向狗展示了人的另一半面部或与训练中使用的完全不同的图像 )” 可知,在训练和

测试阶段,狗看的照片是不一样的。故选 C。

134. 主旨大意题。根据最后一段的““ To us, the most likely explanation appears to be

that the basis lies in their living with humans, which gives them a lot of exposure to human

facial expressions, and this exposure has provided them with many chances to learn to

distinguish between them” Muller said.(Muller 说:“对我们来说,最可能的解释似乎是,

基于他们与人类生活在一起,这让他们有很多机会接触人类的面部表情,而这种接触

为他们提供了很多机会,让他们学会区分他们。” )”可知,最后一段主要讲述了狗能

够辨别人类面部表情的可能原因。故选 B。

16

( 2022· 全 国 ) Goffin’s cockatoos, a kind of small parrot native to Australasia, have

been shown to have similar shape-recognition abilities to a human two-year-old. Though not
known to use tools in the wild, the birds have proved skilful at tool use while kept in the cage.

In a recent experiment, cockatoos were presented with a box with a nut inside it. The clear

front of the box had a “keyhole” in a geometric shape, and the birds were given five

differently shaped “keys” to choose from. Inserting the correct “key” would let out the nut.

In humans, babies can put a round shape in a round hole from around one year of age,

but it will be another year before they are able to do the same with less symmetrical ( 对称的)

shapes. This ability to recognize that a shape will need to be turned in a specific direction

before it will fit is called an “allocentric frame of reference”. In the experiment, Goffin’s

cockatoos were able to select the right tool for the job, in most cases, by visual recognition

alone. Where trial-and-error was used, the cockatoos did better than monkeys in similar tests.

This indicates that Goffin’s cockatoos do indeed possess an allocentric frame of reference

when moving objects in space, similar to two-year-old babies.

The next step, according to the researchers, is to try and work out whether the cockatoos

rely entirely on visual clues ( 线 索 ), or also use a sense of touch in making their shape

selections.

138. How did the cockatoos get the nut from the box in the experiment?

A. By following instructions. B. By using a tool.

C. By turning the box around. D. By removing the lid.

139. Which task can human one-year-olds most likely complete according to the text?

A. Using a key to unlock a door. B. Telling parrots from other birds.

C. Putting a ball into a round hole. D. Grouping toys of different shapes.

140. What does the follow-up test aim to find out about the cockatoos?

A. How far they are able to see.

B. How they track moving objects.

C. Whether they are smarter than monkeys.

D. Whether they use a sense of touch in the test.

141. Which can be a suitable title for the text?

A. Cockatoos: Quick Error Checkers B. Cockatoos: Independent Learners

C. Cockatoos: Clever Signal-Readers D. Cockatoos: Skilful Shape-Sorters

【答案与解析】本文是一篇说明文。文章主要介绍了一种会识别形状的凤头鹦鹉。
138. 细节理解题。根据文章第一段“ Though not known to use tools in the wild, the birds

have proved skilful at tool use while kept in the cage. ( 虽然人们不知道这些鸟在野外会使

用工具,但事实证明,它们在关在笼子里时就能熟练地使用工具 )”以及“the birds were

学科网(北京)股份有限公司
given five differently shaped “keys” to choose from. Inserting, the correct “keys” would let

out the nut. (研究人员给了这些鸟 5 把形状各异的“钥匙”供它们选择。插入,正确的

“钥匙”会让坚果出来 )” 可知,在实验中,凤头鹦鹉是通过使用工具从盒子里取出坚

果的。故选 B。

139. 细节理解题。根据文章第二段“ In humans, babies can put a round shape in a round

hole from around one year of age ( 在人类身上,婴儿从一岁左右就可以把一个圆形的物

品放进一个圆形的洞里 )” 结合选项,可知,一岁儿童最有可能完成“将一个球放进一

个圆形的洞里”的任务。故选 C。

140. 推理判断题。通过文章最后一段“The next step, according to the researchers, is to try

and work out whether the cockatoos rely entirely on visual clues, or also use a sense of touch

in making their shape selections. (根据研究人员的说法,下一步是尝试弄清楚凤头鹦鹉是

完全依靠视觉线索,还是也使用触觉来选择它们的形状 )” 可推知,后续测试的目的是

了解凤头鹦鹉在测试中是否使用触觉。故选 D。

141. 主旨大意题。通读全文,再结合文章第一段“ Coffin’s cockatoos, a kind of small

parrot native to Australasia, have been shown to have similar shape-recognition abilities to a

human two-year-old. ( 科芬的凤头鹦鹉是一种原产于大洋洲的小鹦鹉,它的形状识别能

力 与 两 岁 的 人 类 相 似 )” 可 推 知 , 本 文 主 要 介 绍 了 会 识 别 形 状 的 凤 头 鹦 鹉 。 D 项

“ Cockatoos: Skilful Shape-Sorters ( 凤头鹦鹉:识别形状的熟练工 )” 符合文意,最适合

作为本文标题。故选 D。

17

( 2021· 北 京 ) Hundreds of scientists, writers and academics sounded a warning to

humanity in an open letter published last December: Policymakers and the rest of us must

engage openly with the risk of global collapse. Researchers in many areas have projected the

widespread collapse as “a credible scenario(情景) this century”.

A survey of scientists found that extreme weather events, food insecurity, and freshwater

shortages might create global collapse. Of course, if you are a non-human species, collapse is

well underway.

The call for public engagement with the unthinkable is especially germane in this

moment of still-uncontrolled pandemic and economic crises in the world's most

technologically advanced nations. Not very long ago, it was also unthinkable that a virus

would shut down nations and that safety nets would be proven so disastrously lacking in

flexibility.
The international scholars’ warning letter doesn't say exactly what collapse will look like

or when it might happen. Collapseology, the study of collapse, is more concerned with

identifying trends and with them the dangers of everyday civilization. Among the

signatories( 签 署 者 ) of the warning was Bob Johnson, the originator of the “ecological

footprint” concept, which measures the total amount of environmental input needed to

maintain a given lifestyle. With the current footprint of humanity, “it seems that global

collapse is certain to happen in some form, possibly within a decade, certainly within this

century,” Johnson said in an email.

“Only if we discuss the consequences of our biophysical limits,” the December warning

letter says, “can we have the hope to reduce their speed, severity and harm”. And yet

messengers of the coming disturbance are likely to be ignored. We all want to hope things

will turn out fine. As a poet wrote,

Man is a victim of dope(麻醉品)

In the incurable form of hope.

The hundreds of scholars who signed the letter are intent( 执 着 ) on quieting hope that

ignores preparedness. “Let's look directly into the issue of collapse,” they say, “and deal with

the terrible possibilities of what we see there to make the best of a troubling future.”

68. What does the underlined word “germane” in Paragraph 3 probably mean?

A. Scientific. B. Credible.

C. Original. D. Relevant.

69. As for the public awareness of global collapse, the author is________.

A. worried B. puzzled

C. surprised D. scared

70. What can we learn from this passage?

A. The signatories may change the biophysical limits.

B. The author agrees with the message of the poem.

C. The issue of collapse is being prioritized.

D. The global collapse is well underway.

【答案与解析】这是一篇说明文,文章阐述了全球崩塌( global collapse )的概念。数

百名科学家、作家和学者在去年 12 月发表的一封公开信中向全人类发出了警告:政策

制定者和我们每个人必须直面“全球崩塌”的风险。文章具体阐释了学者们对这一概

念的定义、理解和它的现实意义。

学科网(北京)股份有限公司
68. 词 义 猜 测 题 。 根 据 该 词 所 在 的 具 体 语 境 , 第 三 段 第 一 句 “ The call for public

engagement with the unthinkable is especially germane in this moment of still-uncontrolled

pandemic and economic crises in the world's most technologically advanced nations.” (呼吁

公众对世界不确定性的关注,尤其与此时此刻的情况密切相关:此时此刻,在世界上

技术最先进的国家,仍处于无法控制流行病和经济危机的泥潭中),下文也提到,一

场病毒肆虐,一个国家社会停止了运转,大流行无法控制,经济下行,这样的事情在

不久之前都是无法想象,不可思议( unthinkable )的,即世界充满了不确定性。而此

时此刻呼吁人们对这种 unthinkable 加以关注,正是和此时此刻的世界实况密切相关。

A. Scientific 科学的; B. Credible 可信的,可靠的; C. Original 原来的,原创的; D.

Relevant 相关的,有重大关系的。根据上面的分析,仅有 D 符合语境,故选 D。

69. 推理判断题。本题要求判断作者的情感态度,根据原文第五段“ yet messengers of

the coming disturbance are likely to be ignored. We all want to hope things will turn out

fine.”(然而未来崩溃的先行信号很可能被忽略,我们都希望事情在未来会变好)和倒

数最后一段的呼吁,例如“ Let's look directly into the issue of collapse,” they say, “and

deal with the terrible possibilities of what we see there to make the best of a troubling

future.”(他们说,“让我们直面全球崩塌的议题,真正去解决我们看到的可能的糟糕

情况,以便使未来没那么糟。”)可知,作者认为公众对“全球崩塌”的重视意识不

够,比较担心, A. worried 担忧的;B. puzzled 困惑的,茫然的; C. surprised 惊讶的;

D. scared 害怕的,综合以上的分析,可见作者对此是“担忧的”,故选 A。

70. 推理判断题。原文诗歌“Man is a victim of dope; In the incurable form of hope.”(人

类是麻醉品的受害者;沉迷于无可救药的幻想中)表达的是,人类无视未来全球崩塌

的巨大危险,把头埋进沙子里,假装不知道,充满不切实际的幻想和希望。而诗歌前

面的段落就提到“ yet messengers of the coming disturbance are likely to be ignored. We all

want to hope things will turn out fine.” (然而未来崩溃的先行信号很可能被忽略,我们都

希望事情在未来会变好),结合上下文,这里指的是人们都幻想着未来就会变好。诗

歌之后的最后一段则提到执着于“ quieting hope that ignores preparedness.” (掐灭不做准

备 的 空 有 幻 想 ) , 接 着 又 借 学 者 之 口 , 提 到 “ Let's look directly into the issue of

collapse,” they say, “and deal with the terrible possibilities of what we see there to make the

best of a troubling future.”(他们说,“让我们直面全球崩塌的议题,真正去解决我们看

到的可能的糟糕情况,以便使未来没那么糟。”)可见,上下文一脉相承,表达相同

的一方观点,未对另一方的观点有任何呈现,理解文章后可知,作者有明显的态度倾

向,作者对于这首诗表达的信息是赞同的,故选 B。
18

( 2021· 全国) When the explorers first set foot upon the continent of North America,

the skies and lands were alive with an astonishing variety of wildlife. Native Americans had

taken care of these precious natural resources wisely. Unfortunately, it took the explorers and

the settlers who followed only a few decades to   decimate a large part of these resources.

Millions of waterfowl ( 水 禽 ) were killed at the hands of market hunters and a handful of

overly ambitious sportsmen. Millions of acres of wetlands were dried to feed and house the

ever-increasing populations, greatly reducing waterfowl habitat.

In 1934, with the passage of the Migratory Bird Hunting Stamp Act (Act), an

increasingly concerned nation took firm action to stop the destruction of migratory ( 迁徙的)

waterfowl and the wetlands so vital to their survival. Under this Act, all waterfowl hunters 16

years of age and over must annually purchase and carry a Federal Duck Stamp. The very first

Federal Duck Stamp was designed by J.N. “Ding” Darling, a political cartoonist from Des

Moines, lowa, who at that time was appointed by President Franklin Roosevelt as Director of

the Bureau of Biological Survey. Hunters willingly pay the stamp price to ensure the survival

of our natural resources.

About 98 cents of every duck stamp dollar goes directly into the Migratory Bird

Conservation Fund to purchase wetlands and wildlife habitat for inclusion into the National

Wildlife Refuge System — a fact that ensures this land will be protected and available for all

generations to come. Since 1934, better than half a billion dollars has gone into that Fund to

purchase more than 5 million acres of habitat. Little wonder the Federal Duck Stamp Program

has been called one of the most successful conservation programs ever initiated.

38. What was a cause of the waterfowl population decline in North America?

A. Loss of wetlands. B. Popularity of water sports.

C. Pollution of rivers. D. Arrival of other wild animals.

39. What does the underlined word “decimate” mean in the first paragraph?

A. Acquire. B. Export.

C. Destroy. D. Distribute.

40. What is a direct result of the Act passed in 1934?

A. The stamp price has gone down. B. The migratory birds have flown away.

C. The hunters have stopped hunting. D. The government has collected money.

41. Which of the following is a suitable title for the text?

学科网(北京)股份有限公司
A. The Federal Duck Stamp Story B. The National Wildlife Refuge System

C. The Benefits of Saving Waterfowl D. The History of Migratory Bird Hunting

【答案与解析】本文是一篇说明文。本文讲述了美国鸭票的故事,由于美国移民的大

量流入,对于农地和住房的急需大量的水禽栖息地被破坏导致美国水禽骤减,因此美

国发行了鸭票,狩猎者只有购买了鸭票才能狩猎,而鸭票的部分收入进入到了用于购

买水禽栖息地的基金,从而保护了水禽。
38. 细节理解题。根据第一段“ Millions of acres of wetlands were dried to feed and house

the ever-increasing populations, greatly reducing waterfowl habitat.” 可知,上百万公顷的湿

地被抽干用作农地或者修建住房,极大地减少了水禽的栖息地,故可知,栖息地的减

少导致了水禽数量的下降,故选 A。

39. 词义猜测题。根据前一句“ Native Americans had taken care of these precious natural

resources wisely.”可知,北美的土著人把这些珍贵的自然资源保护的很合理,本句中的

“Unfortunately”可知,本句与上一句形成了转折,前一句陈述北美土著人做的好的地

方,故可知,本句阐述移民者做的不好的地方,即移民者破坏了这些自然资源,故画

线词意思是“破坏”。A. Acquire 获得;B. Export 出口;C. Destroy 破坏;D. Distribute

分配。故选 C。

40. 推理判断题。根据最后一段“Since 1934, better than half a billion dollars has gone into

that Fund to purchase more than 5 million acres of habitat.” 可知,自 1934 年起,超过 5 亿

美元进入到了这个基金会,购买了超过 500 万公顷的水禽栖息地,故可以推出,通过

发行鸭票,美国政府获得了大量的资金,故选 D。

41. 主旨大意题。根据全文可知,由于之前不恰当的发展导致美国水禽骤减,因此美国

发行了鸭票,狩猎者只有购买了鸭票才能狩猎,而鸭票的部分收入进入到了用于购买

水禽栖息地的基金,从而保护了水禽,故可知,本文讲述美国鸭票的故事,故选 A。

19

( 2021· 全 国 ) Popularization has in some cases changed the original meaning of

emotional ( 情感的 ) intelligence. Many people now misunderstand emotional intelligence as

almost everything desirable in a person's makeup that cannot be measured by an IQ test, such

as character, motivation, confidence, mental stability, optimism and “people skills.” Research

has shown that emotional skills may contribute to some of these qualities, but most of them

move far beyond skill-based emotional intelligence.

We prefer to describe emotional intelligence as a specific set of skills that can be used

for either good or bad purposes. The ability to accurately understand how others are feeling
may be used by a doctor to find how best to help her patients, while a cheater might use it to

control potential victims. Being emotionally intelligent does not necessarily make one a

moral person.

Although popular beliefs regarding emotional intelligence run far ahead of what research

can reasonably support, the overall effects of the publicity have been more beneficial than

harmful. The most positive aspect of this popularization is a new and much needed emphasis

( 重视 ) on emotion by employers, educators and others interested in promoting social well-

being. The popularization of emotional intelligence has helped both the public and

researchers re-evaluate the functionality of emotions and how they serve people adaptively in

everyday life.

Although the continuing popular appeal of emotional intelligence is desirable, we hope

that such attention will excite a greater interest in the scientific and scholarly study of

emotion. It is our hope that in coming decades, advances in science will offer new

perspectives ( 视 角 ) from which to study how people manage their lives. Emotional

intelligence, with its focus on both head and heart, may serve to point us in the right direction.

42. What is a common misunderstanding of emotional intelligence?

A. It can be measured by an IQ test. B. It helps to exercise a person’s mind.

C. It includes a set of emotional skills. D. It refers to a person’s positive qualities.

43. Why does the author mention “doctor” and “cheater” in paragraph 2?

A. To explain a rule. B. To clarify a concept.

C. To present a fact. D. To make a prediction.

44. What is the author’s attitude to the popularization of emotional intelligence?

A. Favorable. B. Intolerant.

C. Doubtful. D. Unclear.

45. What does the last paragraph mainly talk about concerning emotional intelligence?

A. Its appeal to the public. B. Expectations for future studies.

C. Its practical application. D. Scientists with new perspectives.

【答案与解析】本文是一篇说明文。文章介绍了情商的定义以及对有关于情商未来研

究的期望。
42. 细 节 理 解 题 。 通 过 文 章 第 一 段 “ Research has shown that emotional skills may

contribute to some of these qualities ( 研究表明,情商技巧可能有助于这些品质的形成 )”

可知,情商指的并不是一个人的积极品质。故选 D 项。

学科网(北京)股份有限公司
43. 推理判断题。通过文章第二段“ The ability to accurately understand how others are

feeling may be used by a doctor to find how best to help her patients, while a cheater might

use it to control potential victims. ( 医生可能利用这种准确理解他人感受的能力来找到最

好的帮助病人的方法,而骗子可能利用这种能力来控制潜在的受害者 )” 可推知,作者

在文章第二段中提到“医生”和“骗子”是举例子来阐明下文的观点——情商高并不

一定能使一个人成为有道德的人。故选 B 项。

44. 推理判断题。通过文章第三段“ the overall effects of the publicity have been more

beneficial than harmful. …The popularization of emotional intelligence has helped both the

public and researchers (宣传的总体效果一直是利大于弊。这种普及最积极的方面是雇主、

教育者和其他对促进社会福利感兴趣的人对情感进行了新的、迫切需要的强调。情商

的普及帮助了公众和研究人员 )” 可推知,作者认为情商普及是对人们有利的。故选 A

项。
45. 推理判断题。通过文章最后一段“ we hope that such attention will excite a greater

interest in the scientific and scholarly study of emotion. It is our hope that in coming decades,

advances in science will offer new perspectives from which to study how people manage their

lives. ( 我们希望这种关注将激发人们对情感科学和学术研究的更大兴趣。我们希望在

未来的几十年里,科学的进步将为研究人们如何管理自己的生活提供新的视角 )”可推

知,本段主要谈了对未来关于情商研究的期望。故选 B 项。

20

(2021·全国)You’ve heard that plastic is polluting the oceans — between 4.8 and 12.7

million tonnes enter ocean ecosystems every year. But does one plastic straw or cup really

make a difference? Artist Benjamin Von Wong wants you to know that it does. He builds

massive sculptures out of plastic garbage, forcing viewers to re-examine their relationship to

single-use plastic products.

At the beginning of the year, the artist built a piece called “Strawpocalypse,” a pair of

10-foot-tall plastic waves, frozen mid-crash. Made of 168,000 plastic straws collected from

several volunteer beach cleanups, the sculpture made its first appearance at the Estella Place

shopping center in Ho Chi Minh City, Vietnam.

Just 9% of global plastic waste is recycled. Plastic straws are by no means the biggest

source ( 来 源 ) of plastic pollution, but they’ve recently come under fire because most

people don’t need them to drink with and, because of their small size and weight, they cannot

be recycled. Every straw that’s part of Von Wong’s artwork likely came from a drink that
someone used for only a few minutes. Once the drink is gone, the straw will take centuries to

disappear.

In a piece from 2018, Von Wong wanted to illustrate (说明) a specific statistic: Every

60 seconds, a truckload’s worth of plastic enters the ocean. For this work, titled “Truckload of

Plastic,” Von Wong and a group of volunteers collected more than 10,000 pieces of plastic,

which were then tied together to look like they’d been dumped (倾倒) from a truck all at

once.

Von Wong hopes that his work will also help pressure big companies to reduce their

plastic footprint.

23. What are Von Wong’s artworks intended for?

A. Beautifying the city he lives in. B. Introducing eco-friendly products.

C. Drawing public attention to plastic waste. D. Reducing garbage on the beach.

24. Why does the author discuss plastic straws in paragraph 3?

A. To show the difficulty of their recycling.

B. To explain why they are useful.

C. To voice his views on modern art.

D. To find a substitute for them.

25. What effect would “Truckload of Plastic” have on viewers?

A. Calming. B. Disturbing.

C. Refreshing. D. Challenging.

26. Which of the following can be the best title for the text?

A. Artists’ Opinions on Plastic Safety

B. Media Interest in Contemporary Art

C. Responsibility Demanded of Big Companies

D. Ocean Plastics Transformed into Sculptures

【答案与解析】本文是一篇说明文。文章介绍了艺术家 Benjamin Von Wong 用塑料垃

圾制作了一个巨大的雕塑作品,让人们通过这个雕塑重新审视自己与一次性塑料制品

的关系。此外他在 2018 的一件作品“Truckload of Plastic”说明了每 60 秒,就有一卡车

塑料进入海洋。Von Wong 通过用塑料垃圾制造巨型雕塑来唤醒和提高人们的环保意识。

23. 推理判断题。根据第一段“ But does one plastic straw or cup really make a difference?

Artist Benjamin Von Wong wants you to know that it does. He builds massive sculptures out

of plastic garbage, forcing viewers to re-examine their relationship to single-use plastic

products.( 但一根塑料吸管或一个塑料杯真的有什么区别吗?艺术家本杰明 · 冯 · 王

学科网(北京)股份有限公司
(Benjamin Von Wong )想让你知道,它确实如此。他用塑料垃圾建造巨大的雕塑,迫

使观众重新审视他们与一次性塑料产品的关系。)”可知,Von Wong 用塑料垃圾制作的

雕塑想让人们重新审视与一次性塑料制品的关系,由此可知他做这个雕塑的目的是为

了引起公众对塑料垃圾的关注。故选 C 项。

24. 推理判断题。根据第三段“ Just 9% of global plastic waste is recycled. Plastic straws

are by no means the biggest source( 来 源 ) of plastic pollution, but they’ve recently come

under fire because most people don’t need them to drink with and, because of their small size

and weight, they cannot be recycled. Every straw that’s part of Von Wong’s artwork likely

came from a drink that someone used for only a few minutes. Once the drink is gone, the

straw will take centuries to disappear.( 全球只有 9%的塑料垃圾被回收。塑料吸管绝不是

最大的塑料污染源,但它们最近却受到了抨击,因为大多数人不需要吸管喝饮料,而

且由于它们体积小、重量轻,无法回收利用。冯 ·王作品中的每一根吸管都很可能来自

只喝了几分钟的饮料。一旦饮料消失了,吸管也要几个世纪才能消失。 )”可知,吸管

由于体积小,重量轻,无法回收利用,由此可推知,作者在第三段讨论吸管是为了展

示它们回收的困难。故选 A 项。

25. 推 理 判 断 题 。 根 据 倒 数 第 二 段 “ In a piece form 2018, Von Wong wanted to

illustrate(说明) a specific statistic: Every 60 seconds, a truckload’s worth of plastic enters the

ocean. For this work, titled “Truckload of Plastic,” Von Wong and a group of volunteers

collected more than 10,000 pieces of plastic, which were then tied together to look like they’d

been dumped( 倾倒 ) from a truck all at once.( 在 2018 年的一个作品中,冯 · 王(Von Wong)

想要说明一个具体的统计数字:每 60 秒,就有一卡车塑料进入海洋。这项名为“一卡

车塑料”的作品,冯 ·王和一组志愿者收集了一万多块塑料,然后把它们绑在一起,让

它们看起来像是同时从卡车上倾倒下来的。 )” 可知,这个作品以创新的方式让人们了

解到塑料垃圾以很快的速度和很大的量倾入海洋,刷新了观众对海洋塑料污染的认知 ,

由此可推断,这个作品会让观众对塑料垃圾进入海洋造成污染这件事感到不安。故选

B 项。

26. 标题判断题。通读全文,结合第一段“But does one plastic straw or cup really make a

difference? Artist Benjamin Von Wong wants you to know that it does. He builds massive

sculptures out of plastic garbage, forcing viewers to re-examine their relationship to single-

use plastic products.( 但一根塑料吸管或一个塑料杯真的有什么区别吗?艺术家本杰明 ·

冯·王(Benjamin Von Wong)想让你知道,它确实如此。他用塑料垃圾建造巨大的雕塑,

迫使观众重新审视他们与一次性塑料产品的关系。 )” 和倒数第二段“ In a piece form


2018, Von Wong wanted to illustrate( 说 明 ) a specific statistic: Every 60 seconds, a

truckload’s worth of plastic enters the ocean. For this work, titled “Truckload of Plastic,” Von

Wong and a group of volunteers collected more than 10,000 pieces of plastic, which were

then  tied together to look like they’d been dumped( 倾倒 ) from a truck all  at once.( 在 2018

年的一个作品中,冯·王(Von Wong)想要说明一个具体的统计数字:每 60 秒,就有一卡

车塑料进入海洋。这项名为“一卡车塑料”的作品,冯 ·王和一组志愿者收集了一万多

块塑料,然后把它们绑在一起,让它们看起来像是同时从卡车上倾倒下来的。 )”可知

艺术家本杰明·冯·王(Benjamin Von Wong)通过利用塑料垃圾制作巨型雕塑的方法来提示

人们重新思考与一次性塑料的关系,唤醒和提高人们循环利用的意识,促进环保的发

展。由此可知,D 项“海洋塑料变成雕塑”符合文章主旨,适合作为标题。故选 D 项。

21

(2021·全国)During an interview for one of my books, my interviewer said something

I still think about often. Annoyed by the level of distraction (干扰) in his open office, he

said, “That’s why I have a membership at the coworking space across the street — so I can

focus”. His comment struck me as strange. After all, coworking spaces also typically use an

open office layout(布局). But I recently came across a study that shows why his approach

works.

The researchers examined various levels of noise on participants as they completed tests

of creative thinking. They were randomly divided into four groups and exposed to various

noise levels in the background, from total silence to 50 decibels (分贝) , 70 decibels, and

85 decibels. The differences between most of the groups were statistically insignificant;

however, the participants in the 70 decibels group — those exposed to a level of noise similar

to background chatter in a coffee shop — significantly outperformed the other groups. Since

the effects were small, this may suggest that our creative thinking does not differ that much in

response to total silence and 85 decibels of background noise.

But since the results at 70 decibels were significant, the study also suggests that the right

level of background noise — not too loud and not total silence — may actually improve one’s

creative thinking ability. The right level of background noise may interrupt our normal

patterns of thinking just enough to allow our imaginations to wander, without making it

impossible to focus. This kind of “distracted focus” appears to be the best state for working

on creative tasks.

So why do so many of us hate our open offices? The problem may be that, in our offices,

学科网(北京)股份有限公司
we can’t stop ourselves from getting drawn into others’ conversations while we’re trying to

focus. Indeed, the researchers found that face-to-face interactions and conversations affect the

creative process, and yet a coworking space or a coffee shop provides a certain level of noise

while also providing freedom from interruptions.

27. Why does the interviewer prefer a coworking space?

A. It helps him concentrate. B. It blocks out background noise.

C. It has a pleasant atmosphere. D. It encourages face-to-face interactions.

28. Which level of background noise may promote creative thinking ability?

A. Total silence. B. 50 decibels C. 70 decibels. D. 8 5 decibels.

29. What makes an open office unwelcome to many people?

A. Personal privacy unprotected. B. Limited working space.

C. Restrictions on group discussion. D. Constant interruptions.

30. What can we infer about the author from the text?

A. He’s a news reporter.

B. He’s an office manager.

C. He’s a professional designer.

D. He’s a published writer.

【答案与解析】这是一篇说明文。作者通过自身经历讲述人们为什么不喜欢开放性办

公室以及有关多少分贝的噪音最有利于人们的创造性思维的研究。
27. 细节理解题。根据第一段“ That’s why I have a membership at the coworking space

across the street - so I can focus. (这就是为什么我在街对面的公用办公空间有会员资格

——这样我就可以集中精力了。)”可知,采访者喜欢共享办公空间的原因是那里可

以帮助他集中精力。故选 A 项。

28. 细 节 理 解 题 。 根 据 第 二 段 “ The differences between most of the groups were

statistically insignificant; however, the participants in the 70 decibels group - those exposed

to a level of noise similar to background chatter in a coffee shop - significantly outperformed

the other groups.(大多数组之间的差异在统计学上是不显著的;然而,音量为 70 分贝

的那组参与者(置身于类似于咖啡店背景噪音的环境中)的表现明显好于其他

组 。 ) ” 和 第 三 段 “ But since the results at 70 decibels were significant, the study also

suggests that the right level of background noise - not too loud and not total silence - may

actually improve one's creative thinking ability. (但由于 70 分贝的结果很显著,该研究还

表明,适当的背景噪音——不要太大声,也不要完全安静——实际上可能会提高一个

人的创造性思维能力。)”可知,70 分贝的那组参与者表现好于其他组,所以 70 分贝
的噪音背景环境更有可能促进创造性思维能力。故选 C 项。

29. 细节理解题。根据最后一段“ So why do so many of us hate our open offices? The

problem may be that, in our offices, we can't stop ourselves from getting drawn into others'

conversations while we're trying to focus. Indeed, the researchers found that face-to-face

interactions and conversations affect the creative process, and yet a coworking space or a

coffee shop provides a certain level of noise while also providing freedom from interruptions.

(那么,为什么我们中有那么多人讨厌开放式办公室呢?问题可能是,在我们的办公

室里,当我们试图集中注意力时,我们无法阻止自己卷入别人的谈话中。的确,研究

人员发现,面对面的互动和对话会影响创作过程,然而,共同工作空间或咖啡馆在提

供一定程度的噪音的同时,也提供不受干扰的自由。)”可知,开放式办公室不受人

们欢迎的原因是让我们不断地卷入别人的谈话中,受到很多干扰。故选 D 项。

30. 推理判断题。根据第一段“ During an interview for one of my books, my interviewer

said something I still think about often. (在一次采访我的一本书时,我的采访者说了一

些我至今还经常想起的话。)”可知,作者提到有人采访自己的书,所以可以推断,

作者是一位作家。故选 D 项。

22

( 2021· 全国) When almost everyone has a mobile phone, why are more than half of

Australian homes still paying for a landline(座机)?

These days you’d be hard pressed to find anyone in Australia over the age of 15 who

doesn’t own a mobile phone. In fact plenty of younger kids have one in their pocket.

Practically everyone can make and receive calls anywhere, anytime.

Still, 55 percent of Australians have a landline phone at home and only just over a

quarter ( 29% ) rely only on their smartphones according to a survey (调查) . Of those

Australians who still have a landline, a third concede that it’s not really necessary and they’re

keeping it as a security blanket — 19 percent say they never use it while a further 13 percent

keep it in case of emergencies. I think my home falls into that category.

More than half of Australian homes are still choosing to stick with their home phone.

Age is naturally a factor(因素)— only 58 percent of Generation Ys still use landlines now

and then, compared to 84 percent of Baby Boomers who’ve perhaps had the same home

number for 50 years. Age isn’t the only factor; I’d say it’s also to do with the makeup of your

household.

Generation Xers with young families, like my wife and I, can still find it convenient to

学科网(北京)股份有限公司
have a home phone rather than providing a mobile phone for every family member. That said,

to be honest the only people who ever ring our home phone are our Baby Boomers parents, to

the point where we play a game and guess who is calling before we pick up the phone (using

Caller ID would take the fun out of it).

How attached are you to your landline? How long until they go the way of gas street

lamps and morning milk deliveries?

19. What does paragraph 2 mainly tell us about mobile phones?

A. Their target users. B. Their wide popularity.

C. Their major functions. D. Their complex design.

20. What does the underlined word “concede” in paragraph 3 mean?

A. Admit. B. Argue.

C. Remember. D. Remark.

21. What can we say about Baby Boomers?

A. They like smartphone games. B. They enjoy guessing callers’ identity.

C. They keep using landline phones. D. They are attached to their family.

22. What can be inferred about the landline from the last paragraph?

A. It remains a family necessity.

B. It will fall out of use some day.

C. It may increase daily expenses.

D. It is as important as the gas light.

【答案与解析】本文是一篇说明文。文章主要介绍了澳大利亚使用固定电话的情况,

并且表达了固定电话是非必需品的观点。
19. 主旨大意题。根据文章第二段“ These days you'd be hard pressed to find anyone in

Australia over the age of 15 who doesn't own a mobile phone. In fact plenty of younger kids

have one in their pocket. Practically everyone can make and receive calls anywhere, anytime.

(现在你很难在澳大利亚找到 15 岁以上的没有手机的人。事实上,很多年幼的孩子口

袋里都有手机。几乎每个人都可以随时随地拨打和接听电话 )”可推知,本段主要说明

手机在澳大利亚广受欢迎。故选 B 项。

20. 词句猜测题。根据划线单词的上文“Of those Australians who still have a landline ( 在

那些仍然有固定电话的澳大利亚人中 )” 可知,这个调查的目标人群是仍然有固定电话

的 澳 大 利 亚 人 ; 根 据 下 文 “ it's not really necessary and they're keeping it as a security

blanket — 19 percent say they never use it while a further 13 percent keep it in case of

emergencies (固定电话并不是必须的,他们将其作为一种安全保障—— 19%的人表示他


们从未使用过固定电话,另有 13%的人保留固定电话以防紧急情况 )” 可知,很多人认

为固定电话并不是必须拥有的,有些人保留固定电话只是为了防止紧急情况。从而推

知,在调查中,他们应该是承认了固定电话的非必要性。由此推知,划线单词

“concede”意为“承认”。故选 A 项。

21. 推理判断题。根据文章第四段“ 84 percent of Baby Boomers who've perhaps had the

same home number for 50 years.( 婴儿潮时代中有 84%的人可能已经有 50 年相同的家庭

号码了)”以及文章第五段“That said, to be honest the only people who ever ring our home

phone are our Baby Boomers parents ( 也就是说,老实说,唯一打过我们家电话的人是婴

儿潮一代的父母)”可推知,婴儿潮时代的人一直用固定电话。故选 C 项。

22. 推理判断题。根据文章最后一段“ How attached are you to your landline? How long

until they go the way of gas street lamps and morning milk deliveries? ( 你有多喜欢你的座

机?它们还要多久才能走上煤气路灯和早晨送牛奶的道路? )”可推知,本段使用类比

的方式,使用煤气路灯以及早晨送牛奶已经被淘汰的例子,侧面说明了固定电话总有

一天会废弃的。故选 B 项。

23

( 2021· 全国 ) Port Lympne Reserve, which runs a breeding ( 繁 育 ) programme, has

welcomed the arrival of a rare black rhino calf ( 犀牛幼崽). When the tiny creature arrived on

January 31, she became the 40th black rhino to be born at the reserve. And officials at Port

Lympne were delighted with the new arrival, especially as black rhinos are known for being

difficult to breed in captivity (圈养).

Paul Beer, head of rhino section at Port Lympne, said: “Obviously we're all absolutely

delighted to welcome another calf to our black rhino family. She's healthy, strong and already

eager to play and explore. Her mother, Solio, is a first-time mum and she is doing a fantastic

job. It's still a little too cold for them to go out into the open, but as soon as the weather

warms up, I have no doubt that the little one will be out and about exploring and playing

every day.”

The adorable female calf is the second black rhino born this year at the reserve, but it is

too early to tell if the calves will make good candidates to be returned to protected areas of

the wild. The first rhino to be born at Port Lympne arrived on January 5 to first-time mother

Kisima and weighed about 32kg. His mother, grandmother and great grandmother were all

born at the reserve and still live there.

According to the World Wildlife Fund, the global black rhino population has dropped as

学科网(北京)股份有限公司
low as 5500, giving the rhinos a “critically endangered” status.

4. Which of the following best describes the breeding programme?

A. Costly. B. Controversial. C. Ambitious. D. Successful.

5. What does Paul Beer say about the new-born rhino?

A. She loves staying with her mother. B. She dislikes outdoor activities.

C. She is in good condition D. She is sensitive to heat.

6. What similar experience do Solio and Kisima have?

A. They had their first born in January. B. They enjoyed exploring new places

C. They lived with their grandmothers. D. They were brought to the reserve young

7. What can be inferred about Porn Lympne Reserve?

A. The rhino section will be open to the public.

B. It aims to control the number of the animals.

C. It will continue to work with the World Wildlife Fund.

D. Some of its rhinos may be sent to the protected wild areas.

【答案与解析】本文是一篇说明文。文章主要介绍了在 Port Lympne 保护区的部分黑犀

牛现状。
4. 推 理 判 断 题 。 通 过 文 章 第 一 段 “ she became the 40th black rhino to be born at the

reserve ( 她成为该保护区出生的第 40 头黑犀牛 )” 以及文章倒数第二段“ His mother,

grandmother and great grandmother were all born at the reserve and still live there. ( 他的母

亲、祖母和曾祖母都出生在保护区,至今仍住在那里 )”可知,保护区的繁育计划使很

多黑犀牛成功存活,可推知,这计划是成功的。故选 D 项。

5. 细 节理 解题 。通 过文 章第 二段 “ She's healthy, strong and already eager to play and

explore. ( 她很健康,很强壮,已经渴望玩耍和探索了 )” 可知,Paul Beer 认为新生的犀

牛身体状况很好。故选 C 项。

6. 细 节 理 解 题 。 通 过 文 章 第 一 段 “ When the tinv creature arrived on January 31, she

became the 40th black rhino to be born at the reserve. (1 月 31 日,当这头小犀牛来到保护

区时,她成为了第 40 头在保护区出生的黑犀牛 )”以及文章倒数第二段“ The first rhino

to be born at Port Lympne arrived on January 5 to first-time mother Kisima and weighed

about 32kg. (1 月 5 日,犀牛妈妈 Kisima 分娩的第一头小犀牛,同时也是第一个出生在

Port Lympne,体重约为 32 公斤)”可知, Solio 和 Kisima 的第一个孩子都是在一月份出

生的。故选 A 项。

7. 推理判断题。通过文章倒数第二段“ it is too early to tell if the calves will make good

candidates to be returned to protected areas of the wild ( 要判断这些小犀牛是否会成为返回


野生保护区的好的候选者还为时过早 )”可推知, Pon Lympne 保护区的一些犀牛可能会

被送到野生保护区。故选 D 项。

2013-2020 题组

1.(2020 年,江苏卷)

For those who can stomach it, working out before breakfast may be more beneficial for

health than eating first, according to a study of meal timing and physical activity.

Athletes and scientists have long known that meal timing affects performance. However,

far less has been known about how meal timing and exercise might affect general health.

To find out, British scientists conducted a study. They first found 10 overweight and

inactive but otherwise healthy young men, whose lifestyles are, for better and worse,

representative of those of most of us. They tested the men’s fitness and resting metabolic ( 新

陈代谢的) rates and took samples (样品) of their blood and fat tissue.

Then, on two separate morning visits to the scientists’ lab, each man walked for an hour

at an average speed that, in theory, should allow his body to rely mainly on fat for fuel.

Before one of these workouts, the men skipped breakfast, meaning that they exercised on a

completely empty stomach after a long overnight fast ( 禁食 ). On the other occasion, they ate

a rich morning meal about two hours before they started walking.

Just before and an hour after each workout, the scientists took additional samples of the

men’s blood and fat tissue.

Then they compared the samples. There were considerable differences. Most obviously,

the men displayed lower blood sugar levels at the start of their workouts when they had

skipped breakfast than when they had eaten. As a result, they burned more fat during walks

on an empty stomach than when they had eaten first. On the other hand, they burned slightly

more calories (卡路里), on average, during the workout after breakfast than after fasting.

But it was the effects deep within the fat cells that may have been the most significant,

the researchers found. Multiple genes behaved differently, depending on whether someone

had eaten or not before walking. Many of these genes produce proteins ( 蛋 白 质 ) that can

improve blood sugar regulation and insulin ( 胰岛素 ) levels throughout the body and so are

associated with improved metabolic health. These genes were much more active when the

men had fasted before exercise than when they had breakfasted.

The implication of these results is that to gain the greatest health benefits from exercise,

it may be wise to skip eating first.

学科网(北京)股份有限公司
1.The underlined expression “stomach it” in Paragraph 1 most probably means “______”.

A.digest the meal easily B.manage without breakfast

C.decide wisely what to eat D.eat whatever is offered

2.Why were the 10 people chosen for the experiment?

A.Their lifestyles were typical of ordinary people.

B.Their lack of exercise led to overweight.

C.They could walk at an average speed.

D.They had slow metabolic rates.

3.What happened to those who ate breakfast before exercise?

A.They successfully lost weight. B.They consumed a bit more calories.

C.They burned more fat on average. D.They displayed higher insulin levels.

4.What could be learned from the research?

A.A workout after breakfast improves gene performances.

B.Too much workout often slows metabolic rates.

C.Lifestyle is not as important as morning exercise.

D.Physical exercise before breakfast is better for health.

【答案】1.B 2.A 3.B 4.D

【解析】本文是说明文。文章介绍了一项研究,结果表明对于那些能忍受的人来说,

不吃早餐锻炼可能对健康更有益。
1. 词句猜测题。根据下文 working out before breakfast may be more beneficial for health

than eating first 可知,早餐前锻炼可能比先吃饭再锻炼对健康更有益,因此推断这里说

的是那些不吃早饭先锻炼的人,因此推断划线词与 B 项“不吃早饭能应付”意思相近。

故选 B。

2. 细节理解题。根据第三段的 They first found 10 overweight and inactive but otherwise

healthy young men, whose lifestyles are far better and worse, representative of those of most

of us.可知,他们首先找到了 10 个超重的,不活跃但健康的年轻人,他们的生活方式可

以说更好,也可以说更糟,代表了我们大多数人。因此可知,实验时选择的 10 个人的

生活方式代表了普通人。故选 A。

3. 细节理解题。根据第六段的 As a result, they burned more fat during walks on an empty

stomach than when they had eaten first. On the other hand. they burned slightly more

calories( 卡路里), on average, during the workout after breakfast than after fasting. 可知,结

果,他们空腹散步时燃烧的脂肪比他们首先吃东西时所燃烧的脂肪要多。 另一方面。

平均而言,他们在早餐后锻炼时燃烧的卡路里略多于禁食后。因此可知,锻炼前吃早
饭消耗更多一点的热量。故选 B。

4. 推理判断题。根据最后一段 The implication of these results is that to gain the greatest

health benefits from exercise, it may be wise to skip eating first. 可知,这些结果的暗示,为

了从运动中获得最大的健康益处,先不吃东西可能更明智。因此推断早饭前的体育锻

炼对健康更有益。故选 D。

2.(2020 年,江苏卷)

Sometimes it’s hard to let go. For many British people, that can apply to institutions and

objects that represent their country’s past-age-old castles, splendid homes… and red phone

boxes.

Beaten first by the march of technology and lately by the terrible weather in junkyards

( 废 品 场 ), the phone boxes representative of an age are now making something of a

comeback. Adapted in imaginative ways, many have reappeared on city streets and village

greens housing tiny cafes, cellphone repair shops or even defibrillator machines ( 除颤器).

The original iron boxes with the round roofs first appeared in 1926. They were designed

by Giles Gilbert Scott, the architect of the Battersea Power Station in London. After

becoming an important part of many British streets, the phone boxes began disappearing in

the 1980s, with the rise of the mobile phone sending most of them away to the junkyards.

About that time, Tony Inglis’ engineering and transport company got the job to remove

phone boxes from the streets and sell them out. But Inglis ended up buying hundreds of them

himself, with the idea of repairing and selling them. He said that he had heard the calls to

preserve the boxes and had seen how some of them were listed as historic buildings.

As Inglis and, later other businessmen, got to work, repurposed phone boxes began

reappearing in cities and villages as people found new uses for them. Today, they are once

again a familiar sight, playing roles that are often just as important for the community as their

original purpose.

In rural areas, where ambulances can take a relatively long time to arrive, the phone

boxes have taken on a lifesaving role. Local organizations can adopt them for l pound, and

install defibrillators to help in emergencies.

Others also looked at the phone boxes and saw business opportunities. LoveFone, a

company that advocates repairing cellphones rather than abandoning them, opened a mini

workshop in a London phone box in 2016.

The tiny shops made economic sense, according to Robert Kerr, a founder of LoveFone.

学科网(北京)股份有限公司
He said that one of the boxes generated around $13,500 in revenue a month and cost only

about $400 to rent.

Inglis said phone boxes called to mind an age when things were built to last. “I like what

they are to people, and I enjoy bringing things back,” he said.

5.The phone boxes are making a comeback ______.

A.to form a beautiful sight of the city

B.to improve telecommunications services

C.to remind people of a historical period

D.to meet the requirement of green economy

6.Why did the phone boxes begin to go out of service in the 1980s?

A.They were not well-designed. B.They provided bad services.

C.They had too short a history. D.They lost to new technologies.

7.The phone boxes are becoming popular mainly because of ______.

A.their new appearance and lower prices B.the push of the local organizations

C.their changed roles and functions D.the big funding of the businessmen

【答案】5.C 6.D 7.C

【解析】这是一篇说明文。文章讲述了在英国,电话亭在 20 世纪 80 年代开始停止使

用。后来,一些商人使电话亭的角色和功能发生了变化,电话亭又变得流行起来。
5.细节理解题。根据第一段 Sometimes it’s hard to let go. For many British people, that can

apply to institutions and objects that represent their country’s past-age-old castles, splendid

homes.. and red phone boxes.( 有时候真的很难放手。对许多英国人来说,这可以适用于

代表他们国家过去历史的机构和物品 --古老的城堡、辉煌的住宅……还有红色的电话

亭)和最后一段 Inglis said phone boxes called to mind an age when things were built to last.

(英格利斯说,电话亭让人想起了一个东西经久耐用的时代)可知,电话亭正在卷土重来,

以提醒人们一个历史时期。故选 C。

6. 细节理解题。根据第三段 After becoming an important part of many British streets, the

phone boxes began disappearing in the 1980s, with the rise of the mobile phone sending most

of them away to the junkyards.( 在成为英国许多街道的重要组成部分后,电话亭在 20 世

纪 80 年代开始消失,随着移动电话的兴起,大部分电话亭被扔到了垃圾场 )可知,电

话亭在 20 世纪 80 年代开始停止使用是因为它们输给了新技术。故选 D。

7.推理判断题。根据第五段 As Inglis and, later other businessmen, got to work, repurposed

phone boxes began reappearing in cities and villages as people found new uses for them.

Today, they are once again a familiar sight, playing roles that are often just as important for
the community as their original purpose.(随着英格利斯和后来的其他商人开始工作,改装

后的电话亭开始在城市和乡村重新出现,人们发现了它们的新用途。今天,它们再次

成为人们熟悉的景象,扮演着与它们最初的目的同样重要的角色 ) 和第六段 In rural

areas, where ambulances can take a relatively long time to arrive, the phone boxes have taken

on a lifesaving role.( 在农村地区,救护车要花相当长的时间才能到达,电话亭就起到了

拯 救 生 命 的 作 用 ) 以 及 第 七 段 Others also looked at the phone boxes and saw business

opportunities.(其他人也在电话亭寻找商机)可推断出,电话亭之所以变得流行,主要是

因为它们的角色和功能发生了变化。故选 C。

3.(2020 年 7 月,浙江卷)

Challenging work that requires lots of analytical thinking, planning and other managerial

skills might help your brain stay sharp as you age, a study published Wednesday in the

journal Neurology suggests.

Researchers from the University of Leipzig in Germany gathered more than 1, 000

retired workers who were over age 75 and assessed the volunteers’ memory and thinking

skills through a battery of tests. Then, for eight years, the scientists asked the same group to

come back to the lab every 18 months to take the same sorts of tests.

Those who had held mentally stimulating( 刺 激 ), demanding jobs before retirement

tended to do the best on the tests. And they tended to lose cognitive( 认知) function at a much

slower rate than those with the least mentally challenging jobs. The results held true even

after the scientists accounted for the participants’ overall health status.

“This works just like physical exercise, ” says Francisca Then, who led the study. “After

a long run, you may feel like you’re in pain, you may feel tired. But it makes you fit. After a

long day at work-sure, you will feel tired, but it can help your brain stay healthy. ”

It's not just corporate jobs, or even paid work that can help keep your brain fit, Then

points out. A waiter’s job, for example, that requires multitasking, teamwork and decision-

making could be just as stimulating as any high-level office work. And “running a family

household requires high-level planning and coordinating( 协 调 ), ” she says. “You have to

organize the activities of the children and take care of the bills and groceries. ”

Of course, our brains can decline as we grow older for lots of reasons-including other

environmental influences or genetic factors. Still, continuing to challenge yourself mentally

and keeping your mind busy can only help.

8.Why did the scientists ask the volunteers to take the tests?

学科网(北京)股份有限公司
A.To assess their health status. B.To evaluate their work habits.

C.To analyze their personality. D.To measure their mental ability.

9.How does Francisca Then explain her findings in paragraph 4?

A.By using an expert’s words. B.By making a comparison.

C.By referring to another study. D.By introducing a concept.

10.Which of the following is the best title for the text?

A.Retired Workers Can Pick Up New Skills

B.Old People Should Take Challenging Jobs

C.Your Tough Job Might Help Keep You Sharp

D.Cognitive Function May Decline As You Age

【答案】8.D 9.B 10.C

【解析】这是一篇说明文。《神经病学》杂志发布的一则研究显示,需要大量的分析

思考、细致规划和其他管理技能有挑战性的工作,有可能会帮助你的大脑随着年龄的

增长而保持敏锐。
8. 细 节 理 解 题 。 根 据 第 二 段 第 一 句 “ Researchers from the University of Leipzig in

Germany gathered more than 1,000 retired workers who were over age 75 and assessed the

volunteers’ memory and thinking skills through a battery of tests.( 来自德国莱比锡大学的研

究人员召集了 1000 多名 75 岁以上的退休老人,并通过一系列的测试对他们的记忆力

和思维能力进行了评估。)” 可知,科学家让志愿者做测试是为了测试他们的心智能力。

故选 D 项。

9. 推 理 判 断 题 。 根 据 第 四 段 中 Francisca Then 说 的 话 “ This works just like physical

exercise,( 这就像体育锻炼一样有效。 )”及接下来进一步的解释“长跑后人就会感到痛

苦和疲惫,但它确实能强身健体。工作一天下来会感到劳累,但这也使你的大脑处于

健康状态”可知,Francisca Then 是通过与“身体锻炼”作比较来解释他的发现的。故

选 B 项。

10. 主 旨 大 意 题 。 第 一 段 “ Challenging work that requires lots of analytical thinking,

planning and other managerial skills might help your brain stay sharp as you age( 需要大量的

分析思考、细致规划和其他管理技能有挑战性的工作,有可能会帮助你的大脑随着年

龄的增长而保持敏锐。 )” 是本文的主题句,结合全文内容可知,文章主要讲述了有挑

战性的工作可能会使大脑保持敏捷,所以 C 项“困难的工作可能有助于大脑保持敏

捷”适合作本文标题。故选 C 项。

4.(2020 年 7 月,浙江卷)

The traffic signals along Factoria Boulevard in Bellevue, Washington, generally don't
flash the same length of green twice in a row, especially at rush hour. At 9:30am, the full

red/yellow/green signal cycle might be 140 seconds. By 9:33am, a burst of additional traffic

might push it to 145 seconds. Less traffic at 9:37am could push it down to 135. Just like the

traffic itself, the timing of the signals changes.

That is by design. Bellevue, a fast-growing city just east of Seattle, uses a system that is

gaining popularity around the US: intersection( 十字路口 ) signals that can adjust in real time

to traffic conditions. These lights, known as adaptive signals, have led to significant declines

in both the trouble and cost of travels between work and home.

“Adaptive signals can make sure that the traffic demand that is there is being addressed,

” says Alex Stevanovic, a researcher at Florida Atlantic University.

For all of Bellevue’s success, adaptive signals are not a cure-all for jammed roadways.

Kevin Balke, a research engineer at the Texas A&M University Transportation Institute, says

that while smart lights can be particularly beneficial for some cities, others are so jammed

that only a sharp reduction in the number of cars on the road will make a meaningful

difference. “It’s not going to fix everything, but adaptive signals have some benefits for

smaller cities,” he says.

In Bellevue, the switch to adaptive signals has been a lesson in the value of welcoming

new approaches. In the past, there was often an automatic reaction to increased traffic: just

widen the roads, says Mark Poch, the Bellevue Transportation Department’s traffic

engineering manager. Now he hopes that other cities will consider making their streets run

smarter instead of just making them bigger.

11.What does the underlined word “that” in paragraph 2 refer to?

A.Increased length of green lights. B.Shortened traffic signal cycle.

C.Flexible timing of traffic signals. D.Smooth traffic flow on the road.

12.What does Kevin Balke say about adaptive signals?

A.They work better on broad roads.

B.They should be used in other cities.

C.They have greatly reduced traffic on the road.

D.They are less helpful in cities seriously jammed.

13.What can we learn from Bellevue’s success?

A.It is rewarding to try new things. B.The old methods still work today.

C.I pays to put theory into practice. D.The simplest way is the best way.

学科网(北京)股份有限公司
【答案】11.C 12.D 13.A

【解析】这是一篇说明文。美国华盛顿州的 Bellevue 采用了能随交通状况而调节交通

灯时间的适应性信号灯,大大缓解了交通压力,也表明人们对于交通阻塞问题不再只

是拓宽道路,而是能够采用新方法。
11.指代猜测题。 That 位于第二段句首,应是指代第一段的内容。根据第一段内容尤其

是第一段最后一句“ Just like the traffic itself, the timing of the signals changes.( 就像交通

本身一样,信号灯的时间也会变化 )”可知,第一段主要讲述的是信号灯的时间会灵活

变化;“ That is by design.” 意为“那是有意为之”,由此可推知, That 指代第一段中

“信号灯的灵活时间”。故选 C 项。

12. 推理判断题。根据第四段第一句中“ adaptive signals are not a cure-all for jammed

roadways( 但自适应信号并不是解决拥堵道路的万能药 )” 及第二句中“ others are so

jammed that only a sharp reduction in the number of cars on the road will make a meaningful

difference(其他城市交通堵塞如此严重,只有减少道路上的车辆才能起有意义的作用 ) ”

可推知,Kevin Balke 认为适应性信号灯对于交通堵塞很严重的城市没有太大帮助。故

选 D 项。

13.推理判断题。根据第二段最后一句“ These lights, known as adaptive signals, have led

to significant declines in both the trouble and cost of travels between work and home.( 这些灯,

被称为适应性信号灯,已经大大减少了通勤的麻烦和花费 )” 最后一段第一句“ In

Bellevue, the switch to adaptive signals has been a lesson in the value of welcoming new

approaches.( 在 Bellevue ,对适应性信号灯的转变是一个在欢迎新方法的价值方面的榜

样)”可推知,从 Bellevue 的成功中可以得出,尝试新事物是值得的。故选 A 项。

5.(2020 年 7 月,浙江卷)

I am an active playgoer and play-reader, and perhaps my best reason for editing this

book is a hope of sharing my enthusiasm for the theater with others. To do this I have

searched through dozens of plays to find the ones that I think best show the power and

purpose of the short play.

Each play has a theme or central idea which the playwright( 剧作家) hopes to get across

through dialogue and action. A few characters are used to create a single impression growing

out of the theme. It is not my intention to point out the central theme of each of the plays in

this collection, for that would, indeed, ruin the pleasure of reading, discussing, and thinking

about the plays and the effectiveness of the playwright. However, a variety of types is

represented here. These include comedy, satire, poignant drama, historical and regional
drama. To show the versatility( 多面性 ) of the short play, I have included a guidance play, a

radio play and a television play.

Among the writers of the plays in this collection, Paul Green, Susan Glaspell, Maxwell

Anderson, Thornton Wilder, William Saroyan, and Tennessee Williams have all received

Pulitzer Prizes for their contributions to the theater. More information about the playwrights

will be found at the end of this book.

To get the most out of reading these plays, try to picture the play on stage, with you, the

reader, in the audience. The houselights dim( 变暗 ). The curtains are about to open, and in a

few minutes the action and dialogue will tell you the story.

14.What do we know about the author from the first paragraph?

A.He has written dozens of plays. B.He has a deep love for the theater.

C.He is a professional stage actor. D.He likes reading short plays to others.

15.What does the author avoid doing in his work?

A.Stating the plays’ central ideas. B.Selecting works by famous playwrights.

C.Including various types of plays. D.Offering information on the playwrights.

16.What does the author suggest readers do while reading the plays?

A.Control their feelings. B.Apply their acting skills.

C.Use their imagination. D.Keep their audience in mind.

17.What is this text?

A.A short story. B.An introduction to a book.

C.A play review. D.An advertisement for a theater.

【答案】14.B 15.A 16.C 17.B

【解析】这是一篇说明文。文章主要介绍了作者的书籍的序言部分,包括其编写目的 ,

书籍内容主体,以及如何实现有乐趣的阅读本书的建议。
14. 细节理解题。根据文章第一段内容 “… and perhaps my best reason for editing this

book is a hope of sharing my enthusiasm for the theater with others” 可知,作者编辑这本书

的最好的原因是希望与他人分享对戏剧的热情,因此可知作者对于戏剧饱含热情。故

选 B 项。

15.细节理解题。根据文章第二段内容“It is not my intention to point out the central theme

of each of the plays in this collection, for that would, indeed, ruin the pleasure of reading,

discussing, and thinking about the plays and the effectiveness of the playwright.” 可知,作者

并不想指出这本书中每一部戏剧的中心主题,因为那样确实会破坏阅读、讨论和思考

学科网(北京)股份有限公司
戏剧以及剧作家的有效性的乐趣。A 项“Stating the plays”central ideas(陈述戏剧的中心

思想)”与原文表达信息一致。故选 A 项。

16.细节理解题。根据文章末尾段内容“To get the most out of the reading these plays, try

to picture the play on stage, with you, the reader, in the audience.” 可知,作者建议读者试着

想象戏剧在舞台上上演,不再是读者而是成为观众,才能实现最佳的阅读乐趣。 C 项

“Use their imagination” 与原文“try to picture the play on the stage” 表达信息一致。故选

C 项。

17.推理判断题。根据文章首段内容“I am an active playgoer and play-reader, and perhaps

my best reason for editing this book is a hope of sharing my enthusiasm for the theater with

others” 可知,该句内容应是书籍的“自序”内容,且结合末尾“To get the most out of

the reading these plays, try to picture the play on stage, with you, the reader, in the audience.”

可知,在讲述如何更好地实现阅读乐趣,再结合全文内容可推知,作者通过该文章介

绍自己的一本书。故选 B 项。

6.(2020 年,天津卷,第一次高考)

Transport has a lot to answer for when it comes to harming the planet. While cars and

trains are moving towards greener, electric power, emissions from air travel are expected to

increase massively by 2050. If we want big green sky solutions, we need blue sky thinking

Fortunately, there's plenty of that happening right now, particularly the short-haul flights

powered by batteries.

Harbour Air is the largest seaplane airline in North America, flying 30, 000 commercial

flights in 40 seaplanes each year. Significantly, all Harbour Air routes last less than 30

minutes, making it perfectly fit for electric engines. “As an airline, we're currently in the

process of turning all our planes into electric airplanes. says CEO Greg Mc Dougall. To make

this happen, the airline has partnered up with MagniX to create the worlds first commercial

flight with an electric engine.

Making the skies electric isn't just good for the environment, it also makes sound

financial sense: a small aircraft uses $400 on conventional fuel for a 100-mile flight, while an

electric one costs $8-12 for the same distance, and that's before you factor in the higher

maintenance costs of a traditional engine. There's also the added bonus that electric planes are

just much more pleasant to fly in. No loud engine noise, no smell of fuel, just

environmentally friendly peace and quiet.

While there has been real progress in the e-plane industry, the technical challenges that
remain are keeping everyone's feet firmly on the ground. A battery, even a lithium one, only

provides 250 watt-hours per kilogram; compare this to liquid fuel, which has a specific

energy of 11, 890 watt-hours per kilogram. Carrying adequate batteries, however, would

make the plane too heavy to get off the ground. In aircraft, where every bit of weight counts,

this can't just be ignored.

The transition (过渡) from gas to electric in the automobile industry has been made

easier by hybrids-vehicles powered by both fuel and electricity. Many believe the same

pattern could be followed in the air. Fuel consumption could be reduced as the electric

component is switched on at key parts of the journey, especially on take-off and landing.

It's certainly an exciting time for electric flying. With companies like Harbour Air taking

the lead, battery-powered planes, especially on short-haul journeys, are set to become a

reality in the next few years.

18.According to Para. l, what is happening in air transport?

A.New explorations of the sky are being launched.

B.Pollution caused by batteries is being controlled.

C.Efforts are being made to make air travel greener.

D.Demand for short-haul flights is increasing massively.

19.Why is Harbour Air fit for electric flights?

A.It runs short route

B.It has a strong partner.

C.Its planes can land on the sea.

D.It has planes with powerful engines.

20.The expression "added bonus" refers to the fact that electric planes___________.

A.give passengers more pleasant views

B.bring airlines more financial benefits

C.offer more enjoyable flying experiences

D.cost less in maintenance than traditional ones

21.What might be the biggest challenge of electric flying?

A.To improve the ground service for e-planes.

B.To find qualified technicians for e-plane industry.

C.To calculate the energy needed to power e-planes.

D.To balance power and weight of batteries in e-planes.

22.What could be done during the transition from gas to electric in air flight?

学科网(北京)股份有限公司
A.To produce new electric components.

B.To increase battery consumption.

C.To use mixed-power technology.

D.To expand the landing field.

23.What is the author's attitude towards the prospect of electric flying?

A.Short-sighted. B.Wait-and-see. C.Optimistic. D.Skeptical.

【答案】18.C 19.A 20.C 21.D 22.C 23.C

【解析】本文是说明文。为了使航空旅行更环保 ,出现了电动飞行。文章介绍了电动飞

行的特点、优点、面临的挑战、解决措施和前景。
18. 推 理 判 断 题 。 根 据 第 一 段 中 If we want big green sky solutions, we need blue sky

thinking. Fortunately, there's plenty of that happening right now, particularly the short-haul

flights powered by batteries( 如果我们想要大的绿色天空解决方案,就需要藍天思维。幸

运的是,现在有很多这样的情况发生,尤其是靠电池供电的短途飞行 )由此判断出,人

们正在努力使航空旅行更加环保。故选 C。

19. 细节理解题。根据第二段 Harbour Air is the largest seaplane airline in North America,

flying30000 commercial flights in 40 seaplanes each year. Significantly, all Harbour Air

routes last less uan30 minutes, making it perfectly fit for electric engines. 可知,Harbour Air

是北美最大的水上飞机航空公司,每年有 40 架水上飞机搭载 3 万次商业航班。值得注

意的是,所有港口航线的飞行时间都不到 30 分钟,这使得它完全适合于电动引擎。也

就是说 Harbour Air 的飞行航线短使电动飞行成了可能。故选 A。

20.猜测词义题。根据下文 that electric planes are just much more pleasant to fly in. No loud

engine noise, no smell of fuel, just environmentally friendly peace and quiet. 可知,电动飞

机没有发动机的噪音,没有燃油的味道,只有环保的宁静等优点,因此为乘坐电动飞

机的乘客提供更愉快的飞行体验。所以 added bonus 指的是乘客获得的更愉快飞行体验

的额外收获。故选 C。

21. 推理判断题。根据倒数第三段 A battery, even a lithium one, only provides 250 watt-

hours per kilogram; compare this to liquid fuel, which has a specific energy of 11, 890 watt-

hours per kilogram Carrying adequate batteries, however, would make the plane too heavy to

get off the ground. In aircraft., where every bit of weight counts, this can' t just be ignored. 可

知电池不能提供足够的燃料, 而携带足够的电池会使飞机太重而无法起飞。在飞机上,

每一点重量都很重要,不能忽视。由此推知平衡电动飞机电池的功率和重量是电动飞

机面临的最大挑战。故选 D。

22. 细 节 理 解 题 。 根 据 倒 数 第 二 段 The transition( 过 渡 ) from gas to electric in the


automobile industry has been made easier by hybrids-vehicles powered by both fuel and

electricity.可知从燃料飞行到电动飞行过渡过程中,可以使用燃料和电力混合动力技术。

故选 C。

23. 推 理 判 断 题 。 根 据 最 后 一 段 It's certainly an exciting time for electric flying. With

companies Harbour Air taking the lead, battery-powered planes, especially on short-haul

journeys, are set to come a reality in the next few years. 可知电动飞行无疑是激动人心的,

电动飞行短途旅行将在 未来几年内成为现实。由此判断出作者对电动飞行的前景是乐

观的。故选 C。

7.(2020 年,天津卷,第一次高考)

Your student ID card identifies you as a student at the University of Bolton. It will

provide you with access to University facilities such as University Libraries, Sports Centre,

and Computing Services. Please carry your card with you at all times. Do not lose your card

or lend it to others. Your card is valid during the whole time of your studies. It remains the

property(财产)of the University of Bolton at all times.

New students—photographs

You will be requested to upload a passport sized photograph as part of the online

registration process. This should be a jpg file with a size of no more than 1 MB .Your

photograph should be a portrait image in proportion (比例) to a 6"×4" portrait photograph.

It should be taken in colour and must not have been beautified with image filters(滤光镜).

Your student ID card will be provided to you by your department during Welcome Week.

Please be aware that you will not receive this smart card during Welcome Week if you do not

upload your image online by 4 September 2020.

Replacement cards

If your student ID card no longer works or has been damaged, or if you have changed

your personal or programme details, you should call into the Student Administration Centre

( SAC ) to request a new one. Please ensure that you bring your existing card with you.

Your replacement card will be produced free of charge.

If your card has been reported to the Police as stolen, please also call into the SAC to

request a replacement, bringing with you a copy of your Police Crime Report paperwork.

Your card will be produced free of charge.

If your card has been lost, a fee of £ 5 is charged for the production of a replacement

card. You are requested to use our online payment service.

学科网(北京)股份有限公司
Note: Replacement cards will be available for you to collect seven working days after

making the request. As a security measure, you are requested to collect your card in person at

the SAC.

24.Which of the following is a function of the student ID card?

A.To remind students to protect public property.

B.To enable students to use resources in the library.

C.To help students demonstrate their computer skills.

D.To provide information about campus facilities.

25.To meet the requirements of the photograph on the student ID card, a new student should

upload__________.

A.an image saved as a PDF file.

B.a colour picture of their passport.

C.a photo beautified by a photographer.

D.a portrait photograph of proper proportion.

26.In which case does a student have to pay for the replacement of the student ID card?

A.The theft of the card has been officially proved.

B.The card holder has changed his or her major.

C.The card dropped off into a lake by accident.

D.The card fails to work properly.

27.To collect your replacement card, you should___________.

A.fetch it at the SAC personally

B.go through a security check first

C.call the police station in advance

D.wait for seven days before getting it.

【答案】24.B 25.D 26.C 27.A

【解析】本文是说明文。 Bolton 大学介绍了学生证的作用,办理和使用学生证过程中

需要注意的问题和处理办法。
24. 细 节 理 解 题 。 根 据 第 一 段 “ Your student ID card identifies you as a student

at the University of Bolton. It will provide you with access to University facilities such as

University Libraries, Sports Centre, and Computing Services.” 可 知 , 学 生 证 表 明 你 是

Bolton 大学的学生。它将为您提供访问大学设施,如大学图书馆、体育中心和计算服

务的机会。即学生证可以让学生使用图书馆的资源。故选 B。

25. 细节理解题。根据 New students-photographs 部分的“ This should be a jpg file with a

size of no more than 1 MB. Your photograph should be a portrait image inproportion(比例)to


a "6×4" portrait photograph.”学校对学生证照片大小、比例的要求,可知新生需要上传

适当比例的肖像照片,故选 D。

26. 推理 判断 题。 根据 倒数 第二 段“ If your card has been lost, a fee of £ 5 is charged

for the production of a replacement card.” 可知如果学生证丢失,补新卡需要支付 5 英镑

的费用。由此判断出,选项 C:卡片不小心掉到湖里去了,属于丢失,应该支付补卡

费用。故选 C。

27. 细节理解题。根据最后一句“ As a security measure, you are requested to collect your

card in person at the SAC.”可知为了安全起见,应该亲自去 SAC 领取学生证。故选 A。

8.(2020 年,天津卷,第二次高考)

For people, who are interested in sound, the field of sound technology is definitely

making noise. In the past, sound engineers worked in the back rooms of recording studios, but

many of today’s sound professionals are sharing their knowledge and experience with

professionals in other fields to create new products based on the phenomenon we call sound.

Sound can be used as a weapon. Imagine that a police officer is chasing a thief. The thief

tries to escape. And the officer can’t let him get away. He pulls out a special device, points it

at the suspect, and switches it on. The thief drops to the ground. This new weapon is called a

Long Range Acoustic Device(LRAD, 远程定向声波发射器). It produces a deafening sound

so painful that it temporarily disables a person. The noise from the LRAD is directed like a

ray of light and travels only into the ears of that person, but it is not deadly.

For those who hunger for some peace and quiet, sound can now create silence. Let’s say

you are at the airport, and the little boy on the seat next to you is humming( 哼 唱 ) a short

commercial song. He hums it over and over again, and you are about to go crazy. Thanks to

the Silence Machine, a British invention, you can get rid of the sound without upsetting the

boy or his parents. One may wonder how the Silence Machine works. Well, it functions by

analyzing the waves of the incoming sound and creating a second set of outgoing waves. The

two sets of waves cancel each other out. Simply turn the machine or point it at the target, and

your peace and quiet comes back.

Directed sound is a new technology that allows companies to use sound in much the

same way spotlights( 聚光灯) are used in the theater. A spotlight lights up only one section of

a stage; similarly, “spotsound” creates a circle of sound in on targeted area. This can be useful

for businesses such as restaurants and stores because it offers a new way to attract customers.

Restaurants can offer a choice of music along with the various food choices on the menu,

学科网(北京)股份有限公司
allowing customers more control over the atmosphere in which they are dining. Directed

sound is also beginning to appear in shopping centers and even at homes.

28.What could be inferred from Paragraph 2 about the effect of the LRAD?

A.It causes temporary hearing loss.

B.It slows down a running man.

C.It makes it easy to identify a suspect.

D.It keeps the suspect from hurting others.

29.The Silence Machine is a device specially designed to ________.

A.silence the people around you

B.remove the sound of commercials

C.block the incoming sound waves

D.stop unwanted sound from affecting you

30.What feature do spotsounds and spotlights share?

A.They travel in circles.

B.They clear the atmosphere.

C.They can be transformed into energy.

D.They can be directed onto a specific area.

31.Directed sound can be used for ________.

A.creative designs of restaurant menus

B.ideal sound effects on the theater stage

C.different choices of music for businesses

D.strict control over any suspicious customer

32.What does the passage focus on?

A.How professionals invented sound products.

B.Inventions in the field of sound technology.

C.The growing interest in the study of sound.

D.How sound engineers work in their studios.

【答案】28.A 29.D 30.D 31.C 32.B

【解析】这是一篇说明文。文章主要介绍了一些声音技术领域的发明创造,包括声音

可以用作武器的远程声学设备、用来阻止有害声音影响你的静音机以及定向音响技术。
28. 推 理 判 断 题 。 根 据 第 二 段 中 的 “ It produces a deafening sound so painful that it

temporarily disables a person. The noise from the LRAD is directed like a ray of light and

travels only into the ears of that person, but it is not deadly.” 可知它发出震耳欲聋的声音,

这种声音令人如此痛苦以至于它会使人暂时丧失能力。 LRAD 发出的噪音像光线一样


被引导,只进入那个人的耳朵,但不会致命。由此可推知, LRAD 会导致暂时性听力

损失。故选 A。

29. 细 节 理 解 题 。 根 据 第 三 段 中 的 “ Well, it functions by analyzing the waves of the

incoming sound and creating a second set of outgoing waves. The two sets of waves cancel

each other out. Simply turn the machine on point it at the target, and your peace and quiet

comes back.”可知,它的功能是通过分析传入声音的波,并产生第二组传出的波。这两

组波会相互抵消。只要打开机器对准目标,你的宁静就会回来。由此可知,静音机是

一种专门设计用来阻止有害声音影响你的设备。故选 D。

30. 细节理解题。根据最后一段中的 “ A spotlight lights up only one section of a stage;

similarly, a “spotsound” creates a circle of sound in one targeted area.” 可知,聚光灯只能照

亮舞台的一个部分;类似地,一个“spotsound”在一个目标区域创建一个声音圈。由此

可知,spotsounds 和聚光灯的共同特性是它们可以被定向到一个特定的区域,故选 D。

31. 细节理解题。根据最后一段中的 “ A spotlight lights up only one section of a stage;

similarly,”spotsound” creates a circle of sound in on targeted area. This can be useful for

businesses such as restaurants and stores because it offers a new way to attract customers.

Restaurants can offer a choice of music along with the various food choice on the menu,

allowing customers more control over the atmosphere in which they are dining.” 可知,聚光

灯只能照亮舞台的一个部分;类似地,一个”spotsound”在一个目标区域创建一个声音

圈。这对餐馆和商店等企业很有用,因为它提供了一种吸引顾客的新方式。餐厅可以

提供音乐选择与菜单上的各种食物选择,让顾客更多地控制气氛,在他们用餐。由此

可知,定向音响可以用于不同的商业音乐选择。故选 C。

32.主旨大意题。根据第一段中的“In the past, sound engineers worked in the back rooms

of recording studios, but many of today's sound professionals are sharing their knowledge and

experience with professionals in other fields to create new products based on the phenomenon

we call sound.”可知,在过去,音响工程师在录音室的后室工作,但今天的许多音响专

业人士正在与其他领域的专业人士分享他们的知识和经验,以我们所谓的声音现象为

基础创造新产品。结合下文具体内容可知,本文着重论述了声音技术领域的发明创造 。

故选 B。

9.(2020 年,天津卷,第二次高考)

How to Use a Modern Public Library

Has it been a while since your last visit to a public library? If so, you may be surprised to

learn that libraries have changed for the better. It’s been years since they were dusty little

学科网(北京)股份有限公司
rooms with books. They have transformed themselves into places where you can develop

your love of knowledge meet interesting people, or find out how to start a business.

Check out a book. While libraries still loan out( 出借 )books, you’ll find it easier to get a

copy of whatever you’re looking for, thanks to a cooperative network of area libraries. Via

such networks, libraries share their books with each other through the use of delivery

vehicles. Once the book you’ve requested is delivered to the nearest branch, they will inform

you by e-mail, so you can pick it up.

Check out other items. The library is now a multimedia zone, loaded with information in

many formats( 载 体 形 式 ). You can borrow movies on DVDs, music on CDs, and popular

magazines. Some libraries even loan out toys and games. If a popular magazine you want

isn’t offered and the library keeps a list of such requests, they may bring it in when enough

interest is shown.

Join targeted reading groups. Libraries will often hold reading-group sessions targeted to

various age groups. Perhaps you’d like to learn a language or improve your English. The

library may sponsor a language group you could join. If you have difficulties reading, ask

about special reading opportunities. Your library might be able to accommodate you. And you

might find it relaxing to bring your small kid to a half-hour Story Time while you sit quietly

in a corner with a good book.

Start a business using the help of your local library. If you want to have a business of

your own, your local library can become a launch space for it. In library books and

computers, you can find information on starting a business. Many libraries will help you with

locally supplied information about business management shared through chambers of

commerce( 商会 )and government agencies, and they will offer printing, faxing and database

services you need.

33.Public libraries connected by a cooperative network benefit readers by______.

A.sharing their books on the Internet

B.giving access to online reading at a library branch

C.sending a needed book to a library branch nearby

D.making the checkout procedures diverse

34.According to Paragraph 3, what items may be checked out from a public library?

A.A magazine and an e-book.

B.A game and an oil painting.

C.A music CD and a kid’s toy.


D.A DVD and a video player

35.As is described in Paragraph 4, taking a small kid to a half-hour Story Time allows

______.

A.the kid to learn a new language

B.the parent to enjoy quiet reading

C.the kid to overcome reading difficulties

D.the parent to meet their program sponsor

36.Your local library can help you start a business by ______.

A.providing relevant information and supporting services

B.offering professional advice on business management

C.supplying useful information of your potential buyers

D.arranging meetings with government officials

37.What is the purpose of the passage?

A.To point out the importance of public libraries.

B.To encourage people to work in public libraries.

C.To introduce the improved services of public libraries.

D.To call for the modernization of public library systems.

【答案】33.C 34.C 35.B 36.A 37.C

【解析】这是一篇说明文。文章主要介绍了如何使用改造升级后现代公共图书馆。
33. 细节理解题。根据文章 Check out a book 中“ Via such networks, libraries share their

books with each other through the use of delivery vehicles. Once the book you've requested is

delivered to the nearest branch, they will inform you by e-mail, so you can pick it up.( 通过

这样的网络,图书馆通过运载车辆的使用彼此分享图书。一旦你要的书送到最近的书

店,他们会通过电子邮件通知你,你就可以去取了。 )”可知,合作网络连接的公共图

书馆通过向附近的图书馆分支机构发送所需的书籍,使读者受益。故选 C 项。

34.细节理解题。根据文章第三段“ You can borrow movies on DVDs, music on CDs, and

popular magazines. Some libraries even loan out toys and games. ( 你 可 以 借 DVD 电 影 、

CD 音乐和流行杂志。一些图书馆甚至借出玩具和游戏。 )”可知,从公共图书馆可以外

借 DVD 电影、CD 音乐、流行杂志、玩具和游戏等物品。故选 C 项。

35.细节理解题。根据文章第四段“ And you might find it relaxing to bring your small kid

to a half-hour Story Time while you sit quietly in a corner with a good book. ( 你可能会发现

带着你的孩子去半个小时的 “故事时间” 是一种放松,那时你安静地坐在角落里看一

本好书,)”可知,带孩子去半小时的“故事时间”,可以让父母享受安静的阅读。故

学科网(北京)股份有限公司
选 B 项。

36. 细节理解题。根据文章最后一段“ Many libraries will help you with locally supplied

information about business management shared through chambers of commerce (商会)and

government agencies, and they will offer printing and database services you need. ( 许多图书

馆会通过商会和政府机构分享本地提供的商业管理信息,并提供你需要的打印和数据

库服务。)”可知,你当地的图书馆可以通过提供相关信息和支持服务来帮助你创业。

故选 A 项。

37.推理判断题。根据文章标题 How to Use a Modern Public Library (如何使用一个现代

公共图书馆)和第一段中的“ you may be surprised to learn that libraries have changed for

the better. They have transformed themselves into places where you can develop your love of

knowledge meet interesting people, or find out how to start a business. (你可能会惊讶地发

现图书馆已经变得更好了。他们已经把自己变成了一个地方,在那里你可以发展你对

知识的热爱,遇见有趣的人,或者发现如何创业)”。再结合几个小标题: Check out

a book (借一本书); Check out other items (借其他物品); Join targeted reading

groups ( 加 入 有 针 对 性 的 阅 读 小 组 ) ; Start a business using the help of your local

library(在当地图书馆的帮助下创业)可知,本文主要介绍了在经过改善后的图书馆里,

人们可以做些什么,即:经过改善后的图书馆可以给人们提供什么服务。故选 C。

10.(2020 年,新高考全国 I 卷[山东])

According to a recent study in the Journal of Consumer Research, both the size and

consumption habits of our eating companions can influence our food intake. And contrary to

existing research that says you should avoid eating with heavier people who order large

portions(份), it's the beanpoles with big appetites you really need to avoid.

To test the effect of social influence on eating habits, the researchers conducted two

experiments. In the first, 95 undergraduate women were individually invited into a lab to

ostensibly(表面上)participate in a study about movie viewership. Before the film began, each

woman was asked to help herself to a snack. An actor hired by the researchers grabbed her

food first. In her natural state, the actor weighed 105 pounds. But in half the cases she wore a

specially designed fat suit which increased her weight to 180 pounds.

Both the fat and thin versions of the actor took a large amount of food. The participants

followed suit, taking more food than they normally would have. However, they took

significantly more when the actor was thin.

For the second test, in one case the thin actor took two pieces of candy from the snack
bowls. In the other case, she took 30 pieces. The results were similar to the first test: the

participants followed suit but took significantly more candy when the thin actor took 30

pieces.

The tests show that the social environment is extremely influential when we're making

decisions. If this fellow participant is going to eat more, so will I. Call it the “I’ll have what

she's having” effect. However, we'll adjust the influence. If an overweight person is having a

large portion, I'll hold back a bit because I see the results of his eating habits. But if a thin

person eats a lot, I'll follow suit. If he can eat much and keep slim, why can't I?

38.What is the recent study mainly about?

A.Food safety. B.Movie viewership.

C.Consumer demand. D.Eating behavior.

39.What does the underlined word “beanpoles” in paragraph 1 refer to?

A.Big eaters. B.Overweight persons.

C.Picky eaters. D.Tall thin persons.

40.Why did the researchers hire the actor?

A.To see how she would affect the participants.

B.To test if the participants could recognize her.

C.To find out what she would do in the two tests.

D.To study why she could keep her weight down.

41.On what basis do we “adjust the influence” according to the last paragraph?

A.How hungry we are. B.How slim we want to be.

C.How we perceive others. D.How we feel about the food.

【答案】38.D 39.D 40.A 41.C

【解析】本文是说明文。最近的研究表明:我们的饮食伙伴的大小和消费习惯都会影

响我们的食物摄入量。文章详述了这个实验的过程。
38.细节理解题。根据第一段中的“According to a recent study in the Journal of Consumer

Research, both the size and consumption habits of our eating companions can influence our

food intake”可知,根据消费者研究杂志最近的一项研究,我们的饮食伙伴的大小和消

费习惯都会影响我们的食物摄入量。因此这项研究是关于饮食行为的。故选 D。

39.词义猜测题。根据前半句“And contrary to existing research that says you should avoid

eating with heavier people who order large portions(份)”可知,现有的研究认为:你应

该避免和体重较重、点大份饭菜的人一起吃饭。后半句认为,你真正应该避免的是 the

beanpoles with big appetites 。由 contrary to 可推断出,画线词和 heavier people (超重的

学科网(北京)股份有限公司
人)相反,结合选项, D 选项(瘦瘦高高的人)正好和 heavier people 正好相反。故选

D。

40. 推理判断题。根据第二段的“ To test the effect of social influence on eating habits, the

researchers conducted two experiments” 可知, 为了测试社会影响对饮食习惯的影响,

研究人员进行了两个实验。根据倒数第三段的内容可知,在两个实验中,胖的和瘦的

演员都吃了大量的食物。参与者也照做,吃的食物比平常多。 然而,当演员是瘦的时

候,参与者们服用的食物更多。由此推断,研究人员雇用演员是为了看看她如何影响

参与者。故选 A。

41.推理判断题。根据最后一段中的“ If an overweight person is having a large portion, I’ll

hold back a bit because I see the results of his eating habits. But if a thin person eats a lot, I’ll

follow suit. If he can eat much and keep slim, why can’t I? ”可知,如果一个超重的人吃很

大一份,我会忍住一点,因为我看到了他饮食习惯的结果。但如果一个瘦的人吃很多 ,

我会跟着做。如果他吃得多保持苗条,为什么我不能呢? 因此推断我们是根据我们对

他人的看法(即:如何看待他人)来调整影响的。故选 C。

11.(2020 年,全国卷 I)

The connection between people and plants has long been the subject of scientific

research. Recent studies have found positive effects. A study conducted in

Youngstown , Ohio , for example, discovered that greener areas of the city experienced less

crime. In another,employees were shown to be 15% more productive when their workplaces

were decorated with houseplants.

The engineers at the Massachusetts Institute of Technology(MIT)have taken it a step

further changing the actual composition of plants in order to get them to perform

diverse , even unusual functions. These include plants that have sensors printed onto their

leaves to show when they’re short of water and a plant that can detect harmful chemicals in

groundwater. " We’re thinking about how we can engineer plants to replace functions of the

things that we use every day,"explained Michael Strano, a professor of chemical engineering

at MIT.

One of his latest projects has been to make plants glow( 发光)in experiments using some

common vegetables. Strano’s team found that they could create a faint light for three-and-a-

half hours. The light ,about one-thousandth of the amount needed to read by ,is just a start.

The technology, Strano said, could one day be used to light the rooms or even to turn trees

into self-powered street lamps.


In the future,the team hopes to develop a version of the technology that can be sprayed

onto plant leaves in a one-off treatment that would last the plant’s lifetime. The engineers are

also trying to develop an on and off " switch " where the glow would fade when exposed to

daylight.

Lighting accounts for about 7% of the total electricity consumed in the US. Since

lighting is often far removed from the power source( 电源)-such as the distance from a power

plant to street lamps on a remote highway-a lot of energy is lost during transmission( 传

输).Glowing plants could reduce this distance and therefore help save energy.

42.What is the first paragraph mainly about?

A.A new study of different plants.

B.A big fall in crime rates.

C.Employees from various workplaces.

D.Benefits from green plants.

43.What is the function of the sensors printed on plant leaves by MIT engineer?

A.To detect plants’ lack of water

B.To change compositions of plants

C.To make the life of plants longer.

D.To test chemicals in plants.

44.What can we expect of the glowing plants in the future?

A.They will speed up energy production.

B.They may transmit electricity to the home.

C.They might help reduce energy consumption.

D.They could take the place of power plants.

45.Which of the following can be the best title for the text?

A.Can we grow more glowing plants?

B.How do we live with glowing plants?

C.Could glowing plants replace lamps?

D.How are glowing plants made pollution-free?

【答案】42.D 43.A 44.C 45.C

【解析】这是一篇说明文。文章主要说明了绿色植物对人们很有好处,因此麻省理工

学院的工程师开发了一种发光植物。文章介绍了他们发明这种植物的过程,以及这种

植物的一些优势,指出在未来发光植物有可能取代路灯,达到节约能源的作用。
42. 主 旨 大 意 题 。 根 据 第 一 段 中 A study conducted in Youngstown, Ohio, for

学科网(北京)股份有限公司
example ,discovered that greener areas of the city experienced less crime. In another,

employees were shown to be 15% more productive when their workplaces were decorated

with houseplants. 可知例如,在俄亥俄州扬斯敦进行的一项研究发现,城市绿化较好的

地区犯罪率较低。在另一项研究中,当员工的工作场所被室内植物装饰时,他们的工

作效率会提高 15%。由此可知,第一段的主旨是关于绿色植物的益处。故选 D。

43. 细 节 理 解 题 。 根 据 第 二 段 中 These include plants that have sensors printed on their

leaves to show when they're short of water and a plant that can detect harmful chemicals in

groundwater.可知这就包括叶子上印有传感器来显示植物缺水的情况的植物,还有一种

植物可以检测到地下水中的有害化学物质。由此可知,麻省理工学院工程师植物叶片

上印上传感器的作用是检测植物缺水的情况。故选 A。

44. 细节理解题。根据最后一段中 Glowing plants could reduce this distance and therefore

help save energy. 可知发光的植物可以缩短这段距离,从而帮助节约能源。由此可知,

这种发光的植物在未来可能有助于减少能源消耗。故选 C。

45. 主旨大意题。根据最后一段中 Lighting accounts for about 7%of the total electricity

consumed in the US. Since lighting is often far removed from the power source-such as the

distance from a power plant to street lamps on a remote highway-a lot of energy is lost during

transmission. Glowing plants could reduce this distance and therefore help save energy. 可知

照明约占美国总耗电量的 7%。由于照明通常远离电源,例如从发电厂到偏僻公路上路

灯的距离,在传输过程中会损失大量能源。发光的植物可以缩短这段距离,从而帮助

节约能源。结合文章主要说明了绿色植物对人们很有好处,因此麻省理工学院的工程

师开发了一种发光植物,文章介绍了他们发明这种植物的过程,以及这种植物的一些

优势,指出在未来发光植物有可能取代路灯,达到节约能源的作用。由此可知, C 选

项“发光的植物能取代路灯吗?”最符合文章标题。故选 C。

12.(2020 年,全国卷 I)

Race walking shares many fitness benefits with running, research shows, while most

likely contributing to fewer injuries. It does, however, have its own problem.

Race walkers are conditioned athletes. The longest track and field event at the Summer

Olympics is the 50-kilometer race walk, which is about five miles longer than the marathon.

But the sport’s rules require that a race walker’s knees stay straight through most of the leg

swing and one foot remain in contact ( 接 触 ) with the ground at all times. It’s this strange

form that makes race walking such an attractive activity, however, says Jaclyn Norberg, an

assistant professor of exercise science at Salem State University in Salem, Mass.


Like running, race walking is physically demanding, she says, According to most

calculations, race walkers moving at a pace of six miles per hour would burn about 800

calories( 卡 路 里 ) per hour, which is approximately twice as many as they would burn

walking, although fewer than running, which would probably burn about 1,000 or more

calories per hour.

However, race walking does not pound the body as much as running does, Dr. Norberg

says. According to her research, runners hit the ground with as much as four times their body

weight per step, while race walkers, who do not leave the ground, create only about 1.4 times

their body weight with each step.

As a result, she says, some of the injuries associated with running, such as runner’s knee,

are uncommon among race walkers. But the sport’s strange form does place considerable

stress on the ankles and hips, so people with a history of such injuries might want to be

cautious in adopting the sport. In fact, anyone wishing to try race walking should probably

first consult a coach or experienced racer to learn proper technique, she says. It takes some

practice.

46.Why are race walkers conditioned athletes?

A.They must run long distances.

B.They are qualified for the marathon.

C.They have to follow special rules.

D.They are good at swinging their legs.

47.What advantage does race walking have over running?

A.It’s more popular at the Olympics.

B.It’s less challenging physically.

C.It’s more effective in body building.

D.It’s less likely to cause knee injuries.

48.What is Dr. Norberg’s suggestion for someone trying race walking?

A.Getting experts’ opinions.

B.Having a medical checkup.

C.Hiring an experienced coach.

D.Doing regular exercises.

49.Which word best describes the author’s attitude to race walking?

A.Skeptical. B.Objective.

C.Tolerant. D.Conservative.

学科网(北京)股份有限公司
【答案】46.C 47.D 48.A 49.B

【解析】这是一篇说明文。短文介绍了竞走相比跑步有诸多的优势,但是之前受过伤

的人,要想从事这样运动要谨慎,最好咨询专家的建议。
46. 细节理解题。根据第二段“ But the sport’s rules require that a race walker’s knees stay

straight through most of the leg swing and one foot remain in contact with the ground at all

times.”可知,但这项运动的规则要求竞走者的膝盖在摆动腿的大部分时间保持伸直,

一只脚始终与地面接触。由此可知,竞走运动员是需要具备某些条件的运动员是因为

运动员需要遵守特殊的规则。故选 C 项。

47.细节理解题。根据最后一段“ As a result, she says, some of the injuries associated with

running, such as runner’s knee, are uncommon among race walkers.” 可知,一些与跑步有关

的损伤,比如跑步者的膝盖,在竞走者中并不常见。由此可知,竞走与跑步相比的优

势是不太可能导致膝盖受伤。故选 D 项。

48. 细节理解题。根据最后一段 Dr. Norberg 说的话“ In fact, anyone wishing to try race

walking should probably first consult a coach or experienced racer to learn proper

technique(事实上,任何想尝试竞走的人都应该首先咨询教练或有经验的竞走运动员,

学习适当的技巧。)” 可知,Dr. Norberg 建议想尝试竞走的人征询专家的建议。故选 A

项。
49. 推理判断题。根据第一段“ Race walking shares many fitness benefits with running,

research shows, while most likely contributing to fewer injuries. It does, however, have its

own problem.” 可知,研究表明,竞走和跑步一样有很多健身益处,而且它还很少导致

受伤。不过,它也有自己的问题。由此判断出作者对于竞走的态度是客观的。故选 B

项。
13.(2020 年,全国卷 I)

Returning to a book you’ve read many times can feel like drinks with an old friend.

There’s a welcome familiarity - but also sometimes a slight suspicion that time has changed

you both, and thus the relationship. But books don’t change, people do. And that’s what

makes the act of rereading so rich and transformative.

The beauty of rereading lies in the idea that our bond with the work is based on our

present mental register. It’s true, the older I get, the more I feel time has wings. But with

reading, it’s all about the present. It’s about the now and what one contributes to the now,

because reading is a give and take between author and reader. Each has to pull their own

weight.
There are three books I reread annually .The first, which I take to reading every spring is

Emest Hemningway’s A Moveable Feast. Published in 1964, it’s his classic memoir of 1920s

Paris. The language is almost intoxicating ( 令人陶醉的),an aging writer looking back on an

ambitious yet simpler time. Another is Annie Dillard’s Holy the Firm, her poetic 1975 ramble

( 随 笔 ) about everything and nothing. The third book is Julio Cortazar’s Save Twilight:

Selected Poems, because poetry. And because Cortazar.

While I tend to buy a lot of books, these three were given to me as gifs, which might add

to the meaning I attach to them. But I imagine that, while money is indeed wonderful and

necessary, rereading an author’s work is the highest currency a reader can pay them. The best

books are the ones that open further as time passes. But remember, it’s you that has to grow

and read and reread in order to better understand your friends.

50.Why does the author like rereading?

A.It evaluates the writer-reader relationship.

B.It’s a window to a whole new world.

C.It’s a substitute for drinking with a friend.

D.It extends the understanding of oneself.

51.What do we know about the book A Moveable Feast?

A.It’s a brief account of a trip.

B.It’s about Hemingway’s life as a young man.

C.It’s a record of a historic event.

D.It’s about Hemingway’s friends in Paris.

52.What does the underlined word "currency" in paragraph 4 refer to?

A.Debt B.Reward. C.Allowance. D.Face value.

53.What can we infer about the author from the text?

A.He loves poetry.

B.He’s an editor.

C.He’s very ambitious.

D.He teaches reading.

【答案】50.D 51.B 52.B 53.A

【解析】这是一篇说明文。短文介绍了重新阅读的意义和益处并向读者介绍了作者每

年重读的三本书。作者鼓励读者去重新阅读书籍。
50. 推理判断题。根据第一段最后两句 “ But books don’t change, people do. And that’s

what makes the act of rereading so rich and transformative.( 但是书没变,人变了。那就是

学科网(北京)股份有限公司
使重新阅读行为如此丰富和富于变化之处 )” 和第二段“ The beauty of rereading lies in

that our bond with the work is based on our present register. It is true, the older I get, the

more I feel time has wings.( 重新阅读的美妙之处在于我们与作品的联系是基于我们现在

的心理状态。真的,我年纪越大,就越觉得时光飞逝。 )” 可推知,作者喜欢重新阅读

是因为重新阅读可以扩展对自己的理解。故选 D 项。

51. 推理判断题。根据倒数第二段 “ Published in 1964, it’s his classic memoir of 1920s

Paris.” 及“an aging writer looking back on an ambitious yet simpler time” 可知,这本书出

版于 1964 年,这是他关于 20 世纪 20 年代在巴黎的经典回忆录,是他老年时对那些野

心勃勃却更简单的日子的回顾。由此可判断出 A Movable Feast 是关于海明威年轻时的

生活。故选 B 项。

52.词义猜测题。根据最后一段中“while money is indeed wonderful and necessary,( 虽然

金钱确实是美妙而必要的 )” 可知,前后句为转折关系,根据上下文的语境可推知,

“ rereading an author’s work is the highest currency a reader can pay them.” 意为“但是但

重新阅读作品是读者能支付给他们的最高回报”,由此判断出划线词的意思是“回

报”。故选 B 项。

53. 推 理 判 断 题 。 根 据 倒 数 第 二 段 “ The third book is Julio Cortázar’s Save Twilight:

selected poems, because poetry.( 第三本书是胡里奥 · 科塔扎的《拯救暮光之城 : 诗歌精

选》,因为诗歌)”可知,作者是由于喜欢诗歌而喜欢这本书。故选 A 项。

13.(2020 年,全国卷 II)

Some parents will buy any high-tech toy if they think it will help their child, but

researchers said puzzles help children with math-related skills.

Psychologist Susan Levine, an expert on mathematics development in young children

the University of Chicago, found children who play with puzzles between ages 2 and 4 later

develop better spatial skills. Puzzle play was found to be a significant predictor of

cognition(认知) after controlling for differences in parents’ income, education and the amount

of parent talk, Levine said.

The researchers analyzed video recordings of 53 child-parent pairs during everyday

activities at home and found children who play with puzzles between 26 and 46 months of

age have better spatial skills when assessed at 54 months of age.

“The children who played with puzzles performed better than those who did not, on

tasks that assessed their ability to rotate( 旋 转 )and translate shapes,” Levine said in a

statement.
The parents were asked to interact with their children as they normally would, and about

half of children in the study played with puzzles at one time. Higher-income parents tended to

have children play with puzzles more frequently, and both boys and girls who played with

puzzles had better spatial skills. However, boys tended to play with more complex puzzles

than girls, and the parents of boys provided more spatial language and were more active

during puzzle play than parents of girls.

The findings were published in the journal Developmental Science.

54.In which aspect do children benefit from puzzle play?

A.Building confidence. B.Developing spatial skills.

C.Learning self-control. D.Gaining high-tech knowledge.

55.What did Levine take into consideration when designing her experiment?

A.Parents’ age. B.Children’s imagination.

C.Parents’ education. D.Child-parent relationship.

56.How do boy differ from girls in puzzle play?

A.They play with puzzles more often.

B.They tend to talk less during the game.

C.They prefer to use more spatial language.

D.They are likely to play with tougher puzzles.

57.What is the text mainly about?

A.A mathematical method. B.A scientific study.

C.A woman psychologist D.A teaching program.

【答案】54.B 55.C 56.D 57.B

【解析】本文是说明文。是关于孩子们玩智力游戏的研究,介绍了研究考虑的因素,

研究过程和结果。
54.细节理解题。根据第二段中… found children who play with puzzles between ages 2 and

4 develop better spatial skill( 在 2 岁到 4 岁之间玩智力游戏的儿童在空间能力方面更好 )

可知,孩子们可以从智力游戏中发展更好的空间技能。B. Developing spatial skills(发展

空间能力)符合以上说法,故选 B 项。

55. 细 节 理 解 题 。 根 据 第 二 段 中 Puzzle play was found to be a significant predictor of

cognition after controlling for difference in parents' income, education and the amount of

parent talk, Levine said.( Levine 说,在父母的收入、教育和父母谈话次数方面控制差异

性之后,拼图游戏被发现是一个重要的认知预测)可知 Levine 在设计这个试验时考虑了

父母的收入、教育程度和父母谈话的次数。C. Parents' education.(父母的教育)符合以上

学科网(北京)股份有限公司
说法,故选 C 项。

56. 细节 理解 题。 根据 倒数 第二 段中 However, boys tended to play with more complex

puzzles than girls, 可知男孩比女孩更喜欢玩复杂的谜题,即他们可能会玩难度更大的谜

题。 D. They are likely to play with tougher puzzles.( 他们有可能玩更复杂的谜题 )符合以

上说法,故选 D 项。

57.主旨大意题。本文是关于孩子们玩智力游戏的研究,介绍了研究考虑的因素,

研究过程和结果。所以是关于科学研究的。 B. A scientific study( 一项科学研究 )符合以

上说法,故选 B 项。

14.(2020 年,全国卷 II)

When you were trying to figure out what to buy for the environmentalist on your holiday

list, fur probably didn’t cross your mind. But some ecologists and fashion ( 时装 )enthusiasts

are trying to bring back the market for fur made from nutria(海狸鼠).

Unusual fashion shows in New Orleans and Brooklyn have showcased nutria fur made

into clothes in different styles. “It sounds crazy to talk about guilt-free fur-unless you

understand that the nutria are destroying vast wetlands every year”, says Cree McCree,

project director of Righteous Fur.

Scientists in Louisiana were so concerned that they decided to pay hunters $5 a tail.

Some of the fur ends up in the fashion shows like the one in Brooklyn last month.

Nutria were brought there from Argentina by fur farmers and let go into the wild. “The

ecosystem down there can’t handle this non-native species( 物 种 ).It’s destroying the

environment. It’s them or us.” says Michael Massimi, an expert in this field.

The fur trade kept nutria in check for decades ,but when the market for nutria collapsed

in the late 1980s,the cat-sized animals multiplied like crazy.

Biologist Edmond Mouton runs the nutria control program for Louisiana. He says it’s

not easy to convince people that nutria fur is green, but he has no doubt about it. Hunters

bring in more than 300,000 nutria tails a year, so part of Mouton’s job these days is trying to

promote fur.

Then there’s Righteous Fur and its unusual fashion. Model Paige Morgan says ,” To

give people a guilt-free option that they can wear without someone throwing paint on them-1

think that’s going to be a massive thing, at least here in New York.” Designer Jennifer

Anderson admits it took her a while to come around to the opinion that using nutria fur for

her creations is morally acceptable. She trying to come up with a lable to attach to nutria
fashions to show it is eco-friendly.

58.What is the purpose of the fashion shows in New Orleans and Brooklyn?

A.To promote guilt-free fur.

B.To expand the fashion market.

C.To introduce a new brand.

D.To celebrate a winter holiday.

59.Why are scientists concerned about nutria?

A.Nutria damage the ecosystem seriously.

B.Nutria are an endangered species.

C.Nutria hurt local cat-sized animals.

D.Nutria are illegally hunted.

60.What does the underlined word “collapsed” in paragraph 5 probably mean?

A.Boomed. B.Became mature. C.Remained stable. D.Crashed.

61.What can we infer abouf wearing fur in New York according to Morgan?

A.It’s formal. B.It’s risky. C.It’s harmful. D.It’s traditional.

【答案】58.A 59.A 60.D 61.B

【解析】本文是说明文。介绍了美国新奥良和布鲁克林举办了不同寻常的时装秀。展

出海狸鼠制成的皮衣。海狸鼠们每年都在破坏大片的湿地,因此设计师称这是一种环

保的举措,科学家们也对海狸鼠损坏生态平衡表示了担忧。
58. 推理判断题。根据第二段 Unusual fashion shows in New Orleans and Brooklyn have

showcased nutria fur made into clothes in different styles. “It sounds crazy to talk about guilt-

free fur-unless you understand that the nutria are destroying vast wetlands every year,”says

Cree McCree, project director of Righteous Fur. 得知,美国新奥尔良和布鲁克林举办了不

同寻常的时装秀,时装秀上展出海狸鼠皮制成的不同风格的衣服,项目总监 Cree

McCree 说:“除非了解海狸鼠正每年破坏大片湿地,否则谈论无罪感皮衣是很疯狂的

事情”,可以判断出由于海狸鼠对生态造成了巨大的破坏,这场海狸鼠皮衣时装秀销

售的是无罪恶感皮衣。故选 A。

59.推理判断题。根据第三段 Scientists in Louisiana were so concerned that they decided to

pay hunters $5 a tail.得知,科学家们如此担心以至于他们决定按照一条海狸鼠尾巴付给

猎人们 5 美元,可以推断出科学家们担忧海狸鼠们严重破坏生态平衡,。故选 A。

60. 词义猜测题。根据第五段 The fur trade kept nutria in check for decades, but when the

market for nutria collapsed in the late 1980s,the cat-sized animals multiplied like crazy. 得知

毛皮贸易使海狸鼠受到了几十年的控制,但是当海狸鼠市场在 20 世纪 80 年代末崩溃

学科网(北京)股份有限公司
时,这种猫大小的动物数量疯长,根据 but 判断,这是转折关系,以前由于皮毛交易,

海狸鼠处于控制,现在这种海狸鼠之所以能够数量激增,是由于市场不再销售海狸鼠

皮毛导致的,可以推断出划线词 collapsed 是和 D.crashed 倒闭的意思最相近。故选 D。

61.推理判断题。根据第二段 Model Paige Morgan says, “To give people a guilt –free option

that they can wear without someone throwing paint on them-I think that’s going to be a

massive thing, at least in New York. ” 得知,为了给人们一种无罪的选择,人们可以穿皮

衣而不被人们泼油漆,我认为在纽约将是一件巨大的事情,根据模特摩根所说得知,

在纽约穿皮质衣服是有风险的。故选 B。

15.(2020 年,全国卷 III)

We are the products of evolution, and not just evolution that occurred billions of years

ago. As scientists look deeper into our genes ( 基 因 ), they are finding examples of human

evolution in just the past few thousand years. People in Ethiopian highlands have adapted to

living at high altitudes. Cattle -raising people in East Africa and northern Europe have gained

a mutation (突变) that helps them digest milk as adults.

On Thursday in an article published in Cell, a team of researchers reported a new kind of

adaptation - not to air or to food, but to the ocean. A group of sea-dwelling people in

Southeast Asia have evolved into better divers. The Bajau, as these people are known, number

in the hundreds of thousands in Indonesia, Malaysia and the Philippines. They have

traditionally lived on houseboats; in recent times, they’ve also built houses on stilts ( 支柱) in

coastal waters. “They are simply a stranger to the land,” said Redney C.Jubilado, a University

of Hawaii researcher who studies the Bajau.

Dr. Jubilado first met the Bajau while growing up on Samal Island in the Philippines.

They made a living as divers, spearfishing or harvesting shellfish. “We were so amazed that

they could stay underwater much longer than us local islanders,” Dr. Jubilado said. “I could

see them actually walking under the sea.”

In2015, Melissa Ilardo, then a graduate student in genetics at the University of

Copenhagen, heard about the Bajau. She wondered if centuries of diving could have led to the

evolution of physical characteristics that made the task easier for them. “it seemed like the

perfect chance for natural selection to act on a population,” said Dr. Ilardo. She also said there

were likely a number of other genes that help the Bajau dive.

62.What does the author want to tell us by the examples in paragraph 1?

A.Environmental adaptation of cattle raisers. B.New knowledge of human evolution.


C.Recent findings of human origin. D.Significance of food selection.

63.Where do the Bajau build their houses?

A.In valleys. B.Near rivers. C.On the beach. D.Off the coast.

64.Why was the young Jubilado astonished at the Bajau?

A.They could walk on stilts all day. B.They had a superb way of fishing.

C.They could stay long underwater. D.They lived on both land and water.

65.What can be a suitable title for the text?

A.Bodies Remodeled for a Life at Sea B.Highlanders’ Survival Skills

C.Basic Methods of Genetic Research D.The World’s Best Divers

【答案】62.B 63.D 64.C 65.A

【解析】这是一篇说明文。最近一项对人类基因的研究发现,人类的进化不仅仅发生

在数十亿年前,而且最近几千年也有。 Bajau 人因为靠海为生,他们的身体已经进化成

更能适应海洋生活。
62.推理判断题。根据第一段的 we are the products of evolution, and not just evolution that

occurred billions of years ago. As scientists look deeper into our genes, they are finding

examples of human evolution in just the past few thousand years.( 我们是进化的产物,而且

不仅仅是数十亿年前的产物。当科学家更深入的研究我们的基因时,他们发现了人类

在过去几千年进化的例子 )可知,作者列举第一段的例子是为了告诉我们关于人类进化

的 一 个 新 信 息 , 那 就 是 人 类 在 最 近 几 千 年 也 在 进 化 。 B. New knowledge of human

evolution.(人类进化的新知识)符合以上说法,故选 B 项。

63.细节理解题。根据第二段的 The Bajau, as these people are known, number in hundreds

of thousands in Indonesia, Malaysia and the Philippines. They have traditionally live on

houseboats; in recent times, they've also built houses on stilts in coastal waters.( 这些人被称

为 Bajau,在印度尼西亚、马来西亚和菲律宾有数十万人。他们一直住在船屋上;最近

他们也把房子建在沿海水域的吊脚楼上 )可知,Bajau 把房子建在沿海区域。 D. Off the

coast.(沿海)符合以上说法,故选 D 项。

64. 细节理解题。根据第三段的 we were so amazed that they could stay underwater much

longer than us local islanders.( 我们很惊讶,他们在水下待的时间比我们当地的岛民要长

的多 ) 可知,让 Jubilado 感到吃惊的是 Bajau 人能在水下待更长的时间。 C. They could

stay long underwater. (他们能在水下待很长时间)符合以上说法,故选 C 项。

65. 主 旨 大 意 题 。 根 据 文 章 的 主 要 内 容 , 尤 其 是 第 二 段 的 On Thursday in an article

published in Cell, a team of researchers reported a new kind of adaption-not to air or to food,

学科网(北京)股份有限公司
but to the ocean.( 周四,在《细胞》杂志上发表的一篇文章中,一群研究人员报道了一

种新的适应——不是空气也不是食物,而是海洋 )可知,本文主要讲述了一种新的进化,

即长期生活在海边,靠海为生的生活方式,让 Bajau 人的身体进化成更适应海洋生活。

A. Bodies Remodeled for a Life at Sea.( 身体为适应海洋生活而重塑 )可以作为本文标题,

故选 A 项。

16.

Your next car might drive itself. After years of trials on city streets, driverless vehicles

are now nearing the live phase. Last month, a driverless bus began carrying passengers

through Lyon, France, Most in the automobile industry think self-driving vehicles will be on

the road by 2020 or before.

Driverless cars will at first be huddled with human-driven cars. But the first places

where they will become dominant( 统治的 ) are dense urban areas - precisely the spots most

damaged by the automobile age. Many advanced cities are already reducing the role of

human-driven cargo. Driverless cars will quicken that process and will bring us enormous

benefits.

Driverless cars will reduce accidents by around 90 percent. That’s big-the annual death

toll on the world’s roads is about 1.2 million a year. Pollution and carbon emissions will drop,

because urban driverless cars will be electric. The old, otherwise they would stay at home

most of the time and the disabled and teenagers will suddenly gain mobility.

On the other hand, driverless cars will bring catastrophe. The best thing about the

automobile age was that it employed tens of millions of people to make, market, insure and

drive vehicles. Over the next 20 years, the mostly low-skilled men who now drive trucks,

taxis and buses will see their jobs reduced. Carmakers are especially scared. The few cars of

the future might be made by tech companies such as Apple, Baidu and Google. Imaging the

impact on Germany, where the automotive sector is the largest industry.

Dramatic change is coming, and driverless cars could arrive by 2020. But governments

have barely begun thinking about it. Only 6 percent of the biggest US cities have factored

them into their long-term planning.

A decade ago anyone hardly saw the Smartphone coming. It has bought an epidemic of

mass addiction. Let’s hope we do a better job of handling the driverless car.

66.The underlined words “be huddled with” in Paragraph 2 can be replaced by “ “.

A.show up B.exist together

C.get rid of D.take the place of


67.Why can driverless cars reduce pollution and carbon emissions?

A.Driverless cars reduce the number of cars.

B.Driverless cars will be powered by electricity.

C.Driverless cars save fuel by driving themselves.

D.Driverless cars will reduce too many accidents.

68.What’s the author’s attitude toward driverless cars?

A.Doubtful. B.Negative.

C.Objective. D.Worried.

【答案】66.B 67.B 68.C

【解析】这是一篇说明文。文章向我们介绍了无人驾驶汽车对未来的影响,说明了无

人驾驶汽车的优势以及可能产生的问题,预测戏剧性的变化正在到来,无人驾驶汽车

可能会在 2020 年到来。但政府几乎还没有开始考虑这个问题。只有 6%的美国大城市

在其长期规划中考虑到了这些问题。
66. 词义猜测题。根据后文 But the first places where they will become dominant( 统治的 )

are dense urban areas - precisely the spots most damaged by the automobile age. 可知但它们

将首先占据主导地位的地方是人口稠密的城市地区,而这些地区正是受汽车时代破坏

最严重的地方。由此可知,无人驾驶汽车首先占据主导的地位的地方是人口稠密的城

市地区,那么意思是无人驾驶汽车一开始将会与人类驾驶的汽车共同存在。划线短语

意思为“共存”。故选 B。

67. 细节理解题。根据第三段中 Pollution and carbon emissions will drop, because urban

driverless cars will be electric. 可知污染和碳排放将会下降,因为城市无人驾驶汽车将是

电动的。由此可知,无人驾驶汽车可以减少污染和碳排放,是因为无人驾驶汽车将由

电力驱动。故选 B。

68. 推理判断题。根据第三段中 Pollution and carbon emissions will drop, because urban

driverless cars will be electric. 可知污染和碳排放将会下降,因为城市无人驾驶汽车将是

电动的。以及第四段中 On the other hand, driverless cars will bring catastrophe. 可知另一

方面,无人驾驶汽车将带来灾难。由此可推知,作者分别说明了无人驾驶汽车的优点

和存在问题,可知作者对无人驾驶汽车的态度是客观的。故选 C。

17.

We all start out with plans about what our life will eventually look like. And those plans,

obviously, don’t include disappointment in the road that each of us will surely face.

When we picture the future, we see only love, success and great friendships.

学科网(北京)股份有限公司
But sooner or later, we realize that life doesn’t go according to plan. Instead, after we’ve

carefully made our plans, we meet the unexpected.

However, unexpected things are to be expected. And once we accept this fact of life,

believe it or not, it is liberating(带来自由的).

Once we understand that no one’s life goes according to plan, then the difference

between successful and unsuccessful people becomes clear. Successful people are good at

handling disappointment, as they know that it’s part of life. And unsuccessful people let pain

and disappointment take over.

I know people who talk about a divorce or the death of a loved one as if the event

happened yesterday, even if it happened a decade or more ago.

It’s reasonable to experience pain at the time of a loss. However, it isn’t reasonable to

keep feeling the pain of an experience that happened a really long time ago.

Most of us have suffered in the past, but past pain should have nothing to do with how

we feel in the present-it is over and done with.

As for me, my focus is always on today and the future- that is how I live every day. I

don’t need to remember the past, because I have accepted that my life doesn’t always go

according to plan.

No one’s life goes according to plan. We all have periods of pain and suffering, which

are natural parts of being alive. So don’t allow your past to define who you are. When you

wake up each morning, just remember to make the most of the day ahead.

69.In what way are successful people different from others, according to the author?

A.They are more hopeful about the future. B.They are working harder than others.

C.They can handle the unexpected better. D.They are good at planning.

70.How does the author think we should treat past pain?

A.We should remember it as a lesson. B.We should forget about it and look ahead.

C.We should keep it to ourselves. D.We should share it with friends to get

comfort from them.

71.What is the main purpose of this article?

A.To tell readers how to plan for their future.

B.To explain why past pain affects people greatly.

C.To show what we should expect from life.

D.To suggest how to handle the unexpected in life.

【答案】69.C 70.B 71.D


【解析】这是一篇说明文。文章主要说明了我们都开始计划我们的生活最终会是什么

样子,但是生活并不是按计划进行的,在我们仔细制定计划之后,我们遇到了意想不

到的事情。文章中作者提出了该如何面对生活中意外的建议。
69. 细节理解题。根据第五段中 Once we understand that no one’s life goes according to

plan, then the difference between successful and unsuccessful people becomes clear.

Successful people are good at handling disappointment, as they know that it’s part of life. 可

知一旦我们明白了没有人的生活是按计划进行的,那么成功者和失败者之间的区别就

变得清晰了。成功的人善于处理失望,因为他们知道这是生活的一部分。由此可知,

成功的人能更好的处理意外,因此不同于其他人。故选 C。

70.推理判断题。根据倒数第三段 Most of us have suffered in the past, but past pain should

have nothing to do with how we feel in the present-it is over and done with. 可知我们大多数

人都经历过痛苦,但过去的痛苦与我们现在的感觉毫无关系 ——已经过去,已经过去

了。以及倒数第二段中 As for me, my focus is always on today and the future- that is how I

live every day.可知对我来说,我的关注点总是在今天和未来——这就是我每天的生活

方式。由此可推知,作者认为我们应该忘记过去的痛苦,向前看。故选 B。

71.推理判断题。根据第三段 But sooner or later, we realize that life doesn’t go according to

plan. Instead, after we’ve carefully made our plans, we meet the unexpected.可知但迟早,我

们会意识到生活并不是按计划进行的。相反,在我们仔细制定计划之后,我们遇到了

意想不到的事情。结合文章主要说明了我们都开始计划我们的生活最终会是什么样子 ,

但是生活并不是按计划进行的,在我们仔细制定计划之后,我们遇到了意想不到的事

情。文章中作者提出了该如何面对生活中意外的建议。由此可推知,这篇文章的主要

目的是建议如何处理生活中的意外。故选 D。

18.(2020 年 1 月,浙江卷)

Today's world is not an easy adjustment for young adults. Key skill set for success is

persistence ( 毅 力 ), a characteristic that researchers say is heavily influenced by fathers.

Researchers from Brigham Young University discovered that fathers are in a unique position

to help their adolescent children learn persistence.

BYU professors Laura Padilla-Walker and Randal Day arrived at these findings after

following 325 American families over several years. And over time , the persistence gained

through fathers led to higher achievement in school.

"There are relatively few studies that stress the unique role of fathers , "Padilla-Walker

said. "This research also helps to prove that characteristics such as persistence-which can be

学科网(北京)股份有限公司
taught-are key to a child's life success.”

Researchers determined that dads need to practice an "authoritative" parenting style.

Authoritative parenting is not authoritarian:rigid , demanding or controlling. Rather , an

authoritative parenting style includes some of the following characteristics:children feel

warmth and love from their father ; responsibility and the reasons behind rules are stressed

children are given an appropriate level of autonomy(自主权).

In the study , about 52 percent of the dads exhibited above-average levels of

authoritative parenting. A key finding is that over time , children raised by an authoritative

father were significantly more likely to develop persistence , which leads to better outcomes

in school.

This particular study examined 11 to 14-year-olds living in two-parent homes. Yet the

researchers suggest that single parents still may play a role in teaching the benefits of

persistence,which is an avenue of future research.

72.What is special about the BYU professors' study?

A.It centered on fathers' role in parenting.

B.It was based on a number of large families.

C.It analyzed different kinds of parenting styles.

D.It aimed to improve kids' achievement in school.

73.What would an authoritative father do when raising his children?

A.Ignore their demands. B.Make decisions for them.

C.Control their behaviors. D.Explain the rules to them.

74.Which group can be a focus of future studies according to the researchers?

A.Single parents.

B.Children aged from 11 to 14.

C.Authoritarian fathers.

D.Mothers in two-parent homes.

75.Which of the following is the best title for the text?

A.Three Characteristics of Authoritative Fathers.

B.Key Skills for Young Adults to Succeed in Future.

C.Children Tend to Learn Determination from Father.

D.Family Relationship Influences School Performance.

【答案】72.A 73.D 74.A 75.C

【解析】这是一篇说明文。研究人员发现,父亲在帮助青春期的孩子学习毅力方面具

有独特的地位。研究人员认为,父亲们需要采用一种“权威”的养育方式,且一项重
要的发现是,随着时间的推移,由权威父亲抚养长大的孩子更有可能培养出坚持不懈

的精神,从而在学校取得更好的成绩。
72. 细 节 理 解 题 。 根 据 第 一 段 最 后 一 句 Researchers from Brigham Young University

discovered that fathers are in a unique position to help their adolescent children learn

persistence. (杨百翰大学的研究人员发现,父亲在帮助青春期的孩子学习毅力方面具

有独特的地位)可知杨百翰大学研究的特别之处在于,它关注的是父亲在养育子女中

的角色。故选 A。

73. 推理判断题。根据第四段中 Rather , an authoritative parenting style includes some of

the following characteristics: children feel warmth and love from their father ; responsibility

and the reasons behind rules are stressed children are given an appropriate level of autonomy.

(相反,权威的养育方式包括以下一些特征:孩子们感受到来自父亲的温暖和爱;强调

责任和规则背后的原因;孩子们被给予适当的自主权)可知一个有权威的父亲在抚养

孩子时会向他们解释规则。故选 D。

74.细节理解题。根据最后一段最后一句 Yet the researchers suggest that single parents still

may play a role in teaching the benefits of persistence , which is an avenue of future

research.(然而,研究人员认为,单亲父母仍然可能在教授坚持不懈的好处方面发挥作

用,这是未来研究的一个方向)可知根据研究人员的说法,单亲父母是未来研究的重

点。故选 A。

75. 主旨大意题。根据文章倒数第二段中 A key finding is that over time , children raised

by an authoritative father were significantly more likely to develop persistence , which leads

to better outcomes in school.(一项重要的发现是,随着时间的推移,由权威父亲抚养长

大的孩子更有可能培养出坚持不懈的精神,从而在学校取得更好的成绩)和文章主要

内容为说明父亲在养育子女中的角色,故选 C 选项“孩子们倾向于向父亲学习决心”

最符合文章标题。故选 C。

19.(2020 年 1 月,浙江卷)

Milwaukee, Wisconsin, is road testing a new way to keep winter roads ice-free – by

spreading on them cheese brine, the salty liquid used to make soft cheese, like mozzarella.

Wisconsin, also called "America's Dairyland," is famous for its cheese. The state

produced 2.8 billion pounds of cheese last year! a result, there was a lot of leftover cheese

brine. Disposing of( 处置 )the brine can be expensive. So what should cheese makers do with

the waste?

Normally, towns use rock salt to de-ice streets. The salt lowers waters' freezing point,

学科网(北京)股份有限公司
causing ice to melt( 融化). But using cheese brine could help both cheese producers and cities

save money, while keeping roads safe. Cheese brine has salt in it, which, like the rock salt,

helps lower water's freezing point.

In addition to saving money, cheese brine could also be a more environment-friendly

option. Many people suspect that all the rock salt used every winter is harming the

environment.

Rock salt is made of sodium chloride, the sane con-pound ( 化合物)in ordinary table salt.

Sounds harmless, right? But while you probably add only a small amount of salt to your food,

road crews spread about 20 million tons of salt on U.S. Roads every year!

The chemical washes off roads and goes into the ground. There it can pollute drinking

water, harm plants. and eat away soil. By spreading cheese brine on streets before adding a

layer of rock salt, Milwaukee may be able to cut its rock salt use by 30 percent.

Cheese brine has a downside too – a shell similar to that of bad milk. "I don't really mind

it," Emil Norby told Modern Farmer magazine. He works for one of Wisconsin's county

highway commissions and came up with the idea of using cheese brine. "Our roads smell like

Wisconsin!" he said.

76.Why can cheese brine help keep winter roads ice-free?

A.It is soft. B.It contains salt. C.It is warm. D.It has milk in it.

77.What is a benefit of using cheese urine on roads?

A.Improving air quality. B.Increasing sales of rock salt.

C.Reducing water pollution. D.Saving the cheese industry.

78.Milwaukee's new way to de-ice streets may be an example of_______________.

A.barking up the wrong tree B.putting the cart before the horse

C.robbing Peter to pay Paul D.killing two birds with one stone

【答案】76.B 77.C 78.D

【解析】这是一篇说明文。主要说明了威斯康辛州的密尔沃基市正在试验一种新的路

面防冻方法——在路面上涂上奶酪盐水。文章说明了这种新方法的好处是,相比于常

用的岩盐,奶酪盐水既可以帮助道路不结冰,同时又很省钱环保。
76.细节理解题。根据第三段最后一句 Cheese brine has salt in it, which, like the rock salt,

helps lower water's freezing point. (奶酪盐水中含有盐,就像岩盐一样,有助于降低水

的冰点)可知因为奶酪盐水含有盐,所以能帮助冬季道路不结冰。故选 B。

77. 推理判断题。根据倒数第二段中 There it can pollute drinking water, harm plants. and

eat away soil. By spreading cheese brine on streets before adding a layer of rock salt,
Milwaukee may be able to cut its rock salt use by 30 percent. (在那里它可以污染饮用水,

伤害植物,侵蚀土壤。通过在街道上撒上奶酪盐水,然后再撒上一层岩盐,密尔沃基

或许可以将岩盐的使用量减少 30%)可知岩盐会污染饮用水,而在道路上使用奶酪盐

水可以减少水污染。故选 C。

78.推理判断题。根据第三段最后一句 Cheese brine has salt in it, which, like the rock salt,

helps lower water's freezing point. (奶酪盐水中含有盐,就像岩盐一样,有助于降低水

的冰点)以及第四段第一句 In addition to saving money, cheese brine could also be a more

environment-friendly option.(除了省钱,奶酪盐水也是一个更环保的选择)可知相比于

常用的岩盐,奶酪盐水既可以帮助道路不结冰,同时又很省钱环保,可知密尔沃基市

的街道除冰新方法是一个“一石二鸟”的成功例子。故选 D。

20.(2018 年 3 月,天津卷)

Most people aren’t good at creative problem solving for two reasons: (1) They are not

trained in how to be creative. (2) They don’t understand group strength well enough to

harness(驾驭)their power to maximize group creativity.

A key element of creativity is applying existing knowledge to a new problem. The

more people getting involved in solving it, the more knowledge there is to work on it.

Unfortunately, research shows that the traditional brainstorming methods fail to achieve that

goal. When groups get together to exchange ideas, they actually come up with fewer ideas

overall than if they each had worked alone.

To fix this problem, you should consider the two stages of group problem-solving:

divergence ( 分 散 ) and convergence ( 集 中 ) . Divergence happens when the group

considers as many different potential solutions as possible. Convergence happens when the

various proposed solutions are evaluated and reduced to a smaller set of candidate solutions

to the current problem.

The essential principle of group creativity is that individuals working alone diverge,

whereas group members working together converge. In groups, once a member states a

potential solution, that makes others think about the problem similarly. That is why groups

working together diverge less than individuals working alone.

Therefore, be aware of when to diverge and when to converge. For example, early in

the problem-solving process, have group members work alone to write down statements

describing the problem. Then get them back to discuss their descriptions. The group

discussion will lead everyone to accept one or a small number of these statements to work

学科网(北京)股份有限公司
on—this is healthy convergence.

When starting to generate solutions, you again want divergence. Have people work

alone to start. Then collect people’s initial ideas and send them around to others and allow

the divergence to continue as everyone individually builds on the ideas of other members.

Finally, let the group discuss the resulting ideas. This discussion will gradually lead the

group to converge on a small number of candidate solutions.

This simple method works effectively, because it respects what individuals and groups

do best.

79.Research shows that the traditional brainstorming methods ______.

A.actually limit group creativity

B.greatly encourage group creativity

C.enable people to form more ideas together

D.prevent people’s involvement in the problems

80.According to Para. 4, when a member presents an idea, others tend to_____.

A.think the other way round

B.follow his way of thinking

C.be more confident in their own ideas

D.be less willing to share their own ideas

81.What should group members first do early in the problem-solving process?

A.Discuss the problem. B.Simplify the problem.

C.Put down group statements together. D.Write down their individual descriptions.

82.How can each group member make changes to his initial solution?

A.By adding in collected evidence. B.By reorganizing his own words.

C.By drawing on others’ ideas. D.By making his statement briefer.

83.What is the author’s purpose in writing the passage?

A.To advocate a way to make group thinking more effective.

B.To demonstrate the difficulty in organizing group thinking.

C.To highlight the differences between divergence and convergence.

D.To show the advantage of group thinking over individual thinking.

【答案】79.A 80.B 81.D 82.C 83.A

【解析】这是一篇说明文。短文介绍了团体不擅长创造性解决问题的原因以及提出了

一种使团体更具有创造力的方法。
79.细节理解题。由第二段“A key element of creativity is applying existing knowledge to a

new problem. The more people getting involved in solving it, the more knowledge there is to
work on it. Unfortunately, research shows that the traditional brainstorming methods fail to

achieve that goal.”可知,创造力的一个关键因素是应用现有知识解决新问题,人们越多

地涉及创新地解决问题, 就会有越多的知识被应有,不幸的是, 研究表明: 传统的

脑力风暴并不能如愿以偿达到目。所以研究表明,传统的头脑风暴方法实际上限制了

团队的创造力。故选 A 项。

80. 细节理解题。由第四段“ In groups, once a member states a potential solution, that

makes others think about the problem similarly.”可知,在团体,一旦一个成员阐述了一个

潜在的解决办法,就会使其他成员国对这个问题有类似的想法。所以根据第 4 段,当

一个成员提出一个想法时,其他人往往会跟随他的思维方式。故选 B 项。

81. 细 节 理 解 题 。 由 第 五 段 “ For example, early in the problem-solving process, have

group members work alone to write down statements describing the problem.”可知,例如,

解决问题的早期过程, 让成员单独写下对问题的叙述。所以在问题解决过程的早期,

团队成员应该首先写下他们各自的叙述。故选 D 项。

82. 细节理解题。由倒数第三段“ Then collect people’s initial ideas and send them around

to others and allow the divergence to continue as everyone individually builds on the ideas of

other members.”可知,然后收集成员的最初想法并发送给其他成员, 允许其他成员基

于别人想法的基础上创新。所以每个组成员可以借鉴别人的想法更改其初始解决方案 。

故选 C 项。

83.推理判断题。由其第一段“Most people aren’t good at creative problem solving for two

reasons” 可 知 , 众 人 不 擅 长 创 造 性 解 决 问 题 的 原 因 有 两 个 。 再 根 据 第 二 段

“ Unfortunately, research shows that the traditional brainstorming methods fail to achieve

that goal. When groups get together to exchange ideas, they actually come up with fewer

ideas overall than if they each had worked alone” 可知,不幸的是,研究表明,传统的头

脑风暴方法未能实现这一目标。当团队聚在一起交流想法时,他们实际上提出的想法

比各自单独工作时要少。所以短文的前两段提出了团体不擅长创造性解决问题的原因

以及现存的团队创造力解决方法的弊端,所以短文从第三段起就上述现象提出了一种

使团体更具有创造力的方法。所以短文的目的是提出了一种使团体更具有创造力的方

法。故选 A 项。

21.(2018 年 3 月,天津卷)

If you were bringing friends home to visit, you could show them the way. You know

the landmarks—a big red house or a bus-stop sign. But what if you were swimming in the

middle of the Atlantic Ocean? Could you still find your way home? A loggerhead turtle (海

学科网(北京)股份有限公司
龟)could.

According to Dr. Ken Lohmann, loggerheads have a magnetic (磁力的) sense based

on Earth’s magnetic field. It helps them locate the best spots for finding food and their home

beaches.

Scientists already know that several other animals, such as whales and honeybees, can

detect(探测到) magnetic fields. The difference between them and loggerheads, however,

is the way they learn to use their magnetic sense. Young whales and honeybees can learn

from adults. Loggerheads are abandoned as eggs.

As newborn loggerheads have no adults to learn from, what helps them figure out how

to use their magnetic sense? Lohmann thinks one of the cues was light on the sea.

Baby loggerheads hatch only at night. However, a small amount of light reflects off the

ocean. The light makes that region brighter. Heading toward the light helps them get

quickly out to sea, where they can find food. Lohmann tested whether newborn loggerheads

use this light source to set their magnetic “compasses” (罗盘). He and his team put some

newborns in a water tank and recorded which way they swam. Around the tank, the

scientists created a magnetic field that matched the Earth’s. They set a weak light to the east

of the magnetic field. Then they let the newborns go.

At first, the newborns swam toward the light. After the scientists turned off the light,

the turtles that had seen the light in the east always swam toward east. When the researchers

reversed (颠倒) the magnetic field, these turtles turned around and swam toward the new

“east”.

This and the follow-up experiments all showed that loggerheads use light from the

outside world to set their magnetic “compasses” and then remember the “correct” direction.

If a turtle hatches on a brightly-lit beach, that would damage its magnetic sense forever and

make survival hard for the turtle.

Lohmann’s work has led others to protect the habitat of this endangered species. Yet

many questions about these creatures remain unanswered, and researchers have a lot to

study.

84.Loggerheads and whales differ in the way they______.

A.bring up their young

B.recognize landmarks

C.detect magnetic fields

D.learn to find directions


85.What is needed for newborn loggerheads to set their magnetic sense?

A.Weak light reflected off the ocean. B.Help from adult loggerheads.

C.Bright sunlight from the sky. D.Food in warmer waters.

86.In the experiment, after the newborns’ magnetic sense was set, their moving direction was

determined by ____.

A.the light

B.the magnetic field

C.other unknown factors

D.the light and the magnetic field

87.What is the significance of Lohmann’s research work?

A.It enables researchers to keep track of turtles.

B.It contributes to the studies of the magnetic field.

C.It offers a new solution to environmental pollution.

D.It helps protect the loggerheads’ living environment.

88.What could be the best title of the passage?

A.Experiments on Loggerheads

B.The Survival of the Sea Turtle

C.The Loggerhead’s Built-in “Compass”

D.Comparison of Loggerheads and Other Animals

【答案】84.D 85.A 86.B 87.D 88.C

【解析】这是一篇说明文。短文介绍了红海龟与其它很多动物一样都可以感知磁场。

与其它动物感知磁场不同的是,海龟依靠光来感知磁场。
84.细节理解题。由第三段“ The difference between them and loggerheads, however, is the

way they learn to use their magnetic sense.”可知,然而,它们和红海龟之间的区别在于它

们学会使用磁感应的方式。所以红海龟和鲸鱼的区别在于它们学习寻找方向的方式。

故选 D 项。

85. 细 节 理 解 题 。 由 第 五 段 “ Baby loggerheads hatch only at night. However, a small

amount of light reflects off the ocean. The light makes that region brighter. Heading toward

the light helps them get quickly out to sea, where they can find food”可知,小红海龟只在晚

上孵化。然而,微弱的光反射到海洋上。光线使那个区域更亮。朝着光走有助于它们

迅速到达大海,在那里它们可以找到食物。所以微弱的光反射到海洋上,来帮助设定

他们的磁场感应。故选 A 项。

86. 细节理解题。由倒数第三段“ At first, the newborns swam toward the light. After the

学科网(北京)股份有限公司
scientists turned off the light, the turtles that had seen the light in the east always swam

toward east. When the researchers reversed (颠倒) the magnetic field, these turtles turned

around and swam toward the new “east””可知,刚开始,新出生的小海龟向光游去。科学

家们把灯关掉后,看到了东方有光的海龟总是向着东方游动。当研究者颠倒磁场当研

究员颠倒磁场之后, 这些海龟调转方向, 向着新的“东方”游去。所以实验中,新

生儿磁感建立后,其运动方向由磁场决定。故选 B 项。

87.细节理解题。由最后一段“ Lohmann’s work has led others to protect the habitat of this

endangered species”可知,Lohmann 的研究使得其他人保护这种濒危物种的栖息地。所

以 Lohmann 研究工作的意义在于它有助于保护红海龟的生存环境。故选 D 项。

88.主旨大意题。通过读全文可知,海龟与其它很多动物一样都可以感知磁场。与其它

动物感知磁场不同的是,海龟依靠光来感知磁场。所以短文最佳标题为“红海龟的内

置“指南针”。故选 C 项。

22.(2018 年 3 月,天津卷)

When you choose a higher education course, here are some tips for you to follow.

Choosing your course

There are a range of subjects and courses available. Many are work-related; others are

academic. You’ll need to look beyond the course title because courses with almost the same

name may differ enormously. For example, if you’re interested in construction but wish to

use your creative skills, you may be better suited to a building design management course

than a building project management course.

Style of learning

Think about the style of learning that best suits you. It’s important to study a subject

that you particularly enjoy, but if you want to follow a career in a different area, you may

consider taking the subjects you want to learn and joining them into one degree. Many

degree courses are made up of different subject modules which deal with particular areas of

interest.

Choosing where to study

Where you study can be almost as important as what you study. One thing to consider

is whether you want to live at home or move away. For some people, living at home offers

the best of both worlds, but it may make sense to move if you want to study a particular

course at a specific university or college. Many students value the experience of living on

campus or in a different part of the country as an important part of university life.

Finding out more about courses


Once you’ve got a list of courses that appeal to you, it’s worth looking at some other

sources of information about them, like independent reports. These provide information on

the numbers of students who finished the course, student satisfaction and more.

Open days

Open days usually take place in spring or autumn. Although you may have an idea

about what a course is like, there’s no substitute (替代) for attending a class yourself. If

you can’t come on the open days, you can sign up for a guided tour instead.

89.You need to look beyond the course title because courses with similar names may be

______.

A.creativity-focused B.work-related

C.far different D.quite similar

90.What are you advised to do if your interest disagrees with your future career?

A.Add preferred subjects to your degree courses.

B.Choose the subjects based on your interest.

C.Earn different degrees.

D.Give up your interest.

91.What do we learn about from Para. 4?

A.Disadvantages of studying on campus.

B.Reasons for choosing where to study.

C.Importance of attending a nearby college.

D.Problems of going to a faraway university.

92.Why is it recommended to check independent reports?

A.To increase applicants’ interest.

B.To better understand a course.

C.To figure out how to complete a course.

D.To find out how different the courses are.

93.On the open days, would-be college students can ______.

A.sign up for a particular course

B.take a guided tour on the campus

C.see whether there is a substitute course

D.have a first-hand experience of a course

【答案】89.C 90.A 91.B 92.B 93.D

【解析】这是一篇说明文。短文介绍了当读者选择高等教育课程时,可以参考的一些

学科网(北京)股份有限公司
建议。
89.细节理解题。由 Choosing your course 中的“You’ll need to look beyond the course title

because courses with almost the same name may differ enormously”可知,你需要看的不仅

仅是课程名称, 因为很多课程名字几乎相同而内容大相径庭。所以你需要看的不仅仅

是课程名称,因为名字相似的课程可能会有很大的不同。故选 C 项。

90. 细 节 理 解 题 。 由 Style of learning 中 的 “ It’s important to study a subject that you

particularly enjoy, but if you want to follow a career in a different area, you may consider

taking the subjects you want to learn and joining them into one degree.” 可知,学习你最喜

爱的科目特别重要,但是如果你想从事不同领域的职业,你可以选你想学和想纳入学

位的课程。所以如果你的兴趣与你未来的职业不一致,你该在你的学位课程中加入你

喜欢的科目。故选 A 项。

91.推理判断题。由第四段“ For some people, living at home offers the best of both worlds,

but it may make sense to move if you want to study a particular course at a specific university

or college. Many students value the experience of living on campus or in a different part of

the country as an important part of university life.”可知, 对于某些人来说,宅在家里比

外面好。 如果你想在一个特色大学或学院学习特别的课程, 在外面学习更能讲得通。

很多学生重视在校经历、 在不同国家的经历作为大学生活的重要部分。由此判断出,

我们从第四段中学到了选择学习地点的原因。故选 B 项。

92. 推理判断题。由 Finding out more about courses 中的“ These provide information on

the numbers of students who finished the course, student satisfaction and more.” 可知,这些

信息提供了完成课程的学生人数、学生满意度等方面的信息。由此判断出为什么建议

检查独立报告是为了更好地理解一门课程。故选 B 项。

93. 细节理解题。由 Open days 中的“ Open days usually take place in spring or autumn.

Although you may have an idea about what a course is like, there’s no substitute (替代)for

attending a class yourself. If you can’t come on the open days, you can sign up for a guided

tour instead.”可知,开放日大多在秋季或者春季。虽然你对一门课有所了解, 但没有

人可以代替你去上课。所以在开放日,准大学生们能够直接体验课程。故选 D 项。

23.(2016 年,全国卷 II)

Reading can be a social activity.Think of the people who belong to book groups.They

choose books to read and then meet to discuss them.Now the website BookCrossing.com

turns the page on the traditional idea of a book group.

Members go on the site and register the books they own and would like to
share.BookCrossing provides an identification number to stick inside the book.Then the

person leaves it in a public place ,hoping that the book will have an adventure ,traveling far

and wide with each new reader who finds it.

Bruce Pederson , the managing director of BookCrossing , says , "The two things that

change your life are the people you meet and books you read.BookCrossing combines both."

Members leave books on park benches and buses , in train stations and coffee

shops.Whoever finds their book will go to the site and record where they found it.

People who find a book can also leave a journal entry describing what they thought of

it.E-mails are then sent to the BookCrossers to keep them updated about where their books

have been found.Bruce Peterson says the idea is for people not to be selfish by keeping a

book to gather dust on a shelf at home.

BookCrossing is part of a trend among people who want to get back to the"real"and not

the virtual ( 虚 拟 ) .The site now has more than one million members in more than one

hundred thirty-five countries.

94.Why does the author mention book groups in the first paragraph? ______

A.To explain what they are.

B.To introduce BookCrossing.

C.To stress the importance of reading.

D.To encourage readers to share their ideas.

95.What does the underlined word"it"in Paragraph 2 refer to? ______

A.The book. B.An adventure.

C.A public place. D.The identification number.

96.What will a BookCrosser do with a book after reading it? ______

A.Meet other readers to discuss it.

B.Pass it on to another reader.

C.Keep it safe in his bookcase.

D.Mail it back to its owner.

97.What is the best title for the text? ______

A.Online Reading:A Virtual Tour

B.Electronic Books:A new Trend

C.A Book Group Brings Tradition Back

D.A Website Links People through Books

【答案】94.B 95.A 96.B 97.D

学科网(北京)股份有限公司
【解析】这是一篇说明文。文章主要介绍了一个叫做 BookCrossing 的网站的功能和设

置该网站的目的。
94.推理判断题。根据 "Now the website BookCrossing.com turns the page on the traditional

idea of a book group."(现在 website BookCrossing.com 为书群的想法翻开了新的篇章)

可知作者在第一段提到书群是为了介绍图书漂流这个网站。故选 B 项。

95.词义猜测题。根据"Then the person leaves it in a public place ,hoping that the book will

have an adventure,traveling far and wide with each new reader who finds it."(然后这个人

把书放在公共场所,希望这本书可以有一个奇旅,因为每一个发现它的阅读者可以走

的更远更宽)根据句意可知,it 是指前半句提到的同一事物"the book"。故选 A 项。

96.推理判断题。根据“ Members leave books on park benches and buses ,in train stations

and coffee shops.Whoever finds their book will go to the site and record where they found

it.”(成员可以把书留在公园长凳上,公交车上,火车站和咖啡厅。发现书的人就去网

站登记他们在哪找到它的)可知,他们把书放在那些地方的目的,就是让其他人能够

找到并且阅读。选项中和该目的相近的是 B 项:把书传递给另一个人。故选 B 项。

97.主旨大意题。这是一篇说明文,目的主要是介绍网站,所以答案应该以网站为中心。

再 根 据 "The two things that change your life are the people you meet and books you

read.BookCrossing combines both"可知该网站可以把人和书联系在一起。故选 D 项。

24.(2017 年,全国卷 I)

Some of the world’s most famous musicians recently gathered in Paris and New Orleans to

celebrate the first annual International Jazz Day. UNESCO( United Nations Educational,

Scientific and Cultural Organization) recently set April 30 as a day to raise awareness of jazz

music, its significance, and its potential as a unifying(联合) voice across cultures.

Despite the celebrations, though, in the U.S. the jazz audience continues to shrink and

grow older, and the music has failed to connect with younger generations.

It’s Jason Moran’s job to help change that. As the Kennedy Center’s artistic adviser for

jazz, Moran hopes to widen the audience for jazz, make the music more accessible, and

preserve its history and culture.

“Jazz seems like it’s not really a part of the American appetite,” Moran tells National

Public Radio’s reporter Neal Conan. “What I’m hoping to accomplish is that my generation

and younger start to reconsider and understand that jazz is not black and write anymore. It’s

actually color, and it’s actually digital.”

Moran says one of the problems with jazz today is that the entertainment aspect of the
music has been lost. “The music can’t be presented today the way it was in 1908 or 1958. It

has to continue to move, because the way the world works is not the same,” says Moran.

Last year, Moran worked on a project that arranged Fats Waller’s music for a dance

party, “Just to kind of put it back in the mind that Waller is dance music as much as it is

concert music,” says Moran. “For me, it’s the recontextualization. In music, where does the

emotion( 情 感 ) lie? Are we, as humans, gaining any insight( 感 悟 ) on how to talk about

ourselves and how something as abstract as a Charlie Parker record gets us into a dialogue

about our emotions and our thoughts? Sometimes we lose sight that the music has a wider

context,” says Moran, “so I want to continue those dialogues. Those are the things I want to

foster.”

98.Why did UNESCO set April 30 as International Jazz Day?

A.To remember the birth of jazz.

B.To protect cultural diversity.

C.To encourage people to study music.

D.To recognize the value of jazz.

99.What does the underlined word “that” in paragraph 3 refer to?

A.Jazz becoming more accessible.

B.The production of jazz growing faster.

C.Jazz being less popular with the young.

D.The jazz audience becoming larger.

100.What can we infer about Moran’s opinion on jazz?

A.It will disappear gradually.

B.It remains black and white.

C.It should keep up with the times.

D.It changes every 50 years.

101.Which of the following can be the best title for the text?

A.Exploring the Future of Jazz.

B.The Rise and Fall of Jazz.

C.The Story of a Jazz Musician.

D.Celebrating the Jazz Day.

【答案】98.D 99.C 100.C 101.A

【解析】本文是一篇说明文。主要讲述 UNESCO 把 4 月 30 号定为国际爵士乐日,希

望意识到爵士乐的价值,但实际收效甚微。

学科网(北京)股份有限公司
98. 细节理解题。根据第一段中的 UNESCO(United Nations Educational, Scientific and

Cultural Organization) recently set April 30 as a day to raise awareness of jazz music, its

significance, and its potential as a unifying( 联合 ) voice across cultures. 可知,UNESCO 把

4 月 30 号定为国际爵士乐日是为了让人们重视爵士乐,意识到它的重要性以及它作为

连接各文化的纽带之声的潜在功能,也就是为了让人们意识到爵士乐的价值。故选

D。

99. 代词指代题。根据前文 Despite the celebrations, though, in the U.S. the jazz audience

continues to shrink and grow older, and the music has failed to connect with younger

generations 可知,尽管 UNESCO 为爵士乐设了纪念日,但美国的爵士乐听众依然在减

少,并且年龄在老化,爵士乐没能将年轻一代人连接起来。再结合 It’s Jason Moran’s

job to help change that( 是 Jason Moran 的工作是帮助改变那一情况)可推测,that 指代的

是前文中爵士乐在年轻一代人中失去吸引力的现象。故选 C。

100. 细节理解题。根据第五段中的 The music can’t be presented today the way it was in

1908 or 1958. It has to continue to move, because the way the world works is not the same

可知,Moran 认为现在爵士乐不能 以 1908 或 1958 年的方式来呈现,因为世界已经不

同了,所以爵士乐必须不断进步,说明随着时代的 发展,爵士乐也要跟上时代才不会

被年轻一代所抛弃。故选 C。

101.标题选择题。通读全文可知本文主要讲 UNESCO 为提高人们对爵士乐的重视而设

立爵士日, 但实际收效甚微。有人认为爵士乐应随着时代的进步而进步,否则将失去

对人们的吸引力,因此本文主要是探索爵士乐的未来,故选 A。

25.(2012 年,安徽卷)

Welcome to your future life!

You get up in the morning and look into the mirror. Your face is firm and young-looking.

In 2035, medical technology is better than ever. Many people at your age could live to be 150,

so at 40, you’re not old at all. And your parents just had an anti-aging( 抗衰老的 ) treatment.

Now, all three of you look the same age!

You say to your shirt, “Turn red.” It changes from blue to red. In 2035, “smart clothes”

contain particles( 粒子 ) much smaller than the cells( 细胞 ) in your body. The particles can be

programmed to change your clothes’ color or pattern(样式).

You walk into the kitchen. You pick up the milk, but a voice says,“ You shouldn’t drink

that!” Your fridge has read the chip ( 芯片 ) that contains information about the milk , and it

knows the milk is old . In 2035, every article of food in the grocery store has such a chip.
It’s time to go to work. In 2035, cars drive themselves. Just tell your “smart car” where

to go. On the way, you can call a friend using your jacket sleeve. Such “smart technology” is

all around you.

So will all these things come true? “For new technology to succeed,” says scientist

Andrew Zolli ,“it has to be so much better that it replaces what we have already.” The

Internet is one example—what will be the next?

102.We can learn from the text that in the future__________.

A.people will never get old

B.everyone will look the same

C.red will be the most popular color

D.clothes will be able to change their pattern

103.What can be inferred from Paragraph 4?

A.Food in the grocery store will carry electronic information.

B.More drinks will be given for sale.

C.Milk in the grocery store will stay fresh much longer.

D.Milk will be harmful to health.

104.Which of the following is mentioned in the text?

A.Cars will be able to drive automatically(自动地).

B.Fridges will know what people need.

C.Jacket sleeves can be used as a guide.

D.Nothing can replace the Internet.

105.What is the text mainly about?

A.Food and clothing in 2035

B.The reason for the success of new technology.

C.Medical treatments of the future.

D.Future technology in everyday life.

【答案】102.D 103.A 104.A 105.D

【解析】本文是说明文。讲述未来随着科学技术的不断发展,和日常生活相关的医疗 ,

衣服材料,食物,汽车等也会发生很大的变化。
102. 推理判断题。根据文章的第二段 The particles can be programmed to change your

clothes’ color or pattern. 进行推断未来的服装能够改变自己的式样。 A 和 B 都与第一段

的内容相关,但不是推断结果;C 与文章内容无关。故 D 正确。

103. 推理判断题。根据本文的第四段的最后一句话 In 2035, every article of food in the

学科网(北京)股份有限公司
grocery store has such a chip. 进行推断出店里的食物里都有这样提醒人们食品新鲜程度

的芯片。故 A 正确。

104. 推理判断题。根据文章的第五段第二句 In 2035, cars drive themselves. Just tell your

“smart car” where to go. 可知在 2035 年,汽车都可以自动驾驶,不再需要司机。故 A 正

确。
105.主旨大意题。本文讲述的是新的科学技术广泛运用于人类生活带来的人们在的衣

食住行等各方面的巨大变化。故 D 正确。

26.(2016 年,安徽卷)

A scientist working at her lab bench and a six-old baby playing with his food might seem

to have little in common.After all,the scientist is engaged in serious research to uncover the

very nature of the physical world,and the baby is,well, just playing…right?Perhaps,but some

developmental psychologists have argued that this “play” is more like a scientific

investigation than one might think.

Take a closer look at the baby playing at the table. Each time the bowl of rice is pushed

over the table edge, it falls in the ground—and, in the process, it belongs out important

evidence about how physical objects interact; bowls of rice do not flood in mid-sit, but

require support to remain stable. It is likely that babies are not born knowing the basic fact of

the universe; nor are they ever clearly taught it. Instead, babies may form an understanding of

object support through repeated experiments and then build on this knowledge to learn even

more about how objects interact. Though their ranges and tools differ, the baby’s investigation

and the scientist’s experiment appear to share the same aim(to learn about the natural world),

overall approach (gathering direct evidence from the world), and logic (are my observations

what I expected?).

Some psychologists suggest that young children learn about more than just the physical

world in this way—that they investigate human psychology and the rules of language using

similar means. For example, it may only be through repeated experiments, evidence

gathering, and finally overturning a theory, that a baby will come to accept the idea that other

people can have different views and desires from what he or she has. For example, unlike the

child, Mommy actually doesn’t like Dove chocolate.

Viewing childhood development as a scientific investigation throws on how children

learn, but it also offers an inspiring look at science and scientists. Why do young children and

scientists seem to be so much alike? Psychologists have suggested that science as an effort—
the desire to explore, explain, and understand our world—is simply something that comes

from our babyhood. Perhaps evolution provided human babies with curiosity and a natural

drive to explain their worlds, and adult scientists simply make use of the same drive that

served them as children. The same cognitive systems that make young children feel good

about figuring something out may have been adopted by adult scientists. As some

psychologists put it, “It is not that children are little scientists but that scientists are big

children.”

106.According to some developmental psychologists, ________.

A.a baby’s play is nothing more than a game.

B.scientific research into babies; games is possible

C.the nature of babies’ play has been thoroughly investigated

D.a baby’s play is somehow similar to a scientist’s experiment

107.We learn from Paragraph 2 that ________.

A.scientists and babies seem to observe the world differently

B.scientists and babies often interact with each other

C.babies are born with the knowledge of object support

D.babies seem to collect evidence just as scientists do

108.Children may learn the rules of language by ________.

A.exploring the physical world

B.investigating human psychology

C.repeating their own experiments

D.observing their parents’ behaviors

109.What is the main idea of the last paragraph?

A.The world may be more clearly explained through children’s play.

B.Studying babies’ play may lead to a better understanding of science.

C.Children may have greater ability to figure out things than scientists.

D.One’s drive for scientific research may become stronger as he grows.

110.What is the author’s tone when he discusses the connection between scientists’ research

and babies’ play?

A.Convincing. B.Confused.

C.Confidence. D.Cautious.

【答案】106.D 107.D 108.C 109.B 110.D

【解析】本文是一篇说明文。文章主要介绍了孩子的玩耍和科学家的研究有共同之处。

学科网(北京)股份有限公司
106. 细节理解题。根据文章第一段的 but some developmental psychologists( 心理学家 )

have argued that this “play” is more like a scientific investigation than one might think 可知,

心理学家觉得孩子的玩耍和科学家的研究有些相似,故选 D。

107. 推理判断题。根据文章第二段的 overall approach (gathering direct evidence from the

world)可知,孩子也会像科学家一样收集证据,故选 D。

108. 细 节 理 解 题 。 根 据 文 章 第 三 段 的 “ For example, it may only be through repeated

experiments, evidence gathering, and finally overturning a theory” 可知,孩子可以通过重复

自己的实验来学习语言的规律,故选 C。

109. 主 旨 大 意 题 。 根 据 最 后 一 段 的 “ Viewing childhood development as a scientific

investigation throws light on how children learn, but it also offers an inspiring look at science

and scientists.”可知,研究孩子可以让人们更好地了解科学,故选 B。

110. 推理判断题。根据作者在文中的用词 appear to 、用 may 来弱化绝对性的表达及通

过引用其他人的观点 some psychologist suggest that 可知,作者的语气是谨慎的。故选

D。

27.(2016 年,全国卷 I)

The meaning of silence varies among cultural groups.Silences may be thoughtful, or

they may be empty when a person has nothing to say. A silence in a conversation may also

show stubbornness, uneasiness, or worry. Silence may be viewed by some cultural groups as

extremely uncomfortable; therefore attempts may be made to fill every gap( 间 隙 ) with

conversation. Persons in other cultural groups value silence and view it as necessary for

understanding a person's needs.

Many Native Americans value silence and feel it is a basic part of communicating

among people, just as some traditional Chinese and Thai persons do. Therefore, when a

person from one of these cultures is speaking and suddenly stops, what may be implied( 暗示)

is that the person wants the listener to consider what has been said before continuing.In these

cultures, silence is a call for reflection.

Other cultures may use silence in other ways, particularly when dealing with conflicts

among people or in relationships of people with different amounts of power. For example,

Russian, French, and Spanish persons may use silence to show agreement between parties

about the topic under discussion. However, Mexicans may use silence when instructions are

given by a person in authority rather than be rude to that person by arguing with him or her.

In still another use, persons in Asian cultures may view silence as a sign of respect,
particularly to an elder or a person in authority.

Nurses and other care-givers need to be aware of the possible meanings of silence when

they come across the personal anxiety their patients may be experiencing. Nurses should

recognize their own personal and cultural construction of silence so that a patient’s silence is

not interrupted too early or allowed to go on unnecessarily. A nurse who understands the

healing(治愈) value of silence can use this understanding to assist in the care of patients from

their own and from other cultures.

111.What does the author say about silence in conversations?

A.It implies anger.

B.It promotes friendship.

C.It is culture-specific.

D.It is content-based.

112.Which of the following people might regard silence as a call for careful thought?

A.The Chinese.

B.The French.

C.The Mexicans.

D.The Russians.

113.What does the author advise nurses to do about silence?

A.Let it continue as the patient pleases.

B.Break it while treating patients.

C.Evaluate its harm to patients.

D.Make use of its healing effects.

114.What may be the best title for the text?

A.Sound and Silence

B.What It Means to Be Silent

C.Silence to Native Americans

D.Speech Is Silver; Silence Is Gold

【答案】111.C 112.A 113.D 114.B

【解析】本文是一篇说明文。主要介绍了沉默在不同文化背景下的不同内涵。
111. 细 节 理 解 题 。 根 据 第 一 段 The meaning of silence varies among cultural groups. A

silence in a conversation may also show stubbornness, uneasiness, or worry. 可知,沉默的

含义因文化群体而异,交谈中的沉默也可能表示固执、不安或担忧。所以作者认为谈

话中的沉默是有文化特性的。故选 C 项。

学科网(北京)股份有限公司
112.细节理解题。根据第二段中 Many Native Americans value silence and feel it is a basic

part of communicating among people, just as some traditional Chinese and Thai persons do.

Therefore, when a person from one of these cultures is speaking and suddenly stops, what

may be implied( 暗 示 ) is that the person wants the listener to consider what has been said

before continuing 可知,许多印第安人珍视沉默,认为这是人与人之间交流的基本组成

部分,就像一些传统的中国人和泰国人一样。因此,当一个来自其中一种文化的人说

话时突然停下来,暗示说话者想要倾听者在继续之前思考一下所说过的话。所以中国

人认为谈话时人们沉默是为了让别人思考。故选 A 项。

113. 细节理解题。根据末段末句 A nurse who understands the healing value of silence can

use this understanding to assist in the care of patients 可知,了解沉默的治疗价值的护士可

以利用这种了解来帮助护理病人。所以作者建议护士要用沉默来治疗病人。故选 D 。

114.主旨大意题。根据第一段“ The meaning of silence varies among cultural groups”可知,

沉默的含义因文化群体而异。以及先问介绍了在不同文化背景下谈话期间的沉默的不

同含义。所以短文的最佳标题是“沉默意味着什么”。故选 B 。

28.(2017 年,北京卷)

Hollywood’s theory that machines with evil( 邪 恶 ) minds will drive armies of killer

robots is just silly. The real problem relates to the possibility that artificial intelligence(AI)

may become extremely good at achieving something other than what we really want. In 1960

a well-known mathematician Norbert Wiener, who founded the field of cybernetics ( 控 制

论) , put it this way: “If we use, to achieve our purposes, a mechanical agency with whose

operation we cannot effectively interfere( 干预 ), we had better be quite sure that the purpose

put into the machine is the purpose which we really desire.”

A machine with a specific purpose has another quality, one that we usually associate

with living things: a wish to preserve its own existence. For the machine, this quality is not

in-born, nor is it something introduced by humans; it is a logical consequence of the simple

fact that the machine cannot achieve its original purpose if it is dead. So if we send out a

robot with the single instruction of fetching coffee, it will have a strong desire to secure

success by disabling its own off switch or even killing anyone who might interfere with its

task. If we are not careful, then, we could face a kind of global chess match against very

determined, super intelligent machines whose objectives conflict with our own, with the real

world as the chessboard.

The possibility of entering into and losing such a match should concentrate the minds of
computer scientists. Some researchers argue that we can seal the machines inside a kind of

firewall, using them to answer difficult questions but never allowing them to affect the real

world. Unfortunately, that plan seems unlikely to work: we have yet to invent a firewall that

is secure against ordinary humans, let alone super intelligent machines.

Solving the safety problem well enough to move forward in AI seems to be possible but

not easy. There are probably decades in which to plan for the arrival of super intelligent

machines. But the problem should not be dismissed out of hand, as it has been by some AI

researchers. Some argue that humans and machines can coexist as long as they work in teams

—yet that is not possible unless machines share the goals of humans. Others say we can just

“switch them off” as if super intelligent machines are too stupid to think of that possibility.

Still others think that super intelligent AI will never happen. On September 11, 1933, famous

physicist Ernest Rutherford stated, with confidence, “Anyone who expects a source of power

in the transformation of these atoms is talking moonshine.” However, on September 12, 1933,

physicist Leo Szilard invented the neutron-induced(中子诱导) nuclear chain reaction.

115.Paragraph 1 mainly tells us that artificial intelligence may .

A.run out of human control

B.satisfy human’s real desires

C.command armies of killer robots

D.work faster than a mathematician

116.Machines with specific purposes are associated with living things partly because they

might be able to .

A.prevent themselves from being destroyed

B.achieve their original goals independently

C.do anything successfully with given orders

D.beat humans in international chess matches

117.According to some researchers, we can use firewalls to .

A.help super intelligent machines work better

B.be secure against evil human beings

C.keep machines from being harmed

D.avoid robots’ affecting the world

118.What does the author think of the safety problem of super intelligent machines?

A.It will disappear with the development of AI.

B.It will get worse with human interference.

学科网(北京)股份有限公司
C.It will be solved but with difficulty.

D.It will stay for a decade.

【答案】115.A 116.A 117.D 118.C

【解析】这是一篇说明文。本文主要介绍了人工智能,随着科技的发展,机器人可能

会超出人类的控制,所以短文对人与机器人之间的关系进行了探讨。
115. 主 旨 大 意 题 。 根 据 第 一 段 The real problem relates to the possibility that artificial

intelligence(AI) may become extremely good at achieving something other than what we

really want.可知人工智能可能会出现的真正问题是 AI 会非常擅长取得某个成就而不是

我们真正想要的东西,也就是说 AI 可能会超出人的控制去做一些事情,故选 A。

116. 细节理解题。根据第二段 A machine with a specific purpose has another quality, one

that we usually associate with living things: a wish to preserve its own existence. 可知,一台

具有特定用途的机器还有另外一个特性,这个特性我们通常与生物联系在一起 : 一种希

望保持自身存在的愿望,也就是说,它具有有生命的物体的一种品质是因为它要保护

自己不被破坏,继续存在下去,故选 A。

117. 细节理解题。根据第三段 using them to answer difficult questions but never allowing

them to affect the real world.可知可以用防火墙来回答那些困难的问题但是却永远不要让

他们影响这个真正的现实世界,故选 D。

118. 推理判断题。根据最后一段 Solving the safety problem well enough to move forward

in AI seems to be possible but not easy. 可知作者认为解决人工智能机器的安全问题是可

能的,但是并不容易,也就是很困难,故选 C。

29.(2017 年,天津卷)

I read somewhere that we spend a full third of our lives waiting. But where are we doing

all of this waiting, and what does it mean to an impatient society like ours? To understand the

issue, let’s take a look at three types of “waits”.

The very purest form of waiting is the Watched-Pot Wait. It is without doubt the most

annoying of all. Take filling up the kitchen sink( 洗碗池 ) as an example. There is absolutely

nothing you can do while this is going on but keep both eyes fixed on the sink until it’s full.

During these waits, the brain slips away from the body and wanders about until the water runs

over the edge of the counter and onto your socks. This kind of wait makes the waiter helpless

and mindless.

A cousin to the Watched-Pot Wait is the Forced Wait. This one requires a bit of

discipline. Properly preparing packaged noodle soup required a Forced Wait. Directions are

very specific. “Bring three cups of water to boil, add mix, simmer three minutes, remove
from heat, let stand five minutes.”I have my doubts that anyone has actually followed the

procedures strictly. After all, Forced Waiting requires patience.

Perhaps the most powerful type of waiting is the Lucky-Break Wait. This type of wait is

unusual in that it is for the most part voluntary. Unlike the Forced Wait, which is also

voluntary, waiting for your lucky break does not necessarily mean that it will happen.

Turning one’s life into a waiting game requires faith and hope, and is strictly for the

optimists among us. On the surface it seems as ridiculous as following the directions on soup

mixes, but the Lucky-Break Wait well serves those who are willing to do it. As long as one

doesn’t come to rely on it, wishing for a few good things to happen never hurts anybody.

We certainly do spend a good deal of our time waiting. The next time you’re standing at

the sink waiting for it to fill while cooking noodle soup that you’ll have to eat until a large

bag of cash falls out of the sky, don’t be desperate. You’re probably just as busy as the next

guy.

119.While doing a Watched-Pot Wait, we tend to ___________.

A.keep ourselves busy

B.get absent-minded

C.grow anxious

D.stay focused

120.What is the difference between the Forced Wait and the Watched-Pot Wait?\

A.The Forced Wait requires some self-control.

B.The Forced Wait makes people passive.

C.The Watched-Pot Wait needs directions.

D.The Watched-Pot Wait engages body and brain.

121.What can we learn about the Lucky-Break Wait?

A.It is less voluntary than the Forced Wait.

B.It doesn’t always bring the desired result.

C.It is more fruitful than the Forced Wait.

D.It doesn’t give people faith and hope.

122.What does the author advise us to do the next time we are waiting?

A.Take it seriously.

B.Don’t rely on others.

C.Do something else.

D.Don’t lose heart.

学科网(北京)股份有限公司
123.The author supports his view by _________.

A.exploring various causes of “waits”.

B.describing detailed processes of “waits”.

C.analyzing different categories of “waits”

D.revealing frustrating consequences of “waits”

【答案】119.B 120.A 121.B 122.D 123.C

【解析】本文属于说明文阅读,作者通过这篇文章主要向我们描述了等待分为三种,

我们不应该在等待时失去信心
119. 细节理解题。根据第二段的句子 During these waits, the brain slips away from the

body and wanders about until the water runs over the edge of the counter and onto your

socks. 可知,在等待的过程中,大脑从身体中溜走,四处游荡,直到水从柜台边缘流

到你的袜子上。所以我们在等待一壶水煮开的时候我们总是心不在焉的。故选 B。

120. 细节理解题。根据第三段 After all , Forced Waiting requires patience 可知强迫等待

需要耐心。所以强迫等待需与被监视的等待的区别是强迫等待需要自控。故选 A。

121. 细节理解题。根据第四段的句子 waiting for your lucky break does not necessarily

mean that it will happen. 可知等待你的好运不一定意味着就会发生,即 e Lucky-Break

Wait 不一定带来渴望的结果 。故选 B。

122.细节理解题。根据第最后一段中 The next time you’re standing at the sink waiting for

it to fill while cooking noodle soup that you’ll have to eat until a large bag of cash falls out of

the sky, don’t be desperate. 可知,下次站在水槽边煮面时,一定不要灰心。所以作者建

议我们下一次等待的时候,我们不要绝望,不要灰心。故选 D。

123. 推 理 判 断 题 。 根 据 第 一 段 To understand the issue , let's take a look at three types

of“waits”.可知,为了理解这个问题,让我们看看三种类型的“等待”。所以判断出作

者是通过分析不同种类的等待来支持自己的观点的。故选 C。

30.(2017 年,全国卷 I)

A build-it-yourself solar still(蒸馏器) is one of the best ways to obtain drinking water in

areas where the liquid is not readily available. Developed by two doctors in the U.S.

Department of Agriculture, it’s an excellent water collector. Unfortunately, you must carry the

necessary equipment with you, since it’s all but impossible to find natural substitutes. The

only components required, though, are a 5 '×5' sheet of clear or slightly milky plastic, six

feet of plastic tube, and a container— perhaps just a drinking cup — to catch the water. These

pieces can be folded into a neat little pack and fastened on your belt.

To construct a working still, use a sharp stick or rock to dig a hole four feet across and
three feet deep. Try to make the hole in a damp area to increase the water catcher’s

productivity. Place your cup in the deepest part of the hole. Then lay the tube in place so that

one end rests all the way in the cup and the rest of the line runs up — and out — the side of

the hole.

Next, cover the hole with the plastic sheet, securing the edges of the plastic with dirt and

weighting the sheet’s center down with a rock. The plastic should now form a cone( 圆锥体 )

with 45-degree-angled sides. The low point of the sheet must be centered directly over, and

no more than three inches above, the cup.

The solar still works by creating a greenhouse under the plastic. Ground water

evaporates ( 蒸 发 ) and collects on the sheet until small drops of water form, run down the

material and fall off into the cup. When the container is full, you can suck the refreshment

zxxk out through the tube, and won’t have to break down the still every time you need a

drink.

124.What do we know about the solar still equipment from the first paragraph?

A.It’s delicate. B.It’s expensive.

C.It’s complex. D.It’s portable.

125.What does the underlined phrase “the water catcher” in paragraph 2 refer to?

A.The tube. B.The still.

C.The hole. D.The cup.

126.What’s the last step of constructing a working solar still?

A.Dig a hole of a certain size. B.Put the cup in place.

C.Weight the sheet’s center down. D.Cover the hole with the plastic sheet.

127.When a solar still works, drops of water come into the cup from ________.

A.the plastic tube B.outside the hole

C.the open air D.beneath the sheet

【答案】124.D 125.B 126.C 127.D

【解析】本文是一篇科普文章。文章主要介绍了一种自己可以亲手制作的简单易行的

太阳能蒸馏器的方法和它的工作原理。这种蒸馏器所需的材料简单,适用于任何缺水

的地方。
124.推理判断题。根据文章第一段最后一句话 These pieces can be folded into a neat little

pack and fastened on your belt.可知,制作蒸馏器的东西可以叠放在一个小包,系在腰间。

说明制作蒸馏器的设备很轻便,portable 表示“轻便的;手提的”,故选 D。

学科网(北京)股份有限公司
125. 猜 测 词 义 题 。 根 据 文 章 Try to make the hole in a damp area to increase the water

catcher’s productivity 可知,最好在潮湿的地方挖洞,以提高接水器的工作效率。接水

器指的是在潮湿地方挖洞,在洞的底部放一个杯子,杯子上方用覆盖膜遮挡。整个装

置构成接水器,而不是某个部分。这一题很容易错选 D。就算在干燥的地方挖洞,杯

子也可以接水,杯子接水的事实并不会因为周围的环境变化而变化,杯子的工作效率

是不变的。故选 B。

126.细节理解题。根据文章第三段中的 Next, cover the hole with the plastic sheet, securing

the edges of the plastic with dirt and weighting the sheet’s center down with a rock. 可知,最

后一步是 weighting the sheet’s center down with a rock“ 放一块石头在覆盖膜的中间,把

它压下去”,故选 C。

127. 推理判断题。根据第四段中的 Ground water evaporates ( 蒸发 ) and collects on the

sheet until small drops of water form, run down the material and fall off into the cup 可知,

地下水蒸发,在覆盖膜上聚集起来直到形成小水滴落在杯子里。覆盖膜是在杯子上面

的,水滴落入杯子里。所以水滴是凝聚在覆盖膜的下面。故选 D。

31.(2015 年,福建卷)

Food festivals around the world

Stilton Cheese Rolling

May Day is a traditional day for celebrations, but the 2,000 English villagers of Stilton

must be the only people in the world who include cheese rolling in their annual plans. Teams

of four, dressed in a variety of strange and funny clothes, roll a complete cheese along a 50-

metre course. On the way, they must not kick or throw their cheese, or go into their

competitors' lane ( 赛 道 ) . Competition is fierce and the chief prize is a complete Stilton

cheese weighing about four kilos (disappointingly, but understandably the cheeses used in the

race are wooden ones). All the competitors arc served with beer or port wine, the traditional

accompaniment for Stilton cheese.

Fiery Foods Festival—The Hottest Festival on Earth

Every year more than 10,000 people head for the city of Albuquerque, New Mexico.

They come from as far away as Australia, the Caribbean and China, but they all share a

common addiction—food that is not just spicy (辛辣) ,but hot enough to make your mouth

burn, your head spin and your eyes water. Their destination is the Fiery Food and BBQ

Festival which is held over a period of three clays every March. You might like to try a

chocolate-covered habanero pepper—officially the hottest pepper in the world—or any one of
the thousands of products that are on show. But one thing's for sure—if you don't like the

feeling of a burning tongue, this festival isn't for you!

La Tomatina—The World's Biggest Food Fight

On the last Wednesday of every August, the Spanish town of Bunol hosts Ea Tomatina—

the world's largest food fight. A week-long celebration leads up to an exciting tomato battle as

the highlight of the week's events. The early morning sees the arrival of large trucks with

tomatoes—official fight-starters get things going by casting tomatoes at the crowd.

The battle lasts little more than half an hour, in which time around 50,000 kilograms of

tomatoes have been thrown at anyone or anything that moves, runs, or fights back. Then

everyone heads down to the river to make friends again—and for a much-needed wash!

128.In the Stilton cheese rolling competition, competitors on each team must .

A.wear various formal clothes

B.roll a wooden cheese in their own lane

C.kick or throw their cheese

D.use a real cheese weighing about four kilos

129.Where is the Fiery Food and BBQ Festival held?

A.In New Mexico.

B.In the Caribbean.

C.In Australia.

D.In China.

130.The celebration of La Tomatina lasts .

A.three days

B.seven days

C.less than three days

D.more than seven days

131.Which of the following is TRUE according to the passage?

A.The chief prize for the Stilton cheese rolling competition is beer or port wine.

B.More than 10,000 Chinese take pail in the Fiery Food and BBQ Festival.

C.Thousands of spicy foods are on show in the Fiery Food and BBQ Festival.

D.An exciting tomato battle takes place at the beginning of La Tomatina.

【答案】128.B 129.A 130.B 131.C

【解析】本文是一篇说明文,分别讲述了英国、新墨西哥以及巴西的几个关于食物的

传统节日。

学科网(北京)股份有限公司
128.细节理解题。根据短文第一段 Teams of four, dressed in a variety of strange and funny

clothes , roll a complete cheese along a 50-metre course. 四个队伍,穿着滑稽搞笑的衣服,

在 50 米的路程中卷一个完整的奶酪,以及后面 but understandably the cheeses used in the

race are wooden ones 这个奶酪是木头的,所以在斯蒂尔顿奶酪滚动比赛中,每个队的

参赛者必须在自己的跑道上滚动一个木制的奶酪。故选 B。

129.细节理解题。根据第二段第一句 Every year more than 10,000 people head for the city

of Albuquerque, New Mexico. 可以知道这是在新墨西哥发生的事情,从段落中第三句

Their destination is the Fiery Eood and BBQ Festival 可以知道人们到新墨西哥是为了参加

Fiery food festival.所以 Fiery Food and BBQ 在新墨西哥州举行。故选 A。

130.细节理解题。根据文章第三段第二句 A week-long celebration leads up to an exciting

tomato battle as the highlight of the week's events. 可知,为期一周的庆祝活动将引发一场

激动人心的番茄大战,这是本周活动的亮点。所以西红柿大战这个西班牙的节日的持

续时间是一周,故选 B。

131. 细节理解题。根据文章第二段 倒数第二个破折号后面的文 字 or any one of the

thousands of products that are on show. 可以知道你可以任意品尝展出的上千种的食物,

所以成千上万的辛辣食物在 Fiery Eood and BBQ 节上展出。故选 C。

32.(2015 年,全国卷 II)

Your house may have an effect on your figure. Experts say the way you design your

home could play a role in whether you pack on the pounds or keep them off. You can make

your environment work for you instead of against you. Here are some ways to turn your home

into part of your diet plan.

Open the curtains and turn up the lights. Dark environments are more likely to

encourage overeating, for people are often less selfconscious (难为情) when they’re in poorly

lit places—and so more likely to eat lots of food. If your home doesn’t have enough window

light, get more lamps and flood the place with brightness.

Mind the colors. Research suggests warm colors fuel our appetites. In one study, people

who ate meals in a blue room consumed 33 percent less than those in a yellow or red room.

Warm colors like yellow make food appear more appetizing, while cold colors make us feel

less hungry. So when it’s time to repaint, go blue.

Don’t forget the clock—or the radio. People who eat slowly tend to consume about 70

fewer calories ( 卡 路 里 ) per meal than those who rush through their meals. Begin keeping

track of the time, and try to make dinner last at least 30 minutes. And while you’re at it,
actually sit down to eat. If you need some help slowing down, turn on relaxing music. It

makes you less likely to rush through a meal.

Downsize the dishes. Big serving bowls and plates can easily make us fat. We eat about

22 percent more when using a 12inch plate instead of a 10inch plate. When we choose a large

spoon over a smaller one, total intake ( 摄入 ) jumps by 14 percent. And we’ll pour about 30

percent more liquid into a short, wide glass than a tall, skinny glass.

132.The text is especially helpful for those who care about ____________.

A.their home comforts B.their body shape

C.house buying D.healthy diets

133.A home environment in blue can help people ______________.

A.digest food better B.reduce food intake

C.burn more calories D.regain their appetites

134.What are people advised to do at mealtimes?

A.Eat quickly. B.Play fast music.

C.Use smaller spoons. D.Turn down the lights.

135.What can be a suitable title for the text?

A.Is Your House Making You Fat? B.Ways of Serving Dinner

C.Effects of SelfConsciousness D.Is Your Home Environment Relaxing?

【答案】132.B 133.B 134.C 135.A

【解析】想要减肥,除了少吃外,还和房间有关系。建议有 4 条:1.房间要明亮;2.用

冷色系会减少食欲;3.放一些舒缓的音乐就餐;4. 用小的碗碟。

132.B 推理判断题。根据全文第一段 Here are some ways to turn your home into part of

your diet plan.可知,这篇文章更可能是对那些在乎自己的体型的人有帮助。故选 B。

133.B 细节理解题。根据第三段第二句话 in one study,… 可知在蓝色的房间内人吃的相

对较少。故选 B。

134.C 推理判断题。最后一段介绍,要想少吃,要用小碗小勺,前面介绍说到,吃得快,

房间暗会增加饮食,放舒缓的音乐有助于减少饮食的速度和进食量。故选 C。

135.A 主旨大意题。根据开头的后文 Here are some ways to turn your home into part of

your diet plan. 可知,本文就房间的布置给那些想减肥的人士提了一些建议。所以选 A

可以更好地概括全文。
33.(2015 年,重庆卷)

In ancient Egypt, a shopkeeper discovered that he could attract customers to his shop

simply by making changes to its environment. Modern businesses have been following his

学科网(北京)股份有限公司
lead, with more tactics (策略).

One tactic involves where to display the goods. For example, stores place fruits and

vegetables in the first section. They know that customers who buy the healthy food first will

feel happy so that they will buy more junk food ( 垃圾食品 ) later in their trip. In department

stores, the women’s shoe section is generally next to the women’s cosmetics ( 化 妆 品 )

section: while the shop assistant is going back to find the right size shoe, bored customers are

likely to wander over and find some cosmetics they might want to try later.

Besides, businesses seek to appeal to customers’ senses. Stores notice that the smell of

baked goods encourages shopping, so they make their own bread each morning and then fan

the bread smell into the store throughout the day. Music sells goods, too. Researchers in

Britain found that when French music was played, sales of French wines went up.

When it comes to the selling of houses, businesses also use highly rewarding tactics.

They find that customers make decisions in the first few seconds upon walking in the door,

and turn it into a business opportunity. A California builder designed the structure of its

houses smartly. When entering the house, the customer would see the Pacific Ocean through

the windows, and then the pool through an open stairway leading to the lower level. The

instant view of water on both levels helped sell these $ 10 million houses.

136.Why do stores usually display fruits and vegetables in the first section?

A.To save customers time. B.To show they are high quality foods.

C.To help sell junk food. D.To sell them at discount prices.

137.According to Paragraph 3, which of the following encourages customers to buy?

A.Opening the store early in the morning. B.Displaying British wines next to French ones.

C.Inviting customers to play music. D.Filling the store with the smell of fresh bread.

138.What is the California builder’s story intended to prove?

A.The house structure is a key factor customers consider.

B.The more costly the house is, the better it sells.

C.An ocean view is much to the customers’ taste.

D.A good first impression increases sales.

139.What is the main purpose of the passage?

A.To explain how businesses turn people into their customers.

B.To introduce how businesses have grown from the past.

C.To report researches on customer behavior.

D.To show dishonest business practices.


【答案】136.C 137.D 138.D 139.A

【解析】这是一篇说明文。自从古埃及的一家店主发现了可以通过改变商场的环境吸

引顾客以来,现代社会中越来越多的商场采用了这种商业战略。即利用商店的布局,

物品的摆设,甚至食品的气味来刺激顾客对商品的购买。此外,房地产的销售部门也

在使用这一高回报的销售策略。
136.细节理解题。根据第二段 They know that customers who buy the healthy food first will

feel happy so that they will buy more junk food( 垃圾食品 ) later in their trip. 可知,他们知

道,首先购买健康食品的顾客会感到高兴,因此他们会在以后的旅行中购买更多的垃

圾食品。所以把水果放在显眼的位置是为了吸引消费者,然后在不知不觉中购买稍后

要看到的那些垃圾食品。故选 C。

137. 推理判断题。根据第三段内容 Stores notice that the smell of baked goods encourages

shopping 可知,商店注意到烘焙食品的味道鼓励购物。由此判断出烤面包的味道能够

起到刺激消费者购买的目的。故选 D。

138. 推理判断题。文章末段提出 They find that customers make decisions in the first few

seconds upon walking in the door, and turn it into a business opportunity. 可知,他们发现顾

客在进门的最初几秒钟就做出决定,这就是一个商机。然后讲述加利福尼亚卖房者的

故事,由此判断加州建筑商的故事想要证明良好的第一印象会增加销售额。故选 D。

139.推理判断题。自从古埃及的一家店主发现了可以通过改变商场的环境吸引顾客以

来,现代社会中越来越多的商场采用了这种商业战略。即利用商店的布局,物品的摆

设,甚至食品的气味来刺激顾客对商品的购买。此外,房地产的销售部门也在使用这

一高回报的销售策略。所以这篇短文的主要目的是解释企业如何把人变成顾客。故选

A 项。

34.(2014 年,广西卷)

Since the first Earth Day in 1970, Americans have gotten a lot “greener” toward the

environment. “We didn’t know at that time there even was an environment, let alone that

there was a problem with it,” says Bruce Anderson, president of Earth Day USA.

But what began as nothing important in public affairs has grown into a social

movement .Business people, political leaders, university professors, and especially millions

of grass-roots Americans are taking part in the movement. “The understanding has increased

many , many times,” says Gaylord Nelson, the former governor from Wisconsin, who

thought up the first. According to US government reports, emissions ( 排 放 )from cars and

trucks have dropped from 10.3 million tons a year to 5.5 tons .The number of cities producing

学科网(北京)股份有限公司
CO beyond the standard has been reduced from 40 to 9. Although serious problems still

remain and need to be dealt with, the world is a safer and healthier place. A kind of “Green

thinking” has become part of practices.

Great improvement has been achieved. In 1988 there were only 600 recycling programs;

today in 1995 there are about 6,600. Advanced lights, motors, and building designs have

helped save a lot of energy and therefore prevented pollution.

Twenty –five years ago, there were hardly any education programs for environment.

Today, it’s hard to find a public school, university, or law school that does not have such a

kind of program. “Until we do that, nothing else will change!” say Bruce Anderson.

140.According to Anderson, before 1970, Americans had little idea about ___.

A.the social movement B.recycling techniques

C.environmental problems D.the importance of Earth Day

141.Where does the support for environmental protection mainly come from?

A.The grass –roots level. B.The business circle.

C.Government officials. D.University professors.

142.What have Americans achieved in environmental protection?

A.They have cut car emissions to the lowest.

B.They have settled their environmental problems.

C.They have lowered their CO levels in forty cities.

D.They have reduced pollution through effective measures.

143.What is especially important for environmental protection according to the last

paragraph?

A.Education. B.Planning

C.Green living D.CO reduction

【答案】140.C 141.A 142.D 143.A

【解析】本文属于说明文。在文章中作者介绍了自从 1970 年第一个地球日成立以来,

美国的环境问题有了很大的改善。以前在公共事物中无足轻重的环境问题现在成了一

项社会关注的问题,从科学家到大学教授到贫民百姓都有了环保意识。此外作者还指

出教育在这种变化方面也起着非常重要的作用。
140.细节理解题。根据文章第一段第 2 句“We didn’t know at that time there even was an

environment, let alone that there was a problem with it,” says Bruce Anderson, president of

Earth Day USA. 可知在 1970 年世界地球日开始的时候,人们甚至都不知道有“环境这

个词”,更不要说知道存在着环境问题了,因此在此之前人们都没有意识到环境问题 。
故选 C。

141.推理判断题。根据文章第二段 Business people, political leaders, university professors,

and especially millions of grass-roots 可知商人,政界领导,大学教授,尤其是平民百姓

们都参与到了保护环境的运动中来,由此判断保护环境的支持主要来自草根阶层,故

选 A。

142. 推 理 判 断 题 。 根 据 文 章 倒 数 第 二 段 内 容 In 1988 there were only 600 recycling

programs …… and building designs have helped save a lot of energy and therefore prevented

pollution .可知回收工作的改进,先进产品和住房设计的研制开发等措施都有效地减少

了污染,改善了环境,因此推断美国人已经通过有效的方法减少了污染的排放。故选

D。

143.推理判断题。四个选项内容对于保护环境都是重要的,但是作者在文章末段提到

目前几乎每所学校都有关于环境保护的课程,而且通过最后 Bruce Anderson 说的话

“Until we do that , nothing else will change! ” 可知教育在改变人们保护环境意识,改善

环境问题方面至关重要,故选 A。

35.(2010 年,福建卷)

Businesses are witnessing a difficult time, which has in turn produced influence on

consumers, desire to go green. However, shoppers are still laying stress on environmental

concerns.

Two thirds of customers say that environmental considerations inform their purchases to

the same degree as they did a year ago, while more than a quarter say that they are now even

better aware of the environmental effect on what they buy.

This may help to influence how shops store goods on their shelves. And the companies

should still make efforts to become more environmentally friendly. Two out of three people

think it is important to buy from environmentally responsible companies, with about one in

seven saying that they had even decided to take their custom elsewhere if they felt a

company’s environmental reputation was not good enough.

Harry Morrison, chief executive ( 主管) of the Carbon Trust, sympathizes: “I understand

this situation where survival is very important now. But from environmental considerations,

the clock is ticking — we don’t have much time. In addition, cutting carbon has an immediate

effect as costs drop and a medium-term benefit for the brand.”

Larger companies have an extra motivation to look at reducing their carbon footprint, as

new rules next year will require businesses to buy carbon allowances to make up for their

学科网(北京)股份有限公司
emissions ( 排放 ). Those that have taken early action will have a head start. More than two

thirds of consumers are not clear about which companies are environmentally responsible.

This suggests that firms that are able to relay clearly their message to the public will be in a

pole position to attract shoppers.

The Carbon Trust believes that it can help by informing customers about the good work

companies are doing. “When companies are granted ( 授予 ) the standard, they can use a logo

( 标 识 ) in all their marketing which makes it clear that they are working towards cutting

emissions,” Mr. Morrison said.

144.What’s the main idea of the passage?

A.Businesses are finding ways to send their message to the shoppers.

B.Companies will soon get information about cutting carbon emissions.

C.Firms are making efforts to encourage customers to keep goods at home.

D.Firms are urged to cut carbon emissions by shoppers, environmental awareness.

145.The underlined word “inform” in Paragraph 2 probably means “ ”.

A.affect B.change

C.disturb D.reject

146.According to Harry Morrison, businesses .

A.will benefit from cutting carbon emissions

B.should buy carbon allowances for shoppers

C.are required to make up for their carbon emissions

D.have encouraged shoppers to take their custom elsewhere

147.We can learn from the passage that businesses will .

A.have a strong desire to reduce costs

B.use the same logo in their marketing

C.gain advantages by taking early action

D.attract more shoppers by storing goods

【答案】144.D 145.A 146.A 147.C

【解析】本文是一篇说明文。文章主要介绍了由于消费者环保意识的增强,各公司不

得不下大力气减少碳的排放量。
144. 主旨大意题。通读全文及第一段 Businesses are witnessing a difficult time, which has

in turn produced influence on consumers, desire to go green.( 企业正经历一个艰难的时期,

这反过来又对消费者产生了影响,促使他们渴望环保 ) 和第二段中的 Two out of three

people think it is important to buy from environmentally responsible companies, with about
one in seven saying that they had even decided to take their custom elsewhere if they felt a

company’s environmental reputation was not good enough.( 三分之二的人认为从对环境负

责的公司购买产品很重要,约七分之一的人表示,如果他们觉得某家公司的环境声誉

不够好,他们甚至决定到其他地方去购买 )可知,文章主要介绍了消费者和环保意识正

敦促企业减少碳排放。故选 D。

145.词义猜测题。根据该词所在句后面的内容 while more than a quarter say that they are

now even better aware of the environmental effect on what they buy.( 而超过四分之一的人

表示,他们现在甚至更加清楚环境对他们所购买产品的影响 ) 可知, Two thirds of

customers say that environmental considerations inform their purchases to the same degree as

they did a year ago 意为“三分之二的消费者表示,环境因素对他们购物的影响与一年

前相同”。由此可知,划线词与选项中的 effect 意思相同。故选 A。

146. 推 理 判 断 题 。 根 据 第 四 段 Harry Morrison 所 说 的 一 番 话 , 特 别 是 最 后 一 句 In

addition, cutting carbon has an immediate effect as costs drop and a medium-term benefit for

the brand.(此外,随着成本下降,降低碳排放会产生立竿见影的效果,对品牌来说也是

中期效益)可推断,企业将从减少碳排放中获益。故选 A

147. 细节理解题。根据第五段第二句 Those that have taken early action will have a head

start.(那些及早采取行动的企业将占得先机)可知,那些通过早期行动的企业将获得优势。

故选 C。

36.(2012 年,陕西卷)

Spring is coming, and it is time for those about to graduate to look for jobs. Competition

is tough, so job seekers must carefully consider their personal choices. Whatever we are

wearing, our family and friends may accept us, but the workplace may not.

A high school newspaper editor said it is unfair for companies to discourage visible

tattoos (纹身), nose rings, or certain dress styles. It is true you can’t judge a book by its cover,

yet people do “cover” themselves in order to convey certain messages. What we wear,

including tattoos and nose rings, is an expression of who we are. Just as people convey

messages about themselves with their appearances, so do companies. Dress standards exist in

the business world for a number of reasons, but the main concern is often about what

customers accept.

Others may say how to dress is a matter of personal freedom, but for businesses it is

more about whether to make or lose money. Most employers do care about the personal

appearances of their employees, because those people represent the companies to their

学科网(北京)股份有限公司
customers.

As a hiring manager I am paid to choose the people who would make the best

impression on our customers. There are plenty of well-qualified candidates, so it is not wrong

to reject someone who might disappoint my customers. Even though I am open-minded, I

can’t expect all our customers are.

There is nobody to blame but yourself if your set of choices does not match that of your

preferred employer. No company should have to change to satisfy a candidate simply because

he or she is unwilling to respect its standards, as long as its standards are legal.

148.Which of the following is the newspaper editor’s opinion according to Paragraph 2?

A.People’s appearances carry message about themselves.

B.Customers’ choices influence dress standards in companies.

C.Candidates with tattoos or nose rings should be fairly-treated.

D.Strange dress styles should not be encouraged in the workplace.

149.What can be inferred from the text?

A.Candidates have to wear what companies prefer for an interview.

B.What to wear is not a matter of personal choice for companies.

C.Companies sometimes have to change to respect their candidates.

D.Hiring managers make the best impression on their candidates.

150.Which of the following would be the best title for the text?

A.Employees Matter B.Personal Choices Matter

C.Appearances Matter D.Hiring Managers Matter

151.The author’s attitude towards strange dress styles in the workplace may best be described

as .

A.enthusiastic B.negative

C.positive D.sympathetic

【答案】148.C 149.B 150.C 151.B

【解析】又是一年毕业季,很多大学毕业生又要开始为寻找一份可以赖以生存的工作

而奔忙了,但是,影响就业成功的因素很多,其中有一点就是面试时的着装。
148. 推理判断题。根据第二自然段的 A high school newspaper editor said it is unfair for

companies to discourage visible tattoos ( 纹身) nose rings, or certain dress styles. It is true

you can’t judge a book by its cover, yet people do “cover” themselves in order to convey ( 传

递)certain messages.可知作者的意思是有纹身或戴鼻环去面试不会被公平的对待,故本

题选 C。
149.推理判断题。通读第三自然段可知本文作者的意思是:员工的穿着不再是员工的

私人问题,而是影响到公司的声誉和发展的大问题。选 B。

150.主旨大意题。本文讨论的主要是在求职的时候,人的外貌和穿着是很重要的,因

为公司都要选择一些在形象上能够代表公司的人,故本题选 C。

151.推理判断题。根据倒数第二段 There are plenty of well-qualified candidates, so it is not

wrong to reject someone who might disappoint my customers.Even though I am open-

minded, I can’t expect all our customers are 说明作者也认为那些穿着奇怪的人是不能被

选中的,说明他对这样的穿着是持否定的态度的,故选 B。

37.(2016 年,北京卷)

California Condor’s Shocking Recovery

California condors are North America’s largest birds, with wind-length of up to 3 meters.

In the 1980s, electrical lines and lead poisoning( 铅 中 毒 ) nearly drove them to dying out.

Now, electric shock training and medical treatment are helping to rescue these big birds.

In the late 1980s, the last few condors were taken from the wild to be bred( 繁殖). Since

1992, there have been multiple reintroductions to the wild, and there are now more than 150

flying over California and nearby Arizona, Utah and Baja in Mexico.

Electrical lines have been killing them off. “As they go in to rest for the night, they just

don’t see the power lines,” says Bruce Rideout of San Diego Zoo. Their wings can bridge the

gap between lines, resulting in electrocution(电死) if they touch two lines at once.

So scientists have come up with a shocking idea. Tall poles, placed in large training

areas, teach the birds to stay clear of electrical lines by giving them a painful but undeadly

electric shock. Before the training was introduced, 66% of set-freed birds died of

electrocution. This has now dropped to 18%.

Lead poisonous has proved more difficult to deal with. When condors eat dead bodies of

other animals containing lead, they absorb large quantities of lead. This affects their nervous

systems and ability to produce baby birds, and can lead to kidney( 肾 ) failures and death. So

condors with high levels of lead are sent to Los Angeles Zoo, where they are treated with

calcium EDTA, a chemical that removes lead from the blood over several days. This work is

学科网(北京)股份有限公司
starting to pay off. The annual death rate for adult condors has dropped from 38% in 2000 to

5.4% in 2011.

Rideout’s team thinks that the California condors’ average survival time in the wild is

now just under eight years. “Although these measures are not effective forever, they are vital

for now,” he says. “They are truly good birds that are worth every effort we put into

recovering them. ”

152.California condors attract researchers’ interest because they _________.

A.are active at night

B.had to be bred in the wild

C.are found only in California

D.almost died out in the 1980s

153.Researchers have found electrical lines are _________.

A.blocking condors’ journey home

B.big killers of California condors

C.rest places for condors at night

D.used to keep condors away

154.According to Paragraph 5, lead poisoning _________.

A.makes condors too nervous to fly

B.has little effect on condors’ kidneys

C.can hardly be gotten rid of from condors’ blood

D.makes it difficult for condors to produce baby birds

155.This passage shows that _________.

A.the average survival time of condors is satisfactory

B.Rideout’s research interest lies in electric engineering

C.the efforts to protect condors have brought good results

D.researchers have found the final answers to the problem

【答案】152.D 153.B 154.D 155.C

【解析】加州兀鹰是北美最大的鸟类,翼展可达 3 米。20 世纪 80 年代,由于触电和铅

中毒,加州兀鹰几乎绝迹。现在,正通过电击和药物治疗拯救加州兀鹰。
152. 推理判断题。根据第三段“ As they go in to rest for the night” 排除 A ;根据第二段

“ In the late 1980s, the last few condors were taken from the wild to be bred( 繁殖 )” 可知,

condors 一直生活在野外,排除 B;根据第二段“more than 150 flying over California and

nearby Arizona, Utah and Baja in Mexico” 可知,不止加利福尼亚州有 condors,排除 C,

根据第一段的 In the 1980s, electrical lines and lead poisoning( 铅中毒 ) nearly drove them
to dying out. 可推断,加州秃鹰吸引了研究人员的兴趣是因为他们在 20 世纪 80 年代几

乎灭绝。故选 D。

153. 细节理解题。根据第三段“ Electrical lines have been killing them off...Their wings

can bridge the gap between lines, resulting in electrocution( 电死 ) if they touch two lines at

once.”可知,兀鹰不知道停在电线上翅膀同时触到两根及两根以上电线会触电身亡,它

们经常停在电线上休息,结果,很多兀鹰被电死了,故选 B。

154. 推 理 判 断 题 。 根 据 “ This affects their nervous systems and ability to produce baby

birds, and can lead to kidney( 肾) failures and death” 可知,摄入大量铅会刺激兀鹰的神经

系统,影响它们的生育能力,导致肾衰竭和死亡,因此推断铅中毒让兀鹰很难繁殖小

鹰。故选 D。

155. 推理判断题。根据“ just under eight years” 可知,在野外生存的兀鹰的平均寿命太

短,排除 A;根据“Although these measures are not effective forever” 可知,目前采用的

措施并不能永远有效,排除 D ;根据第六段“ They are truly good birds that are worth

every effort we put into recovering them.” 可知,Rideout 团队的研究兴趣在于怎样挽救这

一濒危物种,排除 B。根据最后一句“ Although these measures are not effective forever,

they are vital for now,” he says. “They are truly good birds that are worth every effort we put

into recovering them. ” 可知,虽然这些措施不会永远有效,但就目前而言,它们是至关

重要的。 因此推断保护兀鹰故选 C。

38.(2016 年,天津卷)

When John was growing up, other kids felt sorry for him. His parents always had him

weeding the garden, carrying out the garbage and delivering newspapers. But when John

reached adulthood, he was better off than his childhood playmates. He had more job

satisfaction, a better marriage and was healthier. Most of all, he was happier. Far happier.

These are the findings of a 40-year study that followed the lives of 456 teenage boys

from Boston. The study showed that those who had worked as boys enjoyed happier and

more productive lives than those who had not. " Boys who worked in the home or

community gained competence( 能 力 ) and came to feel they were worthwhile members of

society," said George Vaillant, the psychologist( 心理学家) who made the discovery. "And

because they felt good about themselves, others felt good about them."

Vaillant’s study followed these males in great detail. Interviews were repeated at ages

25,31 and 47. Under Vaillant, the researchers compared the men’s mental-health scores with

their boyhood-activity scores. Points were awarded for part-time jobs, housework, effort in

学科网(北京)股份有限公司
school, and ability to deal with problems.

The link between what the men had done as boys and how they turned out as adults was

surprisingly sharp. Those who had done the most boyhood activities were twice as likely to

have warm relations with a wide variety of people, five times as likely to be well paid and 16

times less likely to have been unemployed. The researchers also found that IQ and family

social and economic class made no real difference in how the boys turned out.

Working — at any age — is important. Childhood activities help a child develop

responsibility, independence, confidence and competence — the underpinnings( 基 础 ) of

emotional health. They also help him understand that people must cooperate and work toward

common goals. The most competent adults are those who know how to do this. Yet work isn’t

everything. As Tolstoy once said, " One can live magnificently in this world if one knows

how to work and how to love, to work for the person one loves and to love one’s work. "

156.What do we know about John?

A.He enjoyed his career and marriage.

B.He had few childhood playmates.

C.He received little love from his family.

D.He was envied by others in his childhood.

157.Vaillant’s words in Paragraph 2 serve as    .

A.a description of personal values and social values

B.an analysis of how work was related to competence

C.an example for parents’ expectations of their children

D.an explanation why some boys grew into happy men

158.Vaillant’s team obtained their findings by    .

A.recording the boys’ effort in school

B.evaluating the men’s mental health

C.comparing different sets of scores

D.measuring the men’s problem solving ability

159.What does the underlined word "sharp" probably mean in Paragraph 4?

A.Quick to react. B.Having a thin edge.

C.Clear and definite. D.Sudden and rapid.

160.What can be inferred from the last paragraph?

A.Competent adults know more about love than work.

B.Emotional health is essential to a wonderful adult life.

C.Love brings more joy to people than work does.


D.Independence is the key to one’s success.

【答案】156.A 157.D 158.C 159.C 160.B

【解析】这是一篇说明文。文章通过 John 的例子结合一个研究发现说明童年时期帮助

做一些家务的孩子长大后会更快乐,更成功。
156. 细节理解题。根据第一段的句子 He had more job satisfaction, a better marriage and

was healthier. ,可知,他的工作满意度更高,婚姻更美满,身体也更健康。所以 John

对他的事业和婚姻很满意。故选 A。

157. 推理判断题。根据第二段的句子 Boys who worked in the home or community gained

competence and came to feel they were worthwhile members of society 可知,在家庭或社

区工作的男孩获得足以过温饱生活的收入,并开始觉得自己是有价值的社会成员。由

此判断出第二段 Vaillant 的话是解释为什么一些男孩成长为快乐的人。故选 D。

158. 细节理解题。根据第三段的句子 The researchers compared the men’s mental-health

scores with their boyhood-activity scores ,可知, Vaillant 的团队获得他们的发现是通过

对比不同的得分。故选 C。

159. 词义猜测题。根据第四段的 Those who had done the most boyhood activities were

twice as likely to have warm relations with a wide variety of people, five times as likely to be

well paid.可知,那些在男孩时期参加过最多活动的人,与各种各样的人保持亲密关系

的可能性是其他人的两倍,获得高薪的可能性是其他人的五倍。所以这些人在孩子的

时候做的事情和他们成人后是怎样的人之间的关联是确定),所以通过上下文的语境

可以判断出,这里 sharp 的意思是明确的,一定的。故选 C。

160. 推理判断题。根据最后一段的句子 One can live magnificently in this world if one

knows how to work and how too love, to work for the person one loves and to love one’s

work,可知,如果一个人懂得如何工作,懂得如何去爱,懂得为所爱的人工作,懂得

爱自己的工作,那么他就能在这个世界上活得很精彩。由此判断出情感健康对精彩的

成年人的生活至关重要。故选 B。

39.(2008 年,天津卷)

I love charity( 慈 善 ) shops and so do lots of other people in Britain because you find

quite a few of them on every high street. The charity shop is a British institution, selling

everything from clothes to electric goods, all at very good prices. You can get things you

won’t find in the shops anymore. The thing I like best about them is that your money is going

to a good cause and not into the pockets of profit-driven companies, and you are not

damaging the planet, but finding a new home for unwanted goods.

学科网(北京)股份有限公司
The first charity shop was opened in 1947 by Oxfam. The famous charity’s appeal to aid

postwar Greece had been so successful it had been flooded with donations( 捐 赠 物 ). They

decided to set up a shop to sell some of these donations to raise money for that appeal. Now

there are over 7,000 charity shops in the UK. My favourite charity shop in my hometown is

the Red Cross shop, where I always find children’s books, all 10 or 20 pence each.

Most of the people working in the charity shops are volunteers, although there is often a

manager who gets paid. Over 90% of the goods in the charity shops are donated by the

public. Every morning you see bags of unwanted items outside the front of shops, although

they don’t encourage this, rather ask people to bring things in when the shop is open.

The shops have very low running costs: all profits go to charity work. Charity shops

raise more than £ 110 million a year, funding( 资 助 ) medical research, overseas aid,

supporting sick and poor children, homeless and disabled people, and much more. What

better place to spend your money? You get something special for a very good price and a

good moral sense. You provide funds to a good cause and tread lightly on the environment.

161.The author loves the charity shop mainly because of .

A.its convenient location B.its great variety of goods

C.its spirit of goodwill D.its nice shopping environment

162.The first charity shop in the UK was set up to .

A.sell cheap products B.deal with unwanted things

C.raise money for patients D.help a foreign country

163.Which of the following is TRUE about charity shops?

A.The operating costs are very low. B.The staff are usually well paid.

C.90% of the donations are second-hand. D.They are open twenty-four hours a day.

164.Which of the following may be the best title for the passage?

A.What to Buy at Charity Shops.

B.Charity Shop: Its Origin & Development.

C.Charity Shop: Where You Buy to Donate.

D.The Public’s Concern about Charity Shops.

【答案】161.C 162.D 163.A 164.C

【解析】本文主要介绍了慈善商店的基本特色以及慈善商店的创建与发展。
161.推理判断题。由第一段第四句"The thing I like best about them is that your money is

going to a good cause”“我“最喜欢慈善商店的一点是你的钱用于有用的事业可推断出 C

项为正确答案。"goodwill”含义为“好意”
162. 事 实 细 节 题 。 由 第 二 段 第 二 、 三 两 句 "the famous charity’s appeal to aid postwar

Greece

had been so successful……”可知第一个慈善商店的建立是为了筹钱援助战后的希腊。而

从文章的第一段可知作者是英国人。因此,D 项正确。C 项中的"patients”未提。

163. 事实细节题。由第四段第一句 "The shops have very low running costs” 可知 A 项正

确。 "running” 相当于 "operating” 含义为“运营”由第三段第一句中 "volunteer (志愿

者)"可知 B 项不正确。由第三段第二句可知慈善商店中 90%以上的商品为公众捐赠,

但并不能推出是“二手货”,故 C 项不正确。D 项未提

164. 主 旨 大 意 题 。 本 文 的 最 后 一 句 为 主 题 句 。 由 "a good moral sense ( 好 的 道 德

感)" 和"You provide funds to a good cause (你为一项有意义的事业提供资金) " 可知 C

项为最好题目。 "Charity Shop: Where You Buy To Donate. (慈善商店 ----一个买东西就

相当于捐款的地方。)
40.(2007 年,浙江卷)

People who have lost the ability to understand or use words due to brain damage are

called aphasics ( 失语症患者 ). Such patients can be extremely good at something else. From

the changing expressions on speakers’ faces and the tones of their voices, they can tell lies

from truths.

Doctors studying the human brain have given a number of examples of this amazing

power of aphasics. Some have even compared this power to that of a dog with an ability to

find out the drugs hidden in the baggage.

Recently, scientists carried out tests to see if all that was said about aphasics was true.

THEY STUDIED A MIXED GROUP OF PEOPLE. Some were normal; others were

aphasics. It was proved that the aphasics were far ahead of the normal people in recognizing

false speeches—in most cases, the normal people were fooled by words, but the aphasics

were not.

Some years ago, Dr. Oliver Sacks wrote in his book about his experiences with aphasics.

He mentioned a particular case in a hospital. Some aphasics were watching the president

giving a speech on TV. Since the president had been an actor earlier, making a good speech

was no problem for him. He was trying to put his feelings into every word of his speech.

But his way of speaking had the opposite effect on the patients. They didn’t seem to

believe him. Instead, they burst into laughter. The aphasics knew that the president did not

mean a word of what he was saying. He was lying!

学科网(北京)股份有限公司
Many doctors see aphasics as people who are not completely normal because they lack

the ability to understand words. However, according to Dr. Sacks, they are more gifted than

normal people. Normal people may get carried away by words. Aphasics seem to understand

human expressions better, though they cannot understand words.

165.What is so surprising about aphasics?

A.They can fool other people.

B.They can tell whether people are lying.

C.They can understand language better.

D.They can find out the hidden drugs.

166.How did the scientists study aphasics?

A.By asking them to watch TV together.

B.By organizing them into acting groups.

C.By comparing them with normal people.

D.By giving them chances to speak on TV.

167.What do we learn from this text?

A.What one says reflects how one feels.

B.Aphasics have richer feelings than others.

C.Normal people often tell lies in their speeches.

D.People poor at one thing can be good at another.

【答案】165.B 166.C 167.D

【解析】本文通过对 aphasics(失语症患者)的研究,告诉我们一些在某些方面有缺失的

人在另外的方面会有很大的发展。
165.C 细节题。根据第一段后 3 行 Such patients can be extremely good at something else.

From the changing expressions on speakers' faces and the tones of their voices, they can tell

lies from truths.可知 C 正确。

166.B 细 节 题 。 根 据 第 三 段 They studied a mixed group of people. Some were normal;

others were aphasics. It was proved that the aphasics were far ahead of the normal people in

recognizing false speeches -- in most cases, the normal people were fooled by words, but the

aphasics were not.可知这个研究是通过比较正常人和患病的人的方法来进行的。故 B 正

确。
167.D 推理题。根据文章最后一段 owever, according to Dr. Sacks, they are more gifted

than normal people. Normal people may get carried away by words. Aphasics seem to

understand human expressions better, though they cannot understand words. 可知在某一方面
有所缺失的人在另外一方面会得到一定的补偿,会很擅长,故 D 正确。

41.(2015 年,重庆卷)

The values of artistic works, according to cultural relativism( 相 对 主 义 ), are simply

reflections of local social and economic conditions. Such a view, however, fails to explain the

ability of some works of art to excite the human mind across cultures and through centuries.

History has witnessed the endless productions of Shakespearean plays in every major

language of the world. It is never rare to find that Mozart packs Japanese concert halls, as

Japanese painter Hiroshige does Paris galleries, Unique works of this kind are different from

today’s popular art, even if they began as works of popular art. They have set themselves

apart in their timeless appeal and will probably be enjoyed for centuries into the future.

In a 1757 essay, the philosopher David Hume argued that because“the general principles

of taste are uniform( 不 变 的 ) in human nature,”the value of some works of art might be

essentially permanent. He observed that Homer was still admired after two thousand years.

Works of this type, he believed, spoke to deep and unvarying features of human nature and

could continue to exist over centuries.

Now researchers are applying scientific methods to the study of the universality of art.

For example, evolutionary psychology is being used by literary scholars to explain the long-

lasting themes and plot devices in fiction. The structures of musical pieces are now open to

experimental analysis as never before. Research findings seem to indicate that the creation by

a great artist is as permanent an achievement as the discovery by a great scientist.

168.According to the passage, what do we know about cultural relativism?

A.It introduces different cultural values.

B.It explains the history of artistic works.

C.It relates artistic values to local conditions.

D.It excites the human mind throughout the world.

169.In Paragraph 2, the artists are mentioned in order to show that _____ .

A.great works of art can go beyond national boundaries

B.history gives art works special appeal to set them apart

C.popular arts are hardly distinguishable from great arts

D.great artists are skilled at combining various cultures

170.According to Hume, some works of art can exist for centuries because_____ .

A.they are results of scientific study

学科网(北京)股份有限公司
B.they establish some general principles of art

C.they are created by the world’s greatest artists

D.they appeal to unchanging features of human nature

171.Which of the following can best serve as the title of the passage?

A.Are Artistic Values Universal?

B.Are Popular Arts Permanent?

C.Is Human Nature Uniform?

D.Is Cultural Relativism Scientific?

【答案】168.C 169.A 170.D 171.A

【解析】本文是说明文,主要介绍了根据文化相对主义,艺术作品在永恒受欢迎的原

因是人类的审美具有共同性,并且就此展开说明。
168. 细节理解题。根据第一段第一句话 The values of artistic works, according to cultural

relativism( 相对主义 ), are simply reflections of local social and economic conditions. 可知根

据相对主义,艺术作品就是把艺术的价值与当地的社会经济条件结合在一块,故选 C

项。
169.推理判断题。根据第二段内容可知,在日本的音乐厅里发现莫扎特,就像在巴黎

的画廊发现日本画家弘之治一样,这并不罕见。因此推断,提到这些艺术家是为了说

明艺术作品可以超越国界,故选 A 项。

170. 细 节 理 解 题 。 根 据 第 三 段 第 一 句 话 In a 1757 essay, the philosopher David Hume

argued that because“the general principles of taste are uniform( 不变的) in human nature,”the

value of some works of art might be essentially permanent. 可知 David 认为由于“品味的

一般原则在人性中是不变的”,某些艺术作品的价值可能本质上是永恒的。所以一些

艺术是可能会永生的,故选 D 项。

171.主旨大意题。本文主要从相对论的角度来说,艺术的价值只能反应当时的社会经

济,作者提出不同的观点,探讨了艺术是普遍的观点。故 A 项“艺术价值是普遍的

吗?”故选 A。

42.(2015 年,重庆卷)

There are many places to go on safari (观赏野生动物) in Africa, but riding a horse

through the flooded waters of Botswana's Okavango Delta must rank as one of the world's

most exciting wildlife journeys.

Several safari camps operate as the base for this adventure, providing unique rides twice

a day to explore deep into the delta. The camps have excellent horses, professional guides and

lots of support workers.Theyhave a reputation for providing a great riding experience.


The morning ride, when the guides take you to beautiful, shallow lakes full of water

lilies, tends to be more active. It is unlike any other riding experience. With rainbows forming

in the splashing water around you and the sound of huge drops of water bouncing off your

body and face. It is truly exciting. You are very likely to come across large wild animals, too.

On horseback it is possible to get quite close to elephants, giraffes and many other animals.

The sense of excitement and tension levels rise suddenly though, as does your heart rate, as

you move closer to them.

In the evening, rides are usually at a more relaxed and unhurried pace. With golden light

streaming across the grassy delta and the animals coming out to eat and drink sedate though

they are, rides at this time of day are still very impressive. As the sun's rays pass through the

dust kicked up by the horses, the romance of Africa comes to life.

Back at the camp you can kick off your boots and enjoy excellent food and wine.

Looking back on your day, you will find it hard to deny that a horseback Safari is as close as

you will ever come to answering the call of the wild.

172.What does the underlined word "They" refer to?

A.Flooded waters. B.Wildlife journey.

C.Safari camps. D.Unique rides.

173.What does the author find most exciting about a horse safari?

A.Seeing and feeling the real African life.

B.Enjoying good food and wine at the camp.

C.Hunting large animals just as our ancestors did.

D.Being part of the scene and getting close to animals.

174.What does the underlined word "sedate" probably mean?

A.Wild and romantic. B.Slow and peaceful.

C.Hurry and thirsty. D.Active and excited.

175.The author introduced the riding experience in the OKavango Delta mainly by________.

A.Following space order B.Following time order

C.Making classifications D.Giving examples

【答案】172.C 173.D 174.B 175.B

【解析】本文是一篇说明文。介绍了非洲观赏野生动物的野营项目。
172. 词义猜测题。根据短文第二段 The camps have excellent horses, professional guides

and lots of support workers. 可知这个野营有优秀的马匹,专业的指导和许多人服务工作,

可知这些野营的声望是很好的,故选 C。

学科网(北京)股份有限公司
173. 细 节理 解题 。根 据第 三段 The sense of excitement and tension levels rise suddenly

though, as does your heart rate, as you move closer to them. 可知,骑马的紧张和兴奋让人

心跳加速,因为与动物更亲近,故选 D。

174. 词义猜测题。根据第四段第一句话 In the evening, rides are usually at a more relaxed

and unhurried pace. 可知在晚上骑马很放松,惬意,因此推断划线句句意:金色的光线

穿过长满青草的三角洲,动物出来吃和喝,尽管它们是缓慢平静的,每天的这个时候

仍旧是令人印象深刻,划线词与 B 项符合,故选 B。

175.推理判断题。本文按照早上骑马,晚上骑马来写,可知是按照时间的顺序来介绍

的,故选 B。

43.(2015 年,浙江卷)

Graph can be a very useful tool for conveying information especially numbers ,

percentages, and other data . A graph gives the reader a picture to interpret. That can be a lot

more pages and pages and pages explaining the data .

Graphs can seem frightening, but reading a graph is a lot like reading a story. The graph

has a title ,a main idea ,and supporting details .You can use your active reading skills to

analyze and understand graphs just like any other text .

Most graphs have a few basic parts: a caption or introduction paragraph, a title , a legend

or key, and labeled axes. An active reader looks at each part of the graph before trying to

interpret the data. Captions will usually tell you where the data came from (for example, a

scientific study of 400 African elephants from 1980 to 2005). Captions usually summarize the

author's main point as well. The title is very important. It tells you the main idea of the graph

by stating what kind of information is being shown. A legend, also called a key ,is a guide to

the symbols and colors used in the graph. Many graphs, including bar graphs and line graphs,

have two axes that form a corner, Usually these axes are the left side and the bottom of the

graph .Each axis will always have a label. The label tells you what each axis measures.

Bar Graphs

A bar graph has two axes and uses bars to show amounts. In Graph 1 ,we see that the x-
axis shows grades that students earned, and the y-axis shows bow many students earned each

grade .You can see that 6 students earned an A because the bar for A stretches up to 6 on the

vertical measurement. There is a lot of information we can get from a simple graph like

this(See Graph 1).

Line Graphs

A line graph looks similar to a bar graph ,but instead of Bars, it plots points and connects

them with a line .It has the same parts as a bar graph – two labeled axes –and can be read the

same way .To read a line graph, it’s important to focus on the points of intersection rather

than the line segments between the points, This type of graph is most commonly used to show

how something changes over time.

Here is a graph that charts how far a bird flies during the first Five days of its spring

migration (See Graph 2).

The unit of measurement for the x-axis is days. The unit of measurement for the y-axis is

kilometers. Thus we can see that ,on the first day, the pipit flew 20 kilometers. The line

segment goes up between Day 1 and Day 2,which means that the bird flew farther on Day 2.If

the line segment angled dawn, as between Day 4 and Day 5,it would mean that the bird flew

fewer kilometers than the day before. This line graph is a quick, visual way to tell the reader

about the bird’s migration.

Pie Graphs

A typical pie graph looks like a circular pie. The circle is divided into sections, and each

学科网(北京)股份有限公司
section represents a fraction of the data. The graph is commonly used to show percentages;

the whole pie represents l00 percent, so each piece is a fraction of the whole.

A pie graph might include a legend , or it might use icons or labels within each slice.

This pie graph shows on month’s expense, (See Graph 3 ).

Food $ 25

Movies $ 12

Clothing $ 36

Savings $ 20

Books $ 7

176.When used in a graph,a legend is_____

A.a guide to the symbols and colors

B.an introduction paragraph

C.the main idea

D.the data

177.What is the total number of students who earned a C or better ?

A.4. B.6. C.10. D.20 .

178.The bird covered the longest distance on _____

A.Day 1 B.Day 2 C.Day 3 D.Day 4

179.Which of the following cost Amy most ?

A.Food. B.Books C.Movies D.Clothing.

【答案】176.A 177.D 178.C 179.D

【解析】这是一篇说明文。这是一篇说明文介绍图表的类型以及相关作用,通过对条

形图、线形图和饼形图的介绍,让学生学会读图,解决实际问题。
176.细节理解题。根据第三段第六行 A legend, also called a key ,is a guide to the symbols

and colors used in the graph.” 可知,图例,也称为键,是图形中使用的符号和颜色的指

南。故选 A 项。

177.细节理解题。根据 Bar Graphs 部分中的图表可知,得到 A 等级的学生有 6 人,B 等

级的学生有 10 人,C 等级的学生有 4 人,故 C 等级以上的学生人数为 20(6+l0+4)人。

故选 D 项。

178.细节理解题。根据 Line Graphs 部分第二段并结合图表可知,第三天这只鸟飞翔了

70 千米,这是它这些天飞行最长的距离。故选 C 项。

179.细节理解题。根据扇形图可以得出 Clothing 占比例最多。故选 D。

44.(2015 年,全国卷 I)

Conflict is on the menu tonight at the café La Chope. This evening, as on every
Thursday night, psychologist Maud Lehanne is leading two of France’s favorite pastimes,

coffee drinking and the “talking cure”. Here they are learning to get in touch with their true

feelings. It isn’t always easy. They customers—some thirty Parisians who pay just under $2

(plus drinks) per session-care quick to intellectualize ( 高 谈 阔 论 ), slow to open up and

connect. “You are forbidden to say ‘one feels,’ or ‘people think’,” Lehane told them. “Say ‘I

think,’ ‘Think me’.”

A café society where no intellectualizing is allowed? It couldn’t seem more un-French.

But Lehanne’s psychology café is about more than knowing oneself: It’s trying to help the

city’s troubled neighborhood cafes. Over the years, Parisian cafes have fallen victim to

changes in the French lifestyle-longer working hours, a fast food boom and a younger

generation’s desire to spend more time at home. Dozens of new theme cafes appear to change

the situation. Cafes focused around psychology, history, and engineering are catching on,

filling tables well into the evening.

The city’s psychology cafes, which offer great comfort, are among the most popular

places. Middle-aged homemakers, retirees, and the unemployed come to such cafes to talk

about lover, anger, and dreams with a psychologist. And they come to Lehance’s group just to

learn to say what they feel. There’s a strong need in Paris for communication, says Maurice

Frisch, a cafe La Chope regular who works as religious instructor in a nearby church. “People

have few real friends. And they need to open up” Lehanne says she’d like to see psychology

cafes all over France. “If people had normal lives, these cafes wouldn’t exist”, she says, “If

life weren’t a battle, people wouln’t need a special place just to speak.” But then, it wouldn’t

be France.

180.What are people encouraged to do at the cafe La Chope?

A.Learn a new subject

B.Keep in touch with friends.

C.Show off their knowledge.

D.Express their true feelings.

181.How are cafes affected by French lifestyle changes?

A.They are less frequently visited.

B.They stay open for longer hours.

C.They have bigger night crowds.

D.They start to serve fast food.

学科网(北京)股份有限公司
182.What are theme cafes expected to do?

A.Create more jobs.

B.Supply better drinks.

C.Save the cafe business.

D.Serve the neighborhood.

183.Why are psychology cafes becoming popular in Paris?

A.They bring people true friendship.

B.They give people spiritual support.

C.They help people realize their dreams.

D.They offer a platform for business links.

【答案】180.D 181.A 182.C 183.B

【解析】这是一篇说明文。文章主要介绍了法国精神咖啡馆( psychology cafes)的社

会意义和功能特色,以及在法国越来越受欢迎。
180. 细节理解题。根据第一段第三句 Here they are learning to get in touch with their true

feelings. 可知,在这里他们正在尝试了解他们的真实感受,所以在 La Chope 咖啡馆鼓

励人们表达他们真正的情感。故选 D。

181. 推 理 判 断 题 。 根 据 第 二 段 中 Over the years, Parisian cafes have fallen victim to

changes in the French lifestyle-longer working hours, a fast food boom and a younger

generation’s desire to spend more time at home. 可知,多年来,巴黎的咖啡馆一直是法国

生活方式变化的受害者——工作时间变长、快餐的蓬勃发展以及年轻一代更愿意花更

多的时间在家里。所以法国生活方式使得咖啡馆顾客越来越少。故选 A。

182.推理判断题。根据第二段中 Dozens of new theme cafes appear to change the situation.

Cafes focused around psychology, history, and engineering are catching on, filling tables well

into the evening. 可知,数十家新的主题咖啡馆似乎改变了这一局面。这些咖啡馆专注

于心理学、历史和工程学这些很受欢迎的主题,直到夜晚人们还不愿离去,所以判断

出主题咖啡馆被期望拯救咖啡店的生意。故选 C。

183. 推理判断题。根据最后一段第一句 If people had normal lives, these cafes wouldn’t

exist”, she says, “If life weren’t a battle, people wouldn’t need a special place just to speak.”

可知,“如果人们有正常的生活,这些咖啡馆就不会存在,”她说,“如果生活不是

一场战争,人们就不需要一个专门的地方来说话。”所以判断出心理咖啡馆在巴黎受

欢迎的原因在于他们给予人们精神上的支持。故选 B。

45.(2015 年,天津卷)

University Room Regulations


Approved and Prohibited Items

The following items are approved for use in residential ( 住 宿 的 ) rooms: electric

blankets, hair dryers, personal computers, radios, televisions and DVD players. Items that are

not allowed in student rooms include: candles, ceiling fans, fireworks, waterbeds, sun lamps

and wireless routers. Please note that any prohibited items will be taken away by the Office of

Residence Life.

Access to Residential Rooms

Students are provided with a combination ( 组合密码 ) for their room door locks upon

check-in. Do not share your room door lock combination with anyone. The Office of

Residence Life may change the door lock combination at any time at the expense of the

resident if it is found that the student has shared the combination with others. The fee is $25

to change a room combination.

Cooking Policy

Students living in buildings that have kitchens are only permitted to cook in the kitchen.

Students must clean up after cooking. This is not the responsibility of housekeeping staff.

Kitchens that are not kept clean may be closed for use. With the exception of using a small

microwave oven (微波炉) to heat food, students are not permitted to cook in their rooms.

Pet Policy

No pets except fish are permitted in student rooms. Students who are found with pets,

whether visiting or owned by the student, are subject to an initial fine of $100 and a

continuing fine of $50 a day per pet. Students receive written notice when the fine goes into

effect. If, one week from the date of written notice, the pet is not removed, the student is

referred to the Student Court.

Quiet Hours

Residential buildings must maintain an atmosphere that supports the academic mission

of the University. Minimum quiet hours in all campus residences are 11:00 pm to 8:00 am

Sunday through Thursday. Quiet hours on Friday and Saturday nights are 1:00 am to 8:00 am.

Students who violate quiet hours are subject to a fine of $25.

184.Which of the following items are allowed in student rooms?

A.Ceiling fans and waterbeds.

B.Wireless routers and radios.

C.Hair dryers and candles.

学科网(北京)股份有限公司
D.TVs and electric blankets.

185.What if a student is found to have told his combination to others?

A.The combination should be changed.

B.The Office should be charged.

C.He should replace the door lock.

D.He should check out of the room.

186.What do we know about the cooking policy?

A.A microwave oven can be used.

B.Cooking in student rooms is permitted.

C.A housekeeper is to clean up the kitchen.

D.Students are to close kitchen doors after cooking.

187.If a student has kept a cat in his room for a week since the warning, he will face _____.

A.parent visits B.a fine of $100

C.the Student Court D.a written notice

188.When can students enjoy a party in residences?

A.7:00 am, Sunday. B.7:30 am, Thursday.

C.11:30 pm, Monday. D.00:30 am, Saturday.

【答案】184.D 185.A 186.A 187.C 188.D

【解析】本文是一篇说明文。文章主要介绍了大学宿舍的规范:批准和禁止的项目、

烹饪的政策、宠物政策等规定,让同学们严格遵守。。
184. 细节理解题。根据文章第一段第一句 The following items are approved for use in

residential ( 住 宿 的 ) rooms: electric blankets, hair dryers, personal computers, radios,

televisions and DVD players.可知电视机和电热毯是允许使用的。故选 D。

185. 细节理解题。根据文章第二段 The Office of Residence Life may change the door lock

combination at any time at the expense of the resident if it is found that the student has shared

the combination with others.可知如果发现学生与他人共享门锁组合,居住生活办公室可

随时更换门锁组合,费用由居民承担。故选 A。

186. 细节理解题。根据第三段 With the exception of using a small microwave oven ( 微波

炉) to heat food, students are not permitted to cook in their rooms. 可知除了可以使用小的微

波炉来加热食物外,不允许学生在房间里做饭,故小的微波炉是可以使用的。选 A。

187.细节理解题。根据文章倒数第二段 If, one week from the date of written notice, the pet

is not removed, the student is referred to the Student Court. 可知,学生在接到书面通知一

周之内,没有把宠物移走的,将被移交给学生法庭。故选 C。
188. 推理判断题。根据文章最后一段 Quiet hours on Friday and Saturday nights are 1:00

am to 8:00 am.可知周六的安静时间是早上 1 点到 8,故可以推断出答案。故选 D。

46.(2015 年,四川卷)

Across Britain, burnt toast will be served to mothers in bed this morning as older sons

and daughters rush to deliver their supermarket bunches of flowers. But, according to a new

study, we should be placing a higher value on motherhood all year.

Mothers have long known that their home workload was just as heavy as paid work.

Now, the new study has shown that if they were paid for their parental labours, they would

earn as much as $ 172,000 a year.

The study looked at the range of jobs mothers do, as well as the hours they are working,

to determine the figure. This would make their yearly income $ 30,000 more than the Prime

Minister earns.

By analysing the numbers, it found the average mother works 119 hours a week, 40 of

which would usually be paid at a standard rate and 79 hours as overtime. After questioning

1,000 mothers with children under 18, it found that, on most days, mums started their routine

work at 7am and finished at around 11pm.

To calculate just how much mothers would earn from that labour, it suggested some of

the roles that mums could take on, including housekeeper, part-time lawyer, personal trainer

and entertainer. Being a part-time lawyer, at £ 48.98 an hour, would prove to be the most

profitable of the “mum jobs”, with psychologist (心理学家) a close second.

It also asked mothers about the challenges they face, with 80 percent making emotional

demand as the hardest thing about motherhood.

Over a third of .mums felt they needed more training and around half said they missed

going out with friends.

The study shows mothers matter all year long and not just on Mother’s Day. The

emotional, physical and mental energy mothers devote to their, children can be never-ending,

but children are also sources of great joy and happiness. Investing ( 投 入 ) in time for

parenting and raising relationships is money well spent.

189.How much would a mother earn a year if working as the Prime Minister?

A.£ 30,000. B.£ 142,000.

C.£ 172,000. D.£ 202,000.

190.The biggest challenge for most mothers is from .

学科网(北京)股份有限公司
A.emotional demand B.low pay for work

C.heavy workload D.lack of training

191.What is stressed in the last paragraph?

A.Mothers’ importance shows in family all year long.

B.The sacrifices mothers make are huge but worthwhile.

C.Mothers’ devotion to children can hardly be calculated.

D.Investing time in parenting would bring a financial return.

192.What can we conclude from the study?

A.Mothers,working hours should be largely reduced.

B.Mothers should balance their time for work and rest.

C.Mothers’ labour is of a higher value than it is realised.

D.Mothers should be freed from housework for social life.

【答案】189.B 190.A 191.B 192.C

【解析】这是一篇说明文。母亲是伟大的,人们对母亲的重视不应该只局限于在母亲

节这天给母亲买鲜花礼物上。调查表明母亲在日常生活中的付出如果用钱来支付的话

她们能挣到多达每年£172,000 ,所以人们要尊重母亲,要每天都感恩母亲。

189. 细节理解题。根据第二段 the new study has shown that if they were paid for their

parental labours, they would earn as much as £172,000 a year. 现在,新的研究表明,如果

他们的父母工作得到报酬,他们一年能挣 172,000 美元。和第三段 This would make

their yearly income £30,000 more than the Prime Minister earns. 这将使他们的年收入比首

相多 3 万英镑。可知总理的工资应该是£172,000-£30,000=£142,000,故选 B。

190.细节理解题。根据第六段 It also asked mothers about the challenges they face, with 80

percent making emotional( 情感的 ) demand as the hardest thing about motherhood. 可知,调

查还询问了母亲们面临的挑战,80%的人认为情感需求是做母亲最大的需求。所以大

多数母亲面临的最大挑战是来自情感的需求,故选 A。

191. 推理判断题。根据末段 The emotional, physical and mental energy mothers devote to

their, children can be never-ending, but children are also sources of great joy and happiness.

Investing ( 投入 ) in time for parenting and raising relationships is money well spent. 可知,

母亲为孩子们付出的情感、身体和精神上的能量是无穷无尽的,但孩子们也是巨大的

快乐和幸福的源泉。把时间花在养育子女和培养关系上是值得的,由此判断出最后一

段强调母亲做出的牺牲是巨大的,但也是值得的。故选 B。

192.推理判断题。根据第二段 Now, the new study has shown that if they were paid for their

parental labours, they would earn as much as $ 172,000 a year. 现在,新的研究表明,如果


他们的父母工作得到报酬,他们一年能挣 172,000 美元。根据第四段 By analysing the

numbers, it found the average mother works 119 hours a week, 40 of which would usually be

paid at a standard rate and 79 hours as overtime. After questioning 1,000 mothers with

children under 18, it found that, on most days, mums started their routine work at 7am and

finished at around 11pm. 通过分析这些数字,研究发现,母亲平均每周工作 119 小时,

其中 40 小时通常按标准工资支付, 79 小时为加班费。调查了 1000 名 18 岁以下的母亲

后发现,大多数情况下,母亲早上 7 点开始日常工作,晚上 11 点左右结束。由此推断

出母亲的劳动价值比人们想象的要高。故选 C。

47.(2015 年,上海卷)

Look to many of history’s cultural symbols, and there you’ll find an ancestor of Frosty,

the snowman in the movie Frozen. It appeared on some of the first postcards, starred in some

of the earliest silent movies, and was the subject of a couple of the earliest photos, dating all

the way back to the 1800s. I discovered even more about one of humanity’s earliest forms of

life art during several years of research around the world.

For example, snowmen were a phenomenon in the Middle Ages, built with great skill

and thought. At a time of limited means of expression, snow was like free art supplies

dropped from the sky. It was a popular activity for couples to leisurely walk through town to

view the temporary works of chilly art. Some were created by famous artists, including a 19-

year-old Michelangelo, who in 1494 was appointed by the ruler of Florence, Italy, to build a

snowman in his mansion’s courtyard.

The Miracle of 1511 took place during six freezing works called the Winter of Death.

The city of Brussels was covered in snowmen—an impressive scene that told stories on every

street corner. Some were political in nature, criticizing the church and government. Some

were a reflection of people’s imagination. For the people of Brussels, this was a defining

moment of defining freedom. At least until spring arrived, by which time they were dealing

with damaging floods.

If you fear the heyday of the snowman has passed, don’t worry: I’ve learned that some

explosive snowman history is still being made today. Every year since 1818, the people of

Zurich, Switzerland, celebrate the beginning of spring by blowing up a snowman. On the

third Monday of April, the holiday Sechselauten is kicked off when a cotton snowman called

the Boogg is stuffed with explosive and paraded through town by bakers and other tradesmen

who throw bread to the crowds. The parade ends with the Boogg being placed on a 40-foot

学科网(北京)股份有限公司
pile of firewood. After the bells of the Church of St. Peter have rung six times, representing

the passing of winter, the pile is lit. When the snowman explodes, winter is considered

officially over—the quicker it is burnt down, the longer summer is said to be.

193.According to the passage, why did snowmen become a phenomenon in the Middle Ages?

A.People thought of snow as holy art supplies.

B.People longed to see masterpieces of snow.

C.Building snowmen was a way for people to express themselves.

D.Building snowmen helped people develop their skill and thought.

194.“The heyday of the snowman” (paragraph 4) means the time when___________.

A.snowmen were made mainly by artists

B.snowmen enjoyed great popularity

C.snowmen were politically criticized

D.snowmen caused damaging floods

195.In Zurich, the blowing up of the Boogg symbolizes__________________.

A.the start of the parade

B.the coming of a longer summer

C.the passing of the winter

D.the success of tradesmen

196.What can be concluded about snowmen from the passage?

A.They were appreciated in history

B.They have lost their value

C.They were related to movies

D.They vary in shape and size

【答案】193.C 194.B 195.C 196.A

【解析】本文讲述历史的文化标志:雪人。
193. 细节理解题。根据第二段第二句 At a time of limited means of expression, snow was

like free art supplies dropped from the sky 在那时,表达方式的方法有限,雪就像是来自

于上天的一种自由艺术。 这说明雪的流行是因为它是一种表达的方式,关键词是

express,而且 means 和 way 同义。故选 C。

194. 词义猜测题。根据文章第四段 If you fear the heyday of the snowman has passed ,

don't worry : I've learned that some explosive snowman history is still being made today.可

知如果你担心雪人的全盛期已经过去,不要担心:我已经知道,一些爆炸性的雪人历

史仍然在今天被创造。由此可见, " 雪人的全盛时期 "(第 4 段)是指雪人非常受欢迎

的时期。所以答案就是 B。
195. 细 节 理 解 题 。 根 据 文 章 倒 数 第 二 段 Every year since 1818 , the people of

Zurich,Switzerland, celebrate the beginning of spring by blowing up a snowman. 可知自

1818 年以来,瑞士苏黎世的人们每年都会用吹雪人来庆祝春天的开始。 the passing of

the winter 冬天的过去,符合题意。所以答案是 C。

196.推理判断题。根据文章的第一句话 look to many of history’s cultural symbols 回顾许

多的历史文化标志,另外整篇文章出现过很多的 history,结合上下文可以知道答案是

A (雪人在历史上是受欣赏的。)另外也可以根据排除法,排除其他选项。比如 B 选

项,他们的价值现在依然存在,所以它的表达错误。 C 选项,雪人的确是与电影有关,

但是这个答案不可以通过文章得出来。D 选项在文章中没有提及这个内容。

48.(2015 年,陕西卷)

The production of coffee beans is a huge, profitable business, but, unfortunately, full-sun

production is taking over the industry and bringing about a lot of damage. The change in how

coffee is grown from shade-grown production to full-sun production endangers the very

existence of, certain animals and birds, and even disturbs the world’s ecological balance.

On a local level, the damage of the forest required by full-sun fields affects the area’s

birds and animals. The shade of the forest trees provides a home for birds and other

special(物种) that depend on the trees’ flowers and fruits. Full-sun coffee growers destroy this

forest home. As a result, many special are quickly dying out.

On a more global level, the destruction of the rainforest for full-sun coffee fields also

threatens( 威胁)human life. Medical research often makes use of the forests' plant and animal

life, and the destruction of such species could prevent researchers from finding cures for

certain diseases. In addition, new coffee-growing techniques are poisoning the water locally,

and eventually the world's groundwater.

Both locally and globally, the continued spread of full-sun coffee plantations ( 种 植

园 )could mean the destruction of the rainforest ecology. The loss of shade trees is already

causing a slight change in the world's climate, and studies show that loss of oxygen-giving

trees also leads to air pollution and global warming. Moreover, the new growing techniques

are contributing to acidic(酸性的) soil conditions.

It is obvious that the way much coffee is grown affects many aspects many aspects of

life, from the local environment to the global ecology. But consumers do have a choice. They

can purchase shade-grown coffee whenever possible, although at a higher cost. The future

health of the planet and mankind is surely worth more than an inexpensive cup of coffee.

学科网(北京)股份有限公司
197.What can we learn about full-sun coffee production from Paragraph 4?

A.It limits the spread of new growing techniques.

B.It leads to air pollution and global warming.

C.It slows down the loss of shade trees.

D.It improves local soil conditions.

198.The purpose of the text is to .

A.entertain

B.advertise

C.instruct

D.persuade

199.Where does this text probably come from ?

A.An agricultural magazine.

B.A medical journal.

C.An engineering textbook.

D.A tourist guide.

200.Which of the following shows the structure of the whole text?

A. B.

C. D.

【答案】197.B 198.D 199.A 200.A

【解析】本文是一篇说明文。文章介绍阳光充足的咖啡生产给环境和生态带来的种种

危害,呼吁人们不要购买这种咖啡,而要买对环境有利的产品。
197.细节理解题。根据文章第四段的句子 The loss of shade trees is already causing a slight

change in the world's climate, and studies show that loss of oxygen-giving trees also leads to

air pollution and global warming.(遮阳树的丧失已经引起了世界气候的轻微变化,研究

表明,给氧树的丧失也会导致空气污染和全球变暖。)可知阳光充足的咖啡生产会带

来空气污染和全球气温变暖,故选 B。

198. 推理判断题。根据文章最后一段的句子 But consumers do have a choice. They can

purchase shade-grown coffee whenever possible, although at a higher cost. (但消费者的确

可以选择。他们可以在可能的情况下购买遮阳咖啡,尽管成本较高。)可知,作者的

意图是劝说人们购买阴凉处种植的咖啡,这样对环境是有好处的,故选 D。

199.推理判断题。纵观全文可知,文章介绍阳光充足的咖啡生产会带来的破坏,应该

是出自一本农业杂志,故选 A。

200.推理判断题。文章第一段介绍阳光充足的咖啡生产的话题,二、三、四段介绍了

阳光充足的咖啡生产带来的破坏,第五段说服人们去购买对环境有利的阴凉处种植的

咖啡,所以是总分总的结构,故选 A。

49.(2015 年,江苏卷)

Freedom and Responsibility

Freedom’s challenge in the Digital Age is a serious topic. We are facing today a strange

new world and we are all wondering what we are going to do with it.

Some 2,500 years ago Greece discovered freedom. Before that there was no freedom.

There were great civilizations, splendid empires, but no freedom anywhere. Egypt and

Babylon were both tyrannies , one very powerful man ruling over helpless masses.

In Greece, in Athens ( 雅 典 ), a little city in a little country, there were no helpless

masses. And Athenians willingly obeyed the written laws which they themselves passed, and

the unwritten, which must be obeyed if free men live together. They must show each other

kindness and pity and the many qualities without which life would be very painful unless one

chose to live alone in the desert.The Athenians never thought that a man was free if he could

do what he wanted. A man was free if he was self-controlled. To make yourself obey what

you approved was freedom. They were saved from looking at their lives as their own private

affair. Each one felt responsible for the welfare of Athens, not because it was forced on him

from the outside, but because the city was his pride and his safety. The essential belief of the

first free government in the world was liberty for all men who could control themselves and

would take responsibility for the state.

学科网(北京)股份有限公司
But discovering freedom is not like discovering computers. It cannot be discovered once

for all. If people do not prize it, and work for it, it will go. Constant watch is its price. Athens

changed. It was a change that took place without being noticed though it was of the extreme

importance, a spiritual change which affected the whole state. It had been the Athenian’ s

pride and joy to give to their city. That they could get material benefits from her never entered

their minds. There had to be a complete change of attitude before they could look at the city

as an employer who paid her citizens for doing her work. Now instead of men giving to the

state, the state was to give to them. What the people wanted was a government which would

provide a comfortable life for them; and with this as the primary object, ideas of freedom and

self-reliance and responsibility were neglected to the point of disappearing. Athens was more

and more looked on as a cooperative business possessed of great wealth in which all citizens

had a right to share.

Athens reached the point when the freedom she really wanted was freedom from

responsibility. There could be only one result. If men insisted on being free from the burden

of self-dependence and responsibility for the common good, they would cease to be free.

Responsibility is the price every man must pay for freedom. It is to be had on no other terms.

Athens, the Athens of Ancient Greece, refused responsibility; she reached the end of freedom

and was never to have it again. But, “the excellent becomes the permanent”, Aristotle said.

Athens lost freedom forever, but freedom was not lost forever for the world. A great

American, James Madison, referred to: “The capacity ( 能 力 ) of mankind for self-

government.” No doubt he had not an idea that he was speaking Greek. Athens was not in the

farthest background of his mind, but once man has a great and good idea, it is never

completely lost. The Digital Age cannot destroy it. Somehow in this or that man’s thought

such an idea lives though unconsidered by the world of action. One can never be sure that it is

not on the point of breaking out into action only sure that it will do so sometime.

201.What does the underlined word “tyrannies” in Paragraph 2 refer to?

A.Countries where their people need help.

B.Powerful states with higher civilization.

C.Splendid empires where people enjoy freedom.

D.Governments ruled with absolute power.

202.People believing in freedom are those who________ .

A.regard their life as their own business

B.seek gains as their primary object


C.behave within the laws and value systems

D.treat others with kindness and pity

203.What change in attitude took place in Athens?

A.The Athenians refused to take their responsibility.

B.The Athenians no longer took pride in the city.

C.The Athenians benefited spiritually from the government.

D.The Athenians looked on the government as a business.

204.What does the sentence “There could be only one result.” in Paragraph 5 mean?

A.Athens would continue to be free.

B.Athens would cease to have freedom.

C.Freedom would come from responsibility.

D.Freedom would stop Athens from self-dependence.

205.Why does the author refer to Aristotle and Madison?

A.The author is hopeful about freedom.

B.The author is cautious about self-government.

C.The author is skeptical of Greek civilization.

D.The author is proud of man’s capacity.

206.What is the author’s understanding of freedom?

A.Freedom can be more popular in the digital age.

B.Freedom may come to an end in the digital age.

C.Freedom should have priority over responsibility.

D.Freedom needs to be guaranteed by responsibility.

【答案】201.D 202.C 203.A 204.B 205.A 206.D

【解析】这是一篇历史文化类说明文。文章讲述了自由的起源、演变和它在古代文化

下的形态和缺陷,以及数字化世界给自由带来的挑战。
201.词义猜测题。根据第二段划线词所在句子后半句 one very powerful man ruling over

helpless masses 可知,一个非常强大的人统治着无助的大众。所以通过上下文语境可以

判断出,tyrannies 是指拥有绝对统治权的专制政府。故选 D 项。

202. 细节理解题。根据第三段“ And Athenians willingly obeyed the written laws which

they themselves passed, and the unwritten, which must be obeyed if free men live together.”

可知,雅典人自愿遵守他们自己通过的成文法,和不成文法,如果要自由地生活在一

起,这些法律必须遵守。所以相信自由的人是那些遵守法律和价值体系的人。故选 C

项。

学科网(北京)股份有限公司
203. 细 节理 解题 。根 据第 四段 What the people wanted was a government which would

provide a comfortable life for them; and with this as the primary object, ideas of freedom and

self-reliance and responsibility were neglected to the point of disappearing. 可知,雅典人后

来态度发生了改变,因为他们开始认为政府应该为他们的劳动付费,给他们提供舒适

的生活,而个人的自律和责任意识变得淡薄。所以雅典人拒绝承担他们的责任。故选

A 项。

204. 词句猜测题。根据倒数第二段 If men insisted on being free from the burden of self-

dependence and responsibility for the common good, they would cease to be free. 可知,如果

人们坚持要摆脱自力更生和为共同利益承担负责,他们将不再享有自由。所以通过上

下文的语境可以判断划线“There could be only one result”的意思是雅典人不再享有自由。

故选 B 项。

205. 推 理判 断题 。根 据最 后一 段 Athens lost freedom forever, but freedom was not lost

forever for the world. 雅典永远失去了自由,但世界并没有永远失去自由及“Athens was

not in the farthest background of his mind, but once man has a great and good idea, it is never

completely lost. 雅典并不在他思想的最深处,但是一旦人类有了伟大而美好的想法,它

就永远不会完全消失 The Digital Age cannot destroy it. 数字时代不能摧毁它”。由此判

断出作者为什么提及了 Aristotle and Madison 是为了表明作者并没有因此而失去信心,

相反对自由充满了希望。故选 A 项。

206. 细节理解题。根据第四段 Responsibility is the price every man must pay for freedom.

可知,责任是每个人为了自由必须付出的代价。所以作者对自由的理解是自由需要责

任的保障。故选 D 项。

50.(2015 年,广东卷)

It was once common to regard Britain as a society with class distinction. Each class had

unique characteristics.

In recent years, many writers have begun to speak of the ‘decline of class ’ and ‘classless

society ’ in Britain. And in modern day consumer society everyone is considered to be middle

class.

But pronouncing the death of class is too early. A recent wide-ranging study of pubic

opinion found 90 percent of people still placing themselves in a particular class; 73 percent

agreed that class was still a vital part of British society.; and 52 percent thought there were

still sharp class differences. Thus, class may not be culturally and politically obvious, yet it

remains an important part of British society. Britain seems to have a love of stratification.
One unchanging aspect of a British person’s class position is accent. The words a person

speaks tell her or his class. A study of British accents during the 1970s found that a voice

sounding like a BBC newsreader was viewed as the most attractive voice. Most people said

this accent sounds ‘educated ’ and ‘soft ’. The accents placed at the bottom in this study, on

the other hand, were regional( 地区的 ) city accents. These accents were seen as ‘common ’

and ‘ugly ’. However, a similar study of British accents in the US turned these results upside

down and placed some regional accents as the most attractive and BBC English as the least.

This suggests that British attitudes towards accent have deep roots and are based on class

prejudice.

In recent years, however, young upper middle-class people in London, have begun to

adopt some regional accents, in order to hide their class origins. This is an indication of class

becoming unnoticed. However, the 1995 pop song ‘ Common People ’ puts forward the view

that though a middle-class person may ‘ want to live like common people ’ they can never

appreciate the reality of a working class life.

207.A recent study of pubic opinion shows that in modern Britain_________.

A.it is time to end class distinction

B.most people belong to middle class

C.it is easy to recognize a person’s class

D.people regard themselves socially different

208.The word stratification in Paragraph 3 is closest in meaning to_________.

A.variety B.division

C.authority D.qualification

209.The study in the US showed that BBC English was regarded as _________.

A.regional B.educated

C.prejudiced D.unattractive

210.British attitudes towards accent_________.

A.have a long tradition

B.are based on regional status

C.are shared by the Americans

D.have changed in recent years

211.What is the main idea of the passage?

A.The middle class is expanding.

B.A person’s accent reflects his class.

学科网(北京)股份有限公司
C.Class is a key part of British society.

D.Each class has unique characteristics.

【答案】207.D 208.B 209.D 210.D 211.C

【解析】这是一篇说明文。作者在文中介绍了两项调查,关于 "阶级消亡 "的说法,在

英国对大众进行了一个调查,结果发现 90% 的人仍然把自己划分在一定的阶层中;

73%的人认为阶级是英国社会一个必不可少的部分,52%的人认为仍然有尖锐的阶级差

异.另一项调查是关于人们说话的口音,英国和美国的调查结果正好相反,他们认为一

些地方口音是最吸引人的,而 BBC 的英语是最不吸引人的,也说明了英国阶级划分的

根源非常深.通过调查的结果可知,英国社会的阶级划分没有消失,它是英国社会重要

的一部分。
207. 细节理解题 . 根据第三段的 90percent of people still placing themselves in particular

class ; 73percent agreed that class was still a vital part of British society ; and 52percent

thought there were still sharp class differences." 可知,关于 "阶级消亡" 的说法,在英国对

大众进行了一个调查,结果发现 90%的人仍然把自己划分在一定的阶层中;73%的人

认为阶级是英国社会一个必不可少的部分,52%的人认为仍然有尖锐的阶级差异。所

以最近的一项民意调查显示,在现代英国,人们仍然认为他们在社会中是有区别的。

故选 D.

208. 词义猜测题。根据第三段的 90percent of people still placing themselves in particular

class ; 73percent agreed that class was still a vital part of British society ; and 52percent

thought there were still sharp class differences 可知,根据一项关于英国社会阶级是否开

始消亡的调查可知,绝大多数人仍然认为英国社会中存在不同的阶级,这是英国社会

中重要的一部分。所以英国人好像对阶级划分非常热衷。所以通过上下文的语境可以

判断出第三段中“stratification”是阶层分化。故选 B 项。

209. 细 节 理 解 题 。 根 据 第 四 段 中 However, a similar study of British accents in the US

turned these results upside down and placed some regional accents as the most attractive and

BBC English as the least. 可知,美国关于英语口音的调查结果正好和英国的调查结果相

反,他们认为一些地方口音是最吸引人的,而 BBC 的英语是最不吸引人的。故选 D

210. 推理判断题。根据短文的最后一段的内容可知, In recent years, however, young

upper middle-class people in London, have begun to adopt some regional accents ,可知,然

而,近年来,伦敦年轻的中上层阶级开始接受一些地方口音。由此判断出现在英国人

们对待口音的态度开始改变了。故选 D。

211.主旨大意题。这篇短文给我们讲述了在英国社会中,阶级划分是重要的一部分。
作者在短文中给我们介绍了两项调查,其中一项告诉我们大多数人们觉得阶级是英国

社会必不可少的一部分;另一项调查是关于人们说好的口音,英国和美国的调查结果

正好相反,也说明了英国阶级划分的根源非常深。所以短文主要是是关于阶级是英国

社会中的一个重要部分。故选 C。

51.(2015 年,安徽卷)

Food serves as a form of communication in two fundamental ways. Sharing bread or

other foods is a common human tradition that can promote unity and trust. Food can also

have a specific meaning, and play a significant role in a family or culture's celebrations or

traditions. The foods we eat—and when and how we eat them—are often unique to a

particular culture or may even differ between rural ( 农 村 的 ) and urban areas within one

country.

Sharing bread, whether during a special occasion ( 时刻 ) or at the family dinner table, is

a common symbol of togetherness. Many cultures also celebrate birthdays and marriages with

cakes that are cut and shared among the guests. Early forms of cake were simply a kind of

bread, so this tradition hits its roots in the custom of sharing bread.

Food also plays an important role in many New Year celebrations. In the southern United

States, pieces of corn bread represent blocks of gold for prosperity ( 兴旺) in the New Year. In

Greece, people share a special cake called vasilopita. A coin is put into the cake, which

signifies (预示) success in the New Year for the person who receives it.

Many cultures have ceremonies to celebrate the birth of a child, and food can play a

significant role. In China, when a baby is one month old, families name and welcome their

child in a celebration that includes giving red-colored eggs to guests. In many cultures, round

foods such as grapes, bread, and moon cakes are eaten at welcome celebrations to represent

family unity.

Nutrition is necessary for life, so it is not surprising that food is such an important part of

different cultures around the world.

212.According to the passage, sharing bread______.

A.indicates a lack of food

B.can help to develop unity

C.is a custom unique to rural areas

D.has its roots in birthday celebrations

213.What does the coin in vasilopita signify for its receiver in the New Year?

学科网(北京)股份有限公司
A.Trust. B.Success.

C.Health. D.Togetherness.

214.The author explains the role of food in celebrations by______.

A.using examples B.making comparisons

C.analyzing causes D.describing processes

215.What is the passage mainly about?

A.The custom of sharing food.

B.The specific meaning of food.

C.The role of food in ceremonies.

D.The importance of food in culture.

【答案】212.B 213.B 214.A 215.D

【解析】这是一篇说明文。文章主要讲的是食物和文化的关系。不同的文化包含着不

同的饮食传统和庆祝方式。食物通过两种基本的方式在扮演着沟通的角色。
212.细节理解题。由第一段 Sharing bread or other foods is a common human tradition that

can promote unity and trust. 可知,分享面包或其他食物是人类共同的传统,可以促进团

结和增加信任。所以分享食物可以促进团结。故选 B。

213.细节理解题。由第三段 A coin is put into the cake, which signifies (预示) success in the

New Year...可知,一枚硬币放入蛋糕 ,这预示着在新的一年里取得成功。所以硬币预示

着来年的成功。故选 B。

214. 推 理 判 断 题 。 由 第 三 段 “ Food also plays an important role in many New Year

celebrations. In the southern United States, pieces of corn bread represent blocks of gold for

prosperity ( 兴旺 ) in the New Year. In Greece, people share a special cake called vasilopita.”

可知,食物在许多新年庆祝活动中也扮演着重要的角色。在美国南部,一块块的玉米

面包代表着新的一年的繁荣昌盛。在希腊,人们分享一种叫做 vasilopita 的特殊蛋糕。

所以作者主要使用了举例的方法来解释食物在庆祝中的角色。故选 A。

215. 主旨大意题。通读全文尤其是“ Nutrition is necessary for life, so it is not surprising

that food is such an important part of different cultures around the world. ” 可知,营养是生

命所必需的,所以食物是世界各地不同文化的重要组成部分也就不足为奇了。所以这

篇文章主要讲了食物在文化中的重要性。故选 D。

52.(2015 年,安徽卷)

There are an extremely large number of ants worldwide. Each individual ( 个体的 ) ant

hardly weigh anything, but put together they weigh roughly the same as all of mankind. They

also live nearly everywhere, except on frozen mountain tops and around the poles.For
animals their size, ants have been astonishingly successful, largely due to their wonderful

social behavior.

In colonies ( 群 体 ) that range in size from a few hundred to tens of millions, they

organize their lives with a clear division of labor. Even more amazing is how they achieve

this level of organization. Where we use sound and sight to communicate, ants depend

primarily on pheromone ( 外激素), chemicals sent out by individuals and smelled or tasted by

fellow members of their colony. When an ant finds food, it produces a pheromone that will

lead others straight to where the food is. When an individual ant comes under attack or is

dying, it sends out an alarm pheromone to warn the colony to prepare for a conflict as a

defense unit.

In fact, when it comes to the art of war, ants have no equal. They are completely fearless

and will readily take on a creature much larger than themselves, attacking in large groups and

overcoming their target. Such is their devotion to the common good of the colony that not

only soldier ants but also worker ants will sacrifice their lives to help defeat an enemy.

Behaving in this selfless and devoted manner, these little creatures have survived on Earth,

for more than 140 million years, far longer than dinosaurs. Because they think as one, they

have a collective ( 集 体 的 ) intelligence greater than you would expect from its individual

parts.

216.We can learn from the passage that ants are ____________.

A.not willing to share food

B.not found around the poles

C.more successful than all other animals

D.too many to achieve any level of organization

217.Ants can use pheromones for______.

A.escape B.communication

C.warning enemies D.arranging labor

218.What does the underlined expression "take on" in Paragraph 3 mean?

A.Accept. B.Employ.

C.Play with. D.Fight against.

219.Which of the following contributes most to the survival of ants?

A.Their behavior. B.Their size.

C.Their number. D.Their weight

【答案】216.B 217.B 218.D 219.A

学科网(北京)股份有限公司
【解析】文章主要讲的是蚂蚁。单个的蚂蚁显得微不足道,但成群结队的蚂蚁却蕴含

着巨大的力量。蚂蚁的数量之众,加起来可以与人类的重量相媲美。蚂蚁分布极为广

泛,除了寒冷的山顶和两极地区,几乎到处都有它们的踪影。
216.B 细 节 理 解 题 。 根 据 第 一 段 “ They also live nearly everywhere, except on frozen

mountain tops and around the poles.”可知答案,结冰的山顶和两极没有蚂蚁。故选 B。

217.B 细节理解题。根据第二段“ Where we use sound and sight to communicate, ants

dependprimarily on pheromone ( 外激素)” 可知,我们使用声音和视觉来交流,而蚂蚁却

使用外激素。故选 B。

218.D 猜测词义题。根据“ attacking in large groups and overcoming their target” 成群结队

地攻击并战胜它们的目标可知,这里指与比自己大的敌人战斗。故选 D。

219.A 推理判断题。根据第一段“ ants have been astonishingly successful, largely due to

their wonderfulsocial behavior” 蚂蚁的成功令人吃惊,这主要归功于它们出色的社交行

为,和最后一段“Behaving in this selfless and devoted manner,...they think as one, they have

a collective ( 集体的) intelligence greater” 可知,蚂蚁之所以能够存活下来,是因为它们

的社会行为。故选 A。

53.(2015 年,福建卷)

Life can be so wonderful, full of adventure and joy. It can also be full of challenges,

setbacks and heartbreaks. Whatever our circumstances, we generally still have dreams, hopes

and desires—that little something more we want for ourselves and our loved ones. Yet

knowing we can have more can also create a problem, because when we go to change the way

we do things, up come the old patterns and pitfalls that stopped us from seeking what we

wanted in the first place.

This tension between what we feel we can have and "what were seemingly able to have

is the niggling suffering, the anxiety we feel. This is where we usually think it's easier to just

give up. But we're never meant to let go of the part of us that knows we can have more. The

intelligence behind that knowing is us—the real us. It's the part that believes in life and its

possibilities. If you drop that, you begin to feel a little "dead" inside because you're dropping

"you".

So, if we have this capability but somehow life seems to keep us stuck, how do we break

these patterns?

Decide on a new course and make one decision at a time. This is good advice for a new

adventure or just getting through today's challenges.

While, deep down, we know we can do it, our mind—or the minds of those close to us—
usually says we can't.

That isn't a reason to stop, it's just the mind, that little man or woman on your shoulder,

trying to talk you out of something again. It has done it many times before. It's all about

starting simple and doing it now.

Decide and act before overthinking. When you do this you may feel a little, or large,

release from the jail of your mind and you'll be on your way.

220.It can be inferred from the first two paragraphs that we should .

A.slow down and live a simple life

B.be careful when we choose to change

C.stick to our dreams under any circumstances

D.be content with what we already have

221.What is the key to breaking the old patterns?

A.To focus on every detail.

B.To decide and take immediate action.

C.To listen to those close to us.

D.To think twice before we act.

222.Which of the following best explains the underlined part in the last paragraph?

A.Escape from your punishment

B.Realization of your dreams.

C.Freedom from your tension.

D.Reduction of your expectations.

223.What does the author intend to tell us?

A.It's easier than we think to get what we want.

B.It's important to learn to accept sufferings in life.

C.It's impractical to change our way of thinking.

D.It's harder than we expect to follow a new course.

【答案】220.C 221.B 222.C 223.A

【解析】本文是一篇说明文,人生充满了挑战和乐趣,同时也充满了挑战和挫折、心

碎。但是不管情况怎么样,我们仍然有梦想和希望。然而知道我们可以拥有更多也是

一个问题,因为那会改变我们做事的方式,会阻止我们在第一时间去追求我们想要的

东西。作者认为如果我们遇到挫折,那就重新选择目标,并且迅速作出决定。
220. 推理判断题。根据文章第一段第三句 Whatever our circumstances, we generally still

have dreams, hopes and desires 说不管在什么情况下,我们仍然要有梦想、希望和欲望。

学科网(北京)股份有限公司
以及第二段最后一句 If you drop that, you begin to feel a little "dead" inside because you're

dropping "you".如果你放弃了,你的内在会变得了无生机,因为你放下了你自己。所以

从前两段可以推断,在任何情况下我们都应该坚持我们的梦想。故选 C

221. 细节理解题。根据倒数第三段第一句 Decide on a new course and make one decision

at a time.可以知道为了打破旧格局需要决定一个新目标并且立刻做出决定。故选 B

222.词句猜测题。根据最后一段“Decide and act before overthinking 可知” 在过多思考

之前快速决定和行动。这样才能从过多考虑的焦虑中解放自己。所以可以推测这个划

线句子是指“要从过多考虑的焦虑中解放自己”。故选 C

223.推理判断题。文章开头部分讲到生活中有很多人遇到挫折后往往会放弃他们所追

求的东西,但作者鼓励我们不要放弃,通过一些简单的做法就可以改变这一消极想法 。

因此可以判断出,作者想告诉我们得到我们想要的东西比我们想象的要容易。故选 A

项。
54.(2015 年,上海卷)

One of the executives gathered at the Aspen Institute for a day-long leadership

workshop using the works of Shakespeare was discussing the role of Brutus in the death of

Julius Caesar. “Brutus was not an honorable man,” he said. “He was a traitor (叛徒) . And

he murdered someone in cold blood.” The agreement was that Brutus had acted with cruelty

when other options were available to him. He made a bad decision, they said—at least as it

was presented by Shakespeare—to take the lead in murdering Julius Caesar. And though one

of the executives acknowledged that Brutus had the good of the republic in mind, Caesar was

nevertheless his superior. “You have to endeavor,” the executives said, “our policy is to obey

the chain of command.”

During the last few years, business executives and book writers looking for a new way

to advise corporate America have been exploiting Shakespeare’s wisdom for profitable ends.

None more so than husband and wife team Kenneth and Carol Adelman, well-known advisers

to the White House, who started up a training company called “Movers and Shakespeares”.

They are amateur Shakespeare scholars and Shakespeare lovers, and they have combined

their passion and their high level contacts into a management training business. They conduct

between 30 and 40 workshops annually, focusing on half a dozen different plays, mostly for

corporations, but also for government agencies.

The workshops all take the same form, focusing on a single play as a kind of case

study, and using individual scenes as specific lessons. In Julius Caesar , sly provocation (狡
诈 的 挑 唆 ) of Brutus to take up arms against the what was a basis for a discussion of

methods of team building and grass roots organism.

Although neither of the Adelmans is academically trained in literature, the

programmes, contain plenty of Shakespeare tradition and background. Their workshop on

Henry V, for example, includes a helpful explanation of Henry’s winning strategy at the Battle

of Agincourt. But they do come to the text with a few biases ( 偏向): their reading of Henry V

minimizes his misuse of power. Instead, they emphasize the story of the youth who seizes

opportunity and becomes a masterful leader. And at the workshop on Caesar, Mr. Adelmans

had little good to say about Brutus, saying “the noblest Roman of them all” couldn’t make his

mind up about things.

Many of the participants pointed to very specific elements in the play that they felt

related Caesar’s pride, which led to his murder, and Brutus’s mistakes in leading the after the

murder, they said, raise vital questions for anyone serving as a business when and ho w do

you resist the boss?

224.According to paragraph 1, what did all the executives think of Brutus?

A.Cruel. B.Superior.

C.Honorable. D.Bade

225.According to the passage, the Adelmans set up “Movers and Shakespeares” to ________.

A.help executives to understand Shakespeare’s plays better

B.give advice on leadership by analyzing Shakespeare’s plays

C.provide case studies of Shakespeare’s plays in literature workshops

D.guide government agencies to follow the characters in Shakespeare’s plays.

226.Why do the Adelmans conduct a workshop on Henry V?

A.To highlight the importance of catching opportunities.

B.To encourage masterful leaders to plan strategies to win.

C.To illustrate the harm of prejudices in management.

D.To warn executives against power misuse.

227.It can be inferred from the passage that ____.

A.the Adelmans’ programme proves biased as the roles of characters are maximized.

B.executives feel bored with too many specific elements of Shakespeare’s plays.

C.the Adelmans will make more profits if they are professional scholars.

D.Shakespeare has played an important role in the management field.

228.The best title for the passage is _____.

学科网(北京)股份有限公司
A.Shakespeare’s plays: Executives reconsider corporate culture

B.Shakespeare’s plays: An essential key to business success

C.Shakespeare’s plays: a lesson for business motivation

D.Shakespeare’s plays: Dramatic training brings dramatic results

【答案】224.A 225.B 226.A 227.D 228.D

【解析】这是一篇说明文。文章介绍了莎士比亚戏剧在商业管理方面的运用。
224.细节理解题 根据第一段 The agreement was that Brutus had acted with cruelty 可知,

所有高管都认为,布鲁图斯表现得很残忍。故选 A 项。

225. 推 理 判 断 题 。 根 据 第 二 段 During the last few years, business executives and book

writers looking for a new way to advise corporate .They are amateur Shakespeare scholars

and Shakespeare lovers, and they have combined their passion and their high level contacts

into a management training business. 在在过去几年,商业执行官和作家们找到了一种新

的方式来劝导行政体系。他们是非专业的莎士比亚研究者和推崇者,并将其高昂的激

情用于商业管理培训项目中。由此判断出根据这篇文章,阿德尔曼夫妇成立了

“Movers and Shakespeares”是通过分析莎士比亚的戏剧给领导层提供建议。故选 B 项。

226. 细 节 理 解 题 根 据 第 二 段 Instead, they emphasize the story of the youth who seizes

opportunity and becomes a masterful leader. 相反,他们强调年轻人的故事。这些年轻人

能够抓住机会以及成为有能力的领导。所以为什么阿德尔曼要举办关于亨利五世的研

讨会是为了强调抓住机会的重要性。故选 A 项。

227. 推理判断题。根据第一段 One of the executives gathered at the Aspen Institute for a

day-long leadership workshop using the works of Shakespeare was discussing the role of

Brutus in the death of Julius Caesar. 公司的一名执行官都汇聚在阿斯彭(Aspen)集团参

加为期一天的领导力研讨,这个研讨会其中一个主题就是讨论莎士比亚作品中布鲁图

(Brutus)对凯撒之死所起到的作用。由此推断出莎士比亚在管理领域也起着重要作用。

故选 D 项。

228.主旨大意题。通读全文可知,介绍了莎士比亚戏剧在商业管理方面的运用短文,

对莎士比亚作品中人物的分析,与商业管理结合在一起,产生了戏剧性的结果。所以

短文的最佳标题是“莎士比亚戏剧:戏剧的训练带来戏剧性的结果”,故选 D 项。

55.(2014 年,北京卷)

What is the first thing you notice when you walk into a shop? The products displayed

(展示) at the entrance? Or the soft background music?

But have you ever notice the smell? Unless it is bad, the answer is likely to be no. But

while a shop’s scent may not be outstanding compared with sights and sounds, it is certainly
there. And it is providing to be an increasing powerful tool in encouraging people to

purchase.

A brand store has become famous for its distinctive scent which floats through the fairly

dark hall and out to the entrance, via scent machines. A smell may be attractive but it may not

just be used for freshening air. One sports goods company once reported that when it first

introduced scent into its stores, customers’ intension to purchase increased by 80 percent.

When it comes to the best shopping streets in Pairs, scent is just as important to a brand’s

success as the quality of its window displays and goods on sales. That is mainly because

shopping is a very different experience to what it used to be.

Some years ago, the focus for brand name shopping was on a few people with sales

assistants’ disproving attitude and don’t-touch-what-you-can’t-afford displays. Now the rise

of electronic commerce (e-commerce) has opened up famous brands to a wider audience. But

while e-shops can use sights and sounds, only bricks-and-mortar stores ( 实体店 ) can offer a

full experience from the minute customers step through the door to the moment they leave.

Another brand store seeks to be much more than a shop, but rather a destination. And scent is

just one way to achieve this.

Now a famous store uses complex man-made smell to make sure that the soft scent of

baby powder floats through the kid department, and coconut ( 椰 子 ) scent in the swimsuit

section. A department store has even opened a new lab, inviting customers on a journey into

the store’s windows to smell books, pots and drawers, in search of their perfect scent.

229.According to the passage, what is an increasingly powerful tool in the success of some

brand store?

A.Friendly assistant. B.Unique scents.

C.Soft background music. D.Attractive window display.

230.E-shops are mentioned in the passage to ______.

A.show the advantages of brick-and-mortar stores

B.urge shop assistants to change their attitude

C.push stores to use sights and sounds

D.introduce the rise of e-commerce

231.The underlined word “destination” in Paragraph 5 means ______.

A.a platform that exhibits goods

B.a spot where travelers like to stay

学科网(北京)股份有限公司
C.a place where customers love to go

D.a target that a store expects to meet

232.The main purpose of the passage is to ______.

A.compare and evaluate B.examine and assess

C.argue and discuss D.inform and explain

【答案】229.B 230.A 231.C 232.D

【解析】这是一篇说明文。本文介绍了现在的很多商店都运用一个新方法:味道,来

吸引顾客,并解释了这样做的原因。这是与网店比较起来实体店的一个优势。
229. 细节理解题。根据文章第三段中 A brand store has become famous for its distinctive

scent which floats through the fairly dark hall and out to the entrance, via scent machines. 可

知,这家商店的成功的原因是店里很有特色的味道。故选 B。

230. 推理判断题。根据第五段中 But while e-shops can use sights and sounds, only bricks-

and-mortar stores ( 实 体 店 ) can offer a full experience from the minute customers step

through the door to the moment they leave. 可知,电子网店里可以使用声音和视觉来吸引

顾客,那么实体店里就可以使用味道来吸引顾客。作者举电子网店是为了说明实体店

也有自己的优势。故选 A。

231. 推理判断题。根据本句 Another brand store seeks to be much more than a shop, but

rather a destination.(另外一家实体店通过种种方法努力让自己不仅仅是一个商店,而

成为人们愿意去的目的地)可推知,划线词是指顾客愿意来的地方。故选 C 项。

232.推理判断题。本文介绍了现在的很多商店都运用一个新方法:味道,来吸引顾客,

并解释了这样做的原因。所以本文的目的是为了告诉我们这样的行为并做出解释。故

选 D。

56.(2014 年,浙江卷)

Last summer, two nineteenth-century cottages were rescued from remote farm fields in

Montana, to be moved to an Art Deco building in San Francisco. The houses were made of

wood. These cottages once housed early settlers as they worked the dry Montana soil; now

they hold Twitter engineers.

The cottages could be an example of the industry’ s odd love affair with “low

technology,” a concept associated with the natural world, and with old-school craftsmanship

( 手艺 ) that exists long before the Internet era. Low technology is not virtual ( 虚拟的 ) —so,

to take advantage of it, Internet companies have had to get creative. The rescued wood

cottages, fitted by hand in the late eighteen-hundreds, are an obvious example, but Twitter’s

designs lie on the extreme end. Other companies are using a broader interpretation ( 阐释 ) of
low technology that focuses on nature.

Amazon is building three glass spheres filled with trees, so that employees can “work

and socialize in a more natural, park-like setting.” At Google’s office, an entire floor is

carpeted in glass. Facebook’s second Menlo Park campus will have a rooftop park with a

walking trail.

Olle Lundberg, the founder of Lundberg Design, has worked with many tech companies over

the years. “We have lost the connection to the maker in our lives, and our tech engineers are

the ones who feel impoverished ( 贫乏的 ), because they’re surrounded by the digital world,”

he says. “They’re looking for a way to regain their individual identity, and we’ve found that

introducing real crafts is one way to do that.”

This craft based theory is rooted in history, William Morris, the English artist and writer,

turned back to pre-industrial arts in the eighteen-sixties, just after the Industrial Revolution.

The Arts and Crafts movement defined itself against machines. “Without creative human

occupation, people became disconnected from life,” Morris said.

Research has shown that natural environments can restore( 恢 复 ) our mental capacities. In

Japan, patients are encouraged to “forest-bathe,” taking walks through woods to lower their

blood pressure.

These health benefits apply to the workplace as well. Rachel Kaplvin, a professor of

environmental psychology, has spent years researching the restorative effects of natural

environment. Her research found that workers with access to nature at the office—even

simple views of trees and flowers—felt their jobs were less stressful and more satisfying. If

low-tech offices can potentially nourish the brains and improve the mental health of

employees then, fine, bring on the cottages.

233.The writer mentions the two nineteenth-century cottages to show that ________.

A.Twitter is having a hard time

B.old cottages are in need of protection

C.early settlers once suffered from a dry climate in Montana

D.Internet companies have rediscovered the benefits of low technology

234.Low technology is regarded as something that _______.

A.is related to nature B.is out of date today

C.consumes too much energy D.exists in the virtual world

235.The main idea of Paragraph 5 is that human beings ________.

学科网(北京)股份有限公司
A.have destroyed many pre-industrial arts

B.have a tradition of valuing arts and crafts

C.can become intelligent by learning history

D.can regain their individual identity by using machines

236.The writer’s attitude to “low technology” can best be described as ________.

A.positive B.defensive C.cautious D.doubtful

237.What might be the best title for the passage?

A.Past Glories, Future Dreams

B.The Virtual World, the Real Challenge

C.High-tech Companies, Low-tech Offices

D.The More Craftsmanship, the Less Creativity

【答案】233.D 234.A 235.B 236.A 237.C

【解析】这是一篇说明文。这篇短文讲述的是关于现代科技公司运用 “大自然对人们

创造力产生作用”的理念,积极创设原生态的办公环境。这里提到的低科技指的是工

业化之前的建造技术,它更贴近自然。在自然的办公环境下,职员们不会感到有压力 ,

对他们的健康是有好处的。
233. 细节理解题。作者提到两个 19 世纪的村舍是为了表明什么?根据短文第二段中

Low technology is not virtual ( 虚拟的) —so, to take advantage of it, Internet companies have

had to get creative 可知,低科技不是虚拟的,因此要充分的利用它,高科技公司需要有

创造力。而短文开头提到的两个 19 世纪的小村舍就是低科技技术下的产物。因此推断

现代的高科技公司也需要用低科技的理念创设原生态的办公室。故选 D。

234. 推理判断题。低科技被认为是什么样的事物?根据下文中作者给出的例子 so that

employees can “work and socialize in a more natural, park-like setting 以及最后一段中 Her

research found that workers with access to nature at the office 可以判断出,低科技跟大自

然有关,即在一个天然的环境下,故选 A。B 选项意思是低科技已经过时了,这不符

合文意,现在很多高科技的公司都在打造低科技的办公室,让职员们更接近自然; C

选项的意思是低科技消耗很多能源,其实正好相反,低科技的东西都是很天然的,不

需要消耗很多能源;D 选项的意思说低科技是虚拟的,而短文中说它并不是虚拟的,

故与文意不符。故选 A。

235. 主旨大意题。第五段主要是说人类怎样?根据这一段的第一句话 This craft based

theory is rooted in history, William Morris, the English artist and writer, turned back to pre-

industrial arts in the eighteen-sixties. 可知,工艺基础理论是有其历史根源的,英国的艺

术家、作家 William Morris 又回到了 19 世纪 60 年代的工业化之前的艺术。也就是人类


有珍视工艺的传统。故选 B。

236.推理判断题。作者对低科技的态度可以用哪一个词来形容?A.积极的;B.辩护的;

C. 谨慎的;D. 怀疑的。通过作者给出的这些例子可以看出,利用低科技建造的原生态

办公室对职员们的工作的积极性是有好处的。并且作者还引用了一项研究 Research has

shown that natural environments can restore( 恢复) our mental capacities 来说明自然环境对

我们精神上的有利影响。故选 A。

237.主旨大意题。这篇短文中作者告诉我们低科技不是虚拟的,现代的高科技公司需

要有创造力,所以要充分的利用以前的低科技技术,创设更加原生态的办公环境。故

用 High-tech Companies, Low-tech Offices 作为题目是最合适的。

57.(2014 年,四川卷)

With around 100 students scheduled to be in that 9:00 am Monday morning lecture, it

is no surprise that almost 20 people actually make it to the class and only 10 of them are still

awake after the first 15 minutes; it is not even a surprise that most of them are still in their

pajama’s( 睡衣 ). Obviously, students are terrible at adjusting their sleep cycles to their daily

schedule.

All human beings possess a body clock. Along with other alerting systems, this governs

the sleep/wake cycle and is therefore one of the main processes which govern sleep

behaviour. Typically, the preferred sleep/wake cycle is delayed in adolescents, which leads to

many students not feeling sleepy until much later in the evenings. This typical sleep pattern is

usually referred to as the “night owl” schedule of sleep.

This is opposed to the “early bird” schedule, and is a kind of disorder where the

individual tends to stay up much past midnight. Such a person has great difficulty in waking

up in the mornings. Research suggests that night owls feel most alert and function best in the

evenings and at night. Research findings have shown that about 20 percent of people can be

classified as “night owls” and only 10 percent can be classified as “early birds” ------ the

other 70 percent are in the middle. Although this is clearly not true for all students, for the

ones who are true night owls, this gives them an excellent excuse for missing their lectures

which unfortunately fall before midday.

238.What does the author stress in Paragraph 1?

A.Many students are absent from class.

B.Students are very tired on Monday mornings.

C.Students do not adjust their sleep patterns well.

学科网(北京)股份有限公司
D.Students are not well prepared for class on Mondays.

239.Which of the following is true according to Paragraph 2?

A.Most students prefer to get up late in the morning.

B.Students don’t sleep well because of alerting systems.

C.One’s body clock governs the sleep/wake cycle independently.

D.Adolescents’ delayed sleep/wake cycle isn’t the preferred pattern.

240.Which of the following is closest in meaning to the underlined word “classified”?

A.Criticized B.Grouped C.Organized D.Named

241.What does the text mainly talk about?

A.Functions of the body clock.

B.The “night owl” phenomenon.

C.Human beings’ sleep behaviour.

D.The school schedule of “early birds”.

【答案】238.C 239.D 240.B 241.B

【解析】本文是说明文。在文中作者通过介绍周一早上上课中学生迟到和准时到的情

况,提出了学生不擅长调整自己的生物钟而去适应白天的学业安排。然后在文中分析

了目前青少年睡眠存在的问题,提出了两种不同的睡眠模式,即“ early bird” (早起

型) 和“ night owl” (夜猫子型),并主要介绍了晚上不困,白天困的“ night owl” 现

象。
238.推理判断题。在文章首段作者首先通过数字说明许多学生早晨不能准时起床的一

种情 况, 然后 用一 句话 Obviously, students are terrible at adjusting their sleep cycles to

their daily schedule. 来解释这种现象的原因,提出文章中心,然后围绕此中心展开行文 ,

故答案选 C。

239. 细 节理 解题 。根 据第 二段 Typically , the preferred sleep / wake cycle is delayed in

adolescents … 可知学生们对他们自己的睡眠模式不熟悉,没有能够恰当地去调整适应

自己的生物钟,因而造成了睡眠问题,故答案选 D。

240.词义猜测题。A 批评;B 分类;C 组织;D 命名。根据上文第三段末句 This typical

sleep pattern is usually referred to as the "night owl" schedule of 和 第 四 段 首 句 This is

opposed to the "early bird" schedule, 可知睡眠模式有两种,即 "night owl" 和"early bird" ,

而此处把 20%归到 night owls 类,由此判断该词意思为“分类”,选 B。

241.主旨大意题。本文以学生星期一早上上课时很多学生难以将他们的睡眠周期调整

到正常为引子,解释了“ night owl”这一睡眠周期现象现象进行分析并介绍造成这种情

况的原因,所以答案选 B。A 选项内容“人体生物钟的功能”,C 选项“人体的睡眠行


为”都比较片面,不能反映文章中心;而 D 选项“早起者的学校时间安排”在文中没

有涉及。
58.(2014 年,四川卷)

Women are friendly.But men are more competitive.Why? Researchers have found it's all

down to the hormone oxytocin (荷尔蒙催生素) .Although known as the love hormone, it

affects the sexes differently.

"Women tend to be social in their behavior.They often share with others.But men lend to

be competitive.They are trying to improve their social status," said Professor Ryan.

Generally, people believe that the hormone oxytocin is let out in our body in various

social situations and our body creates a large amount of it during positive social interactions

(互动) such as falling in love or giving birth.

But in a previous experiment Professor Ryan found that the hormone is also let out in

our body during negative social interactions such as envy.

Further researches showed that in men the hormone oxytocin improves the ability to

recognize competitive relationships, but in women it raises the ability to recognize friendship.

Professor Ryan's recent experiment used 62 men and women aged 20 to 37.Half of the

participants(参与者)received oxytocin.The other half received placebo (安慰剂).

After a week, the two groups switched with participants.They went through the same

procedure with the other material.

Following each treatment, they were shown some video pictures with different social

interactions.Then they were asked to analyze the relationships by answering some

questions.The questions were about telling friendship from competition.And their answers

should be based on gestures, body language and facial expressions.

The results indicated that, after treatment with oxytocin, men's ability to correctly

recognize competitive relationships improved, but in women it was the ability to correctly

recognize friendship that got better.

Professor Ryan thus concluded: "Our experiment proves that the hormone oxytocin can

raise people's abilities to better distinguish different social interactions.And the behavior

differences between men and women are caused by biological factors ( 因 素 ) that are

mainly hormonal."

242.What causes men and women to behave differently according to the text?

A.Placebo. B.Oxytocin.

学科网(北京)股份有限公司
C.The gesture. D.The social status.

243.What can we learn from Professor Ryan's previous experiment?

A.Oxytocin affects our behavior in a different way.

B.Our body lets out oxytocin when we are deep in love.

C.Our body produces oxytocin when we feel unhappy about others' success.

D.Oxytocin improves our abilities to understand people's behavior differences.

244.Why did Professor Ryan conduct the recent experiment?

A.To test the effect of oxytocin on the ability to recognize social interactions.

B.To know the differences between friendship and competition.

C.To know people's different abilities to answer questions.

D.To test people's understanding of body language.

245.The author develops the text by______.

A.explaining people's behaviors

B.describing his own experiences

C.distinguishing sexual differences

D.discussing research experiments

【答案】242.B 243.C 244.A 245.D

【解析】本文是一篇说明文。文章介绍科学家通过实验证明,阐述了激素对于男性和

女性在社交上的不同影响。
242. 细 节 理 解 题 。 第 一 段 第 二 句 Researchers have found it's all down to the hormone

oxytocin 研究者已经发现,男人和女人的行为不同取决于荷尔蒙催生素。故选 B。

243. 细节理解题。文章第四段 But in a previous experiment Professor Ryan found that the

hormone is also let out in our body during negative social interactions such as envy.但是在教

授 Ryan 之前的试验中,在负面的社交活动,例如嫉妒的时候,荷尔蒙也会上升。故选

C。

244.推理判断题。根据倒数第二段 The results indicated that, after treatment with oxytocin,

men's ability to correctly recognize competitive relationships improved, but in women it was

the ability to correctly recognize friendship that got better.结果表明,在催生素治疗之后,

男性正确认识竞争性关系的能力提高了,但是,对于女人来说,正确识别友谊的能力

提高了。可以推断这个实验目的在于测试催生素对于人识别社交活动的能力的影响。

故选 A。

245.推理判断题。本文中举出了多个实验以及实验结构,表明作者是通过讨论研究实

验结果来展开全文的,故选 D。
59.(2014 年,上海卷)

If you could be anybody in the world, who would it be? Your neighbour or a super star?

A few people have experienced what it might be like to step into the skin of another person,

thanks to an unusual virtual reality (虚拟现实) device. Rikke Wahl, an actress, model and

artist, was one of the participants in a body swapping experiment at the Be Another lab, a

project developed by a group of artists based in Barcelona. She swapped with her partner, an

actor, using a machine called The Machine to Be Another and temporarily became a man. "As

I looked down, I saw my whole body as a man, dressed in my partner's pants," she said.

"That's the picture I remember best."

The set-up is relatively simple. Both users wear a virtual reality headset with a camera

on the top. The video from each camera is sent to the other person, so what you see is the

exact view of your partner. If she moves her arm, you see it. If you move your arm, she sees

it.

To get used to seeing another person's body without actually having control of it,

participants start by raising their arms and legs very slowly, so that the other can follow

along. Eventually, this kind of slow synchronised ( 同 步 的 ) movement becomes

comfortable, and participants really start to feel as though they are living in another person's

body.

Using such technology promises to alter people's behaviour afterwards-potentially for

the better. Studies have shown that virtual reality can be effective in fighting racism-the

bias(偏见)that humans have against those who don't look or sound like them. Researchers

at the University of Barcelona gave people a questionnaire called the Implicit Association

Test, which measures the strength of people's associations between, for instance, black people

and adjectives such as good, bad, athletic or awkward. Then they asked them to control the

body of a dark skinned digital character using virtual reality glasses, before taking the test

again. This time, the participants' bias scores were lower. The idea is that once you've "put

yourself in another's shoes" you're less likely to think ill of them, because your brain has

internalised the feeling of being that person.

The creators of The Machine to Be Another hope to achieve a similar result. "At the end

of body swapping, people feel like holding each other in their arms," says Arthur Pointeau, a

programmer with the project. "It's a really nice way to have this kind of experience. I would

really, really recommend it to everyone."

学科网(北京)股份有限公司
246.The word "swapping" (paragraph 1) is closest in meaning to______.

A.building B.exchanging C.controlling D.transplanting

247.We can infer from the experiment at the Be Another lab that______.

A.our feelings are related to our bodily experience

B.we can learn to take control of other people's bodies

C.participants will live more passionately after the experiment

D.The Machine to Be Another can help people change their sexes

248.In the Implicit Association Test, before the participants used virtual reality glasses to

control a dark skinned digital character, ______.

A.they fought strongly against racism

B.they scored lower on the test for racism

C.they changed their behaviour dramatically

D.they were more biased against those unlike them

249.It can be concluded from the passage that______.

A.technology helps people realize their dreams

B.our biases could be eliminated through experiments

C.virtual reality helps promote understanding among people

D.our points of view about others need changing constantly

【答案】246.B 247.A 248.D 249.C

【解析】这是一篇说明文。文章介绍了一种 Be another lab 的体验,可以让人们互换角

色,这样可以一定程度上消除人们的偏见,也可以促进人与人的相互理解。
246. 词句猜测题。根据第一段的最后的描写: She swapped with her partner, an actor,

using a machine called The Machine to Be Another and temporarily became a man. "As I

looked down, I saw my whole body as a man, dressed in my partner's pants," she said. "That's

the picture I remember best." 可知,她和她的搭档,一个演员,用一台叫 The Machine to

Be Another 交换了身体。”她说:“当我往下看时,我看到我整个身体都是男人,穿

着我搭档的裤子。”这是我记得最清楚的照片。所以女演员和男演员互换了角色,故

根据上下文语境可以判断出,swapping 的意思是“交换”,故选 B。

247. 推 理 判 断 题 。 根 据 文 章 第 三 段 的 最 后 一 句 话 : Eventually, this kind of slow

synchronised(同步的)movement becomes comfortable, and participants really start to feel

as though they are living in another person's body. 可知,最终,这种缓慢的同步运动变得

舒适起来,参与者真的开始觉得他们好像生活在另一个人的身体里。从 Be another lab

的体验可以推断出我们的感觉和身体的经验有关,故选 A
248. 细节理解题。根据文章第四段 Studies have shown that virtual reality can be effective

in fighting racism-the bias (偏见) that humans have against those who don't look or sound

like them. 可知,研究表明,虚拟现实可以有效地打击种族主义,即人类对那些长得或

听起来不像他们的人的偏见。所以在 the Implicit Association test 中,在参与者使用虚拟

现实的眼镜控制一个黑色皮肤的人之前,他们对不像他们的人物更有偏见,选 D。

249. 推理判断题。根据文章第三段和第四段 The idea is that once you've "put yourself in

another's shoes" you're less likely to think ill of them, because your brain has internalised the

feeling of being that person. At the end of body swapping, people feel like holding each other

in their arms," says Arthur Pointeau, a programmer with the project 可知,这个想法是,一

旦你“设身处地地处在别人的位置时”,你就不太可能对他们不好,因为你的大脑已

经内化了成为那个人的感觉。在身体交换后,人们想要张开双臂,彼此拥抱。故判断

出虚拟现实帮助促进人们之间的理解,故选 C。

60.(2014 年,山东卷)

Elizabeth Freeman was born about 1742 to African American parents who were slaves.

At the age of six months she was acquired, along with her sister, by John Ashley, a wealthy

Massachusetts slaveholders. She became known as “Mumbet” or “Mum Bett.”

For nearly 30 years Mumbet served the Ashley family. One day, Ashley’s wife tried to

strike Mumbet’s sister with a spade. Mumbet protected her sister and took the blow instead.

Furious, she left the house and refused to come back. When the Ashleys tried to make her

return, Mumbet consulted a lawyer, Theodore Sedgewick. With his help, Mumbet sued( 起诉)

for her freedom.

While serving the Ashleys, Mumbet had listened to many discussions of the new

Massachusetts constitution. If the constitution said that all people were free and equal, then

she thought it should apply to her. Eventually, Mumbet won her freedom---- the first slave in

Massachusetts to do so under the new constitution.

Strangely enough, after the trial, the Ashleys asked Mumbet to come back and work for

them as a paid employee. She declined and instead went to work for Segdewick. Mumbet

died in 1829, but her legacy lived on in her many descendants( 后 裔 ). One of her great-

grandchildren was W.E.B.Du Bois, one of the founder of the NAACP, and an important writer

and spokesperson for African American civil rights.

Mumbet’s tombstone still stands in the Massachusetts cemetery where she was buried. It

reads, in part: “She was born a slave and remained a slave and remained a slave for nearly

学科网(北京)股份有限公司
thirty years. She could neither read nor write, yet in her own sphere she had no superior or

equal.”

250.What do we know about Mumbet according to Paragraph 1?

A.She was born a slave

B.She was a slaveholder

C.She had a famous sister

D.She was born into a rich family

251.Why did Mumbet run away from the Ashleys?

A.She found an employer

B.She wanted to be a lawyer

C.She was hit and got angry

D.She had to take care of her sister

252.What did Mumbet learn from discussions about the new consititution?

A.She should always obey her owners’ orders

B.She should be as free and equal as whites

C.How to be a good servant

D.How to apply for a job

253.What did Mumbet do after the trial?

A.She chose to work for a lawyer

B.She found the NAACP

C.She continued to serve the Ashleys

D.She went to live with her grandchildren

254.What is the test mainly about?

A.A story of a famous writer and spokesperson

B.The friendship between a lawyer and a slave

C.The life of a brave African American woman

D.A trial that shocked the whole world

【答案】250.A 251.C 252.B 253.A 254.C

【解析】这是一片人物传记。本文讲述了一位勇敢的奴隶为了获得自由和平等权而努

力斗争的故事。
250.A 考查细节理解。由第一句“Elizabeth Freeman was born about …who were slaves.”

可知,Mumbet 出生在一个奴隶家庭里,所以她也是一个奴隶。故选 A。

251.C 考查细节理解。由第二段的第三 句“ Mumbet protected her sister…” 和第四句

“Furious, she left the house and refused to come back.” 可知, Mumbet 在奴隶主暴打姐姐
的时候因为保护姐姐而被打,她因为愤怒而离开了奴隶主家。故选 C。

252.B 考查细节理解。由第三段的第二句“ If the constitution said that all…she thought it

should apply to her” 可知, Mumbet 在新宪法的讨论中知道了所有人都是自由和平等的,

故选 B。

253.A 考查细节理解。由第四段可知, Mumbet 在审判之后拒绝了 Ashleys 家的聘用,

去为律师 Sedgewick 工作,故选 A。

254.C 考查主旨要义。通过阅读文章可知,本文讲述的是一位勇敢的奴隶为了获得自由

而做出的努力。故选 C。

61.(2014 年,山东卷)

How fit are your teeth? Are you lazy about brushing them? Never fear: An inventor is on

the case. An electric toothbrush senses how long and how well you brush, and it lets you track

your performance on your phone.

The Kolibree toothbrush was exhibited at the International Consumer Electronics Show

in Las Vegas this week. It senses how it is moved and can send the information to an Android

phone or iPhone via a Bluetooth wireless connection.

The toothbrush will be able to teach you to brush right (don’t forget the insides of the

teeth!) and make sure you’re brushing long enough. “It’s kind of like having a dentist actually

watch your brushing on a day-to-day basis,” says Thomas Serval, the French inventor.

The toothbrush will also be able to talk to other applications on your phone, so

developers could, for instance, create a game controlled by your toothbrush. You could score

points for beating monsters among your teeth. “We try to make it smart but also fun,” Several

says.

Serval says he was inspired by his experience as a father. He would come home from

work and ask his kids if they had brushed their teeth. They said “yes,” but Serval would find

their toothbrush heads dry. He decided he needed a brush that really told him how well his

children brushed.

The company says the Kolibree will go on sale this summer, for $99 to $199, developing

on features. The U.S. is the first target market.

Serval says that one day, it’ll be possible to replace the brush on the handle with a

brushing unit that also has a camera. The camera can even examine holes in your teeth while

you brush.

255.Which is one of the feature of the Kolibree toothbrush?

学科网(北京)股份有限公司
A.It can sense how users brush their teeth.

B.It can track users’school performance.

C.It can detect users’fear of seeing a dentist.

D.It can help users find their phones.

256.What can we learn from Serval’s words in Paragraph 3?

A.You will find it enjoyable to see a dentist.

B.You should see your dentist on a day-to –day basis.

C.You can brush with the Kolibree as if guided by a dentist.

D.You’d like a dentist to watch you brush your teeth every day.

257.Which of the following might make the Kolibree toothbrush fun?

A.It can be used to update mobile phones.

B.It can be used to play mobile phone games

C.It can send messages to other users

D.It can talk to its developers.

258.What is Paragraph 5 mainly about?

A.How Serval found out his kids lied to him.

B.Why Serval thought brushing teeth was necessary.

C.How Serval taught his kids to brush their teeth.

D.What inspired Serval to invent the toothbrush

259.What can we infer about Serval’s children?

A.They were unwilling to brush their teeth

B.They often failed to clean their toothbrushes.

C.They preferred to use a toothbrush with a dry head.

D.They liked brushing their teeth after Serval came home.

260.What can we learn about the future development of the Kolibree?

A.The brush handle will be removed.

B.A mobile phone will be built into it.

C.It will be used to fill holes in teeth

D.It will be able to check users’ teeth

【答案】255.A 256.C 257.B 258.D 259.A 260.D

【解析】本文是一篇说明文。文章介绍了一种新的发明: the Kolibree toothbrush; 介绍

了这种牙刷的特点及功能。以及这个发明的起因和具体的用法。
255. 细节理解题。根据第一段提到 An electric toothbrush senses how long and how well

you brush, and it lets you track your performance on your phone. 可知这种牙刷可以感觉到
牙刷的移动并向手机发射与此有关信息的信号。也就是说这种牙刷可以感觉到用户如

何刷牙。故 A 正确。

256.细节理解题。根据第三段 It’s kind of like having a dentist actually watch your brushing

on a day-to-day basis 可知这种牙刷会让你感觉到有一个牙医在注意着你如何刷牙并指

导你如何刷牙。故 C 正确。

257.推理判断题。根据文章第四段 developers could, for instance, create a game controlled

by your toothbrush. You could score points for beating monsters among your teeth. “We try to

make it smart but also fun 可知还可以用这个来玩游戏,让它具有娱乐性。故 B 正确。

258. 段 落 大 意 题 。 根 据 文 章 第 五 段 Serval says he was inspired by his experience as a

father. He would come home from work and ask his kids if they had brushed their teeth. They

said “yes,” but Serval would find their toothbrush heads dry. 可知作为父亲的他经常发现孩

子不刷牙。所以他想找一个方法来监督孩子刷牙,他才有了发明这样的事物的想法。

故本段是为了说明他有这样的发明的原因,也就是:是什么鼓励着他发明出 The

Kolibree toothbrush。故 D 正确。

259.推理判断题。根据第五段 He would come home from work and ask his kids if they had

brushed their teeth. They said “yes,” but Serval would find their toothbrush heads dry. 可知

Serval 问孩子们有没有刷牙,孩子说“ yes”,但是牙刷还是干的。什么吗孩子没有刷牙。

是因为他们不喜欢刷牙,这是他们不喜欢做的事情。故 A 正确。

260. 推理判断题。根据文章最后一句 The camera can even examine holes in your teeth

while you brush. 可知牙刷上装的相机甚至能够检查你的牙齿上的小洞。说明它可以检

查你的牙齿。故 D 项正确。

62.(2014 年,江西卷)

Close your eyes for a minute and imagine what life would be like if you had a hundred

dollars less.Also imagine what it would be like spending the rest of your life with you eyes

closed.Imagine having to read this page, not with your eyes but with your finger-tips.

With existing medical knowledge and skills, two-thirds of the world’s 42 million blind

should not have to suffer.Unfortunately, rich countries posses most of this knowledge, while

developing countries do not.

ORBIS is an international non-profit organization which operates the world’s only flying

teaching eye hospital.ORBIS intends to help fight blindness worldwide.Inside a DC-8

aircraft, there is a fully-equipped teaching hospital with television studio and

classroom.Doctors are taught the latest techniques of bringing sight back to people

学科网(北京)股份有限公司
there.Project ORBIS also aims at promoting peaceful cooperation(合作) among countries.

ORBIS tries to help developing countries by providing training during three-week

medical programs.ORBIS has taught sight-saving techniques to over 35,000 doctors and

nurses, who continue to cure tens of thousands of blind people every year.ORBIS has

conducted 17 plane programs is China so far.For the seven to ten million blind in China

ORBIS is planning to do more for them.At the moment an ORBIS is working on a long-term

plan to develop a training center and to provide eye care service to Shanxi Province.ORBIS

needs your help to continue their work and free people from blindness.

For just US$38,you can help one person see; for $380 you can bring sight to 10 people;

$1,300 helps teach a doctor new skills; and for $13,000 you can provide a training program

for a group of doctors who can make thousands of blind people see again.Your money can

open their eyes to the world.Please help ORBIS improve the quality of life for so many

people less fortunate than ourselves.

261.The first paragraph is intended to ______.

A.introduce a new way of reading

B.advise the public to lead a simple life

C.direct the public’s attention to the blind

D.encourage the public to use imagination

262.What do we learn about existing medical knowledge and skills in the world?

A.They are adequate

B.They have not been updated.

C.They are not equally distributed

D.They have benefited most of the blind

263.ORBIS aims to help the blind by ______.

A.teaching medical students

B.training doctors and nurses

C.running flying hospitals globally

D.setting up non-profit organization

264.What does the author try to do in the last paragraph?

A.Appeal for donations B.Make an advertisement

C.Promote training programs D.Show sympathy for the blind

265.What can be the best title for the passage?

A.ORBIS in China B.Fighting Blindness

C.ORBIS Flying Hospital D.Sight-saving Techniques


【答案】261.A 262.C 263.B 264.A 265.C

【解析】世界盲人多达 4 千万。为使贫富国家获得均等的医疗知识和技术, ORBIS 组

织举办 flying teaching eye hospital ,并且为之做了许多贡献。要让这个教学机构发挥更

大作用,大家都不免出一份力吧!
261.C 写作意图题。该段有三句话连续用了三个 imagine (imagine what it would be like

spending the rest of your life with you eyes closed. Imagine having to read this page, not with

your eyes but with your finger-tips.),要读者想象一下:闭着眼睛度过余生将会怎样;用

指尖不用眼睛阅读此文又会怎样。作者显然要大家体会一下盲人生活,从而引起读者

阅读下文话题的兴趣。故选 C.

262.C 细 节 理 解 题 。 根 据 文 章 第 二 段 Unfortunately, rich countries posses most of this

knowledge, while developing countries do not 可知当今世界医疗知识如此丰富,医疗技

巧如此发达,4 千万的盲人,2/3 的不应该遭受失明的痛苦。然而医疗知识和技巧的拥

有发达居多,而贫穷国家则无。显然是医疗知识和技巧的分配不均。故选 C.

263.B 细节理解题。根据文章第三段 Doctors are taught the latest techniques of bringing

sight back to people there 可知在 ORBIS 组织中医生被教给使盲人恢复视力的最新技术;

根 据 第 四 段 ORBIS tries to help developing countries by providing training during three-

week medical programs 可知 ORBIS 组织通过提供为期三个星期的培训课程来帮助发展

中国家;且由 ORBIS has taught sight-saving techniques to over 35,000 doctors and nurses

可知 ORBIS 组织已教了三万五千多名医生护士相关的恢复视力技巧。故选 B.

264.A 主旨要义题。根据最后一段列举 US$38/$380/ $1,300 分别可以为盲人恢复视力做

些什么,然后在结束部分提出协助 ORBIS 组织改善盲人境遇的请求。作者显然在向读

者 呼 吁 捐 款 ( Please help ORBIS improve the quality of life for so many people less

fortunate than ourselves.)故选 A.

265.C 大意归纳题。全文先以吸引读者关注盲人入手,然后介绍 ORBIS 组织运营的

flying teaching eye hospital 给全世界培训医生和护士所做出的贡献。 最后倡导大家捐

款来帮助 ORBIS 组织运营的 flying teaching eye hospital 。唯有 C 选项符合文章大意,A

选项以及 B 选项范畴太大,不贴切。故选 C.

63.(2014 年,湖南卷)

The behaviour of a building’s users may be at least as important as its design when it

comes to energy use, according to new research from the UK Energy Research Centre

(UKERC). The UK promises to reduce its carbon emissions ( 排放) by 80 percent by 2050,

part of which will be achieved by all new homes being zero-carbon by 2016. But this report

学科网(北京)股份有限公司
shows that sustainable building design on its own — though extremely important- is not

enough to achieve such reductions: the behaviour of the people using the building has to

change too.

The study suggests that the ways that people use and live in their homes have been

largely ignored by existing efforts to improve energy efficiency ( 效率 ) , whichinstead focus

on architectural and technological developments.

‘Technology is going to assist but it is not going to do everything,’explains Katy Janda, a

UKERC senior researcher , ‘ consumption patterns of building users can defeat the most

careful design. ’In other words,old habits die hard, even in the best-designed eco-home.

Another part of the problem is information. Households and bill-payers don’t have the

knowledge they need to change their energy-use habits. Without specific information , it’s

hard to estimate the costs and benefits of making different choices. Feedback ( 反 馈 )

facilities, like smart meters and energy monitors , could help bridge this information gap by

helping people see how changing their behaviour directly affects their energy use; some

studies have shown that households can achieve up to 15 percent energy savings using smart

meters.

Social science research has added a further dimension ( 方 面 ) , suggesting that

individuals’behaviour in the home can be personal and cannot be predicted 一 whether

people throw open their windows rather than turn down the thermostat ( 恒 温 器 ) , for

example. Janda argues that education is the key. She calls for a focused programme to teach

people about buildings and their own behaviour in them.

266.As to energy use, the new research from UKERC stresses the importance of________.

A.zero-carbon homes

B.the behaviour of building users

C.sustainable building design

D.the reduction of carbon emissions

267.The underlined word “which” in Paragraph 2 refers to”________.”

A.the ways B.their homes C.developments D.existing efforts

268.What are Katy Janda’s words mainly about?

A.The importance of changing building users, habits.

B.The necessity of making a careful building design.

C.The variety of consumption patterns of building users.

D.The role of technology in improving energy efficiency.


269.The information gap in energy use _______.

A.can be bridged by feedback facilities

B.affects the study on energy monitors

C.brings about problems for smart meters

D.will be caused by building users’ old habits

270.What does the dimension added by social science research suggest?

A.The social science research is to be furthered.

B.The education programme is under discussion.

C.The behaviour of building users is unpredictable.

D.The behaviour preference of building users is similar.

【答案】266.B 267.D 268.A 269.A 270.C

【解析】这是一篇科学叙述文,主要叙述来自 UKERC 的最新研究:房屋使用者对能

源使用起着重要影响及介绍研究的内容。
266.B.细节理解题。本题问关于能源使用, UKERC 的新研究强调了什么的重要性。定

位 到 第 一 段 第 一 句 “ The behavior of a building’s users…important…when it comes to

energy use.” 当提及能源使用,房屋使用者的行为至少和房屋的设计一样重要。由此可

知选 B。

267.D.考查定语从句先行词指代的意思。此句是个非限制性定语从句,由 which 引导的

定语从句意思“which 反而关注建筑和科技的进步”可知 which 指代前句中的“existing

efforts”故选 D。

268.A.推理题。考查考生对文中特殊句子的理解。本题问 Katy Janda 话的主要表达什么。

根 据 Katy Janda 定 位 到 第 三 段 , “ Technology is going to assist but is not going to do

everything ,(科技打算帮助可它并不能做每件事) consumption patterns of building

users can defeat the most careful design. (房屋使用者的消费形式可以打败最细心的设

计)”。可知选 A 选项(改变房屋使用者的习惯的重要性)。

269.A. 细节理解题。由“ information gap” 定位到文中第四段第四行,由“ Feedback

facilities could help bridge this information gap ” 可知 A 选项正确。 bridge information gap

意为“填平信息鸿沟”
270.C.推理题。考查考生对语意理解。本题问被社会科学研究加入的方面表明了什么。

结合文中第五段首句“ suggesting that individuals’ behavior in the home can be personal

and cannot be predicted”可知个人在家的行为不可预知。可知 D 选项正确。

64.(2014 年,广西卷)

学科网(北京)股份有限公司
One of the latest trends( 趋 势 ) in American Childcare is Chinese au pairs. Au Pair in

Stamford, Conn, for example, has got increasing numbers of request for Chinese au pairs

from aero to around 4, 000 since 2004. And that’s true all across the country.

“I thought it would be useful for him to learn Chinese at an early age” Joseph Stocke, the

managing director of a company, says of his 2-year old son. “I would at least like to give him

the chance to use the language in the future,” After only six months of being cared by 25-

year-old woman from China, the boy can already understand basic Chinese daily expressions,

his dad says.

Li Drake, a Chinese native raising two children in Minnesota with an American husband,

had another reason for looking for an au pair from China. She didn’t want her children to

miss out on their roots.” Because I am Chinese, my husband and I wanted the children to

keep exposed to(接触) the language and culture.” she says.

“Staying with a native speaker is better for children than simply sitting in a classroom,”

says Suzanne Flynn, a professor in language education of Children. “But parents must

understand that just one year with au pair is unlikely to produce wonders. Complete mastery

demands continued learning until the age of 10 or 12.”

The popularity if au pairs from China has been strengthened by the increasing numbers

of American parents who want their children who want their children to learn Chinese. It is

expected that American demand for au pairs will continue to rise in the next few years.

275.What does the term “au pair” in the text mean?

A.A mother raising her children on her own

B.A child learning a foreign language at home

C.A professor in language education of children

D.A young foreign woman taking care of children.

276.Li Drake has her children study Chinese because she wants them ________.

A.to live in China some day B.to speak the language at home

C.to catch up with other children D.to learn about the Chinese culture

277.What can we infer from the text?

A.Learning Chinese is becoming popular In America

B.Educated woman do better in looking after children

C.Chinese au pairs need to improve their English Skills.

D.Children can learn a foreign language well in six months.

【答案】275.D 276.D 277.A


【解析】这是一篇说明文。文章讲述们目前在美国流行为孩子找一个中国保姆,这会

让孩子及早接触到中文,了解中国文化。这说明在美国学习中文越来越流行了。
275. 词 义 猜 测 题 。 根 据 第 二 段 中 After only six months of being cared by 25-year-old

woman from China, the boy can already understand basic Chinese daily expressions, his dad

says. 以及第四段 Staying with a native speaker is better for children than simply sitting in a

classroom 可知,au pair 是指来自中国的照顾孩子的年轻女性,她们承担这教育孩子中

文的任务。故选 D。

276. 细节理解题。根据第二段中 She didn’t want her children to miss out on their roots.”

Because I am Chinese, my husband and I wanted the children to keep exposed to( 接触 ) the

language and culture.” she says. 可知,她是一个中国人,她和丈夫都希望不要忘根,接

触中文并了解中国文化。故选 D。

277. 推理判断题。根据文章第一句 One of the latest trend( 趋势 ) in American Childcare is

Chinese au pairs. 和最后一段 The popularity if au pairs from china has been strengthened by

the increasing numbers of American parents who want their children who want their children

to learn Chinese. It is expected that American demand for au pairs will continue to rise in the

next few years. 可知,美国人对于学习中文的兴趣越来越大,为孩子寻找一个会讲中文

保姆已经成为了一个趋势。这些都说明学习中文变得很流行了。故选 A。

65.(2014 年,福建卷)

Walt Disney is credited for creating such wonderful things as Donald Duck and Mickey

Mouse. However, he cannot take the credit for creating other well-loved characters, such as

Cinderella and Snow White.Theyare almost automatically associated with Disney because

Disney turned old fables(寓言) into cartoon movies.

The original Cinderella varies very much from the Disney version we know today. It

started off with the girl mourning her mother’s death and going to her tomb three times a day.

In addition, there were only birds that helped Cinderella; there was no such thing as a fairy

godmother or helpful mice, nor was there mention of a horse and carriage.

The stepsisters were cruel: they always threw Cinderella’s food into the ashes of the fire

and made her sleep on the ashes on the floor, hence(因此)her name.

In the original story, the king’s ball actually lasted for three days. With the help of the birds,

the girl, beautifully dressed, danced with the prince on all three nights and the prince fell in

love with her. However, she broke away from him to rush back home each night. On the last

night, the prince placed soothing sticky on the stairs; as Cinderella made her escape, a shoe

学科网(北京)股份有限公司
got stuck on it.

Here now is where the story becomes unpleasant: when the prince went to the house

looking for the girl whose foot fit the shoe, the wicked( 邪恶的 ) stepmother told one of her

two daughters to cut off her big toe to fit into the shoe. The daughter did as told. So the prince

took her away to be his bride. But when they passed the tomb of Cinderella’s mother, the

birds called out to the prince,

“Turn and peep, there’s blood in the shoe; the shoe is too small, the true bride waits for

you.” ZXXK

Realizing he had been tricked, the prince returned the daughter to her mother. The other

then had to cut off part of her heel in order to fit into the shoe, with the same result. Only

Cinderella’s foot fit perfectly and so the prince chose to marry her. The story ends with the

wedding day: as Cinderella’s two stepsisters followed her, pretending to be devoted to her so

that they could enjoy the king’s riches, two birds flew by and plucked( 啄 ) out their eyes.

Because of their wickedness and falsehood, they had to spend the rest of their days blind.

The original Cinderella is so different from the Disney version. Thank goodness Disney

made such changes; it indeed was a wise move.

278.What dose the underlined word “They” in the first paragraph refer to?

A.Such wonderful things.

B.Other well-loved characters.

C.Old fables.

D.Cartoon movies.

279.How did Cinderella get her name?

A.The Birds came up with it.

B.It was given by Disney.

C.It came from the word “ash”.

D.She got it from her mother.

280.Which of the following is TRUE according to the original story?

A.Helpful mice got Cinderella a beautiful dress.

B.The ball was held to celebrate the prince’s wedding.

C.Cinderella left her shoe on the stairs on purpose.

D.The birds told the prince that he had been cheated.

281.The moral of the original story is that ____________.

A.a wicked person cannot escape punishment.


B.a devoted person certainly deserves respect.

C.a well-behaved child earns a great reward.

D.a dishonest child cannot get mother love.

282.What does the author think of the Disney version?

A.Excellent. B.Ordinary C.Dull. D.Ridiculous.

【答案解析】

这是一篇说明文。这篇文章主要讲了迪斯尼创造了许多很有趣的角色,但是有一些深

受喜爱的角色不能归功于迪斯尼,例如灰姑娘。灰姑娘的原始版本与迪斯尼中的灰姑

娘很不同,作者认为迪斯尼版本的灰姑娘非常好。
278.B. 猜词题:前一句提到其他一些深受喜爱的角色不能归功于迪斯尼。所以“ they”

指的是其他一些深受喜爱的角色,故选 B。

279.C 细节题:根据 the stepsisters were cruel: they always threw Cinderella’s food into the

ashes of the fire ,and made her sleep on the ashes on the floor, hence her name 。 可 知

Cinderella 的名词来源于灰土,故选 C。

280.D. 细节题:根据第五段最后一句话 the birds called out to the prince 可知是小鸟们告

诉王子,他被欺骗了,故选 D。

281.A 细节题:根据第六自然段最后一句话 Because of their wickedness and falsehood,

they had to spend the rest of their days blind. 可知最初这个故事的寓意是邪恶的人不能逃

脱惩罚,故选 A。

282.A. 推 断 题 : 根 据 全 文 最 后 一 句 话 The original Cinderella is so different from the

Disney version. Thank goodness Disney made such changes; it indeed was a wise move. 可知,

作者认为迪斯尼版本的灰姑娘非常好,故选 A。

66.(2014 年,安徽卷)

You are the collector in the gallery of your life. You collect. You might not mean to but

you do. One out of three people collects tangible (有形的) things such as cats, photos and

noisy toys.

There are among some 40 collections that are being shown at “The Museum Of”—the

first of several new museums which, over the next two years, will exhibit the objects

accumulated by unknown collectors. In doing so, they will promote a popular culture of

museums, not what museums normally represent.

Some of the collections are fairly common—records, model houses. Others are strangely

beautiful—branches that have fallen from tree, for example. But they all reveal (显露)a lot

学科网(北京)股份有限公司
of things: ask someone what they collect and their answers will tell you who they are.

Other on the way include “The museum of Collectors” and “The Museum of Me.”These

new ones, it is hoped, will build on the success of “The Museum Of.” The thinkers behind the

project want to explore why people collect, and what it means to do so. They hope that

visitors who may not have considered themselves collectors will begin to see they, too,

collect.

Some collectors say they started or stopped making collections at important point: the

beginning or end of adolescence—“it’s a growing-up thing; you stop when you grow up,”says

one. Other painful times are mentioned, such as the end of a relationship. For time and life

can seem so uncontrollable that a steady serial(顺序排列的)arrangement is comforting.

283.How will the new museums promote a popular culture of museums?

A.By collecting more tangible things.

B.By showing what ordinary people have collected.

C.By correcting what museums normally represent.

D.By accumulating 40 collections two years from now.

284.What can be learned about collectors from their collections?

A.Who they are.

B.How old they are.

C.Where they were born.

D.Why they might not mean to collect.

285.Which of the following is an aim of the new museums?

A.To help people sell their collections.

B.To encourage more people to collect.

C.To study the significance of collecting.

D.To find out why people visit museums.

286.According to the last paragraph, people may stop collecting when they

A.become adults

B.feel happy with life

C.are ready for a relationship

D.feel time to he uncontrollable

【答案】
283.B

284.A

285.C
286.A

【解析】文章介绍一些新博物馆,收藏的是普通人的收藏品,他们这么做的目的是想

通过展示普通人收集的东西来促进博物馆文化的,研究收藏的重要性。
283. 细节题:根据文章第二段的第二、三行的句子: the first of several new museums

which, over the next two years, will exhibit the objects accumulated by unknown collectors.

In doing so, they will promote a popular culture of museums, not what museums normally

represent. 可知新的博物馆是通过展示普通人收集的东西来促进博物馆文化的,所以选

B。

284. 细节题:根据文章第三段的最后一个句子: But they all reveal (显露) a lot of

things: ask someone what they collect and their answers will tell you who they are. 可知从人

们的收藏中可以看出他们是什么样的人,选 A

285. 细节题:根据文章第四段第四行的句子: The thinkers behind the project want to

explore why people collect, and what it means to do so. They hope that visitors who may not

have considered themselves collectors will begin to see they, too, collect. 可知新的博物馆是

想探索人们为什么收藏,收藏的意义是什么,所以它们的目的是研究收藏的重要性,

所以选 C

286.细节题:根据文章最后一段第二行的句子:“ it’s a growing-up thing; you stop when

you grow up,”says one.可知人们在成年后会停止收集东西,所以选 A。

67.(2014 年,安徽卷)

Recordings of angry bees are enough to send big, tough African elephants running away,

a new study says. Beehives ( 蜂 窝 )—either recorded or real—may even prevent elephants

from damaging farmer’s crops.

In 2002, scientist Lucy King and her team found that elephants avoid certain trees with

bees living in them. Today, Lucy wants to see if African honeybees might discourage

elephants from eating crops. But before she asked farmer to go to the trouble of setting up

beehives on their farms, she needed to find out if the bees would scare elephants away.

Lucy found a wild beehive inside a tree in northern Kenya and set up a recorder. Then

she threw a stone into the beehive, which burst into life. Lucy and her assistant hid in their car

until the angry bees had calmed down. Next , Lucy searched out elephant families in

Samburu National Reserve in northern Kenya and put a speaker in a tree close to each family.

From a distance, Lucy switched on the pre-recorded sound of angry bees while at the

same time recording the elephants with a video camera. Half the elephant groups left the area

学科网(北京)股份有限公司
within ten seconds. Out of a total of 17 groups, only one group ignored the sound of the angry

bees. Lucy reported that all the young elephants immediately ran to their mothers to hide

under them. When Lucy played the sound of a waterfall ( 瀑布 ) instead of the angry bees to

many of the same elephant families, the animals were undisturbed. Even after four minutes,

most of the groups stayed in one place.

Lucy is now studying whether the elephants will continue to avoid the sound of angry

bees after hearing it several times. She hasn’t tested enough groups yet to know, but her initial

( 最 初 的 ) results were promising enough to begin trials with farmers. She has now begun

placing speakers in the fields to see if elephants are frightened away.

287.We know from the passage that elephants may be frightened of .

A.loud noises B.some crops

C.video cameras D.angry bees

288.As mentioned in the passage, Lucy .

A.works by herself in Africa

B.needs to test more elephant groups

C.has stopped elephants eating crops

D.has got farmers to set up beehives on their farms

289.Why did Lucy throw a stone into a wild beehive?

A.To record the sound of bees.

B.To make a video of elephants.

C.To see if elephants would run away.

D.To find out more about the behavior of bees.

290.Which of the following is true according to the passage?

A.Young elephants ignore African honeybees.

B.Waterfalls can make elephants stay in one place.

C.Elephants do not go near trees with bees living in them.

D.Farmers do not allow Lucy to conduct tests in their fields.

【答案与解析】一项最新的研究表明大象会害怕蜜蜂,为了让农民利用蜜蜂的声音防

止大象破坏庄稼,Lucy 做了实验,实验表明大象对于蜜蜂的声音是畏惧的,但是为了

更有效的证明它的效果,Lucy 还要进一步进行研究。

287.D 细节题:根据文章第一段的第一个句子: Recordings of angry bees are enough to

send big, tough African elephants running away, a new study says. 可知大象可能害怕愤怒的

蜜蜂的声音。选 D
288.B 细节题:根据文章最后一段的第二行的句子: She hasn't tested enough groups yet

to know, but her initial ( 最初的 ) results were promising enough to begin trials with farmers.

可知 Lucy 需要检测更多的大象群,所以选 B

289.A 细 节 题 : 根 据 文 章 第 三 段 的 句 子 : Lucy found a wild beehive inside a tree in

northern Kenya and set up a recorder. Then she threw a stone into the beehive, which burst

into life. 可知 Lucy 向野生蜂窝扔石头,是让它们发疯从而记录下蜜蜂的声音,所以选

290.C 细节题:根据文章第二段的第一句: In 2002, scientist Lucy King and her team

found that elephants avoid certain trees with bees living in them. 可知大象不会靠近有蜜蜂

的树,根据倒数第二段的句子:Lucy reported that all the young elephants immediately ran

to their mothers to hide under them. 可知小象也害怕蜜蜂,根据 When Lucy Played the

sound of a waterfall ( 瀑布 ) instead of the angry bees to many of the same elephant families,

the animals were undisturbed. Even after four minutes, most of the groups stayed in one

place.可知是瀑布的声音让大象呆在同样的地方,不是瀑布,而文章也没有说农民不让

Lucy 在他们的田地做实验。所以选 C。

68.(2014 年,安徽卷)

You may not have heard of Ashoka, but for the past 27 years,this association, founded

by Bill Drayton, has fought poverty ( 贫 穷 )and sickness, promoted education and

encouraged small businesses. To support these worthy causes, Ashoka provides money for the

world's most promising "changemakers" seeking to solve ( 解决) urgent problems and would

like to create a world in which every citizen is a changemaker.

Drayton believes that anyone can become an agent for change. The important thing is to

simply give yourself permission. If you see a problem that you care about, you can help solve

it. The young in particular are willing to accept this concept because at heart every child

wants to grow into a happy, healthy, contributing adult. In fact It is many young people's

ambition to set up programmes or businesses that improve social conditions. An excellent

example is an Ashoka project started in 1995 in Dhaka, which handled the rubbish problem

facing the city ,helped local farmers and provided an income for poor people there .

When Masqsood and Iftekhar began to study the problem of all the uncollected rubbish

that lay in Dhaka’s streets,Attracting tats and disease , they discovered that 80% of it was

natural waste . So they educated the poor people in the city to compost ( 把……制成堆粪)this

waste . They knew that they would have a market for the end product because local farmers

学科网(北京)股份有限公司
were struggling with chemical ferntilisers ( 化肥 ) which were expensive and had reduced the

natural minerals in the soil over the years . At first , they were refused ,but once they were

able to persuade____that there was money to be made , the project took off. In 2009 sales

were $14,000.

Drayton is optimistic that in ten years Ashoka will be making really serious ,practical

progress in bringing about social change by changing the way we look at economic

development.

291.Which of the following could be the best title for the passage?

A.changemakers B.Businessmen

C.Social Conditions D.Rubbish Problem

292.The underlined word "them" in Paragraph 3 probably refers to" "

A.the local farmers B.Masqsood and Iftekhar

C.Drayton and his team D.the poor people in Dhaka

293.It can be concluded from the passage that anyone can become a changemaker if he .

A.considers Drayton's concept

B.gets permission from Ashoka

C.tries to improve social conditions

D.is a young, happy and healthy adult

294.The authors attitude towards Ashoka's program can be described as

A.changing B.forgiving C.cautious D.Positive

【答案】
291.A

292.D

293.C

294.D

【解析】这是一篇说明文。文章介绍了 Bill Drayton 创办的一个组织,旨在对抗贫困疾

病,提高教育,鼓励小产业。并具体介绍了 Ashoka 在 1995 年在 Dhaka 解决垃圾问题

所取得的成就。
291. 主旨大意题:根据文章第一段的内容: Ashoka provides money for the world's most

promising "changemakers" seeking to solve ( 解决 ) urgent problems and would like to create

a world in which every citizen is a changemaker. 可知这篇文章介绍了 Bill Drayton 创办的

一个组织,旨在对抗贫困疾病,提高教育,鼓励小产业,为了让每个人都变成

“changemakers”,故选 A
292. 词义猜测题:根据第三段的内容: So they educated the poor people in the city to

compost (把……制成堆粪)this waste .可知 Ashoka 鼓励当地的穷人将垃圾变成堆肥,可

以卖给购买昂贵的化肥当地农民,有人接受,但是有人拒绝,所以这里 they 指的是

“当地的穷人”。故选 D。

293. 细节理解题:根据文章第二段的第四行的句子: In fact It is many young people's

ambition to set up programmes or businesses that improve social conditions. 可知每个人都能

成为“changemaker”,只要他们努力提高社会状况,故选 C

294.推理判断题:根据文章的内容:可知作者介绍 Ashoka 项目的宗旨,它对于促进社

会进步的做法和措施,并具体介绍了 Ashoka 在 1995 年在 Dhaka 解决垃圾问题所取得

的成就,还有从最后一段可以看出作者对于 Ashoka 这个项目是乐观的,故选 D

69.(2014 年,重庆卷)

The idea of being able to walk on water has long interested humans greatly. Sadly,

biological facts prevent us ever accomplishing such a thing without artificial aid---we simply

weigh too much, and all our mass pushes down through our relatively small feet, resulting in

a lot of pressure that makes us sink.

However, several types of animals can walk on water. One of the most interesting is the

common basilisk Basilicus basilicus, a lizard ( 蜥蜴 )native to Central and South America. It

can run across water for a distance of several meters, avoiding getting wet by rapidly hitting

the water’s surface with its feet. The lizard will take as many as 20 steps per second to keep

moving forward. For humans to do this, we , d need huge feet that we could bring up to our

ears in order to create adequate w hitting. ’’

But fortunately there is an alternative : cornflour. By adding enough of this common

thickening agent to water (and it does take a lot), you can create a “non-Newtonian” liquid

that doesn’t behave like normal water. Now, if the surface of the water is hit hard enough,

particles( 粒子) in the water group together for a moment to make the surface hard. Move

quickly enough and put enough force into each step, and you really can walk across the

surface of an adequately thick Liquid of cornflour.

Fun though all this may sound, it’s still rather messy and better read about in theory than

carried out in practice. If you must do it, then keep the water wings handy in case you start to

sink--and take a shower afterward!

295.Walking on water hasn’t become a reality mainly because humans______.

A.are not interested in it

学科网(北京)股份有限公司
B.have biological limitations

C.have not invented proper tools

D.are afraid to make an attempt

296.What do we know about Basilicus basilicus from the passage?

A.It is light enough to walk on water.

B.Its huge feet enable it to stay above water.

C.It can run across water at a certain speed.

D.Its unique skin keeps it from getting wet in water.

297.What is the function of the cornflour according to the passage?

A.To create a thick liquid.

B.To turn the water into solid.

C.To help the liquid behave normally.

D.To enable the water to move rapidly.

298.What is the author’s attitude toward the idea of humans’ walking on water?

A.It is risky but beneficial.

B.It is interesting and worth trying.

C.It is crazy and cannot become a reality.

D.It is impractical though theoretically possible.

【答案】
295.B

296.C

297.A

298.D

【解析】本文是一篇说明文。人类有一个一直感兴趣但一直未能实现的愿望:水上行

走。不能实现的原因是“生物性局限”。本文主要介绍了一种理论上可行实际上难以

实现的人类水上行走的方法:借助玉米粉(cornflour)增加水面浓度。

295. 细节理解题。从第一段 Sadly, biological facts prevent us ever accomplishing such a

thing without artificial aid …可知,人类至今不能在水面行走是“生物”原因。选择 B。

296. 细 节 理 解 题 。 从 第 二 段 It can run across water for a distance of several meters,

avoiding getting wet by rapidly hitting the water’s surface with its feet. 一句可知,这种蜥蜴

能在水面行走是因为速度快(每秒钟达到 20 步)。选择 C。

297.细节理解题。第三段 this common thickening agent 明确这是一种“常见的增稠剂”。

选择 A。

298. 推理判断题。从第二段 it does take a lot ,最后一段 it’s still rather messy and better
read about in theory than carried out in practice. 等句推断,借助增稠剂实现人类在水面行

走,理论上可行但真正实现很难做到。选择 D 项。

70.(2014 年,浙江卷)

Here is some must-know information from a handbook on how people behave in doing

business in some countries.

In Brazil

Brazilians are warm and friendly. They often stand close when talking and it is common

for them to touch the person on the shoulder. People often greet each other (particularly

women) with light cheek kisses. Schedules tend to be flexible, with business meetings

sometimes starting later than planned. But to be safe, be on time. Meals can stretch for hours

— there’s no such thing as rushing a meal in Brazil. Lunches also can start in the mid to late

afternoon. Brazilians are social, preferring face-to-face communication over mails or phone

calls.

In Singapore

Singaporeans shake hands when they meet and often also greet each other with a small,

polite bow. Business cards should be offered and received with two hands. Arriving late is

considered disrespectful. So be on time. Efficiency( 效 率 ) is the goal, so meetings and

dealings often are fast-paced. Singaporeans are direct in their discussions, even when the

subject is about money. Rank is important and authority is respected. This determines both

people interact in meetings. For example, people avoid disagreeing outright with someone

with a higher rank.

In the United Arab Emirates

In the UAE, status is important, so the most senior or oldest should be greeted first with

their titles. The handshake seems to be longer than elsewhere. So, do not pull away the

handshake. Women should cover themselves when it comes to dress. Men also tend to be

covered from neck to elbows( 肘部 ) and down to the knees. People do not avoid entertaining

in their homes, but they also hold business meals at restaurants. Touching or passing food or

eating with your left hand is to be avoided. When meetings are one-to-one, if your host offers

you coffee, you should refuse. It might seem odd, but it is a cultural tradition. Coffee should

only be accepted if it is always set out or presented.

In Switzerland

The Swiss tend to be formal and address each other by last name. They also are

学科网(北京)股份有限公司
respectful of private lives. You should be careful not to ask about personal topics. Punctuality

( 守 时 ) is vital, something that comes from a deep respect for others’ time. Arrive at any

meeting or event a few minutes early to be safe. They also have clear structure in their

companies. Higher-ups make the final decisions, even if others might disagree. Neat, clean

dress is expected. The Swiss follow formal table manners. They also keep their hands visible

at the table and their elbows off the table. It is polite to finish the food on your plate.

299.The passage is mainly about _____________.

A.communication types

B.the workplace atmosphere

C.customs and social manners

D.living conditions and standards

300.Why do Singaporeans avoid arguing with their boss?

A.They put efficiency in the first place.

B.They dislike face-to-face communication.

C.They want to finish meetings as quickly as possible.

D.They are supposed to obey the person of a higher rank.

301.In the UAE, when should you refuse the coffee if it is offered?

A.When greeting seniors.

B.When meeting the host alone.

C.When attending a presentation.

D.When dining with business partners.

302.In which country do people care about where to put their hands at the dinner table?

A.In Brazil. B.In Singapore.

C.In the United Arab Emirate. D.In Switzerland.

【答案与解析】这是一篇说明文。这篇短文是一本手册上摘录下来的,是关于不同国

家里的风俗习惯和社交礼仪的。在这篇短文中一共出现了四个国家或地区。
299. 主 旨 大 意 题 。 这 篇 短 文 主 要 讲 述 的 什 么 ? 由 第 一 段 “ Here is some must-know

information from a handbook on how people behave in doing business in some countries” 可

知,这里是一个从一本手册上摘录下来的,关于不同的国家中,人们的行为举止是怎

样的。所以第一段就是文章的中心句。故短文主要是关于不同的国家风俗习惯和社交

礼仪。故选 C。

300.细节理解题。为什么新加坡人不会跟老板发生争吵。由 In Singapore

“Rank is important and authority is respected” 可知,在新加坡等级是很重要的,权威的人


物必须受到尊敬。由此我们可以知道,新加坡人不会跟他们的老板争吵。故选 D。

301. 细节理解题。在阿联酋,什么时候你应该拒绝咖啡?根据短文第四段中 When

meetings are one-to-one, if your host offers you coffee, you should refuse.可知,当独自见面

时,如果主人给你咖啡,你应该拒绝。故选 B。

302.细节理解题。在哪个国家人们把手放在餐桌上?根据短文的最后一段“ The Swiss

follow formal table manners. They also keep their hands visible at the table and their elbows

off the table. It is polite to finish the food on your plate.” 可知,在瑞士这个国家里,人们

遵循着正式的餐桌礼仪,他们在吃饭的时候,要把手放在餐桌的上面,胳膊肘放在下

面,而且要吃光盘里的东西。故选 D。

71.(2014 年,湖北卷)

London’s newest skyscraper ( 摩天大楼)is called the Shard and it cost about 430 million

pounds to build. At a height of almost 310 metres, it is the tallest building in Europe. The

Shard has completely changed the appearance of London. However, not everyone thinks that

it is a change for the better.

The Shard was designed by the famous Italian architect Renzo Piano. When he began

designing the Shard for London, Piano wanted a very tall building that looked like a spire ( 尖

顶 )• He wanted the glass surfaces to reflect the sky and the city. The sides of the building

aren’t regular. So the building has an unusual shape. It looks like a very thin ,sharp piece of

broken glass. And that is how the building got the name: the Shard. Piano says that the spire

shape of the Shard is part of a great London tradition. The shape reminds him of the spires of

the churches of London or the tall masts ( 桅 杆 ) of the ships that were once on the river

Thames.

The Shard has 87 floors. At the top, there is an observatory. At the moment the building

is empty, but eventually there will be a five-star hotel. There will also be top quality

restaurants, apartments and offices.

Before building work began, a lot of people didn’t want the Shard though the plans were

approved. Now they are still unhappy about the Shard. Some critics say that such a tall

skyscraper might be good in a city like New York, but not in London. They say that the best

thing about the Shard is its spire shape. But that is the only thing. There is no decoration, only

flat surfaces. The Egyptians did that 4,500 years ago. They also think the Shard is too big for

London. It destroys the beauty of the city.

Other critics don’t like what the Shard seems to represent. They say that the Shard shows

学科网(北京)股份有限公司
how London is becoming more unequal. Only very rich people can afford to buy the

expensive private apartments and stay in the hotel. But the people who live near the Shard are

among the poorest in London. So the Shard seems a symbol of the division in society

between the very rich and the poor.

The Shard now dominates the London skyline. It is not certain, however, that ordinary

London citizens will ever accept it as a valuable addition to the city.

303.London’s newest skyscraper is called the Shard because of ._____

A.its cost

B.its size

C.its shape

D.its height

304.When he designed the Shard, Piano wanted it to _____

A.change London’s skyline

B.inherit London’s tradition

C.imitate the Egyptian style

D.attract potential visitors

305.The critics who refer to social division think the Shard_______ .

A.is only preferred by the rich

B.is intended for wealthy people

C.is far away from the poor area

D.is popular only with Londoners

306.Which would be the best title for the passage?

A.The Shard: Cheers and Claps

B.The Shard: Work of a Great Architect

C.The Shard: New Symbol of London?

D.The Shard: A Change for the Better?

【答案与解析】这是一篇说明文。文章介绍了伦敦新建了欧洲最高的被称之为 “ the

Shard” 的摩天大楼,成为了伦敦的新标志,然而对此却有不同的观点,尤其是对新的

摩天大楼代表的阶层更是有络绎不绝的批评之声。
303.细节理解题。根据文章第二段 The sides of the building aren’t regular. So the building

has an unusual shape. It looks like a very thin , sharp piece of broken glass. And that is how

the building got the name: the Shard. 可知, 这座建筑的侧面不规则。所以这座建筑有一个

不寻常的形状。它看起来像一块非常薄、锋利的碎玻璃。这就是这座建筑得名的原因 。

所以正是因为它的外形新的摩天大楼才被称作为“the Shard”。故选 C 项。
304.细节理解题。根据文章第二段“Piano says that the spire shape of the Shard is part of a

great London tradition ” 可知 Piano 说,Shard 的尖顶形状是伦敦伟大传统的一部分。所

以当 Piano 设计 Shard 时,Piano 希望它能继承伦敦的传统,故选 B 项。

305. 细节理解。根据文章第五段“ Only very rich people can afford to buy the expensive

private apartments and stay in the hotel. …So the Shard seems a symbol of the division in

society between the very rich and the poor.” 可知批评者认为它只是代表了有钱的富人,也

成为了穷人与富人的区分和界限,所以提到社会分化的批评家认为 Shard 是为富人设

计的。故 B 正确。

306.主旨大意题。根据文章第一段 However, not everyone thinks that it is a change for the

better 可知文章是在探究与阐述对新的摩天大楼所带来的变化究竟是怎样的,故利用这

样的话题作为标题很鲜明直接,所以短文的最佳标题为“碎片大厦:更好的改变?”

故 D 正确。

72.(2012 年,上海卷)

Researchers in the psychology department at the University of California at Los Angeles

(UCLA) have discovered a major difference in the way men and women respond to

stress.This difference may explain why men are more likely to suffer from stress related

disorders.

Until now , psychological research has maintained that both men and women have the

same “fightorflight” reaction to stress.In other words , individuals either react with

aggressive behavior , such as verbal or physical conflict (“fight”) , or they react by

withdrawing from the stressful situation (“flight”).However,the UCLA research team found

that men and women have quite different biological and behavioral responses to stress.While

men often react to stress in the fightorflight response , women often have another kind of

reaction which could be called “tend and befriend.” That is , they often react to stressful

conditions by protecting and nurturing their young (“tend”) , and by looking for social

contact and support from others—especially other females (“befriend”).

Scientists have long known that in the fightorflight reaction to stress ,an important role

is played by certain hormones( 激素) released by the body.The UCLA research team suggests

that the female tendorbefriend response is also based on a hormone.This hormone , called

oxytocin , has been studied in the context of childbirth , but now it is being studied for its

role in the response of both men and women to stress.The principal investigator , Dr.Shelley

E.Taylor,explained that “animals and people with high levels of oxytocin are calmer ,more

学科网(北京)股份有限公司
relaxed,more social,and less anxious.” While men also secrete( 分泌) oxytocin,its effects

are reduced by male hormones.

In terms of everyday behavior , the UCLA study found that women are far more likely

than men to seek social contact when they are feeling stressed.They may phone relatives or

friends,or ask directions if they are lost.

The study also showed how fathers and mothers responded differently when they came

home to their family after a stressful day at work.The typical father wanted to be left alone to

enjoy some peace and quiet.For a typical mother , coping with a bad day at work meant

focusing her attention on her children and their needs.

The differences in responding to stress may explain the fact that women have lower

frequency of stressrelated disorders such as high blood pressure or aggressive behavior.The

tendandbefriend regulatory( 调节的) system may protect women against stress ,and this may

explain why women on average live longer than men.

307.The UCLA study shows that in response to stress , men are more likely than women to

______.

A.turn to friends for help

B.solve a conflict calmly

C.find an escape from reality

D.seek comfort from children

308.Which of the following is true about oxytocin according to the passage?

A.Men have the same level of oxytocin as women do.

B.Oxytocin used to be studied in both men and women.

C.Both animals and people have high levels of oxytocin.

D.Oxytocin has more of an effect on women than on men.

309.What can be learned from the passage?

A.Male hormones help build up the body's resistance to stress.

B.In a family a mother cares more about children than a father does.

C.Biological differences lead to different behavioral responses to stress.

D.The UCLA study was designed to confirm previous research findings.

310.Which of the following might be the best title of the passage?

A.How men and women get over stress

B.How men and women suffer from stress

C.How researchers overcome stress problems

D.How researchers handle stress related disorders


【来源】2012 年普通高等学校招生全国统一考试英语(上海卷带解析)

【答案与解析】本篇阅读是关于男女对压力不同反应的研究。
307.C 细节理解题。此题可借用排除法。 A、B、D 皆是讲女性对待压力的特点,故选

308.D 细节理解题。由“ While men also secrete (分泌) oxytocin, its effects are reduced

by male hormones.”可知 Oxytocin 对女性的作用更大。故选 D。

309.C 推理判断题。本篇主要讨论的是男女对待压力的不同反应,并解释了其中的生理

原因。故选 C。

310.A 主旨大意题。根据文章主旨判断,最佳标题应与男女对压力的反应,即如何克服

压力有关,故选 A。

73.(2013 年,江苏卷)

If a diver surfaces too quickly, he may suffer the bends. Nitrogen( 氮) dissolved(溶解) in

his blood is suddenly liberated by the reduction of pressure. The consequence, if the bubbles

( 气 泡 )accumulate in a joint, is sharp pain and a bent body—thus the name. If the bubbles

form in his lungs or his brain, the consequence can be death.

Other air-breathing animals also suffer this decompression( 减压) sickness if they surface

too fast: whales, for example. And so, long ago, did ichthyosaurs. That these ancient sea

animals got the bends can be seen from their bones. If bubbles of nitrogen form inside the

bone they can cut off its blood supply. This kills the cells in the bone, and consequently

weakens it, sometimes to the point of collapse. Fossil ( 化石)bones that have caved in on them

selves are thus a sign that the animal once had the bends.

Bruce Rothschild of the University of Kansas knew all this when he began a study of

ichthyosaur bones to find out how widespread the problem was in the past. What he

particularly wanted to investigate was how ichthyosaurs adapted to the problem of

学科网(北京)股份有限公司
decompression over the 150 million years. To this end, he and his colleagues traveled the

world’s natural-history museums, looking at hundreds of ichthyosaurs from the Triassic

period and from the later Jurassic and Cretaceous periods.

When he started, he assumed that signs of the bends would be rarer in younger fossils,

reflecting their gradual evolution of measures to deal with decompression. Instead, he was

astonished to discover the opposite. More than 15% of Jurassic and Cretaceous ichthyosaurs

had suffered the bends before they died, but not a single Triassic specimen( 标 本 ) showed

evidence of that sort of injury.

If ichthyosaurs did evolve an anti-decompression means, they clearly did so quickly—

and, most strangely, they lost it afterwards. But that is not what Dr Rothschild thinks

happened. He suspects it was evolution in other animals that caused the change.

Whales that suffer the bends often do so because they have surfaced to escape a predator

(捕食动物) such as a large shark. One of the features of Jurassic oceans was an abundance of

large sharks and crocodiles, both of which were fond of ichthyosaur lunches. Triassic oceans,

by contrast, were mercifully shark- and crocodile-free. In the Triassic, then, ichthyosaurs

were top of the food chain. In the Jurassic and Cretaceous, they were prey( 猎物 ) as well as

predator—and often had to make a speedy exit as a result.

311.Which of the following is a typical symptom of the bends?

A.A twisted body. B.A gradual decrease in blood supply.

C.A sudden release of nitrogen in blood. D.A drop in blood pressure.

312.The purpose of Rothschild’s study is to see________ .

A.how often ichthyosaurs caught the bends B.how ichthyosaurs adapted to

decompression

C.why ichthyosaurs bent their bodies D.when ichthyosaurs broke their bones

313.Rothschild’s finding stated in Paragraph 4________ ..

A.confirmed his assumption B.speeded up his research process

C.disagreed with his assumption D.changed his research objectives

314.Rothschild might have concluded that ichthyosaurs ________ .

A.failed to evolve an anti decompression means

B.gradually developed measures against the bends

C.died out because of large sharks and crocodiles

D.evolved an anti decompression means but soon lost it

【答案与解析】本文是说明文,文章介绍了对鱼龙等海洋生物变弯曲的原因的研究。
311.细节理解题。第一段第三句提到遭遇 bend 的症状是"sharp pain and a bent body",此

处 bent 等于 twisted,意为"扭曲的,弯曲的",故答案选 A。

312. 推理判断题。根据第三段第二句 "What he particularly wanted to investigate was how

ichthyosaurs adapted to the problem of decompression over the 150million years." 可 知

Rothschild 教授的研究目的是鱼龙如何适应减压问题,答案为 B。

313. 推 理 判 断 题 。 根 据 第 四 段 第 二 句 "Instead , he was astonished to discover the

opposite."可知研究结果与他原本的假设相反,因此答案为 C。

314. 推 理 判 断 题 。 根 据 文 章 倒 数 第 二 段 第 一 句 If ichthyosaurs did evolve an anti-

decompression means , they clearly did so quickly-and , most strangely , they lost it

afterwards.可以推断出答案,Rothschild 可能已经得出结论:鱼龙类生物未能进化出反

减压器官,故选 A。

74.(2013 年,天津卷)

Guide to Stockholm University Library

Our library offers different types of studying places and provides a good studying

environment.

Zones

The library is divided into different zones.The upper floor is a quiet zone with over a

thousand places for silent reading , and places where you can sit and work with your own

computer.The reading places consist mostly of tables and chairs.The ground floor is the zone

where you can talk.Here you can find sofas and armchairs for group work.

Computers

You can use your own computer to connect to the wifi specially prepared for notebook

computers ; your can also use library computers ; which contain the most commonly used

applications , such as Microsoft Office.They are situated in the area known as the

Experimental Field on the ground floor.

Groupstudy Places

If you want to discuss freely without disturbing others , you can book a study room or

sit at a table on the ground floor.Some study rooms are for 23 people and others can hold up

to 68 people.All rooms are marked on the library maps.

There are 40 groupstudy rooms that must be booked via the website.To book , you need an

active University account and a valid University card.You can use a room three hours per

day,nine hours at most per week.

学科网(北京)股份有限公司
Storage of Study Material

The library has lockers for students to store course literature.When you have obtained at

least 40 credits (学分),you may rent a locker and pay 400 SEK for a year’s rental period.

Rules to be Followed

Mobile phone conversations are not permitted anywhere in the library.Keep your phone

on silent as if you were in a lecture and exit the library if you need to receive calls.

Please note that food and fruit are forbidden in the library ,but you are allowed to have

drinks and sweets with you.

315.The library’s upper floor is mainly for students to________.

A.read in a quiet place

B.have group discussions

C.take comfortable seats

D.get their computers fixed

316.Library computers on the ground floor________.

A.help students with their field experiments

B.contain software essential for schoolwork

C.are for those who want to access the wifi

D.are mostly used for filling out application forms

317.What condition should be met to book a groupstudy room?

A.A group must consist of 8 people.

B.Threehour use per day is the minimum.

C.One should first register at the university.

D.Applicants must mark the room on the map.

318.A student can rent a locker in the library if he________.

A.can afford the rental fee

B.attends certain courses

C.has nowhere to put his books

D.has earned the required credits

319.What should NOT be brought into the library?

A.Mobile phones. B.Orange juice.

C.Candy. D.Sandwiches.

【答案】315.A 316.B 317.C 318.D 319.D

【解析】本文是说明文,主要介绍了斯德哥尔摩大学图书馆的不同区域、图书馆的电

脑、小组学习地点、学习材料的存放和需要遵守的规定。
315. 解析:选 A 。细节理解题。联系文章 Zones 部分中的“ The upper floor is a quiet

zone with over a thousand places for silent reading...” 可知,上层是一个供学生阅读的安静

的地方。
316. 解析:选 B 。细节理解题。从文章 Computers 部分中的“ ...which contain the most

commonly used applications ,such as Microsoft Office.” 可知,在一楼的电脑中包含必备

的办公软件,而对于学生来说,用这些办公软件就是为了做作业。故选 B 项。

317.解析:选 C。推理判断题。从文章 Groupstudy Places 部分中的“To book,you need

an active University account and a valid University card.” 可知,要想在这里预订房间,首

先需要一个激活的大学账号和有效的大学卡片。换言之,就是首先要在大学进行注册 。

故选 C 项。

318.解析:选 D。细节理解题。联系文章 Storage of Study Material 部分中的“When you

have obtained at least 40 credits (学分),you may rent a locker and...” 可知,只有达到了要

求的学分,学生才可以在这里租寄存柜。
319. 解析:选 D 。细节理解题。从文章 Rules to be Followed 部分中的“ Please note that

food and fruit are forbidden in the library...” 可知,食物 sandwiches“ 三明治”是不能带进

图书馆的。文中提到在图书馆内是不能打电话的,但是可以带着手机,把手机调至静

音,故排除 A 项。而 B 项和 C 项属于饮料和糖果,是可以带入图书馆的,故也被排除。

75.(2013 年,全国卷 I)

The baby is just one day old and has not yet left hospital. She is quiet but alert ( 警觉的).

Twenty centimeters from her face researchers have placed a white card with two black spots

on it. She stares at it carefully. A researcher removes the card and replaces it by another, this

time with the spots differently spaced. As the cards change from one to the other, her gaze( 凝

视 ) starts to lose its focus—until a third, with three black spots, is presented. Her gaze

returns: she looks at it for twice as long as she did at the previous card. Can she tell that the

number two is different from three, just 24 hours after coming into the world?

Or do newborns simply prefer more to fewer? The same experiment, but with three spots

shown before two, shows the same return of interest when the number of spots changes.

Perhaps it is just the newness? When slightly older babies were shown cards with pictures of

objects (a comb, a key, an orange and so on), changing the number of objects had an effect

separate from changing the objects themselves. Could it be the pattern that two things make,

as opposed to three? No again. Babies paid more attention to squares moving randomly( 随意

地)on a screen when their number changed from two to three, or three to two. The effect even

学科网(北京)股份有限公司
crosses between senses. Babies who were repeatedly shown two spots became more excited

when they then heard three drumbeats than when they heard just two; likewise( 同样地) when

the researchers started with drumbeats and moved to spots.

320.The experiment described in Paragraph 1 is related to the baby's ________.

A.sense of hearing B.sense of sight

C.sense of touch D.sense of smell

321.Babies are sensitive to the change in ________.

A.the size of cards B.the colour of pictures

C.the shape of patterns D.the number of objects

322.Why did the researchers test the babies with drumbeats?

A.To reduce the difficulty of the experiment.

B.To see how babies recognize sounds.

C.To carry their experiment further.

D.To keep the babies' interest.

323.Where does this text probably come from?

A.Science fiction. B.Children’s literature.

C.An advertisement. D.A science report.

【答案】320.B 321.D 322.C 323.D

【解析】这是一篇说明文。章对刚出生一天的婴儿做实验,通过变化纸上的黑点及鼓

的敲打次数对婴儿的视觉、听觉进行的一个实验 .主要介绍了研究人员通过一些卡片黑

点的数量来检验婴儿对于数量变化的敏感程度,发现当数量相对多的时候,比较容易

引起婴儿的注意,使他们变得更加兴奋。
320.细节理解题。根据文章第一段文中的 She stares it carefully. Her gaze starts to lose its

focus.可知,她仔细地盯着它,目光开始失去焦点。所以本段是对孩子的视觉实验,故

第一段所描述的实验主要是关于视觉的。故选 B 项。

321.细节理解题。根据第一段中的 As the cards change from one to the other ,her gaze(凝

视 )starts to lose its focus—until a third , with three black spots , is presented.Her gaze

returns...可知,随着卡片上黑点数量的变化,婴儿的注视也发生变化,表明了婴儿对于

数量的变化比较敏感,故选 D 项。

322.细节理解题。第二段中研究人员先是通过卡片上的黑点来测验婴儿对于数量的变

化的敏感程度;根据第二段中的 The effect even crosses between senses...three drumbeats

than when they heard just two ; likewise( 同 样 地 )when the researchers started with

drumbeats and moved to spots. 可知,研究人员又做了击鼓的实验,这样做的目的是更加


深入地研究婴儿对于数量变化的敏感程度,故选 C

323.推理判断题。本文提到了 experiment 和 researchers 等,可知是一些研究人员做了一

个实验,来检验婴儿们对于数量的变化是不是敏感,所以本文属于一个科学研究报告 。

故选 D

76.(2013 年,四川卷)

Fear may be felt in the heart as well as in the head, according to a study that has found a

link between the cycles of a beating heart and the chance of someone feeling fear.

Tests on healthy volunteers found that they were more likely to feel a sense of fear at the

moment when their hearts are contracting( 收 缩 ) and pumping blood around their bodies,

compared with the point when the heartbeat is relaxed. Scientists say the results suggest that

the heart is able to influence how the brain responds to a fearful event, depending on which

point it is at in its regular cycle of contraction and relaxation.

Sarah Garfinkel at the Brighton and Sussex Medical School said: “Our Study shows for

the first time that the way in which we deal with fear is different depending on when we see

fearful pictures in relation to our heart.”

The study tested 20 healthy volunteers on their reactions to fear as they were shown

pictures of fearful faces. Dr Garfinkel said, “The study showed that fearful faces are better

noticed when the heart is pumping than when it is relaxed. Thus our hearts can also affect

what we see and what we don’t see 一 and guide whether we see fear.”

To further understand this relationship, the scientists also used a brain scanner( 扫描仪 )

to show how the brain influences the way the heart changes a person’s feeling of fear.

“We have found an importantmechanismby which the heart and brain ‘speak’ to each

other to change our feelings and reduce fear,” Dr Garfinkel said.

“We hope that by increasing our understanding about how fear is dealt with and ways

that it could be reduced, we may be able to develop more successful treatments for anxiety

disorders, and also for those who may be suffering from serious stress disorder.”

324.What is the finding of the study?

A.One’s heart affects how he feels fear.

B.Fear is a result of one’s relaxed heartbeat.

C.Fear has something to do with one’s health.

D.One’s fast heartbeats are likely to cause fear.

325.The study was carried out by analyzing .

学科网(北京)股份有限公司
A.volunteers’ heartbeats when they saw terrible pictures

B.the time volunteers saw fearful pictures and their health conditions

C.volunteers’ reactions to horrible pictures and data from their brain scans

D.different pictures shown to volunteers and their heart—brain communication

326.Which of the following is closest in meaning to “mechanism” in Paragraph 6?

A.Order. B.System.

C.Machine. D.Treatment.

327.This study may contribute to .

A.treating anxiety and stress better.

B.explaining the cycle of fear and anxiety

C.finding the key to the heart-brain communication

D.understanding different fears in our hearts and heads

【答案】324.A 325.C 326.B 327.A

【解析】文章大意:本文属于科普文,介绍了心脏与恐惧的关系。通过两次试验,研

究者发现这两者之间的关系,同时借助这一发现,研究者希望能够减轻或者治疗恐惧

和压力所带来的诸多问题。
324.根据文章第一段第一句 Fear may be felt in the heart as well as in the head, according to

a study that has found a link between the cycles of a beating heart and the chance of someone

feeling fear.可以得出本题答案 A。C 选项表述与本文无关,D 选项表述不正确。

325.根据文章第四段第一句 The study tested 20 healthy volunteers on their reactions to fear

as they were shown pictures of fearful faces. 以及第五段第一句 To further understand this

relationship, the scientists also used a brain scanner( 扫 描 仪 ) to show how the brain

influences the way the heart changes a person's feeling of fear. 可以得出本题答案 C。

326. 从 后 面 的 句 子 : by which the heart and brain ‘speak’ to each other to change our

feelings and reduce fear," 可知大脑和心脏对话来改变我们的感觉和减少恐惧,这是一个

系统,选 B。

327. 主旨题:根据文章最后一段 "We hope that by increasing our understanding about how

fear is dealt with and ways that it could be reduced, we may be able to develop more

successful treatments for anxiety disorders, and also for those for those who may be suffering

from serious stress disorder."可以得出这个研究可以导致更好的治疗焦虑和压力。本题答

案是 A。

77.(2013 年,陕西卷)

According to sociologists( 社会学家 ), every modern industrial society has some form of
social stratification( 阶层 ). Class, power and status are important in deciding people's rank in

society.

Class means a person's economic position in society. A commonly used classification is

lower class, middle class and upper class. While sociologists disagree on how these terms

should be exactly defined, they do describe societies like the United States quite well. One

study shows that 53% of Americans belong to the lower class, 46% the middle class, and 1%

the upper class. Interestingly, a surgeon earning $500,000 a year and a bus driver earning

$50,000 a year both regard themselves as the middle class!

Power refers to the amount of control a person has over other people. Obviously, people in

positions of great power (such as governors) exercise( 行使 )big power, but people who take

orders from others have less power. Power and class do not always go hand in hand, however.

For example, the governor of a state has great power, but he or she may not belong to a

corresponding ( 相应的 )economic class. Generally, however, there is a relationship between

power and class. To our knowledge, there aren't too many people who aren't millionaires in

the U.S. Senate!

Status is the honor or respect attached to a person's position in society. It can also be

affected by power and class, but not necessarily so. For example, a university professor may

have a high status but not belong to a high social class or have a lot of power over others.

328.What can we learn about “the middle class” from Paragraph 2?

A.People earning $50,000 a year belong to the middle class.

B.Nearly half Americans belong to the middle class.

C.People generally consider bus drivers as the middle class.

D.Sociologists have a clear definition of the middle class.

329.According to the text, we know that ________.

A.Power and class do not always correspond with each other

B.Status refers to a person's economic position in society

C.People with high status have a lot of control over others

D.Class is less important in deciding a person's social rank

330.Which of the following shows the structure of the whole text?

学科网(北京)股份有限公司
A. B.

C. D.

【答案】328.B 329.A 330.A

【解析】本文是一篇说明文。阐明了阶级、权力、地位的定义以及三者之间的关系。
328. 细节理解题。由第二段第四句话 46% the middle class 可知美国中产阶级占 46% ,

差不多一半,故 B 项正确。由第五句话 Interestingly, a surgeon earning $500,000 a year

and a bus driver earning $50,000 a year both regard themselves as the middle class!“ 年收入

五十万美元的外科医生和五万美元的公交车司机都自认为是中产阶级”,是“自认

为”,所以 A 、C 项都不对。由第三句话 While sociologists disagree on how these terms

should be exactly defined, 可知社会学家对如何准确定义这些术语存在分歧,D 项错误,

故选 B。

329. 细节理解题。由第三段第三句话 Power and class do not always go hand in hand ,可

知权力和阶级并不总是相辅相成的,A 项正确。由第四段第一句话 Status is the honor or

respect attached to a person's position in society. 可知社会地位是一个人在社会中的地位所

带来的荣誉或尊重,并不是一个人在社会中的经济地位。可知 B 项错误。由第四段最

后一句话 For example, a university professor may have a high status but not belong to a high

social class or have a lot of power over others.可知地位高的人不一定对别人有很大的控制

权, C 项 错误 。由 第一 段第 二 句 话 Class, power and status are important in deciding

people's rank in society.可知阶级、权力和地位是决定人们社会地位的重要因素, D 项错

误。故选 A。

330.推理判断题。第一段总体介绍阶级、权力、地位在确定一个人的社会等级方面都

很重要,第二三四段分别说明了阶级、权力、和地位的定义和其中的关系,采用了总

分结构。故选 A 项。

78.(2013 年,辽宁卷)
Here is an astonishing and signficant fact:Mental work alone can’t make us tired. It

sounds absurd. But a few years ago, scientists tried to find out how long the human could

labor without reaching a stage of fatigue( 疲劳 ). To the amazement of these scientists, they

discovered thett blood passing through the brain, when it is active, shows no fatigue at all! If

we took a drop of blood from a day labourer, we could find it full of fatigue toxins( 毒素) and

fatigue products. But if we took blood from the brain of Albert Einstein, it would show no

fatigue toxing at the end of the day.

So far as the brain is concerned, it can work as well and swiftly at the end of eight or even

twelve hours f efforts as at the beginning. The brain is totally tireless. So what makes us tired.

Some scientists declare that most of our fatigue come from our mental and emotional( 情感

的 ) attitudes. One of England’s most outstanding scientists. J. A.Hadfield,says,“The greater

part of the fatigue from which we suffer is of mental origin. In fact,fatigue of purely physical

origin is rare.” Dr. Brill, a famous American scientist, goes even further. He declares,“One

hundred percent of the fatigue of a sitting worker in good health is due to emotional

problems.”

What kinds of emotions make sitting workers tired?Joy?Satifaction?No!A feeling of being

bored,anger,anxiety,tenseness,worry,a feeling of not being appreciated---those are emotions

that tire sitting workers.Hard work by itself seldom causes fatigue.We get tired because our

emotions produce nervousness in the body.

331.What surprised the scientists a few years ago?

A.Fatigue toxins could hardly be found in a labour’s blood.

B.Albert Eistein didn’t feel worn out after a day’s work.

C.The brain could work for many hours without fatigue.

D.A mental worker’s blood was filled with fatigue toxins.

332.According to the author,which of the following can make sitting worker tired?

A.Challenge mental work.

B.Unpleasant emotions.

C.Endless tasks.

D.Physical labor.

333.What’s the authour’s attitude towards the scientists’ ideas?

A.He agrees with them.

B.He doubts them

学科网(北京)股份有限公司
C.He argues against them.

D.He hesitates to accept them.

334.We can infer from the passage that in order to stay energetic, sitting workers need to

______.

A.have some good blood

B.enjoy their work

C.exercise regularly

D.discover fatigue toxin

【答案】331.C 332.B 333.A 334.B

【解析】本文为说明文。研究发现,单纯的脑力劳动并不会令人感觉疲劳,而真正令

人感觉疲劳的是心理和情感态度,如烦躁、气愤、焦虑、紧张、担心、不被欣赏等。

这些情感都是令脑力劳动者感觉疲劳的因素。
331. 细节理解题。根据第一段中 To the amazement of these scientists, they discovered the

blood passing through the brain, when it is active, shows no fatigue at all!” 可知,令这些科

学家们惊讶的是,大脑活跃时流经大脑的血液并没有表现出疲劳。所以大脑可以连续

工作好几个小时而不感到疲劳。C 项正确。

332. 细节理解题。根据倒数第二段 Some scientists declare that most of our fatigue come

from our mental and emotional( 情感的 ) attitudes. 可知,使脑力劳动者疲劳的主要原因是

情感问题。所以不愉快的情绪会使坐着工作的人感到疲倦。故 B 项正确。

333.推理判断题。通读全文可知,研究发现,单纯的脑力劳动并不会令人感觉疲劳,

而真正令人感觉疲劳的是心理和情感态度,如烦躁、气愤、焦虑、紧张、担心、不被

欣赏等。这些情感都是令脑力劳动者感觉疲劳的因素。根据最后一段 Hard work by

itself seldom causes fatigue. We get tired because our emotions produce nervousness in the

body.可知,努力工作本身很少引起疲劳。我们感到疲劳是因为我们的情绪在体内产生

了紧张感觉。由此判断出作者对科学家的观点持支持的态度。故选 A 项。

334. 推理判断题。根据倒数第二段 He declares, “One hundred percent of the fatigue of a

sitting worker in good health is due to emotional problems.” 可知,他宣称 :“ 一个健康状况

良好的在职员工的疲劳百分之百是由情绪问题造成的。由此我们可以从文章中推断,

为了保持精力充沛,坐着的工人需要喜欢他们的工作。故选 B 项。

79.(2008 年,天津卷)

Sandra Cisneros was born in Chicago in 1954 to a Mexican American family. As the

only girl in a family of seven children, she often felt like she had “seven fathers,” because her

six brothers, as well as her father, tried to control her. Feeling shy and unimportant, she
retreated( 躲 避 ) into books. Despite her love of reading, she did not do well in elementary

school because she was too shy to participate.

In high school, with the encouragement of one particular teacher, Cisneros improved her

grades and worked for the school literary magazine. Her father encouraged her to go to

college because he thought it would be a good way for her to find a husband. Cisneros did

attend college, but instead of searching for a husband, she found a teacher who helped her

join the famous graduate writing program at the University of Iowa. At the university’s

Writers’ Workshop, however, she felt lonely----a Mexican American from a poor

neighborhood among students from wealthy families. The feeling of being so different helped

Cisneros find her “Creative voice.”

“It was not until this moment when I considered myself truly different that my writing

acquired a voice. I knew I was a Mexican woman, but I didn’t think it had anything to do with

why I felt so much imbalance in my life, but it had everything to do with it! That’s when I

decided I would write about something my classmates couldn’t write about.”

Cisneros published her first work, The House on Mango Street, when she was twenty-

nine. The book tells about a young Mexican American girl growing up in a Spanish-speaking

area in Chicago, much like the neighborhoods in which Cisneros lived as a child. The book

won an award in 1985 and has been used in classes from high school through graduate school

level. Since then, Cisneros has published several books of poetry, a children’s book, and a

short-story collection.

338.Which of the following is TRUE about Cisneros in her childhood?

A.She had seven brothers. B.She felt herself a nobody.

C.She was too shy to go to school. D.She did not have any good teachers.

339.The graduate program gave Cisneros a chance to _____.

A.work for a school magazine B.run away from her family

C.make a lot of friends D.develop her writing style

340.According to Cisneros, what played the decisive role in her success?

A.Her early years in college. B.Her training in the Workshop.

C.Her feeling of being different. D.Her childhood experience.

341.What do we learn about The House on Mango Street?

A.It is quite popular among students.

B.It is the only book ever written by Cisneros.

学科网(北京)股份有限公司
C.It wasn’t success as it was written in Spanish.

D.It won an award when Cisneros was twenty-nine.

【答案】338.B 339.D 340.C 341.A

【解析】这是一篇说明文。本篇文章为我们介绍了一个人物——有一个害羞的小女孩

成为知名作家的故事,着重描述了她独特写作风格形成的原因。
338. 细节理解题。由第一段的第三句话 Feeling shy and unimportant, she retreated( 躲避)

into books.可知,她感到害羞,不重要,她躲到书本中。由此可知, Cisneros 在童年时

感到自己是个不重要的人。故选 B 项。

339.细节理解题。由第二段最后一句 The feeling of being so different helped Cisneros find

her “Creative voice.”在大学的文学社,她感到孤独……这种与别人不同的认知帮助她找

到了她的“独创声音”即写作风格。所以研究生课程给了西斯内罗斯一个机会来发展

她的写作风格。故选 D 项。

340. 细 节 理 解 题 。 由 第 三 段 第 一 句 她 It was not until this moment when I considered

myself truly different that my writing acquired a voice 直到我认识到我真的与众不同我的

写作才获得声音。所以她的与众不同在她的成功中扮演着重要的角色。故选 C 项。

341. 推理判断题。由第四段第二句 The book won an award in 1985 and has been used in

classes from high school through graduate school level. 这本书在 1985 年获得了一个奖项,

并在从高中到研究生阶段的课堂上使用。由此判断出 The House on Mango Street 很受学

生的欢迎。故选 A 项。

80.(2012 年,广东卷)

“Have a nice day!” may be a pleasant gesture or a meaningless expression. When my

friend Maxie says “Have a nice day” with a smile, I know she sincerely cares about what

happens to me. I feel loved and secure since another person cares about me and wishes me

well.

“Have a nice day. Next!” This version of the expression is spoken by a salesgirl at the

supermarket who is rushing me and my groceries out the door. The words come out in the

same tone( 腔调 )with a fixed procedure._________________Obviously, the concern for my

day and everyone else’s is the management’s attempt to increase business.

The expression is one of those behaviors that help people get along with each other.

Sometimes it indicates the end of a meeting. As soon as you hear it, you know the meeting is

at an end. Sometimes the expression saves us when we don’t know what to say. “Oh, you just

had a tooth out? I’m terribly sorry, but have a nice day.”

The expression can be pleasant. If a stranger says “Have a nice day” to you, you may
find it heart-warming because someone you don’t know has tried to be nice to you.

Although the use of the expression is an insincere, meaningless social custom at times, there

is nothing wrong with the sentence except that it is a little uninteresting. The salesgirl, the

waitress, the teacher, and all the countless others who speak it without thinking may not really

care about my day. But in a strange and comfortable way, it’s nice to know they care enough

to pretend they care when they really don’t care all that much. While the expression may not

often be sincere, it is always spoken. The point is that people say it all the time when they

like.

342.How does the author understand Maxie’s words?

A.Maxie shows her anxiety to the author.

B.Maxie really wishes the author a good day.

C.Maxie encourages the author to stay happy.

D.Maxie really worries about the author’s security.

343.What does the underlined sentence in Paragraph 2 mean?

A.The salesgirl is rude.

B.The salesgirl is bored.

C.The salesgirl cares about me.

D.The salesgirl says the words as a routine.

344.By saying “Have a nice day,” a stranger may   .

A.try to be polite to you B.express respect to you

C.give his blessing to you D.share his pleasure with you

345.According to the last paragraph, people say “Have a nice day”   .

A.sincerely B.as thanks

C.as a habit D.encouragingly

346.What is the best title of the passage?

A.Have a Nice Day — a Social Custom

B.Have a Nice Day — a Pleasant Gesture

C.Have a Nice Day — a Heart-warming Greeting

D.Have a Nice Day — a Polite Ending of a Conversation

【答案】342.B 343.D 344.A 345.C 346.A

【解析】主要介绍了人们相互之间常说“Have a nice day”这一习惯。

342.细节理解题。由第一段“...I know she sincerely cares about what happens to me.” 及最

后一句可知 Maxie 希望我有美好的一天。故选 B。

学科网(北京)股份有限公司
343.词句理解题。由第二段最后一句可知女售货员说这些话只是例行公事。故选 D。

344.细节理解题。由“...you may find it heartwarming because someone you don't know has

tried to be nice to you.”可知选 A。

345.细节理解题。由第一句和最后一段的最后两句可知,人们常说“ Have a nice day”已

成为一种习惯了。
346.主旨大意题。整篇文章在讲人际交往,社会风俗。

81.(2012 年,广东卷)

Sports account for a growing amount of income made on the sales of commercial time

by television companies. Many television companies have used sports to attract viewers from

particular sections of the general public, and then they have sold audiences to advertisers.

An attraction of sport programs for the major U. S. media companies is that events are

often held on Saturday and Sunday afternoons—the slowest time periods of the week for

general television viewing. Sport events are the most popular weekend programs, especially

among male viewers who may not watch much television at other times during the week. This

means the television networks are able to sell advertising time at relatively high prices during

what normally would be dead time for programming.

Media corporations also use sports to attract commercial sponsors that might take their

advertising dollars elsewhere if television stations did not report certain sports. The people in

the advertising departments of major corporations realize that sports attract made viewers.

They also realize that most business travelers are men and that many men make family

decisions on the purchases of computers, cars and life insurance.

Golf and tennis are special cases for television programming. These sports attract few

viewers, and the ratings( 收视率 )are unusually low. However, the audience for these sports is

attractive to certain advertisers. It is made up of people from the highest income groups in the

United States, to certain advertisers. It is made up of people from the highest income groups

in the United States, including many lawyers and business managers. This is why television

reporting of golf and tennis is sponsored by companies selling high-priced cars, business and

personal computer, and holiday trips. This is also why the networks continue to carry these

programs regardless of low ratings. Advertisers are willing to pay high fees to reach high-

income consumers and those managers who make decisions to buy thousands of “company

cars” and computer, with such viewers, these programs don’t need high ratings to stay on the

air.
347.Television sport programs on weekend afternoons .

A.result in more sport event

B.get more viewers to play sports

C.make more people interested in television

D.bring more money to the television networks

348.Why would weekend afternoons become dead time without sport programs?

A.Because there would be few viewers

B.Because the advertisers would be off work

C.Because television programs would go slowly

D.Because viewers would pay less for watching television

349.In many families, men make decision on .

A.holiday B.sports viewing C.television shopping D.expensive purchases

350.The ratings are not important for golf and tennis programs because .

A.their advertisers are carmakers

B.their viewers are attracted by sports

C.their advertisers target at rich people

D.their viewers can afford expensive cars

351.What is the passage mainly about?

A.Television viewers are determined by male viewers

B.Rich viewers contribute most to television companies

C.Sports are gaining importance in advertising on television

D.Commercial advertisers are the major sponsors of sport event

【答案】347.D 348.A 349.D 350.C 351.C

【解析】本文是说明文,说明运动占据很大一部分商业广告。
347. 细 节 理 解 题 。 由 本 文 第 二 段 This means the television networks are able to sell

advertising time at relatively high prices during what normally would be dead time for

programming.可知。其中 sell high price 与 bring more money 是同义重现。

348.细节理解题。结合 41 题和排除法可以得出答案。其中 BCD 项的表达都不合常理。

349. 细节理解题。由 They also realize that most business travelers are men and that many

men make family decisions on the purchases of computers, cars and life insurance. 可知,其

中 expensive purchases 与 on the purchases of computers, cars and life insurance 属于上下义

重现。
350. 细节理解题。由 It is made up of people from the highest income groups in the United

学科网(北京)股份有限公司
States,可知,其中 highest income 和 rich people 同义重现。此外,此题也可以根据常识

解答:一般而言,高尔夫球是贵族运动,是有钱人的娱乐活动。
351.主旨大意题。根据全文不难得出答案。另外也可以用排除法解答:A 项中电视收视

率只是其中的一个细节,不能作为全文主旨,错在以偏概全。 B 项的 most 和 D 项的

major 一样,都是表达有误。

82.(2012 年,湖北卷)

How is it that siblings ( 兄弟姐妹 ) can turn out so differently? One answer is that in fact

each sibling grows up in a different family. The firstborn is, for a while, an only child, and

therefore has a completely different experience of the parents than those born later. The next

child is, for a while, the youngest, until the situation is changed by a new arrival. The mother

and father themselves are changing and growing up too. One sibling might live in a stable and

close family in the first few years; another might be raised in a family crisis, with a

disappointed mother or an angry father.

Sibling competition was identified as an important shaping force as early as in 1918. But

more recently, researchers have found many ways in which brothers and sisters are a lasting

force in each others’ lives. Dr. Annette Henderson says firstborn children pick up vocabulary

more quickly than their siblings. The reason for this might be that the later children aren’t

getting the same one-on-one time with parents. But that doesn’t mean that the younger

children have problems with language development. Later-borns don’t enjoy that much

talking time with parents, but instead they harvest lessons from bigger brothers and sisters,

learning entire phrases and getting an understanding of social concepts such as the difference

between “I” and “me”.

A Cambridge University study of 140 children found that siblings created a rich world of

play that helped them grow socially. Love-hate relationships were common among the

children. Even those siblings who fought the most had just as much positive communication

as the other sibling pairs.

One way children seek more attention from parents is by making themselves different

from their siblings, particularly if they are close in age. Researchers have found that the first

two children in a family are typically more different from each other than the second and

third. Girls with brothers show their differences to a maximum degree by being more

feminine than girls with sisters. A 2003 research paper studied adolescents from 185 families

over two years, finding that those who changed to make themselves different from their
siblings were successful in increasing the amount of warmth they gained from their parents.

352.The underlined part “in a different family” (in Para. 1) means “_______”.

A.in a different family environment

B.in a different family tradition

C.in different family crises

D.in different families

353.In terms of language development, later-borns ________.

A.get their parents’ individual guidance

B.learn a lot from their elder siblings

C.experience a lot of difficulties

D.pick up words more quickly

354.What was found about fights among siblings?

A.Siblings hated fighting and loved playing.

B.Siblings in some families fought frequently.

C.Sibling fights led to bad sibling relationships.

D.Siblings learned to get on together from fights.

355.The word “feminine” (in Para. 4) means “_______”.

A.having qualities of parents

B.having qualities of women

C.having defensive qualities

D.having extraordinary qualities

【答案】352.A 353.B 354.D 355.B

【解析】同一家庭出身的兄弟姐妹性格为何不同?那是因为他们生活于不同的家庭环

境:对于父母的体验,第一个出生的孩子与以后出生的孩子会迥然不同;在语言发展

上,后出生的孩子更喜欢向哥哥姐姐学习而不是向父母学习……
352.词义猜测题。难度中等。

【解题思路】根据第一段第二句中的“… different experience…”以及第一段最后一句中

“…might live in a stable and close family in the first few years; another might be raised in a

family crisis, with a disappointed mother and angry father.” 可知,该短语意为“生活在不

同的家庭环境中”,故 A 项正确。

353.细节理解题。难度中等。

【解题思路】根据第二段最后一句话中“ Later-borns don’t enjoy that much talking time

with parents, but instead they harvest lessons from bigger brothers and sister…” 可知后出生

学科网(北京)股份有限公司
者更倾向于向兄弟姐妹学习,故 B 项正确。

354.细节理解题。难度中等。

【解题思路】根据第三段最后一句“ Even those siblings who fought the most had just as

much posistive communication as the other sibling pairs.” 可知,即使兄弟姐妹间争吵不断,

他们也在此中学习相处之道,故 D 项正确。

355.词义猜测题。难度中等。

【解题思路】根据语境可知,有兄弟的女孩比有姐妹的女孩更具有女性品质特征,故

B 项正确。

83.(2012 年,湖北卷)

It was a simple letter asking for a place to study at Scotland’s oldest university which

helped start a revolution in higher education. A 140-year-old letter written by a lady calling

for her to be allowed to study medicine at St Andrews University has been discovered by

researchers. Written by Sophia Jex-Blake in 1873, the seven-page document, which urged the

university to allow women to study medicine at the institution, was released yesterday on

International Women’s Day.

The document was discovered buried in the universityarchives( 档 案 ) by part-time

history student Lis Smith, who is completing her PhD at St Andrews Institute of Scottish

Historical Research. She said: “We knew that Sophia Jex-Blake and her supporters, in their

effort to open up university medical education for women, had written to theSenatus

Academicus(校评议委员会) at St Andrews in an attempt to gain permission to attend classes

there, but we didn’t know documentary evidence existed. While searching the archives for

information about the university’s higher certificate for women, I was astonished to come

across what must be the very letter Jex-Blake wrote.”

In the letter, Sophia and her supporters offered to hire teachers or build suitable

buildings for a medical school and to arrange for lectures to be delivered in the subjects not

already covered at St Andrews. Although her letter was not successful, it eventually led to the

establishment of the Ladies Literate in Arts at St Andrews, a distance-learning degree for

women. The qualification, which ran from 1877 until the 1930s, gave women access to

university education in the days before they were admitted as students. It was so popular that

it survived long after women were admitted as full students to St Andrews in 1892.

Ms Jex-Blake went on to help establish the London School of Medicine for Women in

1874. She was accepted by the University of Berne, where she was awarded a medical degree
in January 1877. Eventually, she moved back to Edinburgh and opened her own practice.

356.Sophia wrote a letter to St Andrews University because she wanted _______.

A.to carry out a research project there

B.to set up a medical institute there

C.to study medicine there

D.to deliver lectures there

357.Lis Smith found Sophia’s letter to St Andrews University _______.

A.by pure chance

B.in the school office

C.with her supporters’ help

D.while reading history books

358.Sophia’s letter resulted in the establishment of _______.

A.the London School of Medicine for Women

B.a degree programme for women

C.a system of medical education

D.the University of Berne

359.When did St Andrews University begin to take full-time women students?

A.In 1873. B.In 1874. C.In 1877. D.In 1892.

【答案】356.C 357.A 358.B 359.D

【解析】文章介绍了一封改变女性学习医学历史的信。
356.C 细 节 题 。 根 据 第 一 段 A 140-year-old letter written by a lady calling for her to be

allowed to study medicine at St Andrews University has been discovered by researchers. 可知

她写这封信就是为了想再那里学医。
357.A 细节题。根据第 2 段最后三行 While searching the archives for information about

the university’s higher certificate for women, I was astonished to come across what must be

the very letter Jex-Blake wrote.”可知他找到这封信,纯属偶然。故 A 正确。

358.B 细 节 题 。 根 据 第 三 段 2,3 行 it eventually led to the establishment of the Ladies

Literate in Arts at St Andrews, a distance-learning degree for women.可知 B 正确。

359.D 细节题。根据第三段最后一句 . It was so popular that it survived long after women

were admitted as full students to St Andrews in 1892.可知 D 正确。

83.(2012 年,浙江卷)

Below is a selection about Guinness(吉尼斯) World Records.

Top 6 Unusual Guinness World Records

学科网(北京)股份有限公司
♦ Fastest 100 m running on all fours

The 2008 Guinness World Records Day was, according to CWR, their biggest day of

record-breaking ever, I- h more than 290.000 people taking put in record attempts in 15

different countries. Kenichi Ito's record attempt was port of this special day. He is just another

example of Japanese with "super powers". His "super power" is to run with great speed on all

fours. Kenichi Ito ran 100 m on all fours in 18.58 seconds. The Japanese set this record at

Setagaya Kuritsu Sogo Undojyo, Tokyo, in 2008.

♦ Most people inside a soap bubble

The Discovery Science Center in Santa Ana, Califomia celebrated this year the 15th

anniversary of the Bubble ( 泡 泡 ) Festival. A bubble's math principles and science were

presented and demonstrated at the three-week-long exhibition. The intriguing Bubble Show

was also part of the program. Fan Yang and Deni Yang impressed the audience with their

awesome skills for bubble making. The Yang family cooperated with the Discovery Science

Center to set a new Guinness World Record for mow people inside a scup bubble and they

succeeded.

The family that has been working with soap bubbles for 27 years created a huge soap

bubble and got 118 people inside it. The record was set or. April 4, 2011.

♦ Longest ears on a dog

A bloodhound from Illinois has the longest ears ever measured a dog. The right ear is

13.75 inches long and the left one 13.5 inches. The dog named Tigger earned this title in 2004

and is owned by Christina and Bryan Flessner.

Mr. Jeffries is the previous record holder of this title. Each of his ears measured

approximately 11. 5 inches long. His grandfather used to hold this amazing world record, but

when he died Mr.Jeffries look over.

♦ Most living generations

Did you ever wonder what is the Guinness World Record for most living generation in

one family? Seven is the answer.

The ultimate authority on record-breaking mentions on the website that the youngest

great-great- great-great grandparent of this family was Augusta Bung "aged 109 years 97

days, followed by her daughter aged 89, her granddaughter aged 70, her great grand-daughter

aged 52, her great-great grand-daughter aged 33 and her great-great-great granddaughter aged

15 on the birth of her great-great-great-great grandson on January 21, 1989".

♦ Most T shirts worn at once


Believe it or not, there is a record also for this category. Krunoslav Budiseli set a new

world record on May 22, 2010 for wearing 245 T-shirts at the same time. The nun from

Croatia was officially recognized as the new record bolder by Guinness World Records after

he managed to put on 245 different T-shirts in 1ess than two hours. . The T-shirts weighted 68

KG and Budiseli said he began struggling around T-shirt No. 120. He dethroned the Swedish

Guinness record holder who wore 238 T-shirts.

♦Heaviest pumpkin

Guinness World Records confirmed on October 9. 2010 that a gigantic pumpkin ( 南

瓜 ) grown in Wisconcin was officially the world’s heaviest. It weighed 1,810 pounds 8

ounces and was unveiled by Chris Stevens at the Stillwater Harvest Festival in Minnesota.

Stevens' pumpkin was 85 pounds Javier than the previous re I, another huge pumpkin grown

in Ohio. The proud farmer said his secret is a precise of rain, cow mature, good soil, sea grass

and fish emulsion. Some of the world's heaviest pumpkins, including the record bolder, were

on public display at the Bronx Botanical Gardens in New Yost for a dozen days.

360.Why is Kenichi Ito described «s a man with a "super power"?

A.He set a good example to all Japanese.

B.He made record attempts in 15 different countries.

C.He set a new record for "Fastest 100 m running on all fours".

D.He participated in the 2008 Guinness World Records Day activities.

361.Jeffries is the name of .

A.the owner of the dog with the longest ears

B.the grandfather of the dog with the longest care

C.the present holder of the record for "Longest care on a dog"

D.the former holder of the record for " longest care on a dog'

362.How many T-shirts had Krunoslav Budiseli put on before he felt it difficult to go on?

A.68. B.120. C.238. D.245.

363.According to the given information. which Guinness World Record was most recently

set?

A.The record for "Most people inside a soup bubble".

B.The record for "Most living generations'".

C.The record for "Most T-shirts worn at once".

D.The record for " Heaviest pumpkin".

【答案】360.C 361.D362.B 363.A

学科网(北京)股份有限公司
【解析】能够上吉尼斯纪录是多么了不起的事情!那我们就来了解一下吉尼斯世界纪

录的前六名吧,看看究竟有什么奇迹发生呢?
360. 细节理解题。根据第一部分的倒数第二句: His "super power" is to run with great

speed on all fours. Kenichi Ito ran 100 m on all fours in 18.58 seconds.可知答案 C 符合文意。

361. 细 节 理 解 题 。 根 据 第 三 部 分 的 标 题 Longest ears on a dog 和 第 二 段 的 开 头 Mr.

Jeffries is the previous record holder of this title. 可知答案 D 符合文意。

362. 细节理解题。根据第五部分的倒数第二句 The T-shirts weighted 68 KG and Budiseli

said he began struggling around T-shirt No. 120.可知答案 B 符合文意。

363.细节理解题。根据文章中讲述的 6 大吉尼斯世界纪录的时间提示,可以得知 答案

A,The record was set or. April 4, 2011.是最新的时间,故符合文意。

84.(2012 年,浙江卷)

Easter( 复活节 ) is still a great day for worship, randy in baskets and running around the

yard finding eggs, but every year it gets quite a bit worse for bunnies.

And no, not because the kids like to pull their ears. Theculpritis climate change, and

some researchers found that rising temperatures arc having harmful effects on at least five

species of rabbit in the US.

Take the Lower Keys March rabbit, for instance. An endangered species that lives in the

Lower Florida Keys, this species of cottontail is a great swimmer — it lives on the islands! —

but it is already severely affected by development and now by rising levels. According to the

Center for Biological Diversity, an ocean level rise of only 0. 6 meters will send these guys

jumping to higher ground and a 0.9-meter rise would wipe out their habitat ( 栖 息 地 )

completely.

The snowshoe hare, on the other hand, has a color issue. Most of these rabbits change

their fur color from white in the wintertime to brown in the summer, each designed to give

them better cover from predators (捕食者) . As the number of days with snow decreases

all across the country, however, more and more bunnies arc being left in white fur during

brown dirt days of both fall and spring, making them an easier mark for predators.

Researchers know that the color change is controlled by the number of hours of sunlight, but

whether the rabbit will be able to adapt quick enough to survive is a big question. The

National Wildlife Federation has reported that hunters have noticed their numbers are already

markedly down.

American pikas or rock rabbits, a relative of rabbits and hares, might be the firs' of these

species to go extinct due to climate change. About 7-8 inches long, pikas live high in the cool,
damp mountains west of the Rocky Mountains. As global temperatures rise, they would

naturally migrate (迁徙) to higher ground — but they already occupy the mountaintops. They

can't go any higher. The National Wildlife Federation reports that they might not be able to

stand the new temperatures as their habitat beats up.

The volcano rabbit has the same problem. These rabbits live on the slopes of volcanoes

in Mexico, and recent studies have shown that the lower range of their habitat has already

shifted upward about 700 meters, but there are not suitable plants for them to move higher, so

they are stuck in the middle. Scientists are concerned about their populations.

Native to the US, pygmy rabbits weigh less than 1 pound and live in the American West.

They are believed to be the smallest rabbits in the world. Their habitats have been destroyed

by development. Several populations, such as the Columbia Basin pygmy, almost went

extinct and were saved by zoo breeding programs. Pygmy rabbits also rely on winter cover by

digging tunnels through the snow to escape predators, but lesser snowfall is leaving them

exposed.

All of this gives new meaning to dressing up in a giant bunny costume this Easter.

364.The writer mentions Easter at the beginning of the passage in order to_______.

A.show the importance of Easter Day

B.introduce the issue about bunnies

C.remind people of Easter traditions

D.discuss the relationship between Easter and bunnies

365.The word "culprit" ( Paragraph 2) is closest in meaning to______.

A.criminal B.judge C.victim D.producer

366.According to the passage, some rabbits can now be easily Uncovered by predators

because they_______

A.are exposed lo more skillful hunters

B.have moved to habitats with fewer plants

C.haven't adapted themselves to climate change

D.can't change their fur color into white in the fall and the spring

367.The problem faced by volcano rabbets and rock rabbits is that________.

A.both are affected by 1ess snow

B.both are affected by rising sea levels

C.neither can find enough food

D.neither can migrate to higher places

学科网(北京)股份有限公司
368.Which best describes the writer's tone in the postage?

A.Approving. B.Concerned. C.Enthusiastic. D.Doubtful.

【答案】364.B 365.A 366.C 367.D 368.B

【解析】每年复活节可爱兔子给大家带来了多少欢乐,然而随着全球温暖趋势的加剧 ,

这种给我们带来的愉悦兔子也不可避免地受到了一定的影响。不久的将来,复活节上

兔子的服饰也会因此而有所变化。
364.目的意图题。根据第一段在全文中的作用可以看出,本段主要目的是导入主题,

即:关于兔子的问题。
365. 词义猜测题。根据文章第一段最后一句后半句: but every year it gets quite a bit

worse for bunnies. 和第二段的第一句: And no, not because the kids like to pull their ears.

可知答案 A,罪过,符合文意。

366.细节理解题。根据第三段的整体理解和倒数第二句 but whether the rabbit will be able

to adapt quick enough to survive is a big question.的总结,可知答案 C 符合文意。

367. 细节理解题。根据文章倒数第三段的第三句 they would naturally migrate ( 迁徙 ) to

higher ground — but they already occupy the mountaintops. They can't go any higher. 和倒数

第二段第二句的后半部分 but there are not suitable plants for them to move higher, 可知,

故答案 D 符合文意。

368. 作者态度题。根据文章的整体理解,可知推断出作者对兔子这一问题很是关切

(concerned),故答案 B 符合文意。 Approving 赞同的; Enthusiastic 热情的; Doubtful 质

疑的,均不符合语境, 故排除。
85.(2012 年,全国卷 II)

You may think that sailing is a difficult sport, but it is really not hard to learn it. You do

not need to be strong. But you need to be quick. And you need to understand a few basic rules

about the wind.

First, you must ask yourself, “Where is the wind coming from? Is it coming from ahead

or behind or from the side?” You must think about this all the time on the boat. The wind

direction tells you what to do with the sail.

Let’s start with the wind blowing from the behind. This means the wind and the boat are

going in the same direction. Then you must always keep the sail outside the boat. It should be

at a 90° angle (角度) to the boat. Then it will catch the wind best.

If the wind is blowing from the side, it is blowing across the boat. In this case, you must

keep the sail half way outside the boat. It should be at a 45° angle to the boat.Itneeds to be out

far enough to catch the wind, but it shouldn’t flap ( 摆动). It shouldn’t look like on a flagpole.
If it is flapping, it is probably out too far, and the boat will slow down.

Sailing into the wind is not possible. If you try, the sail will flap and the boat will stop.

You may want to go in that direction. It is possible, but you can’t go in a straight line. You

must go first in one direction and then in another. This is called tacking. When you are

tacking, you must always keep the sail inside the boat.

369.What should you consider first while sailing?

A.Sailors’ strength. B.Wave levels.

C.Wind directions. D.Size of sails.

370.What does the word “It” underlined in Paragraph 4 refer to?

A.The boat. B.The wind. C.The sail. D.The angle.

371.What do you have to do when sailing against the wind?

A.Move in a straight line. B.Allow the sail to flap.

C.Lower the sail. D.Tack the boat.

372.Where can you probably find the text?

A.In a popular magazine. B.In a tourist guidebook.

C.In a physics textbook. D.In an official report.

【答案】369.C 370.C 371.D 372.A

【解析】本文主要介绍了帆船运动的一些常识。就帆船顺风、侧风和逆风行驶进行了

详细的阐述。
369. 细 节 题 : 从 第 二 段 的 句 子 : “ Where is the wind coming from? Is it coming from

ahead or behind or from the side?”可知是先要考虑风向。选 C

370.猜词题:从前面的句子: you must keep the sail half way outside the boat 可知这里的

it 指代风帆。选 C

371. 细节题:从最后一段的句子: You must go first in one direction and then in another.

This is called tacking.可知选 D

372.文章出处题:从文章整体看是给大家一些帆船的知识,应该出自流行杂志。选 A

86.(2012 年,全国卷 II)

Cold weather can be hard on pets, just like it can be hard on people. Sometimes

owners forget that their cats are just as used to the warm shelter ( 住所 ) as they are. Some

owners will leave their animals outside for a long period of time, thinking that all animals

are used to living outdoors. This can put their pets in danger of serious illness. There are

things you can do to keep your animal warm and safe.

学科网(北京)股份有限公司
Keep your pets inside as much as you can when the weather is bad. If you have to

take them out, stay outside with them. When you’re cold enough to go inside, they

probably are too. If you must leave them outside for a long time, make sure they have a

warm, solid shelter against the wind, thick bedding, and plenty of non-frozen water.

If left alone outside, dogs and cats can be very smart in their search for warm shelter.

They can dig into snow banks or hide somewhere. Watch them closely when they are left

outdoors, and provide them with shelter of good quality. Keep an eye on your pet’s water.

Sometimes owners don’t realize that a water bowl has frozen and their pet can’t get

anything to drink. Animals that don’t have clean and unfrozen water may drink dirty water

outside, which may contain something unhealthy for them.

373.What do we learn about pets from Paragraph 1?

A.They are often forgotten by their owners. B.They are used to living outdoors.

C.They build their own shelters. D.They like to stay in warm places.

374.Why are pet owners asked to stay with their pets when they are out in cold weather?

A.To know when to bring them inside. B.To keep them from eating bad food.

C.To help them find shelters. D.To keep them company.

375.If pets are left on their own outdoors in cold weather, they may .

A.be short of clean water B.dig deep holes for fun

C.dirty the snow nearby D.get lost in the wild

376.What is the purpose of this text?

A.To solve a problem. B.To give practical advice.

C.To tell an interesting story. D.To present a research result.

【答案】373.D 374.A 375.A 376.B

【解析】试题分析:本文是一篇说明文。文章给养宠物的人们提供了一些实用的建议 。

天气恶劣的时候,把宠物放在家里;天气寒冷的时候,注意让宠物有干净的水喝。
373. 考查判断推理。根据“ Sometimes owners forget that their cats are just as used to the

warm shelter (住所) as they are.” 可知,猫等宠物也像人一样习惯于暖和的住所。所以 D

正确。
374. 考查细节理解。根据第二段中的“ If you have to take them out, stay outside with

them. When you’re cold enough to go inside, they probably are too.” 可知:当你感到冷的时

候就会进房间,它们也是如此。所以 A 正确。

375.考查细节理解。根据文章第三段中的“ a water bowl has frozen and their pet can’t get

anything to drink”可知,当水盆被冻之后,宠物就可能找不到干净的水喝。故选 A。
376. 考查作者的写作意图。根据第一段的中心句“ There are things you can do to keep

your animal warm and safe.” 可知,这是一篇说明文,其目的是为了给养宠物的人们一些

实用的建议。
87.(2012 年,全国卷)

Honey( 蜂 蜜 )from the African forest is not only a kind of natural sugar, it is also

delicious. Most people, and many animals, like eating it. However, the only way for them to

get that honey is to find a wild bees' nest( 巢)and take the honey from it. Often, these nests are

high up in trees, and it is difficult to find them. In parts of Africa, though, people and animals

looking for honey have a strange and unexpected helper 一 a little bird called a honey guide.

The honey guide does not actually like honey, but it does like the wax ( 蜂 蜡 ) in the

beehives (蜂房). The little bird cannot reach this wax, which is deep inside the bees’ nest. So,

when it finds a suitable nest, it looks for someone to help it. The honey guide gives a loud cry

that attracts the attention of both passing animals and people. Once it has their attention, it

flies through the forest, waiting from time to time for the curious animal or people as it leads

them to the nest. When they finally arrive at the nest,______reaches in to get at the delicious

honey as the bird patiently waits and watches. Some of the honey, and the wax, always falls

to the ground, and this is when the honey guide takes its share.

Scientists do not know why the honey guide likes eating the wax, but it is very

determined in its efforts to get it. The birds seem to be able to smell wax from a long distance

away. They will quickly arrive whenever a beekeeper is taking honey from his beehives, and

will even enter churches when beeswax candles are being lit.

382.Why is it difficult to find a wild bees' nest?

A.It's small in size.

B.It's hidden in trees.

C.It's covered with wax.

D.It's hard to recognize.

383.What do the words "the follower" in Paragraph 2 refer to?

A.A bee. B.A bird.

C.A honey seeker. D.A beekeeper.

384.The honey guide is special in the way________.

A.it gets its food

B.it goes to church

学科网(北京)股份有限公司
C.it sings in the forest

D.it reaches into bees' nests

385.What can be the best title for the text?

A.Wild Bees

B.Beekeeping in Africa

C.Wax and Honey

D.Honey-Lover's Helper

【答案】382.B 383.C 384.A 385.D

【解析】382.细节题,由第一段 these nests are high up in trees 可知蜂巢藏在树上,很难

被发现。故选 B.

383.细节题,此处描述的是 honey guide 找到蜂巢然后大声鸣叫来吸引那些寻找蜂巢的

人或动物帮助它们打开蜂巢,所以 the follower 是指寻找蜂巢的人或动物,即 A honey

seeker。

384.推理题,最后一段是对 honey guide 是怎样找到蜂巢然后怎样得到食物的,可推测

出它们寻找食物的方式很特别。故选 A。

385.推理题,文章主要描述了 honey guide 是如何帮助那些喜欢蜂蜜的人或动物找到蜂

巢的,所以 D 项最符合题意。

88.(2012 年,全国卷)

Are you looking for some new and exciting places to take your kids to? Try some of

these places:

• visit art museums. They offer a variety of activities to excite your kids’ interest. Many

offer workshops for making hand-made pieces, traveling exhibits, book signings( 签名) by

children’ s favorite writers, and even musical performances and other arts.

• Head to a natural history museum. This is where kids can discover the past from

dinosaur models to rock collections and pictures of stars in the sky. Also, ask what kind of

workshops and educational programs are prepared for kids and any special events that are

coming up.

• Go to a Youtheater. Look for one in your area offering plays for children and family

visitors. Pre-show play shops are conducted by area artists and educators where kids can

discover the secret about performing arts. PuPPet ( 木 偶) making and stage make-up are

just a couple of the special offerings you might find.

• Try hands-on science. Visit one of the many hands-on science museums around the

Country • those science play-lands are great fun for kids and grown-ups alike. They will keep
your child mentally and physically active the whole day through while pushing buttons,

experimenting, and building. When everyone is tired, enjoy a fun family science show,

commonly found in these museums.

386.If a child is interested in the universe, he probably will visit .

A.a Youtheater B.an art museum

C.a natural history museum D.a hands-on science museum

387.What can kids do at a Youtheater?

A.Look at rock collections. B.See dinosaur models.

C.Watch puppet making. D.Give performances.

388.What do the words “hands-on science'' mean in the last paragraph?

A.Science games designed by kids. B.Learning science by doing things.

C.A show of kids' science work. D.Reading science books.

389.Whore does this text probably come from?

A.A science textbook. B.A tourist map.

C.A museum guide. D.A news report.

【答案】386.C 387.C 388.B 389.C

【解析】这是一篇说明文。文章介绍了孩子会感兴趣的一些博物馆以及博物馆里一些

孩子会感兴趣的地方和博物馆中孩子可以参加的活动的介绍。
386.细节理解题。由 Head to a natural history museum. “This is where kids can discover the

past from dinosaur( 恐龙 ) models to rock collections and pictures of stars in the sky” 可知,

在这里,孩子们可以发现过去,从恐龙模型到岩石收藏和天空中星星的图片。所以如

果一个孩子对宇宙感兴趣,他可能会去一个自然历史博物馆。故选 C 项。

387. 细节理解题。由 Go to a Youtheater“Puppet( 木偶 )making and stage make-up are just a

couple of the special offerings you might find” 可知,木偶制作和舞台化妆是你可能会发

现的一些特别的项目。所以孩子们在 Youtheater 可以看到木偶制作。故选 C 项。

388. 词句猜测题。由最后一段“ They'll keep your child mentally and physically active the

whole day through while pushing buttons, experimenting, and building” 可知,他们会让你

的孩子一整天都保持精神和身体上的活跃,孩子们在按钮,进行实验和建造东西。所

以孩子们是通过动手做东西学习科学。故通过上下文可以判断出,最后一段中的

“hands-on science”是“通过动手做东西学习科学”。故选 B 项。

389. 推理判断题。由第一段 “ Are you looking for some new and exciting places to take

your kids to? Try some of these places” 可知,你是否正在寻找一些新的和有趣的地方要

学科网(北京)股份有限公司
带你的孩子去呢?试试这些地方。以及下文对博物馆的具体介绍可以判断出,这篇文章

来自于博物馆指南。故选 C 项。

89.(2012 年,山东卷)

The Pacific island nation of Nauru used to be a beautiful place. Now it is an ecological

disaster area. Nauru’s heartbreaking story could have one good consequence — other

countries might learn from its mistakes.

For thousands of years, Polynesian people lived the remote island of Nauru, far from

western civilization. The first European to arrive was John Fearn in 1798. He was the British

captain of the Hunter, a whaling ship. He called the island Pleasant Island.

However, because it was very remote, Nauru had little communication with Europeans at

first. The whaling ships and other traders began to visit, bringing guns and alcohol. These

elements destroyed the social balance of the twelve family groups on the island. A ten-year

civil war started, which reduced the population from 1,400 to 900.

Nauru’s real troubles began in 1899 when a British mining company discovered

phosphate( 磷 酸 盐 )on the island. In fact, it found that the island of Nauru was nearly all

phosphate, which a very important fertilizer for farming. The company began mining the

phosphate.

A phosphate mine is not a hole in the ground; it is a strip mine. When a company strip-

mines, it removes the top layer of soil. Then it takes away the material it wants. Strip mining

totally destroys the land. Gradually, the lovely island of Nauru started to look like the moon.

In 1968, Nauru became one of the richest countries in the world. Every year the

government received millions and millions of dollars for its phosphate.

Unfortunately, the leaders invested the money unwisely and lost millions of dollars. In

addition, they used millions more dollars for personal expenses. Soon people realized that

they had a terrible problem — their phosphate was running out. Ninety percent of their island

was destroyed and they had nothing. By 2000, Nauru was financially ruined. Experts say that

it would take approximately $433,600,000 and more than 20 years to repair the island. This

will probably never happen.

390.What might be the author's purpose in writing the text?

A.To seek help for Nauru's problems.

B.To give a warning to other countries.

C.To show the importance of money.

D.To tell a heartbreaking story of a war.


391.What was Nauru like before the Europeans came?

A.Rich and powerful. B.Modern and open.

C.Peaceful and attractive. D.Greedy and aggressive.

392.The ecological disaster in Nauru resulted from _______.

A.soil pollution B.phosphate overmining

C.farming activity D.whale hunting

393.Which of the following was a cause of Nauru's financial problem?

A.Its leaders misused the money.

B.It spent too much repairing the island.

C.Its phosphate mining cost much money.

D.It lost millions of dollars in the civil war.

394.What can we learn about Nauru from the last paragraph?

A.The ecological damage is difficult to repair.

B.The leaders will take the experts'words seriously.

C.The island was abandoned by the Nauruans.

D.The phosphate mines were destroyed.

【答案】390.B 391.C 392.B 393.A 394.A

【解析】文章是一篇说明文。主要讲述了太平洋岛国瑙鲁如何从一个环境优美的天堂

岛变成一个生态失衡、满目疮痍的小岛。
390. 推 理 判 断 题 。 由 文 章 第 一 段 的 Now it is an ecological disaster area. Nauru's

heartbreaking story could have one good consequence—other countries might learn from its

mistakes.(现在它是一个生态灾区。瑙鲁令人心碎的故事可能会有一个好的后果——其

他国家可能会从它的错误中吸取教训。 )可以看出,作者的目的是给当前很多只注重发

展经济而忽略环境保护的国家一个警告。故选 B。

391.推理判断题。从第一段提到的“太平洋上的国家瑙鲁岛曾经是一个美丽的地方”

看出,瑙鲁岛 attractive;由第二段第一句以及第三段第一句可知,上千年来,瑙鲁岛

的人们生活在偏远的小岛上,过着平静的生活。故选 C。

392. 细节理解题。由最后一段的 Soon people realized that they had a terrible problem -

their phosphate was running out. Ninety percent of their island was destroyed and they had

nothing.可以看出“磷酸盐逐渐减少,开采殆尽”。由此可以看出,“磷酸盐的过量开

采”导致了生态灾难。故选 B。

393. 细节理解题。由最后一段的“ Unfortunately, the leaders invested the money unwisely

and lost millions of dollars. In addition, they used millions more dollars for personal

学科网(北京)股份有限公司
expenses. ”看出,是领导人不正确使用资金导致了财政问题。故选 A。

394. 推 理 判 断 题 。 文 章 最 后 告 诉 我 们 Experts say that it would take approximately

$433,600,000 and more than 20 years to repair the island ,也就是说:瑙鲁岛上的生态损

失弥补起来需要大量的资金和时间,即选项所说的“修复起来很难”。故选 A。

90.(2012 年,北京卷)

Decision-making under Stress

A new review based on a research shows that acute stress affects the way the brain

considers the advantages and disadvantages, causing it to focus on pleasure and ignore the

possible negative (负面的) consequences of a decision.

The research suggests that stress may change the way people make choices in

predictable ways.

“Stress affects how people learn,” says Professor Mara Mather. “People learn better

about positive than negative outcomes under stress.”

For example, two recent studies looked at how people learned to connect images( 影像 )

with either rewards or punishments. In one experiment, some of the participants were first

stressed by having to give a speech and do difficult math problems in front of an audience; in

the other, some were stressed by having to keep their hands in ice water. In both cases, the

stressed participants remembered the rewarded material more accurately and the punished

material less accurately than those who hadn’t gone through the stress.

This phenomenon is likely not surprising to anyone who has tried to resist eating cookies

or smoking a cigarette while under stress –at those moments, only the pleasure associated

with such activities comes to mind. But the findings further suggest that stress may bring

about a double effect. Not only are rewarding experiences remembered better, but negative

consequences are also easily recalled.

The research also found that stress appears to affect decision-making differently in men

and women. While both men and women tend to focus on rewards and less on consequences

under stress, their responses to risk turn out to be different.

Men who had been stressed by the cold-water task tended to take more risks in the

experiment while women responded in the opposite way. In stressful situations in which risk-

taking can pay off big, men may tend to do better, when caution weighs more, however,

women will win.

This tendency to slow down and become more cautious when decisions are risky might

also help explain why women are less likely to become addicted than men: they may more
often avoid making the risky choices that eventually harden into addiction.

395.We can learn from the passage that people under pressure tend to ______.

A.keep rewards better in their memory

B.recall consequences more effortlessly

C.make risky decisions more frequently

D.learn a subject more effectively

396.According to the research, stress affects people most probably in their ______.

A.ways of making choices B.preference for pleasure

C.tolerance of punishments D.responses to suggestions

397.The research has proved that in a stressful situation, ______.

A.women find it easier to fall into certain habits

B.men have a greater tendency to slow down

C.women focus more on outcomes

D.men are more likely to take risks

【答案】395.A 396.A 397.D

【解析】本文是一篇说明文。文章介绍了在压力之下人们作出决定时的考虑是不一样

的,并通过实验说明人们在有压力的时候通常想到的是积极的一面比较多,同时也介

绍了男人和女人在面对不同的压力时反映也是不同的。
395. 推 理 判 断 题 。 根 据 文 章 第 二 段 “ People learn better about positive than negative

outcomes under stress.( 人们在压力之下记住更多的是积极方面而不是消极后果。 )”和第

三 段 “ In both cases, the stressed participants remembered the rewarded material more

accurately and the punished material less accurately than those who hadn’t gone through the

stress.( 在以上两个案例中,受到压力的这些参与者相对于那些没有经历压力的人来说

记得更多更清楚的是奖励的物资而不是惩罚。 )”由此可知,人们在压力下往往会更好

地记住奖励。故选 A。

396.细节理解题。根据文章的标题及第一自然段的总结可知压力影响的是人们做出决

定的方式,即如何作出选择。第一自然段为文章的中心, 一项新的研究表明压力影响

了人们对有利条件和不利条件的考虑方式,导致了人们更多的去思考好的一面而忽视

了消极的一面,故选 A。

397. 细节理解题。根据文章最后一段“ Men who had been stressed by the cold-water task

tended to take more risks in the experiment while women responded in the opposite way. ”实

验中,在受到冰冷的水的压力之下男士趋向于冒更大的险,而女士的反映则相反,故

学科网(北京)股份有限公司
选 D。

91.(2012 年,江西卷)

For those who make journeys across the world, the speed of travel today has turned the

countries into a series of villages.Distances between them appear no greater to a modern

traveler than those which once faced men as they walked from village to village. Jet planes

fly people from one end of the earth to the other, allowing them a freedom of movement

undreamt of a hundred years ago.

Yet some people wonder if the revolution in travel has gone too far. A price has been

paid, they say, for the conquest ( 征服 ) of time and distance._______________________( 忍

受 ). The boat offers leisure and time enough to appreciate the ever-changing sights and

sounds of a journey. A journey by train also has a special charm about it. Lakes and forests

and wild, open plains sweeping past your carriage window create a grand view in which time

and distance mean nothing. On board a plane, however, there is just the blank blue of the sky

filling the narrow window of the airplane. The soft lighting, in-flight films and gentle music

make up the only world you know, and the hours progress slowly.

    Then there is the time spent being “processed” at a modern airport. People are

conveyed like robots along walkways; baggage is weighed, tickets produced, examined and

produced yet again before the passengers move to another waiting area. Journeys by rail and

sea take longer, yes, but the hours devoted to being “processed” at departure and arrival in

airports are luckily absent. No wonder, then, that the modern high-speed trains are winning

back passengers from the airlines.

    Man, however, is now a world traveler and cannot turn his back on the airplane. The

working lives of too many people depend upon it; whole new industries have been built

around its design and operation. The holiday maker, too, with limited time to spend, patiently

endures the busy airports and limited space of the flight to gain those extra hours and even

days, relaxing in the sun. speed controls people’s lives; time saved, in work or play, is the

important thing—or so we are told. Perhaps those first horsemen, riding free across the wild,

open plains, were enjoying a better world than the one we know today. They could travel at

will, and the clock was not their master.

398.What does the writer try to express in Paragraph 1?

A.Travel by plane has speeded up the growth of villages.

B.The speed of modern travel has made distances relatively short.


C.The freedom of movement has helped people realize their dreams.

D.Man has been fond of traveling rather than staying in one place.

399.How does the writer support the underlined statement in Paragraph 2?

A.By giving instructions.

B.By analyzing cause and effect.

C.By following the order of time.

D.By giving examples.

400.According to Paragraph 3, passengers are turning back to modern high-speed trains

because______.

A.they pay less for the tickets

B.they feel safer during the travel

C.they can enjoy higher speed of travel

D.they don’t have to waste time being “processed”

401.What does the last sentence of the passage mean?

A.They could enjoy free and relaxing travel.

B.They needed the clock to tell the time.

C.They preferred traveling on horseback.

D.They could travel with their master.

402.What is the main idea of the passage?

A.Air travel benefits people and industries.

B.Train Travel has some advantages over air travel.

C.Great changes have taken place in modern travel.

D.The high speed of air travel is gained at a cost.

【答案】398.B 399.D 400.D 401.A 402.D

【解析】在现代,人们可以乘坐飞机环游世界,体验飞机带来的高速度。然而,也有

人怀疑,旅行方式的发展是否有些过度了。因为,在体验高速度的同时,人们也付出

了比较大的代价。
398. 根据第一段“ the speed of travel today has turned the countries into a series of villages.

Distances between them appear no greater...” 可知,第一段讲的是,现代化的旅行大大缩

小了世界的距离,故选 B。

399. 根据第二段“ The boat offers leisure and time ...A journey by train also has a special

charm about it. ...Lakes and forests and wild, open plains sweeping past” 可知,作者举例来

论证“旅行是享受的而不是忍受的过程”,故选 D。

学科网(北京)股份有限公司
400. 根 据 第 三 段 “ Then there is the time spent being “processed” at a modern

airport. ...baggage is weighed, tickets produced, examined and produced yet again before the

passenger move again to another waiting area. ...but the hours devoted to being “processed”

at departure and arrival in airports are luckily absent.” 可知,第三段讲的是,乘坐飞机时,

大量的时间被浪费在了排队、检票、候机这些事情上,故选 D。

401. 根据最后一段“ Perhaps those first horsemen, riding free across the wild, open plains,

were enjoying a better world than the one we know today. They could travel at will, and the

clock was not their master.” 可知,骑马在广阔的草原上游荡,欣赏到的会是一个更美好

的世界。骑马的人可以任意驰骋,而不用担心时间问题。故选 A。

402. 根据第一段“ For those who make journeys across the world, the speed of travel today

has turned the countries into a series of villages.” 和第二段“ Yet some people wonder if the

revolution in travel has gone too far. A price has been paid, they say, for the conquest (征

服) of time and distance.” 可知,在获得高速度的同时,人们也付出了一定的代价,故

选 D。

92.(2012 年,江西卷)

Big Brothers Big Sisters is based on the simplicity and power of friendship. It is a

program which provides friendship and fun by matching vulnerable young people(ages 7-17)

with a volunteer adult who can be both a role model and a supportive friend.

   Volunteer tutors come from all walks of life—married, single, with or without children.

Big Brothers and Big Sisters are not replacement parents or social workers. They are tutors:

someone to trust, to have fun with, to talk and go to when needed.

A Big Sister and Little Sister will generally spend between one and four hours together three

or four times each month for at least twelve months. They enjoy simple activities such as a

picnic at a park, cooking, playing sport or going to a football match. These activities improve

the friendship and help the young person develop positive self-respect, confidence and life

direction.

Big Brothers Big Sisters organizations exist throughout the world. It is the largest and

most well-known provider of tutor services internationally and has been operating for 25

years.

Emily and Sarah have been matched since 2008. Emily is a 10-year-old girl who has

experienced some difficulties being accepted by her schoolmates at school. “ I was pretty sure

there was something wrong with me.”


Emily’s mum came across Big Brothers Big Sisters and thought it would be of benefit to

Emily by “providing different feedback ( 反 馈 ) about herself other than just relying on

schoolmates to measure her self-worth.

Sarah wanted to get involved in a volunteer program. “I googled it and found out how to be a

part of it. I thought it would be fun for me to get involved in making time to do something

because sometimes it is all work and no play.”

Big Brothers Big Sisters has been of great benefit and enjoyment to both Emily and Sarah.

They love and look forward to their time together and the partnership has certainly helped

Emily be more comfortable in being the wonderful, happy and unique girl she is!

403.What is the aim of Big Brothers Big Sisters?

A.To offer students public services.

B.To help students improve their grades.

C.To organize sport activities for young people.

D.To provide partnership and fun for young people.

404.A volunteer is usually expected to work within a year for at least______.

A.24 hours B.36 hours C.48 hours D.72 hours

405.According to Emily’s mother, this program may provide Emily with______.

A.advice from her teachers

B.a new way to assess herself

C.a new way to judge her schoolmates

D.more comments from her schoolmates

406.Why did Sarah want to get involved in the program?

A.She used to be a volunteer.

B.She needed a part-time job.

C.She felt a bit bored with her life.

D.She wanted to get a challenging job.

407.According to the passage, “vulnerable young people” are probably those who are

_________.

A.popular at school B.rather weak physically

C.easily hurt emotionally D.confident in themselves

【答案】403.D 404.B 405.B 406.B 407.C

【解析】 Bigger Brothers Bigger Sisters (公益项目)存在 25 年且遍及世界各地。其宗

旨是通过老少联谊活动,加强友谊。从而帮助脆弱的 7—17 岁未成年人增强自信,把

学科网(北京)股份有限公司
握人生方向。Emily 就是其中的一个受益者。

403. 细节理解题 根据第一段的第二句话“ It is a program which provides friendship and

fun by matching vulnerabile young people (7—17) with a volunteer aldult…” ,可以得知该

项目宗旨是通过老少联姻活动,加强友谊获得乐趣。故选 D。

404.细节理解题 根据第三段第一句话可知,该项目的志愿者与未成年人相处的时间,

一年中每月 3—4 次,每次 1—4 小时。所以 B 选项正确。

405.细节理解题 根据第 6 段可知,母亲认为 Emily 参与这一项目可以让 Emily 获得不

同的自我价值反馈途径,而不仅仅是一种方式(从同学中获得)。所以选 B.

406.推理判断题 从倒数第二段“I thought it would be fun for me to get involved in making

time to do something …it is all work and no play.” 可以得知 Sarah 觉得花点时间做别的事

情是很有乐趣的。它也是一种工作方式(兼职)。故 B 选项正确。

407. 推理判断题 从第三段最后一句得知,该项目举办一些活动来培养 vulnerabile

young people 的自尊感,自信感以及积极人生心态;从第五段得知 Emily (vulnerabile

young people 的例子)在学校难以得到同学的认同;从最后一段最后一句可以得知

Emily 参加该项目以后受益匪浅,从此变成一个快乐而富有个性的好女孩。所以从正反

两方面可推知 C 选项正确。

93.(2012 年,全国卷 II)

You may think that sailing is a difficult sport, but it is really not hard to learn it. You do

not need to be strong. But you need to be quick. And you need to understand a few basic rules

about the wind.

First, you must ask yourself, “Where is the wind coming from? Is it coming from ahead

or behind or from the side?” You must think about this all the time on the boat. The wind

direction tells you what to do with the sail.

Let’s start with the wind blowing from the behind. This means the wind and the boat are

going in the same direction. Then you must always keep the sail outside the boat. It should be

at a 90° angle (角度) to the boat. Then it will catch the wind best.

If the wind is blowing from the side, it is blowing across the boat. In this case, you must

keep the sail half way outside the boat. It should be at a 45° angle to the boat.It needs to be

out far enough to catch the wind, but it shouldn’t flap ( 摆 动 ). It shouldn’t look like on a

flagpole. If it is flapping, it is probably out too far, and the boat will slow down.

Sailing into the wind is not possible. If you try, the sail will flap and the boat will stop.

You may want to go in that direction. It is possible, but you can’t go in a straight line. You

must go first in one direction and then in another. This is called tacking. When you are
tacking, you must always keep the sail inside the boat.

1.What should you consider first while sailing?

A.Sailors’ strength. B.Wave levels.

C.Wind directions. D.Size of sails.

2.What does the word “It” underlined in Paragraph 4 refer to?

A.The boat. B.The wind. C.The sail. D.The angle.

3.What do you have to do when sailing against the wind?

A.Move in a straight line. B.Allow the sail to flap.

C.Lower the sail. D.Tack the boat.

4.Where can you probably find the text?

A.In a popular magazine. B.In a tourist guidebook.

C.In a physics textbook. D.In an official report.

【答案】1.C 2.C 3.D 4.A

【解析】本文主要介绍了帆船运动的一些常识。就帆船顺风、侧风和逆风行驶进行了

详细的阐述。
1. 细节题:从第二段的句子:“ Where is the wind coming from? Is it coming from ahead

or behind or from the side?”可知是先要考虑风向。选 C

2.猜词题:从前面的句子:you must keep the sail half way outside the boat 可知这里的 it

指代风帆。选 C

3.细节题:从最后一段的句子: You must go first in one direction and then in another. This

is called tacking.可知选 D

4.文章出处题:从文章整体看是给大家一些帆船的知识,应该出自流行杂志。选 A

考点:考查日常生活类短文
94.(2012 年,全国卷 II)

Cold weather can be hard on pets, just like it can be hard on people. Sometimes

owners forget that their cats are just as used to the warm shelter ( 住所 ) as they are. Some

owners will leave their animals outside for a long period of time, thinking that all animals

are used to living outdoors. This can put their pets in danger of serious illness. There are

things you can do to keep your animal warm and safe.

Keep your pets inside as much as you can when the weather is bad. If you have to

take them out, stay outside with them. When you’re cold enough to go inside, they

probably are too. If you must leave them outside for a long time, make sure they have a

学科网(北京)股份有限公司
warm, solid shelter against the wind, thick bedding, and plenty of non-frozen water.

If left alone outside, dogs and cats can be very smart in their search for warm shelter.

They can dig into snow banks or hide somewhere. Watch them closely when they are left

outdoors, and provide them with shelter of good quality. Keep an eye on your pet’s water.

Sometimes owners don’t realize that a water bowl has frozen and their pet can’t get

anything to drink. Animals that don’t have clean and unfrozen water may drink dirty water

outside, which may contain something unhealthy for them.

5.What do we learn about pets from Paragraph 1?

A.They are often forgotten by their owners. B.They are used to living outdoors.

C.They build their own shelters. D.They like to stay in warm places.

6.Why are pet owners asked to stay with their pets when they are out in cold weather?

A.To know when to bring them inside. B.To keep them from eating bad food.

C.To help them find shelters. D.To keep them company.

7.If pets are left on their own outdoors in cold weather, they may .

A.be short of clean water B.dig deep holes for fun

C.dirty the snow nearby D.get lost in the wild

8.What is the purpose of this text?

A.To solve a problem. B.To give practical advice.

C.To tell an interesting story. D.To present a research result.

【答案】5.D 6.A 7.A 8.B

【解析】本文是一篇说明文。文章给养宠物的人们提供了一些实用的建议。天气恶劣

的时候,把宠物放在家里;天气寒冷的时候,注意让宠物有干净的水喝。
5. 考 查判 断推 理。 根据 “ Sometimes owners forget that their cats are just as used to the

warm shelter (住所) as they are.” 可知,猫等宠物也像人一样习惯于暖和的住所。所以 D

正确。
6. 考查细节理解。根据第二段中的“ If you have to take them out, stay outside with them.

When you’re cold enough to go inside, they probably are too.” 可知:当你感到冷的时候就

会进房间,它们也是如此。所以 A 正确。

7. 考查细节理解。根据文章第三段中的“ a water bowl has frozen and their pet can’t get

anything to drink”可知,当水盆被冻之后,宠物就可能找不到干净的水喝。故选 A。

8.考查作者的写作意图。根据第一段的中心句“ There are things you can do to keep your

animal warm and safe.”可知,这是一篇说明文,其目的是为了给养宠物的人们一些实用

的建议。
95.(2012 年,四川卷)

Plants are flowering faster than scientists predicted (预测) in reaction to climate change,

which could have long damaging effects on food chains and ecosystems.

Global warming is having a great effect on hundreds of plant and animal species around

the world, changing some living patterns, scientists say.

Increased carbon dioxide (CO2) in the air from burning coal and oil can have an effect

on how plants produce oxygen, while higher temperatures and changeable rainfall patterns

can change their patterns of growth.

“Predicting species, reaction to climate change is a major challenge in ecology,” said the

researchers of several U.S. universities. They said plants had been the key object of study

because their reaction to climate change could have an effect on food chains and ecosystem

services.

The study, published on the Nature website, uses the findings from plant life cycle

studies and experiments across four continents and 1,634 species. It found that some

experiments had underestimated ( 低估 ) the speed of flowering by 8.5 times and leafing by 4

times.

“Across all species, the experiments under-predicted the speed of the advance—for both

leafing and flowering— that results from temperature increase,” the study said.

The design of future experiments may need to be improved to better predict how plants

will react to climate change, it said.

Plants are necessary for life on the Earth. They are the base of the food chain, using

photosynthesis ( 光 合 作 用 ) to produce sugar from carbon dioxide and water. They let out

oxygen which is needed by nearly every organism on the planet.

Scientists believe the world’s average temperature has risen by about 0.8 ℃ since 1900,

and nearly 0.2℃ every ten years since 1979.

So far, efforts to cut emissions ( 排放 ) of planet-warming greenhouse gases are not seen

as enough to prevent the Earth heating up beyond 2 ℃ this century—a point scientists say

will bring the danger of a changeable climate in which weather extremes are common,

leading to drought, floods, crop failures and rising sea levels.

40.What is the main idea of Paragraph 1?

A.Plants’ reaction to weather could have damaging effects on ecosystems.

B.The increasing speed of flowering is beyond scientists’ expectation.

学科网(北京)股份有限公司
C.Climate change leads to the change of food production patterns.

D.Food chains have been seriously damaged because of weather.

41.We can learn from the study published on the Nature website that .

A.plants’ flowering is 8.5 times faster than leafing

B.there are 1,634 plant species on the four continents

C.scientists should improve the design of the experiments

D.the experiments failed to predict how plants react to climate change

42.Scientists pay special attention to the study of plants because .

A.they can prove the climate change clearly

B.they are very important in the food chains

C.they play a leading role in reducing global warming

D.they are growing and flowering much faster than before

43.What can be inferred from the last two paragraphs about the world’s temperature?

A.It needs to be controlled within 2℃ in this century.

B.Its change will lead to weather extremes.

C.It is 0.8℃ higher in 1979 than that of 1900.

D.It has risen nearly 0.2℃ since 1979.

【答案】40.B 41.C 42.B 43.A

【解析】本文为科普类文章。文章论述了全球变暖的气候环境会让植物提前生长开花 ,

进而影响整个食物链及生态系统。
40.细节理解题。第一段话给读者传达的主要信息存在于主句,而不是 which 所引导的

定语从句上。根据“ Plants are flowering faster than scientists predicted” 可知,作者是要

告知读者植物开花的增速超出了科学家的预期。故选 B。A 为次要信息。

41. 推 理 判 断 题 。 第 五 段 话 “ It found that some experiments had underestimated ( 低

估) the speed of flowering by 8.5 times and leafing by 4 times” 暗示读者:对于植物所设

计的实验应该有所改进,故答案选 C。

42. 细节理解题。根据第四段“ They said plants had been the key object of study because

their reaction to climate change could have an effect on food chains and ecosystem services”

可知植物对气候变化的反应对整个食物链和生态系统都至关重要。故答案选 B。

43.推理判断题。从倒数第二段“… has risen by about 0.8℃ since 1900, and nearly 0.2 ℃

every ten years since 1979” 可知 A、C 项错误;从最后一段叙述可知,如果本世纪地球

变暖超过 2℃,便会出现 weather extremes。暗示我们这个世纪地球温度的上升需要控

制在 2℃的范围之内。故答案选 D。
96.(2012 年,江苏卷)

Deputy Agriculture Secretary Kathleen Merrigan sees an epidemic ( 流行病 ) sweeping

across America’s farmland. It has little to do with the usual challenges, such as flood, rising

fuel prices and crop-eating insects. The country’s farmers are getting older, and there are

fewer people standing in line to take their place. National agricultural census ( 普查 ) figures

show that the fastest-growing group of farmers is the part over 65. Merrigan is afraid the

average age will be even higher when the 2012 statistics are completed.

Merrigan, a former college professor, is making stops at universities across the country

in hopes of encouraging more students to think about careers in agriculture. Aside from trying

to stop the graying of America’s farmers, her work is made tougher by a recent blog posting

that put agriculture at No. 1 on a list of “useless” college degrees. Top federal agriculture

officials are talking about the posting, and it has the attention of agricultural organizations

across the country.

“There couldn’t be anything that’s more incorrect,” Merrigan said. “We know that there

aren’t enough qualified graduates to fill the jobs that are out there in American agriculture.”

In addition, a growing world population that some experts predict will require 70% more

food production by 2050, she said.

“I truly believe we’re at a golden age of agriculture. Global demand is at an all-time

record high, and global supplies are at all-time record lows,” said Matt Rush, director of the

Texas Farm Bureau. “Production costs are going to be valuable enough that younger people

are going to have the opportunity to be involved in agriculture.”

The Department of Agriculture has programs aimed at developing more farmers and at

increasing interest in locally grown food. The National Young Farmers’ Coalition has also

been pushing for state and federal policy changes to make it easier for new farmers.

Ryan Best, president of Future Farmers of America, has been living out of a suitcase,

traveling the country and visiting with high school students about careers in agriculture. The

21-year-old Best hopes his message—that this is a new time in agriculture—will motivate the

next generation to turn around the statistics. “Never before have we had the innovations ( 创

新) in technology which have led to agriculture in this country being the most efficient it has

ever been,” he said. “There’s really a place for everybody to fit in.”

44.What is the new challenge to American agriculture?

A.Fewer and older farmers. B.Higher fuel prices.

学科网(北京)股份有限公司
C.More natural disasters. D.Lower agricultural output.

45.Why is Merrigan visiting universities across the country?

A.To draw federal agriculture officials’ attention.

B.To select qualified agriculture graduates.

C.To clarify a recent blog posting.

D.To talk more students into farming careers.

46.According to Matt Rush, American agriculture will provide opportunities for younger

people because__________..

A.the government will cover production costs

B.global food supplies will be even lower

C.investment in agriculture will be profitable

D.America will increase its food export

47.What do the underlined words “to turn around the statistics” in the last paragraph mean?

A.To re-analyze the result of the national census.

B.To increase agricultural production.

C.To bring down the average age of farmers.

D.To invest more in agriculture.

【答案】44.A 45.D 46.C 47.C

【解析】美国的农业遭遇到了危机。最大的问题是从业人员不多,而且年龄越来越大 。

农业专家预测将来从事与农业有关的职业会收益颇丰的。所以年轻人应该把农业作为

一项事业来做。
44. 细节理解题。由“ The country’s farmers are getting older, and there are fewer people

standing in line to take their place. ” 可知农业面临的最大问题是更少并逐渐年迈的农民。

故选 A。

45. 细 节 理 解 题 。 由 第 二 段 第 一 句 , “ Merrigan, a former college professor, is making

stops at universities across the country in hopes of encouraging more students to think about

careers in agriculture.可知,Merrigan 想说服更多的学生考虑从事农业方面的工作。故选

D。

46.推理判断题。麦特•拉什说:农业生产成本使生产有足够的利益,年轻人有机会从

事农业工作。与 C 项的意思(投资农业会有钱可赚的。)一致。故选 C。

47.词意猜测题。根据本句和下面的意思:21 岁的贝斯特希望“现在是农业的新时代”

这一信息能够激励下一代改变以上的数据。“以前从未有过的技术创新使这个国家的

农业达到空前的高效,这里给每个人都提供了发展空间。”贝斯特说道。判断,与 C
的意思:(年轻人从事农业方面的工作会使农场主老龄化的情况得以扭转。)一致。

故选 C。

97.(2012 年,江苏卷)

Medical drugs sometimes cause more damage than they cure. One solution to this

problem is to put the drugs inside a capsule, protecting them from the body—and the body

from them—until they can be released at just the right spot. There are lots of ways to trigger

(引发) this release, including changing temperature, acidity, and so on. But triggers can come

with their own risks—burns, for example. Now, researchers in California have designed what

could be a harmless trigger to date: shining near-infrared light (NIR, 近红外线 ) on the drug

in the capsule.

The idea of using light to liberate the drug in the capsule isn’t new. Researchers around

the globe have developed polymers ( 聚合物 ) and other materials that begin to break down

when they absorb either ultraviolet (UV, 紫 外 线 ) or visible light. But tissues also readily

absorb UV and visible light, which means the drug release can be triggered only near the

skin, where the light can reach the capsule. NIR light largely passes through tissues, so

researchers have tried to use it as a trigger. But few compounds ( 化合物 ) absorb NIR well

and go through chemical changes.

That changed last year when Adah Almutairi, a chemist at the University of California,

San Diego, reported that she and her colleagues had designed a polymer that breaks down

when it absorbs NIR light. Their polymer used a commercially available NIR-absorbing

group called o-nitrobenzyl (ONB). When they catch the light, ONB groups fall off the

polymer, leading to its breakdown. But ONB is only a so-so NIR absorber, and it could be

poisonous to cells when it separates from the polymer.

So Almutairi and her colleagues reported creating a new material for capsules that’s even

better.This one consists of a long chain of compounds called cresol groups linked in a

polymer. Cresol contains reactive( 易反应的 ) components that make it highly unstable in its

polymeric form, a feature Almutairi and her colleagues use to their advantage. After

polymerizing the cresols, they cap each reactive component with a light-absorbing compound

called Bhc. When the Bhcs absorb NIR light, the reactive groups are exposed and break the

long polymer into two short chains. Shining additional light continues this breakdown,

potentially releasing any drugs in the capsule. What’s more, Almutairi says, Bhc is 10 times

better at absorbing NIR than is ONB and is not poisonous to cells.

学科网(北京)股份有限公司
48.According to the passage, which of the following could be the best trigger?

A.Temperature change. B.NIR light. C.Acidity change. D.UV light.

49.Why is ONB unsatisfactory?

A.It breaks down when it absorbs NIR light.

B.It falls off the polymer and triggers drug release.

C.It has not come onto the market up till now.

D.It is not effective enough and could be poisonous.

50.Which word can be used to complete the following process of changes?

A.protected B.formed C.exposed D.combined

【答案】48.B 49.D 50.C

【解析】这一篇科普文章,讲的是“怎样使用胶囊,让吃药更加安全、疗效好”。
48.细节推理题。根据首段最后两句“ But triggers can come with their own risks-…”“Now,

…a harmless trigger…near-infrared light (NIR, 近红外线)”可知答案是 B。

49.细节理解题。根据第三段尾句“ But ONB is only a so-so NIR absorber, and it could be

poisonous to cells when it separates from the polymer.”

50. 细 节 归 纳 题 。 根 据 尾 段 关 键 词 、 句 : “ a long chain of compounds called cresol

groups”“Cresol contains reactive( 易 反 应 的 ) components that…”“When…, the reactive

groups are exposed and break…”“…continues this breakdown” 可知答案是 C。

98.(2012 年,陕西卷)

Eating too much fatty food, exercising too little and smoking can raise your future risk

of heart disease. But there is another factor that can cause your heart problems more

immediately:the air you breathe.

Previous studies have linked high exposure ( 暴 露 ) to environmental pollution to an

increased risk of heart problems,but two analyses now show that poor air quality can lead to

heart attack or stroke ( 中风 ) within as little as a few hours after exposure. In one review of

the research, scientists found that people exposed to high levels of pollutants ( 污染物 ) were

up to 5% more likely to suffer a heart attack within days of exposure than those with lower

exposure. A separate study of stroke patients showed that even air that the U. S.

Environmental Protection Agency (EPA)considers to be of “moderate” ( 良 好 ) quality and

relatively safe for our health can raise the risk of stroke as much as 34% within 12 to 14 hours

of exposure.

The authors of both studies stress that these risks are relatively small for healthy people

and certainly modest compared with other risk factors such as smoking and high blood
pressure. However, it is important to be aware of these dangers because everyone is exposed

to air pollution regardless of lifestyle choices. So stricter regulation by the EPA of pollutants

may not only improve environmental air quality but could also become necessary to protect

public health.

51.The text mainly discusses the relationship between ________.

A.heart problems and air quality

B.heart problems and exercising

C.heart problems and smoking

D.heart problems and fatty food

52.The underlined word “modest” in Paragraph 3 most probably means ________.

A.relatively high B.extremely low

C.relatively low D.extremely high

53.What can we learn from the text?

A.Eating fatty food has immediate effects on your heart.

B.The EPA conducted many studies on air quality.

C.Moderate air quality is more harmful than smoking.

D.Stricter regulations on pollutants should be made.

54.The author's purpose of writing the text is most likely to ________.

A.inform B.persuade

C.describe D.entertain

【答案】51.A 52.C 53.D 54.A

【解析】
51.主旨大意。分析文章的内容,、可知本文主要讲述了吸入了受污染的空气对人类的

心脏造成了极大的影响,结合各个选项的意思,可知应选 A。

52.词义猜测,根据 modest 前的 relatively small,可知它的意思与之相近,应选 C。

53.细节理解,根据文章中第三段的 So stricter regulation by the EPA of pollutants may not

only improve environmental air quality but could also become necessary to protect public

health,可知 D 选项的意思与之相符,故选 D。

54.细节理解,根据文章的内容,可知本文主要介绍了吸入了受污染的空气对人类的心

脏造成了极大的影响,因此,本文的作用主要是客观地给读者讲述了这个问题,应选

A 正确。

99.(2011 年,湖北卷)

Howling is a behavior commonly observed among a wolf nark. An animals, wolves

学科网(北京)股份有限公司
work together to hunt and rely on howling was an important means of communication each

other. There are different explanations of a wolf’s howl and it appears that there may be more

to discover. One theory is that wolves howl to bend better together. It’s almost as if howling

together helps the pack stay together. Perhaps something similar to people feeling a sense of

involvement with each other when singing a song together . But this theory may be wrong,

explains Fred H. Harington, a professor who studies wolf behavior. Indeed, there have been

tines when wolves have been seen one moment howling in a exhorts, and the next, quarreling

anions each other. It appears that usually the lowest-tanking menthes of the pack may actually

be “punished” for

Joining in the churs at times. So is howling a way to strengthen a social bond or just a

way to reconfirm status among its members? ——Why do wolves howl for sure?

What is clear, however, is that howling is often used among packmates to locate each

other. Hunting grounds are distant and it happens that wolves may separate from one another

at times. When this happens, howling appears to be an excellent means of gathering.

Howling, interestingly, is a contagious behaviour. When one wolf starts to howl, very likely

others will follow. This is often seen to occur in the morning, as if wolves were doing some

sort of “roll rall”where wolves all howl together to howl, very likely others will follow. This

is often seen to occur in the morning, as if wolves were doing w some sotr of “roll call”where

wolves all howl together to report their pretence.

55..What the por similarity between wolves’ how humans ting in chorus?

A.The act of calling each other.

B.the sense of accomplishment.

C.The act of hunting for something.

D.The sense of belonging to a group.

56..Why does Harrington think the“social bond”theory may be wrong?

A.Wolves separate from each other after howling.

B.Wolves tend to protect their hunting grounds.

C.Wolves sometimes have quarrels after howling together.

D.Wolves of low rank are encouraged to join in the chorus.

57.Researchers are sure that wolves often howl to______.

A.show their ranks

B.find their companion

C.report the missing ones


D.express their loveliness

58.“Howling… is a contagious behaviour”(in the last paragraph)means_ ______.

A.howling is a signal for hunting

B.howling is a way of communication

C.howling often occurs in the morning

D.howling spreads from one to another

【答案】55.D 56.C 57.B 58.D

【解析】
55.细节理解题。根据文章第二段的 One theory is that wolves howl to bend better together.

It’s almost as if howling together helps the pack stay together. Perhaps something similar to

people feeling a sense of involvement with each other when singing a song together . 可知答

案。
56.细节理解题。定位关键词 wrong,接下来文章 there have been tines when wolves have

been seen one moment howling in a exhorts, and the next, quarreling anions each other. 可知。

57.细节理解题。定位 sure 和文章第 4 段的 what is clear 相对应,接下来介绍了狼的嚎叫

是狼在召唤同伴。
58.猜测词义题。定位 contagious behaviour,从该词下文举的例子“When one wolf starts

to howl, very likely others will follow.” 说明“ contagious behaviour” 的意思的解释,可以

看 出 Howling… is a contagious behaviour 意 思 很 可 能 是 “ howling spreads from one to

another”的意思。

100.(2011 年,辽宁卷)

About 21,000 young people in 17 American states do not attend classes in school

buildings.

Instead, they receive their elementary( 初 等 )and high school education by working at

home on computers.The Center for Education Reform says the United States has 67 public

“cyberschools.”and that is about twice as many as two years ago.

The money for students to attend a cyberschool comes from the governments of the

states where they live. Some educators say cyberschools receive money that should support

traditional public schools. They also say it is difficult to know if students are learning well.

Other educators praise this new form of education for letting students work at their own

speed. These people say cyberschools help students who were unhappy or unsuccessful in

traditional schools. They say learning at home by computer ends long bus rides for children

学科网(北京)股份有限公司
who live far from school.

Whatever the judgement of cyberschools,they are getting more and more popular. For

example, a new cyberschool called Commonwealth Connections Academy will take in

students this fall. It will serve children in the state of Pennsylvania from ages five through

thirteen.

Children get free equipment for their online education. This includes a computer,a

printer,books and technical services. Parents and students talk with teachers by telephone or

by sending emails through their computers when necessary.

Students at cyberschools usually do not know one another. But 56 such students who

finished studies at Western Pennsylvania Cyber Charter School recently met for the first time.

They were guests of honor at their graduation.

63.What do we know from the text about students of a cyberschool?

A.They have to take long bus rides to school.

B.They study at home rather than in classrooms.

C.They receive money from traditional public schools.

D.They do well in traditional school programs.

64.What is a problem with cyberschools?

A.Their equipment costs a lot of money.

B.They get little support from the state government.

C.It is hard to know students' progress in learning.

D.The students find it hard to make friends.

65.Cyberschools are getting popular became

A.they are less expensive for students

B.their students can work at their own speed

C.their graduates are more successful in society

D.they serve students in a wider age range

66.We can infer that the author of the text is .

A.unprejudiced in his description of cyberschools

B.excited about the future of cyberschools

C.doubtful about the quality of cyberschoois

D.disappointed at the development of cyberschools

【答案】63.B 64.C 65.B 66.A

【解析】
63.B 细节理解题。根据第二段的第一句话的 by working at home on computers 可知 B 项
正确。
64.C 细节理解题。第三段的最后一句话 it is difficult to know if students are learning well

提到了网络课程的缺点,C 项正确。

65.B 细节理解题。根据第四段的第一句话说的是网络课程的优点,即学生们可以按自

己的水平选择课程,这也是它之所以受欢迎的原因,选 B。

66.A 主旨大意题。作者并无表明自己对网络学校的态度,仅是对网络学校作了客观的

描述,因此选 A。

101.(2011 年,北京卷)

Submission Guidelines

Before sending us a manuscript (稿件) , look through recent issues (刊物) of the

Post to get an idea of the range and style of articles we publish. You will discover that our

focus has broadened to include well-researched, timely and informative articles on finance,

home improvement, travel, humor, and many other fields.

The Post’s goal is to remain unique, with content that provides additional understandings

on the ever-evolving American scene.

In addition to feature-length (专题长度的)articles, the Post buys anecdotes, cartoons,

and photos. Payment ranges from $25 to $400.

Our nonfiction needs include how-to, useful articles on gardening, pet care and training,

financial planning, and subjects of interest to a 45-plus, home-loving readership. For

nonfiction articles, indicate any special qualifications you have for writing about the subject,

especially scientific material. Include one or two published pieces with your article. We prefer

typed articles between 1000 and 2000 words in length. We encourage you to send both

printed and online versions.

We also welcome new fiction. A light, humorous touch is appreciated. We are always in

need of straight humor articles. Make us laugh , and we’ll buy it.

Feature articles average about 1000 to 2000 words. We like positive, fresh angles to Post

articles, and we ask that they be thoroughly researched.

We normally respond to article submissions within six weeks. You are free to submit the

article elsewhere at the same time.

Please submit all articles to Features Editor, The Saturday Evening Post, 1100 Waterway

Boulevard, Indianapolis, IN 46202, (317)634-1100.

67.Before sending a manuscript to the Post, a contributor is advised to .

学科网(北京)股份有限公司
A.get a better understanding of American issues

B.find out the range of the articles in the post

C.increase his knowledge in many fields

D.broaden his research focus

68.to submit nonfiction articles,a contributor must_____.

A.provide his special qualification

B.be a regular reader of the post

C.produce printed version

D.be over 45 year old

69.from the passage we can learn that the post

A.allows article submission within six weeks

B.favor science articles within 2,000 words

C.have a huge demand for humorous works

D.prefers nonfiction to fiction article

【答案】67.B 68.A 69.C

【解析】文章是一篇说明文。就投稿指南作了一下相关介绍和注意事项。
67.细节理解题。根据第一段 Before sending us a manuscript (稿件), look through recent

issues (刊物) of the Post to get an idea of the range and style of articles we publish. You

will discover that our focus has broadened to include well-researched, timely and informative

articles on finance, home improvement, travel, humor, and many other fields. 可知,在将稿

件发送到邮局之前,我们的关注范围已经扩大到包括充分研究,及时和信息丰富的文

章,金融,家居装修,旅游,幽默,和许多其他领域。故可知在将稿件发送到邮局之

前,建议投稿者了解一下邮局内文章的范围。故选 B。

68.细节理解题。根据第四段 For nonfiction articles, indicate any special qualifications you

have for writing about the subject, especially scientific material. 可知,对于非小说类的文章,

请注明你在写作方面的任何特殊资格,尤其是科学材料。故推知 要提交非小说类文章,

投稿者必须提供他的特殊资格。故选 A。

69.推理判断题。通读全文可知文章就如何投稿作了一些相关建议,并根据第五段 We

also welcome new fiction. A light, humorous touch is appreciated. We are always in need of

straight humor articles. Make us laugh, and we'll buy it. 我们可以了解到,邮报对幽默作品

有着巨大的需求。故选 C。

102.(2011 年,福建卷)

Driving a car is not just handling controls and judging speed and distance. It requires
you to predict what other road users will do and get ready to react to something unexpected.

When alcohol is consumed, it enters your bloodstream and acts as a depressant ( 抑 制 药 ),

damaging eyesight, judgment and co-ordination( 协 调 ), slowing down reaction time and

greatly increasing the risk of accidents. Even below the drink driving link, driving will be

affected.

Alcohol may take a few minutes to be absorbed into the bloodstream and start action on

the brain. Absorption rate is increased when drinking on an empty stomach or when

consuming drinks mixed with fruit juice. To get rid of alcohol from the body is a very slow

process and it is not possible to speed it up with any measures like taking a shower or having

a cup of tea or coffee.

The present Road Traffic Ordinance states clearly that the limit of alcohol concentration

is:

● 50 milligrams of alcohol per 100ml of blood; or

● 22 micrograms of alcohol per 100ml of breath; or

● 67 milligrams of alcohol per 100ml of urine (尿液).

Drivers who cause traffic accidents, or who commit a moving traffic offence or are being

suspected of drink driving will be tested.

Any driver found drinking beyond the limit will be charged. The driver declared guilty

may be fined a maximum of HK $25,000 and be sentenced to up to 3 years in prison and

punished for 10 driving-offence points; or temporarily banned from driving.

The same punishment applies to failing to provide specimens ( 样本) for breath, blood or

urine tests without good excuse.

Drink driving is a criminal offence (违法犯罪行为) . Be a responsible driver, think

before you drink. For the safety of yourself and other road users, never drive after consuming

alcohol.

70.The first paragraph is mainly about ________.

A.the introduction of driving skills

B.the damage of drinking to your body

C.the effect of drinking on driving

D.the process of alcohol being absorbed

71.The underlined word “it” in the second paragraph refers to “________”.

A.alcohol B.absorption C.blood D.process

学科网(北京)股份有限公司
72.Which of the following is TRUE according to the passage?

A.Drinking below the drink driving limit has no effect on driving.

B.Alcohol is taken in more quickly when drunk with fruit juice.

C.Having a cup of tea helps to get rid of alcohol from the body.

D.50 milligrams of alcohol per 100ml of breath is below the drink limit.

73.A driver suspected of (被怀疑) drink driving ________.

A.should provide specimens for testing

B.will be forbidden to drive for 3years

C.will be punished for 10 driving-offence points

D.should pay a maximum fine of HK $25,000

【答案】70.C 71.D 72.B 73.A

【解析】本文介绍的是酒后驾车的危害,以及对酒后驾车的处罚等,呼吁人们为了本

人和家人、其他人的健康,一定不要酒后驾车。
70. 主 旨 大 意 题 。 根 据 文 章 第 一 段 的 “ When alcohol is consumed, it enters your

bloodstream and acts as a depressant ( 抑 制 药 ), damaging eyesight, judgment and co-

ordination( 协 调 ), slowing down reaction time and greatly increasing the risk of accidents.

Even below the drink driving link, driving will be affected” 可知喝酒会损害人的视力、判

断力、减慢反应力,可知本段重要讲述酒后驾车的影响。故选 C 项。

71. 词义猜测题。根据文章第二段的“ To get rid of alcohol from the body is a very slow

process 驱除体内的酒精是非常缓慢的过程”可知,后半句表示“采取像淋浴或喝一杯

茶、咖啡等这样的办法来加速这个过程是不可能的”,故 it 代指“process 过程”。故

选 D 项。

72. 细节理解题。根据第二段中的 Absorption rate is increased when drinking on an empty

stomach or when consuming drinks mixed with fruit juice 当空腹饮用或饮用混合果汁的饮

料时,吸收率会增加。故选 B 项。

73. 细 节 理 解 题 。 根 据 文 章 倒 数 第 四 段 中 Drivers who cause traffic accidents, or who

commit a moving traffic offence or are being suspected of drink driving will be tested. 可知会

对造成交通事故、违反交通规则或涉嫌酒后驾驶的司机进行酒精测试。所以涉嫌酒后

驾车的司机应提供检测样本。故选 A 项。

103.(2011 年,福建卷)

For five days, Edmonton's Downtown Park is transformed into one huge stage where

artists are able to share their talents, and where people are able to celebrate and enjoy

themselves. Since its beginning in 1980, the Edmonton Folk Music Festival has been
commemorating ( 纪 念 ) the true feeling of what folk music is all about and that's the

traditional togetherness ( 友 爱 ) that is felt when people gather to share stories and feelings

through song.

This year will be the sixth year when volunteer Riedel will be offering up her time to the

festival. “People coming off a busy spring and summer have a moment of relaxation , ”

Riedel said. “It's really easy to relax, and it's great seeing family and friends have fun

together.” These families and friends come from all different kinds of musical tastes. People

who take pleasure in Blues are there, so are people who love Bluegrass. This festival does its

best to develop everyone's musical interests.

With so many years of experience, the festival has become a welloiled machine, and

does whatever it can to make attendees feel as comfortable as possible. There are free water

stations throughout the venue ( 举办地 ) for people to fill up their travel cups. When people

buy food, reusable dishes are given a $2 plate fee, but that is returned when the plate is

brought back.

The festival has completely sold out of tickets, and in record time. But with big names

such as Van Morrison and Jakob Dylan ,it's easy to see how that was going to happen. There

is no parking area during the festival, so using the Park & Ride system or Edmonton Transit is

highly recommended. A bike lockup area is provided and will be available Thursday until

Sunday one hour before the gates open until 45 minutes after the gates close.

The Edmonton Folk Music Festival begins on Wednesday, Aug. 4 with Van Morrison

playing the special donation fund (基金) concert,and will finish up on Sunday,Aug. 8.

74.The Edmonton Folk Music Festival is held mainly to ________.

A.gather people with different musical tastes

B.remind people of the real sense of folk music

C.exhibit the good voices of great talents in folk music

D.collect old stories of folk music

75.Which of the following is TRUE according to the passage?

A.Riedel has volunteered for the festival for at least 5 years.

B.It's hard for people to appreciate Blues.

C.It costs people a little to fill up their cups from water stations.

D.People have to pay $2 for a plate of food.

76.We can learn from the passage that ________.

学科网(北京)股份有限公司
A.people can get tickets easily for the festival

B.the Edmonton Folk Music Festival is well organized

C.driving one's own car to the festival is highly recommended

D.bikes are available at the festival from Wednesday to Sunday

77.What would be the best title for this passage?

A.Folk Music of Blues

B.One Festival for All

C.Festival for Family Gathering

D.Edmonton's Downtown Park

【答案】74.B 75.A 76.B 77.B

【解析】
74.B 细节题。根据第一段 the Edmonton Folk Music Festival has been commemorating ( 纪

念 ) the true feeling of what folk music is all about and that's the traditional togetherness that

is felt when people gather to share stories and feelings through song.说明 B 正确。

75.A 推理题。根据第二段第一句 This year will be the sixth year when volunteer Riedel

will be offering up her time to the festival.说明这是他第六年做志愿者,故他最起码做了 5

年的志愿者,故 A 正确。

76.B 推理题。根据文章第三段第一行 With so many years of experience, the festival has

become a well-oiled machine, and does whatever it can to make attendees feel as comfortable

as possible.说明这个活动已经举办多年,经验丰富,有很好的组织。故 B 正确。

77.B 主旨大意题。文章介绍了著名的 The Edmonton Folk Music Festival,故 B 正确。

104.(2011 年,陕西卷)

Ever since they were first put on the market in the early 1990s, genetically modified

(GM, 转基因) foods have been increasingly developed and marketed in many countries in the

word,mainly on the basis of their promise to end the worldwide food crisis. But can GM

technology solve world hunger problems? Even if it would, is it the best solution?

Despite what it promises, GM technology actually has not increased the production

potential of any corp. In fact, studies show that the most widely grown GM crop. GM

soybeans, has suffered reduced productivity. For instance, a report than analysed nearly two

decades of research on mojor GM food crops shows that GM engineering has failed to

significantly increase US crop production.

Something else, however, has been on the rise, While GM seeds are expensive, GM

companies tell farmers that they will make good profits by saving money on pesticides (杀
虫剂) . On the contrary, US government data show that GM crops in the US have produced

an overall increase in pesticide use compared to traditional crops. “The promise was that you

could use less chemicals and boost production. But nether is true,” said Bill Christison,

President of the US National Farm Coalition.

At the same time, the authors of the book World Hunger: Twelve Myths argue that there

actually is more than enough food in the world and that the hunger crisis is not caused by

production, but by problems in food distribution and politics. These indeed deserve our

efforts and money. Meanwhile, the rise in food prices results from the increased use of crops

for fuel rather than food, according to a 2008 World Bank report.

As a matter of fact, scientists see better ways to feed the world. Another World Bank

report concluded that GM crops have little to offer to the challenges of worldwide poverty

and hunger, because better ways out are available, among which “green” farming is supposed

to be the first choice.

78.The author develops the second paragraph mainly .

A.by classification B.by comparison

C.by example D.by process

79.What does the underlined word “boost ” in the third paragraph probably mean?

A.Control. B.Evaluate C.Obtain. D.Increase.

80.GM companies promise farmers that they will benefit from ______________.

A.practicing “green” farming B.use of less chemicals

C.fair distribution of their crops D.using more crops for fuel

81.Which of the following best describes the attitude of the author towards GM technology?

A.Optimistic B.Defensive C.Disapproving D.Casual

【答案】78.C 79.D 80.B 81.C

【解析】本文是一篇说明文。文章说明了转基因粮食对于解决世界饥荒的作用和存在

的问题。
78. 推理判断题。根据第二段“ For instance, a report than analysed nearly two decades of

research on major GM food crops shows that GM engineering has failed to significantly

increase US crop production.”可知,该段采用的是举例子的方法。故选 C。

79. 词义猜测题。根据前句“ On the contrary, US government data show that GM crops in

the US have produced an overall increase in pesticide use compared to traditional crops.( 相

反,美国政府的数据显示,与传统作物相比,美国转基因作物的农药使用量总体上有

学科网(北京)股份有限公司
所增加。)”可知,他们承诺为了帮助农民解决粮食问题,通过减少化学品的使用和提

高(increase)粮食产量的方式实现。从而推出 boost 为“增加”之意。故选 D。

80. 细节理解题。根据第三段““ The promise was that you could use less chemicals and

boost production. But nether is true,” said Bill Christison, President of the US National Farm

Coalition.( 我们的承诺是,你可以使用更少的化学品,提高产量。但事实并非如此,”

美国国家农场联盟主席比尔•克里斯蒂森表示。 )”可知。基因公司向农民承诺,他们将

从减少化学品的使用中受益。故选 B。

81. 推理判断题。根据最后一段“ Another World Bank report concluded that GM crops

have little to offer to the challenges of worldwide poverty and hunger, because better ways

out are available, among which “green” farming is supposed to be the first choice. ”可知,有

很多更好的方法可以采用,所以对于转基因粮食持不赞成态度。故选 C。

105.(2011 年,江苏卷)

According to the US government, wind farms off the Pacific coast could produce 900

gig watts of electricity every year. Unfortunately, the water there is far too deep for even the

tallest windmills(see picture)to touch bottom. An experiment under way off the coast of

Norway, however, could help put them anywhere.

The project, called Hywind, is the world’s first large-scale deepwater wind turbine( 涡轮

发电机 ).Although it uses a fairly standard 152-ton,2.3-megawatt turbine, Hywind represents

totally new technology. The turbine will be fixed 213 feet above the water on a floating

spar(see picture),a technology Hywind’s creator, the Norwegian company StatoilHydro, has

developed recently. The steel spar, which is filled with stones and goes 328 feet below the sea

surface, will be tied to the ocean floor by three cable( 缆索 ) ; these will keep the spar stable

and prevent the turbine from moving up and down in the waves. Hywind’s stability( 稳 定

性 )in the cold and rough sea would prove that even the deepest corners of the ocean are

suitable for wind power. If all goes according to plan, the turbine will start producing

electricity six miles off the coast of southwestern Norway as early as September.

To produce electricity on a large scale, a commercial wind farm will have to use bigger

turbines than Hywind does, but it’s difficult enough to balance such a large turbine so high on

a floating spar in the middle of the ocean. To make that turbine heavier, the whole spar’s to

design a new kind of wind turbine, one whose gearbox( 变速箱 ) sits at sea level rather than

behind the blades (see picture )

Hywind is a test run, but the benefits for perfecting floating wind-farm technology could
be extremely large. Out at sea, the wind is often stronger and steadier than close to shore,

where all existing offshore windmills are planted. Deep-sea farms are invisible from land,

which helps overcome the windmill-as-eyesore objection. If the technology catches on, it will

open up vast areas of the planet’s surface to one of the best low-carbon power sources

available.

82.The Hywind project uses totally new technology to ensure the stability of _______.

A.the cables which tie the spar to the ocean floor

B.the spar which is floating in deep-sea water

C.the blades driven by strong and steady sea wind

D.the stones filled in the spar below the sea surface

83.To balance a bigger turbine high on a flatting spar, a new type of turbine is to be designed

with its gearbox sitting ____________.

A.on the sea floor B.on the spar top

C.at sea level D.behind the blades

84.Wide applications of deepwater wind power technology can ____________.

A.solve the technical problems of deepwater windmills

B.make financial profits by producing more turbines

C.settle the arguments about environmental problems

D.explore low-carbon power resources available at sea

【答案】82.B 83.C 84.D

【解析】
82.细节理解题。根据第二段倒数第三句可知,该工程采用了全新的技术,是为了确保

漂浮在深海水域中的圆材的稳定。
83.细节理解题。根据第三段最后一句可知,为了使较大的涡轮发电机能在圆材上保持

平衡,公司计划设计新的发电机,其变速箱在海平面上。
84.细节理解题。根据最后一段最后一句可知,深海风能技术的广泛应用可能会在海洋

上开发出可使用的低碳能源。

学科网(北京)股份有限公司
106.(2011 年,湖南卷)

A recent study of ancient and modern elephants has come up with the unexpected

conclusion that the African elephant is divided into two distinct (不同的) species

The discovery was made by researchers at York and Harvard universities when they

were examining the genetic relationship between the ancient woolly mammoth and mastodon

to modern elephants—the Asian elephant, African forest elephant and African savanna

elephant

Once they obtained DNA sequences ( 序列 ) from two fossils ( 化石 ) , mammoths and

mastodons the team compared them with DNA from modern elephants. They found to their

amazement that modern forest and savanna elephants are as distinct from each other as Asian

elephants and mammoths.

The scientists used detailed genetic analysis to prove that the African savanna elephants

and the African forest elephants have been distinct species for several million years. The

divergence of the two species took place around the time of the divergence of Asian elephants

and woolly mammoths. This result amazed all the scientists.

There has long been debate in the scientific community that the two might be separate

species but this is the most convincing scientific evidence so far that they are indeed different

species.

Previously, many naturalists believed that African savanna elephants and African forest

elephants were two populations of the same species despite the elephants’ significant size

differences. The savanna elephant has an average shoulder height of 3.5metres while the

forest elephant has an average shoulder height of 2.5metres. The savanna elephant weighs

between six and seven tons, roughly double the weight of the forest elephant. But the fact that

they look so different does not necessarily mean they are different species. However, the

proof lay in the analysis of the DNA.

Alfred Roca, assistant professor in the department of Animal Sciences at the University

of Minois, said, “We now have to treat the forest and savanna elephants as two different units

for conservation purpose. Since 1950 all African elephants have been conserved as one

species. Now that we know the forest and savanna elephants are two very distinct animals,

the forest elephant should become a bigger priority (优先)for conservation purpose .”

85.One of the fossils studied by the researchers is that of ________.

A.the Asian elephant B.the forest elephant

C.the savanna elephant D.the mastodon elephant


86.The underlined word “divergence” in paragraph 4means “________”

A.evolution B.exhibition C.separation D.examination

87.The researcher’s conclusion was based on a study of the African elephant’s ____________

A.DNA B.height C.weight D.population

88.What were Alfred Roca’s words mainly about?

A.The conversation of African elephants.

B.The purpose of studying African elephants

C.The way to divide African elephants into two units

D.The reason for the distinction of African elephants

89.Which of the following can be the best title for the passage?

A.Naturalist’s Belief about Elephants.

B.Amazing Experiment about Elephants

C.An Unexpected Finding about Elephants

D.A Long scientific Debate about Elephants

【答案】85.D 86.C 87.A 88.A 89.C

【解析】本文主要介绍关于大象的最新研究成果,非洲大象被分成两种完成不同的品

种,非洲草原大象和非洲森林大象;
85.D 细节理解题。根据第三段提到 Once they obtained DNA sequences ( 序列 ) from two

fossils ( 化石) ,mammoths and mastodons the team compared them with DNA from modern

elephants.可知选 D 项。

86.C 词义猜测题。根据第四段提到 The scientists used detailed genetic analysis to prove

that the African savanna elephants and the African forest elephants have been distinct species

for several million years. 可知科学家已证实,非洲的草原大象与非洲的森林大象几百万

年来不属于同一品种,要吧推断下句话为两种大学的分离应发生在。。。,故选 C 项。

87.A 细节理解题。根据第三段提到 Once they obtained DNA sequences ( 序列 ) from two

fossils ( 化石) ,mammoths and mastodons the team compared them with DNA from modern

elephants. They found to their amazement that modern forest and savanna elephants are as

distinct from each other as Asian elephants and mammoths 可知判断研究者结论是建立 在

对非洲大象的 DNA 的研究结果的基础上的,故选 A 项。

88.A 推断题。根据最后一段提到 Now that we know the forest and savanna elephants are

two very distinct animals, the forest elephant should become a bigger priority ( 优 先 )for

conservation purpose 可知他的讲话是围绕着对非洲大象的保护开展的,故选 A 项。

89.C 主旨大意。根据第一段可知,本文主要介绍关于非洲大象的最新研究成虹,非洲

学科网(北京)股份有限公司
草原大象和非洲森林大象两个完成不同的品种,故选 C 项。

107.(2011 年,辽宁卷)

The Coalition for the Homeless is an organization that seeks to address the needs of

the homeless population in the United States. It is a network of offices, some of which

provide food and houses for the homeless population, and some of which fight for the passing

of laws that of over two hundred million people living in the United States,up to three million

are homeless—and the number is still growing. Since the late 1970s,fast rising house

prices,large cuts in government supported housing programs, and economic recession( 经济衰

退 )have made it impossible for many Americans to meet housing costs. Sadly, this has

resulted in a number of persons being forced to leave their homes and/or unable to find new

affordable homes. According to another research,families with children appear to be the

fastest-growing part of the homeless population, making up 39% of it.The old idea of a

homeless person,that of the single man who gets drunk all the time,is no longer true. A much

lager part of the population now finds itself homeless. Even worse, once a person becomes

homeless, he often finds it impossible to find a job,since most employers require anyone who

wants a job from them to provide a home address on a job application.

90.The word “address” in the first line probably means .

A.talk about B.deal with C.fight for D.write to

91.How many people are homeless in the U.S. according to the Coalition studies?

A.39% of the population. B.200 million people.

C.About 3 million people. D.About one-fifth of the population.

92.Homeless people often have difficulty finding a job because________.

A.they have no home addresses

B.they mostly have a drinking problem

C.they aren't supported by government programs

D.they often don't have enough work experience

93.What is the main cause of the rising number of the homeless in the U.S.?

A.The passing of new housing laws.

B.The fast growth of family size.

C.The slow construction of houses.

D.The ever-rising price of housing.

【答案】90.B 91.C 92.A 93.D

【解析】本文是一篇说明文。文章介绍了一个无家可归者提供帮助的公益组织。
90. 词义猜测题。由后文 It is a network of offices, some of which provide food and houses

for the homeless population, and some of which fight for the passing of laws that of over two

hundred million people living in the United States, 可得知,无家可归者联盟是一个致力于

解决美国无家可归者需求的组织。故选 B。

91. 细节理解题。由 up to three million are homeless 可知多达 300 万人无家可归。故选

C。

92. 细节理解题。由 he often finds it impossible to find a job, since most employers require

anyone who wants a job from them to provide a home address on a job application. 可知,因

为大多数雇主无法从他们那里得到工作的人提供的家庭住址。故选 A。

93. 细 节 理 解 题 。 由 fast rising house prices, large cuts in government supported housing

programs, and economic recession(经济衰退)可知,房价上涨快,大规模削减政府住房计

划的支持和经济衰退等原因美国无家可归者人数上升。故选 D。

108.(2011 年,北京卷)

As the railroads and the highways shaped the American West in the past centuries, a new

electhical generating (发电)and transmission(输送)systen for the 21th century will leave

a lasting mark on the West, for better or worse. Much of the real significance of railroads and

highways is not in their direct physical effect on the scenery, but in the ways that they affect

the suurouding community. The same is true of big solar ppants and the power lines that will

be laid dowm to move electricity around.

The 19thcentury saw land grants (政府拨地) offered to railroad companies to build the

transcontinental railroads ,leaving public land in between privately owned land . In much of

the west ,some of the railroad sections were developed while others remained

undeveloped ,and in both cases the landownership has presented unique challenges to land

management ,with the completion or the interstate highway system ,many of the small towns

which sprang up as railway stops and developed well ,have lost their lifeblood and died .

Big solar plants and their power lines will also have effects far beyond their direct

footprint in the west .this is not an argument against building then ,we need alternative energy

badly .and to really take advantage of it we need to be able to move electricity around far

more readily than we can now .

So trade-offs will have to be made .some scenic sport will be sacrificed .some species

( 物种 )will be forced to move ,or will be carefully moved to special accommodations ,deals

will be struck to reduce the immediate effects .

学科网(北京)股份有限公司
The lasting effects of these trade-offs are another matter .the 21 stcentury development of

the American west as an ideal place for alternative energy is going to throw off a lot of power

and money to do a lot of good .but it is just as likely that they will be spent wastefully and

will leave new problems behind ,just like the railroad and the highway .

The money set aside in negotiated trade –offs and the institution that control will shape

the west far beyond the immediate footprint of power plants and transmission lines .so let’s

remember the effects of the railroad and the highways as we construct these new power plants

in the west .

94.what was the problem caused by the construction of the railways ?

A.small towns along the railways became abandoned .

B.some railroad stops remained .

C.land in the west was hard to manage .

D.land grants went into private hands.

95.what is the major concern in the development of alternative energy according to the last

two paragraphs ?

A.the transmission of power B.the use of money and power

C.the conservation of solar energy D.the selection of an ideal place

96.what is the author ‘s attitude towards building solar plants ?

A.cautious B.approving C.doubtful D.disapproving

97.which is the best title for the passage ?

A.how the railways have affected the west

B.how solar energy could reshape the west

C.how the effects of power plants can be reduced

D.how the problems of the highways have been settled

【答案】94.C 95.B 96.A 97.B

【解析】本文为说明文。文章说明的是一种新的太阳能电力系统是如何或好或坏的影

响美国西部地区的。
94.细节理解题。由第二段 The 19th century saw land grants(政府拨地) offered to railroad

companies to build the transcontinental railroads, leaving public land in between privately

owned land. 和 … the landownership has presented unique challenges to land

management ,with the completion or the interstate highway system 可知铁路的建设使政府

拨地落到私人的手里,西部的土地很难管理。故答案为 C。

95.细节理解题。从倒数第二段的句子: There are chances for that power and money to do a


lot of good. But it is just as likely that they will be spent wastefully and will leave new

problems behind, just like the railroads and the highways. 可知在新能源的使用方面主要担

心的是钱和 能源的使用。故答案为 B。

96.作者态度题。作者从铁路和公路在西部出现的好处和坏处,说明 solar plants 在西部也

会出现同样的问题,故答案为 A。

97. 主旨 大意 题。 从文 章的 第一 段的 句子 : The same is true of big solar plants and the

power lines that will be laid down to move electricity around. 可知这篇文章讲的是太阳能会

如何塑造西部故答案为 B。

109.(2010 年,湖北卷)

Have you winterized your horse yet? Even though global warming may have made our

climate more mild, many animals are still hibernating( 冬眠 ) .It’s too bad that humans can’t

hibernate. In fact, as a species, we almost did.

Apparently, at times in the past , peasants in France liked a semi-state of human

hibernation . So writes Graham Robb, a British scholar who has studied the sleeping habits of

the French peasants. As soon as the weather turned cold people all over France shut

themselves away and practiced the forgotten art of doing nothing at all for months on end.

In line with this, Jeff Warren, a producer at CBC Radio’s The Current, tells us that the

way we sleep has changed fundamentally since the invention of artificial( 人 造的 ) lighting

and the electric bulb.

When historians began studying texts of the Middle Ages, they noticed something

referred to as “first sleep”, which was not clarified, though. Now scientists are telling us our

ancestors most likely slept in separate periods. The business of eight hours’ uninterrupted

sleep is a modern invention.

In the past , without the artificial light of the city to bathe in, humans went to sleep when

it became dark and then woke themselves around midnight. The late night period was known

as ”The Watch” It was when people actually kept watch against wild animals ,although many

of them simply moved around or visited family and neighbours .

According to some sleep researchers, a short period of insomnia( 失眠) at midnight is not

a disorder .It is normal . Humans can experience another state of consciousness around their

sleeping, which occurs in the brief period before we fall asleep or wake ourselves in the

morning .This period can be an extraordinarily creative time for some people .The impressive

inventor, Thomas Edison , used this state to hit upon many of his new ideas.

学科网(北京)股份有限公司
Playing with your sleep rhythms can be adventurous ,as anxiety may set in. Medical

science doesn’t help much in this case. It offers us medicines for a full night’s continuous

sleep, which sounds natural ; however, according to Warren’s theory ,it is really the opposite

of what we need.

106.The example of the French peasants shows the fact that________.

A.people might become lazy as a result of too much sleep

B.there were signs of hibernation in human sleeping habits

C.people tended to sleep more peacefully in cold weather

D.winter was a season for people to sleep for months on end

107.The late night was called “The Watch”because it was a time for people______.

A.to set traps to catch animals

B.to wake up their family and neighbours

C.to remind others of the time

D.to guard against possible dangers

108.What does the author advise people to do ?

A.Sleep in the way animals do.

B.Consult a doctor if they can’t sleep.

C.Follow their natural sleep rhythm.

D.Keep to the eight-hour sleep pattern.

109.What is the author’s purpose in writing the passage?

A.To give a prescription for insomnia.

B.To urge people to sleep less.

C.To analyze the sleep pattern of modern people.

D.To throw new light on human sleep.

【答案】106.B 107.D 108.C 109.D

【解析】本文是一篇说明文。文章是对人类睡眠的新阐述。
106.细节理解题。本题是对“论点——论据”的考查,即考查该事例用来证明何种观

点。题干的关键词是 example of the French peasants ,回原文定位到第二自然段。该论

据与第一自然段的论点之问有一个衔接过渡句, In fact,as a species,we almost did. 这

里的 did 指代上文提到的 hibernate,因此本题正确答案为 B。

107. 推理判断题。根据第五段中 The late night period was known as "The Watch". It was

when people actually kept watch against wild animals, although many of them simply moved

around or visited family and neighbours. 可知,深夜被称为"The Watch",因为这是人们防

范可能的危险的时刻。故选 D。
108. 推 理 判 断 题 。 根 据 第 六 段 According to some sleep researchers, a short period of

insomnia ( 失眠 ) at midnight is not a disorder. It is normal. Humans can experience another

state of consciousness around their sleeping, which occurs in the brief period before we fall

asleep or wake ourselves in the morning. This period can be an extraordinarily creative time

for some people.可知,作者建议人们遵循他们的自然睡眠节奏。故选 C。

109. 目 的 意 图 题 。 根 据 It's too bad that humans can't hibernate. In fact, as a species, we

almost did.

和第一段最后一段 Playing with your sleep rhythms can be adventurous, as anxiety may set

in. Medical science doesn't help much in this case. It offers us medicines for a full night's

continuous sleep, which sounds natural; however, according to Warren's theory, it is really the

opposite of what we need.可知,本文是对人类睡眠的新阐述。故选 D。

学科网(北京)股份有限公司

You might also like